Sie sind auf Seite 1von 192

School Year 2015-2016 REMEDIAL LAW REVIEW II - ATTY.

BRONDIAL

I. Provisional Remedies (Rules 57-71)


2. Requirements
A. Preliminary Attachment (Rule 57)
Sec. 3. Affidavit and bond required.
 A provisional remedy issued upon the order of the court where an
action is pending to be levied upon the property of the defendant so An order of attachment shall be granted only when it appears by the
the property may be held by the sheriff as a security for the affidavit of the applicant, or of some other person who personally
satisfaction of whatever judgment may be rendered by the court. knows the facts, that a sufficient cause of action exists, that the case
is one of those mentioned in section 1 hereof, that there is no other
1. Grounds (MEMORIZE) sufficient security for the claim sought to be enforced by the action,
Section 1. Grounds upon which attachment may issue. and that the amount due to the applicant, or the value of the property
At the commencement of the action or at any time before entry of the possession of which he is entitled to recover, is as much as the
judgment, a plaintiff or any proper party may have the property of the sum for which the order is granted above all legal counterclaims. The
adverse party attached as security for the satisfaction of any judgment affidavit, and the bond required by the next succeeding section, must
that may be recovered in the following cases: be duly filed with the court before the order issues.
(a) In an action for the recovery of a specified amount of money or
damages, other than moral and exemplary, on a cause of action There are 4 requirements under Section 3:
arising from law, contract, quasi-contract, delict or quasi-delict against
a party who is about to depart from the Philippines which intent to First, valid cause of action. A provisional remedy cannot exist without
defraud his creditors; a principal action. The valid cause of action means there must be a
(b) In an action for money or property embezzled or fraudulently principal action.
misapplied or converted to his own use by a public officer, or an
Second, it must be on the basis of any of the grounds provided for in
officer of a corporation, or an attorney, factor, broker agent, or clerk,
Section 1 of Rule 57. Outside of the 6 items enumerated, you cannot
in the course of his employment as such, or by other person in a
apply for a writ of preliminary attachment.
fiduciary capacity, or for a willful violation of duty;
(c) In an action to recover the possession of property unjustly or Third, there must be no sufficient security. Attachment is to secure
fraudulently taken, detained or converted, when the property, or any the satisfaction of the judgment. If there is already a security (like a
part thereof, has been concealed, removed, or disposed of to prevent its mortgage), then you cannot apply anymore for preliminary
being found or taken by the applicant or an authorized person;
attachment.
(d) In an action against a party who has been guilty of a fraud in
contracting the debt or incurring the obligation upon which the action Finally, the value of the property must be equal to that of the property
is brought, or in the performance thereof; sought to be attached less all other counterclaims. This applies to
(e) In an action against a party who has removed or disposed of his the concept of redemptioner.
property, or is about to do so, with intent to defraud his creditors; or
(f) In an action against a party who does not reside and is not found in Stages in the issuance of writ
the Philippines, or on whom summons may be served by publication. 1. The court issues an order granting the application;

1 Aicka Singson
Notes in Remedial Law Review II
Provisional Remedies, Special Civil Actions, Special Proceedings and Evidence
School Year 2015-2016 REMEDIAL LAW REVIEW II - ATTY. BRONDIAL

2. The writ is issued pursuant to the order; dispose of his properties. So even if you win, you will have nothing to
3. The writ is implemented. execute. This is where third party claims come in.

“At the commencement of the action or at any time before entry of In actual practice, the unwritten purpose of preliminary attachment, is
judgment” – you can apply for a writ of preliminary attachment even not to secure the satisfaction of judgment, is to force the parties to
with the SC because a case may reach the SC and there is no entry come to the negotiating table. Example: A businessman is engaged in
of judgment yet. General rule: It is only the trial court that can execute merchandising. A filed a case against the businessman and availed of
a judgment as a matter of right. Even if the case started with the MTC, a preliminary attachment. His hands will be ties and he cannot do
it went up and up to the SC, when you try to execute it, you have to business anymore. What should he do? He will come to the
do so with the MTC. Only when there is an entry of judgment can you negotiating table and would want to settle.
execute it. Before entry of judgment, you can always apply for a writ
of preliminary attachment. Thus, you can apply it with the SC if there (2) for the court to acquire jurisdiction over the res. Jurisdiction over
is no entry of judgment yet. the res is not necessary for the case to continue because as long as
the court has jurisdiction over the person of the defendant, the case
For what purpose? will proceed. But if the court cannot acquire jurisdiction over the
(1) “as security for the satisfaction of any judgment.” You are not to person of the defendant, then at least, if it is a recovery action, you
satisfy the judgment because satisfaction of judgment is with Rule 39. attach the property and even if the court doe not acquire jurisdiction
Here, you only want to secure the satisfaction of a judgment. over the person of the defendant, the case can continue. The
limitation is when you execute, you are limited only to the value of the
How? When you file a case and you want to secure the satisfaction of property. You cannot go beyond that which has been attached.
a judgment, even at the commencement of the action, you can
already ask for a writ of preliminary attachment. Because if there is Example: The obligation is P1,000,000. You were able to sell the
already an attached property and the judgment has already been property for P800,000. You can claim only up to the attached
entered, you don’t go to Rule 39. You go to sale on attachment and property. It is only the res that is answerable. But if you file a case and
not sale on execution. you were able to summon the defendant, and then you levy the
property, you can go beyond the value of the attached property
Preliminary attachment is seldom used in small cases. If the debt is because the court has jurisdiction over the person.
P1,000,000, forget about preliminary attachment. But if someone
owes you P10,000,000, attach the property so that when you have an 3. Manner of Attaching
entry of judgment, you’ll have nothing more to execute. Sec. 5. Manner of attaching property.
The sheriff enforcing the writ shall without delay and with all
Example: In 1990, you filed a case against Henry Sy. Attach the reasonable diligence attach, to await judgment and execution in the
building of Henry Sy at the commencement of the action. The case action, only so much of the property in the Philippines of the party
continued until 2000. Between 1990 and 2000, a lot can happen. If against whom the writ is issued, not exempt from execution, as may
the lawyer of Sy sees that they will lose, the lawyer will advice him to be sufficient to satisfy the applicant's demand, unless the former

2 Aicka Singson
Notes in Remedial Law Review II
Provisional Remedies, Special Civil Actions, Special Proceedings and Evidence
School Year 2015-2016 REMEDIAL LAW REVIEW II - ATTY. BRONDIAL

makes a deposit with the court from which the writ is issued, or gives and a notice stating that the stock or interest of the party against
a counterbond executed to the applicant, in an amount equal to the whom the attachment is issued is attached in pursuance of such writ.
bond fixed by the court in the order of attachment or to the value of
the property to be attached, exclusive of costs. No levy on DEBTS AND CREDITS, including bank deposits, financial interest,
attachment pursuant to the writ issued under section 2 hereof shall royalties, commissions and other personal property not capable of
be enforced unless it is preceded, or contemporaneously manual delivery shall be attached by leaving with the person owing
accompanied, by service of summons, together with a copy of the such debts, or in possession or control of such credits or other
complaint, the application for attachment, the applicant's affidavit and personal property, or with his agent, a copy of the writ, and notice
bond, and the order and writ of attachment, on the defendant within that such properties are attached.
the Philippines.
As to interest of the party against whom attachment is issued in
The requirement of prior or contemporaneous service of summons
property BELONGING TO THE ESTATE OF THE DECEDENT are
shall not apply where the summons could not be served personally
attached by giving a copy of the writ and notice to the executor or
or by substituted service despite diligent efforts, or the defendant is a
administrator and the office of the clerk of court where is the estate is
resident of the Philippines temporarily absent therefrom, or the
being settled.
defendant is a non-resident of the Philippines, or the action is one in
rem or quasi in rem. • When the writ of preliminary attachment is issued, that should be
Notes: implemented after the court acquires jurisdiction over the person of
the defendant. As far as implementation is concerned, the court must
The sheriff enforcing the writ shall without delay and with all
have acquired jurisdiction over the person of the defendant. Without
reasonable diligence attach, to await judgment and execution in the
that, the implementation is null and void. This is the doctrine laid
action, only so much of the property in the Philippines of the party
down in Mangila vs CA.
against whom the writ is issued, not exempt from execution, as may
be sufficient to satisfy the applicant's demand.
4. Discharge of Attachment
In attaching REAL PROPERTY, or growing crops thereon or any
Sec. 12. Discharge of attachment upon giving counterbond.
interest therein, a copy of the order shall be filed with the registry of
deeds along with a description of the property attached and by After a writ of attachment has been enforced, the party whose
leaving a copy of such order with the occupant of the property, if any property has been attached, or the person appearing on his behalf,
or such other person or his agent if found within the province. may move for the discharge of the attachment wholly or in part on
the security given. The court shall, after due notice and hearing,
In attaching PERSONAL PROPERTY capable of manual delivery -
order the discharge of the attachment if the movant makes a cash
by taking and safely keeping it in his custody after issuing the
deposit, or files a counter-bond executed to the attaching party with
corresponding receipt therefor.
the clerk of the court where the application is made, in an amount
As to STOCKS OR SHARES, or an interest thereon, by leaving with equal to that fixed by the court in the order of attachment, exclusive
the president or managing agent of the company, a copy of the writ, of costs. But if the attachment is sought to be discharged with

3 Aicka Singson
Notes in Remedial Law Review II
Provisional Remedies, Special Civil Actions, Special Proceedings and Evidence
School Year 2015-2016 REMEDIAL LAW REVIEW II - ATTY. BRONDIAL

respect to a particular property, the counter-bond shall be equal to (1) Under section 5 and section 12, the way to discharge is
the value of that property as determined by the court. In either case, through the posting of a counterbond.
the cash deposit or the counter-bond shall secure the payment of
any judgment that the attaching party may recover in the action. A How much is the counterbond? Equal to the bond posted by the
notice of the deposit shall forth with be served on the attaching party. attaching creditor. This is the only kind of counterbond (under
Upon the discharge of an attachment in accordance with the Section 5), in the Rules of Court, that answers for the judgment. It is
provisions of this section, the property attached, or the proceeds of not a bond that just answers damages.
any sale thereof, shall be delivered to the party making the deposit or
Example: Judgment is for P10,000,000 which is the principal
giving the counter-bond, or to the person appearing on his behalf,
obligation. Damages is for P1,000,000. The totality is P11,000,000.
the deposit or counter-bond aforesaid standing in place of the
Other bonds answer only for the P1,000,000. But the counterbond in
property so released. Should such counter-bond for any reason to be
preliminary attachment answers for the P11,000,000 – it includes the
found to be or become insufficient, and the party furnishing the same
entire judgment
fail to file an additional counter-bond, the attaching party may apply
for a new order of attachment. (2) Under section 13, you file a motion questioning or assailing
the propriety or regularity of the attachment. This is possible when
Sec. 13. Discharge of attachment on other grounds.
the party whose property has been attached files a motion to set
The party whose property has been ordered attached may file a aside or discharge the attachment and during the hearing of the
motion with the court in which the action is pending, before or after motion, he proves that:
levy or even after the release of the attached property, for an order to
1) The attachment was improperly or irregularly issued or
set aside or discharged the attachment on the ground that the same
enforced; or
was improperly or irregularly issued or enforced, or that the bond is
2) The bond of the attaching creditor is insufficient; or
insufficient. If the attachment is excessive, the discharge shall be
3) The attachment is excessive and must be discharged as to
limited to the excess. If the motion be made on affidavits on the part
the excess; or
of the movant but not otherwise, the attaching party may oppose the
4) The property is exempt from execution, and as such is also
motion by counter-affidavits or other evidence in addition to that on
exempt from preliminary attachment.
which the attachment was made. After due notice and hearing, the
court shall order the setting aside or the corresponding discharge of
• “Improperly” (e.g. writ of attachment was not based on the grounds in
the attachment if it appears that it was improperly or irregularly
Sec. 1)
issued or enforced, or that the bond is insufficient, or that the
• “Irregularly” (e.g. writ of attachment was executed without previous
attachment is excessive, and the defect is not cured forthwith.
or contemporaneous service of summons)
NOTES:
5. Third Party Claim
There are 2 ways of discharging or lifting the writ of preliminary Sec. 14. Proceedings where property claimed by third person.
attachment: Sections 5, 12 and 13

4 Aicka Singson
Notes in Remedial Law Review II
Provisional Remedies, Special Civil Actions, Special Proceedings and Evidence
School Year 2015-2016 REMEDIAL LAW REVIEW II - ATTY. BRONDIAL

If the property attached is claimed by any person other than the party of such right or title. The affidavit must be served upon the sheriff
against whom attachment had been issued or his agent, and such and the attaching party. The sheriff shall not be bound to keep the
person makes an affidavit of his title thereto, or right to the property under attachment except if the attaching party files a bond
possession thereof, stating the grounds of such right or title, and approved by the court.
serves such affidavit upon the sheriff while the latter has possession b) The third person may invoke the court’s authority in the same
of the attached party, and a copy thereof upon the attaching party, case and move for a summary hearing on his claim to decide if the
the sheriff shall not be bound to keep the property under attachment, sheriff has acted correctly or not.
unless the attaching party or his agent, on demand of the sheriff, c) The third party may file a separate action to nullify the levy with
shall file a bond approved by the court to indemnify the third-party damages resulting from the unlawful levy and seizure. This action
claimant in a sum not less than the value of the property levied upon. may be totally distinct from the case in which the attachment was
In case of disagreement as to such value, the same shall be decided issued.
by the court issuing the writ of attachment. No claim for damages for
the taking or keeping of the property may be enforced against the Redemptioner - creditor who has a lien subsequent to the lien under
bond unless the action therefor is filed within one hundred twenty which the property is sold. If you are an attaching creditor and the
(120) days from the date of the filing of the bond. property is sold pursuant to a lien on sale on attachment not
The sheriff shall not be liable for damages for the taking or keeping execution, there will be a lot of redemptioners because you applied
of such property, to any such third-party claimant, if such bond shall for the writ of preliminary attachment at the commencement of the
be filed. Nothing herein contained such prevent such claimant or any action.
third person from vindicating his claim to the property, or prevent the
Example: You file a case against Henry Sy and you attach the
attaching party from claiming damages against a third-party claimant
building worth P100,000,000. His debt is only P50,000,000. There is
who filed a frivolous or plainly spurious claim, in the same or a
a balance of P50,000,000. A lot of other people will attach that
separate action.
building also. B, whose credit is P10,000,000 will attach the property.
When the writ of attachment is issued in favor of the Republic of the
The balance is now P40,000,000. C will attach for P30,000,000. This
Philippines, or any officer duly representing it, the filing of such bond
complies with section 3; value of the property less all the
shall not be required, and in case the sheriff is sued for damages as
counterclaims.
a result of the attachment, he shall be represented by the Solicitor
General, and if held liable therefor, the actual damages adjudged by When you sell the property on attachment, there is now sale on
the court shall be paid by the National Treasurer out of the funds to attachment, the purchaser buys it. It was redeemed by redemptioner.
be appropriated for the purpose. Who are the other redemptioners? Subsequent lien holders (B and
Notes: C). Lien holders subsequent to the lien under which the property was
The third party may resort to any of the following remedies which are sold. The lien now under which the property is being sold is not the
cumulative and thus could be resorted independently and separately lien on execution but the lien on attachment, which happened a long
from the others: time ago.
a) He may avail of the remedy of terceria – by making an affidavit of
his title thereto or his right to possession thereof, stating the grounds

5 Aicka Singson
Notes in Remedial Law Review II
Provisional Remedies, Special Civil Actions, Special Proceedings and Evidence
School Year 2015-2016 REMEDIAL LAW REVIEW II - ATTY. BRONDIAL

6. Claim for Damages CASES:

Sec. 20. Claim for damages on account of improper, irregular or a. Mangila vs. CA (387 SCRA)
excessive attachment. Facts: Petitioner contracted the freight forwarding services of private
respondent for shipment of petitioner’s products to Guam. Petitioner
An application for damages on account of improper, irregular or agreed to pay private respondent cash on delivery. However,
excessive attachment must be filed before the trial or before the trial Petitioner failed to pay private respondent shipping charges.
or before appeal is perfected or before the judgment becomes
executory, with due notice to the attaching party and his surety or Despite several demands, petitioner never paid private respondent.
sureties, setting forth the facts showing his right to damages and the Thus, private respondent filed Civil Case for collection of sum of
amount thereof. Such damages may be awarded only after proper money.
hearing and shall be included in the judgment on the main case. On Sheriff’s Return, there was showing that summons was not
served on petitioner for the reason that petitioner transferred her
If the judgment of the appellate court be favorable to the party
residence and further, that petitioner had left the Philippines for
against whom the attachment was issued, he must claim damages
Guam.
sustained during the pendency of the appeal by filing an application
Construing petitioner’s departure from the Philippines as done with
in the appellate court, with notice to the party in whose favor the
intent to defraud her creditors, private respondent filed a Motion for
attachment was issued or his surety or sureties, before the judgment
Preliminary Attachment. On September 26, 1988, the trial court
of the appellate court becomes executory. The appellate court may
issued an Order of Preliminary Attachment against petitioner. The
allow the application to be heard and decided by the trial court.
following day, the trial court issued a Writ of Preliminary Attachment.
Nothing herein contained shall prevent the party against whom the On October 24, 1988, Sheriff Alfredo served on petitioner’s
attachment was issued from recovering in the same action the household help the Notice of Levy with the Order, Affidavit and Bond.
damages awarded to him from any property of the attaching party
not exempt from execution should the bond or deposit given by the Petitioner filed an Urgent Motion to Discharge Attachment without
submitting herself to the jurisdiction of the trial court pointing out that
latter be insufficient or fail to fully satisfy the award.
she had not been served a copy of the Complaint and the summons.
NOTE: A claim for cannot be brought in a separate action. You bring Hence, petitioner claimed the court had not acquired jurisdiction over
up claim for damages as long as the judgment has not been entered her person.
yet. You can file it with the trial court or appellate court.
The trial court granted the Motion to Discharge Attachment upon
It does not mean that if you lose in the provisional remedy, that you filing of petitioner’s counter-bond. The trial court, however, did not
will lose the principal action. They are two separate and distinct rule on the question of jurisdiction and on the validity of the writ of
matters. Likewise, it does not mean that if you win in the provisional preliminary attachment.
remedy of preliminary attachment, that you will win the principal Private respondent applied for an alias summons it was only on
action. January 26, 1989 that summons was finally served on petitioner.

6 Aicka Singson
Notes in Remedial Law Review II
Provisional Remedies, Special Civil Actions, Special Proceedings and Evidence
School Year 2015-2016 REMEDIAL LAW REVIEW II - ATTY. BRONDIAL

Trial Court: Ordering petitioner to pay respondent P109,376.95 plus manner against the defendant. Any order issuing from the Court will
18 percent interest per annum, 25 percent attorney’s fees and costs not bind the defendant.
of suit.
CA affirmed the decision of the trial court. The Court of Appeals In the instant case, the Writ of Preliminary Attachment was issued on
upheld the validity of the issuance of the writ of attachment. September 27, 1988 and implemented on October 28, 1988.
However, the alias summons was served only on January 26, 1989
ISSUE: WHETHER RESPONDENT COURT ERRED IN NOT or almost three months after the implementation of the writ of
HOLDING THAT THE WRIT OF ATTACHMENT WAS attachment.
IMPROPERLY ISSUED AND SERVED The trial court had the authority to issue the Writ of Attachment on
September 27 since a motion for its issuance can be filed "at the
HELD: Improper Issuance and Service of Writ of Attachment commencement of the action." However, on the day the writ was
In Davao Light & Power Co., Inc. v. Court of Appeals, this Court implemented, the trial court should have, previously or
clarified the actual time when jurisdiction should be had: simultaneously with the implementation of the writ, acquired
"It goes without saying that whatever be the acts done by the Court jurisdiction over the petitioner. Yet, as was shown in the records of
prior to the acquisition of jurisdiction over the person of defendant - the case, the summons was actually served on petitioner several
issuance of summons, order of attachment and writ of attachment - months after the writ had been implemented.
do not and cannot bind and affect the defendant until and unless Private respondent, nevertheless, claims that the prior or
jurisdiction over his person is eventually obtained by the court, either contemporaneous service of summons contemplated in Section 5 of
by service on him of summons or other coercive process or his Rule 57 provides for exceptions. Among such exceptions are "where
voluntary submission to the court’s authority. Hence, when the sheriff the summons could not be served personally or by substituted
or other proper officer commences implementation of the writ of service despite diligent efforts or where the defendant is a resident
attachment, it is essential that he serve on the defendant not only a temporarily absent therefrom x x x." Private respondent asserts that
copy of the applicant’s affidavit and attachment bond, and of the when she commenced this action, she tried to serve summons on
order of attachment, as explicitly required by Section 5 of Rule 57, petitioner but the latter could not be located. Furthermore,
but also the summons addressed to said defendant as well as a copy respondent claims that petitioner was not even in Pampanga; rather,
of the complaint xxx." she was in Guam purportedly on a business trip.
Furthermore, we have held that the grant of the provisional remedy Private respondent never showed that she effected substituted
of attachment involves three stages: first, the court issues the order service on petitioner after her personal service failed. Likewise, if it
granting the application; second, the writ of attachment issues were true that private respondent could not ascertain the
pursuant to the order granting the writ; and third, the writ is whereabouts of petitioner after a diligent inquiry, still she had some
implemented. For the initial two stages, it is not necessary that other recourse under the Rules of Civil Procedure.
jurisdiction over the person of the defendant be first obtained. In conclusion, we hold that the alias summons belatedly served on
However, once the implementation of the writ commences, the court petitioner cannot be deemed to have cured the fatal defect in the
must have acquired jurisdiction over the defendant for without such enforcement of the writ. The trial court cannot enforce such a
jurisdiction, the court has no power and authority to act in any coercive process on petitioner without first obtaining jurisdiction over

7 Aicka Singson
Notes in Remedial Law Review II
Provisional Remedies, Special Civil Actions, Special Proceedings and Evidence
School Year 2015-2016 REMEDIAL LAW REVIEW II - ATTY. BRONDIAL

her person. The preliminary writ of attachment must be served after provided for the transfer to the government of certain assets of PNB
or simultaneous with the service of summons on the defendant in exchange for which the government would assume certain
whether by personal service, substituted service or by publication as liabilities of PNB. Among those liabilities assumed were L/C in favor
warranted by the circumstances of the case. The subsequent service of Chuidian in the amount of US$4,400,000.00.
of summons does not confer a retroactive acquisition of jurisdiction
over her person because the law does not allow for retroactivity of a On July 30, 1987, the government filed before the Sandiganbayan
belated service. Civil Case against the Marcos spouses Chuidian [et al.] for the
reconveyance, and restitution of all forms of wealth allegedly
b. Chuidian vs Sandiganbayan (349 SCRA) procured illegally and stashed away by the defendants.

FACTS: The instant petition of Chuidian arises from transactions that While the case was pending, the Republic filed a motion for issuance
were entered into by the government in the penultimate days of the of a writ of attachment over L/C.
Marcos administration. As a favored business associate of the
Accordingly, an order of attachment was issued ordering the SB
Marcoses, Chuidian allegedly induced the officers of
Sheriff to attach PNB L/C for safekeeping as security for the
PHILGUARANTEE to facilitate the issuance of a loan guarantee in
satisfaction of judgment in SB Civil Case. Chuidian filed a motion to
favor of the Asian Reliability Company, Incorporated (ARCI). ARCI,
lift the attachment.
98% of which was allegedly owned by Chuidian, was granted a loan
guarantee of US$25,000,000.00. Subsequently, Chuidian filed a motion to require the Republic to
deposit the dormant L/C in an interest bearing account. He proposed
With the advent of the Aquino administration, the PCGG exerted
that it would be to the benefit of all if the Sandiganbayan requires
earnest efforts to search and recover illegally acquired money and
PNB to deposit the full amount to a Sandiganbayan trust account at
other assets.
any bank in order to earn interest while awaiting judgment of the
Petitioner Chuidian was among those whose assets were action.
sequestered. On May 30, 1986, the PCGG issued a Sequestration
The Sandiganbayan denied Chuidian's motion to lift attachment.
Order directing the PNB to place under its custody, for and in behalf
of the PCGG, the irrevocable L/C (No. SFD-005-85). On the same day, the Sandiganbayan issued another Resolution
denying Chuidian's motion to require deposit of the attached L/C in
In the meantime, Philguarantee filed a motion before the Superior
an interest bearing account.
Court of Santa Clara County of California seeking to vacate the
stipulated judgment containing the settlement between Philguarantee The Sandiganbayan [acting on Chuidian's motion for reconsideration]
and Chuidian. gave due course to Chuidian's plea for the attached L/C to be
deposited in an interest bearing account, on the ground that it will
Meanwhile, a Deed of Transfer was executed between the Republic
redound to the benefit of both parties. The Sandiganbayan declared
thru then Secretary of Finance Ongpin and then PNB President
the national government as the principal obligor of the L/C even
Espiritu, to facilitate the rehabilitation of PNB. The said Deed

8 Aicka Singson
Notes in Remedial Law Review II
Provisional Remedies, Special Civil Actions, Special Proceedings and Evidence
School Year 2015-2016 REMEDIAL LAW REVIEW II - ATTY. BRONDIAL

though the liability remained in the books of the PNB for accounting cases, there is no actual substantial relief to which petitioner would
and monitoring purposes. be entitled to and which would be negated by the dismissal of the
petition.
The Sandiganbayan, however, denied Chuidian's motion for
reconsideration of the denial of his motion to lift attachment. c. Alejando Ng Wee vs. Tankiansee (545 SCRA)

ISSUE: WON the attachment should be lifted Petitioner Alejandro Ng Wee, a valued client of Westmont Bank (now
United Overseas Bank), made several money placements totaling
HELD: Chuidian cannot ask for the discharge or lifting of the writ of P210,595,991.62 with the bank's affiliate, Westmont Investment
preliminary attachment pursuant to section 13. Had you wanted it Corporation (Wincorp).
discharged, you could have easily posted a counterbond under
section 12. But you did avail of section 13 and that is not possible Petitioner received disturbing news on Wincorp's financial condition
because if the ground in the principal action as well as in the prompting him to inquire about and investigate the company's
provisional remedy of preliminary attachment is the same, we cannot operations and transactions with its borrowers. Discovering that the
discharge the writ pursuant to section 13 because we will be company extended a loan equal to his total money placement to a
resolving the principal action. You cannot resolve a principal action corporation [Power Merge] with a subscribed capital of only P37.5M.
by a mere motion.
Under the scheme, petitioner's money placements were transferred
It bears to stress at this juncture that the Sandiganbayan's October without his knowledge and consent to the loan account of Power
8, 2002 Resolution and July 10, 2003 Order juxtaposed with its Merge through an agreement that virtually freed the latter of any
assailed October 30, 2003 resolution to veritably enforce and liability. Allegedly, through the false representations of Wincorp and
implement the Court's January 19, 2001 Decision in G.R. No. its officers and directors, petitioner was enticed to roll over his
139941. In a very real sense, therefore, the Sandiganbayan had placements so that Wincorp could loan the same to Virata/Power
already granted what petitioner Chuidian was then asking that court Merge. Finding that Virata purportedly used Power Merge as a
and, presently, this Court, i.e., the execution of said January 19, conduit and connived with Wincorp's officers and directors to
2001 Decision, albeit a formal writ of execution has not been issued. fraudulently obtain for his benefit without any intention of paying the
Nonetheless, the October 30, 2003 ruling already commanded the said placements, petitioner instituted a case for damages with the
Sheriff of the Sandiganbayan to take steps to ensure that PNB, at (RTC). One of the defendants impleaded in the complaint is herein
the risk of contempt, comply immediately with this Court's directive. respondent Manuel Tankiansee, Vice-Chairman and Director of
Wincorp.
At bottom then, Chuidian's petition in G.R. No. 156383 has been
rendered moot and academic by the Sandiganbayan's October 30, On October 26, 2000, on the basis of the allegations in the complaint
2003 Resolution aforestated and its earlier resolution of the same Affidavit of petitioner, the trial court ordered the issuance of a writ of
tenor. A moot and academic case or issue is one that ceases to preliminary attachment against the properties not exempt from
present a justifiable controversy by virtue of supervening events, so execution of all the defendants in the civil case subject, among
that a determination thereof would be of no practical value. In such

9 Aicka Singson
Notes in Remedial Law Review II
Provisional Remedies, Special Civil Actions, Special Proceedings and Evidence
School Year 2015-2016 REMEDIAL LAW REVIEW II - ATTY. BRONDIAL

others, to petitioner's filing of aP50M-bond. The writ was, of respondent is Section 1(d) of Rule 57 of the Rules of Court which
consequently, issued on November 6, 2000. pertinently reads:

Arguing that the writ was improperly issued and that the bond (d) In an action against a party who has been guilty of a fraud in
furnished was grossly insufficient, respondent, moved for the contracting the debt or incurring the obligation upon which the action
discharge of the attachment which was denied. is brought, or in the performance thereof.

Respondent filed before the trial court another Motion to Discharge For a writ of attachment to issue under this rule, the applicant must
Attachment, re-pleading the grounds he raised in his first motion but sufficiently show the factual circumstances of the alleged fraud
raising the following additional grounds: (1) that he was not present because fraudulent intent cannot be inferred from the debtor's mere
in Wincorp's board meetings approving the questionable non-payment of the debt or failure to comply with his obligation. The
transactions;21 and (2) that he could not have connived with applicant must then be able to demonstrate that the debtor has
Wincorp and the other defendants because he and Pearlbank intended to defraud the creditor. In Liberty Insurance Corporation v.
Securities, Inc., in which he is a major stockholder, filed cases Court of Appeals, we explained as follows:
against the company as they were also victimized by its fraudulent
schemes which the trial court denied. To sustain an attachment on this ground, it must be shown that the
debtor in contracting the debt or incurring the obligation intended to
With the denial of its motion for reconsideration, respondent filed a defraud the creditor. The fraud must relate to the execution of the
certiorari petition before the CA and rendered the assailed Decision agreement and must have been the reason which induced the other
reversing and setting aside the aforementioned orders of the trial party into giving consent which he would not have otherwise given.
court and lifting the Writ of Preliminary Attachment to the extent that To constitute a ground for attachment in Section 1 (d), Rule 57 of the
it concerned respondent's properties. Petitioner moved for the Rules of Court, fraud should be committed upon contracting the
reconsideration of the said ruling, but the CA denied. obligation sued upon. A debt is fraudulently contracted if at the time
of contracting it the debtor has a preconceived plan or intention not
ISSUE: WON THE CA ERRED IN LIFTING THE WRIT to pay, as it is in this case. Fraud is a state of mind and need not be
proved by direct evidence but may be inferred from the
HELD: NO. Respondent counters that nowhere in the said affidavit
circumstances attendant in each case.
does petitioner mention the name of respondent and any specific act
committed by the latter to defraud the former. A writ of attachment In the instant case, petitioner's Affidavit is bereft of any factual
can only be granted on concrete and specific grounds and not on statement that respondent committed a fraud. The affidavit narrated
general averments quoting perfunctorily the words of the Rules only the alleged fraudulent transaction between Wincorp and Virata
and/or Power Merge but as to the participation of respondent in the
We agree with respondent's contentions and deny the petition.
said transaction, the affidavit merely states that respondent, an
In the case at bench, the basis of petitioner's application for the officer and director of Wincorp, connived with the other defendants in
issuance of the writ of preliminary attachment against the properties the civil case to defraud petitioner of his money placements. No
other factual averment or circumstance details how respondent

10 Aicka Singson
Notes in Remedial Law Review II
Provisional Remedies, Special Civil Actions, Special Proceedings and Evidence
School Year 2015-2016 REMEDIAL LAW REVIEW II - ATTY. BRONDIAL

committed a fraud or how he connived with the other defendants to claimed that Solar has already paid the entire purchase price,
commit a fraud in the transaction sued upon. In other words, however Nicanor only remitted P9, 000,000 out of the P28, 000,000
petitioner has not shown any specific act or deed to support the sum they are entitled to and that Nicanor had acquired a house and
allegation that respondent is guilty of fraud. lot and a car (which he registered in the names of his children).
Despite the repeated verbal and written demands, Nicanor failed to
The affidavit, being the foundation of the writ, must contain such remit the balance prompting the petitioners to file a complaint for
particulars as to how the fraud imputed to respondent was committed sum of money against the family Satsatin.
for the court to decide whether or not to issue the writ. Absent any
statement of other factual circumstances to show that respondent, at Petitioners filed an Ex Parte Motion for the Issuance of a Writ of
the time of contracting the obligation, had a preconceived plan or Attachment, alleging among other things, that respondent was about
intention not to pay, or without any showing of how respondent to depart the country and that they are willing to post a bond fixed by
committed the alleged fraud, the general averment in the affidavit court. After filing a Motion for Deputation of Sheriff, which the RTC
that respondent is an officer and director of Wincorp who allegedly granted, it issued a Writ of Attachment (WOA) on November 15. On
connived with the other defendants to commit a fraud, is insufficient November 19, after serving a copy of the WOA upon the Satsatins,
to support the issuance of a writ of preliminary attachment. In the the sheriff levied their real and personal properties. On November
application for the writ under the said ground, compelling is the need 21, the summons and copy of complaint was served upon the
to give a hint about what constituted the fraud and how it was respondents. Respondents filed their answer and a Motion to
perpetrated38 because established is the rule that fraud is never Discharge Writ of Attachment, claiming, among others, that: the bond
presumed.39 Verily, the mere fact that respondent is an officer and was issued before the issuance of WOA, the WOA was issued
director of the company does not necessarily give rise to the before the summons was received. Respondents posted a counter-
inference that he committed a fraud or that he connived with the bond for the lifting of WOA, which was denied along with MR.
other defendants to commit a fraud. While under certain Aggrieved, they filed with CA a Petition for Certiorari, Mandamus and
circumstances, courts may treat a corporation as a mere Prohibition with Preliminary Injunction and TRO under Rule 65. CA
aggroupment of persons, to whom liability will directly attach, this is ruled in favor of respondents and denied petitioners’ MR hence the
only done when the wrongdoing has been clearly and convincingly petition for review on certiorari with the SC.
established.
Issue: W/N CA erred in finding that RTC was guilty of GADALEJ
d. Torres vs. Satsatin (605 SCRA) in the issuance and implementation of the WOA

Facts: Siblings Torres (petitioners) each owned adjacent 20,000 Held: No. A writ of preliminary attachment is defined as a provisional
square meters track of land in Dasmariñas, Cavite. Nicanor Satsatin, remedy issued upon order of the court where an action is pending to
through petitioners’ mother Agripina Aledia, was able to convince the be levied upon the property or properties of the defendant therein,
siblings to sell their property and authorize him via SPA, to negotiate the same to be held thereafter by the sheriff as security for the
for its sale. Nicanor offered to sell the properties to Solar Resources, satisfaction of whatever judgment that might be secured in the said
to which Solar allegedly agreed to buy the three parcels of land plus action by the attaching creditor against the defendant.
the property of one Rustica Aledia for P35, 000,000. Petitioners

11 Aicka Singson
Notes in Remedial Law Review II
Provisional Remedies, Special Civil Actions, Special Proceedings and Evidence
School Year 2015-2016 REMEDIAL LAW REVIEW II - ATTY. BRONDIAL

In the case at bar, the CA correctly found that there was grave abuse implementation of the writ of attachment, served a copy of the
of discretion amounting to lack of or in excess of jurisdiction on the summons upon the respondents in order for the trial court to have
part of the trial court in approving the bond posted by petitioners acquired jurisdiction upon them and for the writ to have binding
despite the fact that not all the requisites for its approval we re effect. Consequently, even if the writ of attachment was validly
complied with. In accepting a surety bond, it is necessary that all t issued, it was improperly or irregularly enforced and, therefore,
cannot bind and affect the respondents.n
he requisites for its approval are met; otherwise, the bond should be
rejected. Moreover, again assuming arguendo that the writ of attachment was
validly issued, although the trial court later acquired jurisdiction over
Moreover, in provisional remedies, particularly that of preliminary the respondents by service of the summons upon them, such belated
attachment, the distinction between the issuance and the service of summons on respondents cannot be deemed to have
implementation of the writ of attachment is of utmost importance to cured the fatal defect in the enforcement of the writ. The trial court
the validity of the writ. The distinction is indispensably necessary to cannot enforce such a coercive process on respondents without first
determine when jurisdiction over the person of the defendant should obtaining jurisdiction over their person. The preliminary writ of
be acquired in order to validly implement the writ of attachment upon attachment must be served after or simultaneous with the service of
his person. summons on the defendant whether by personal service, substituted
service or by publication as warranted by the circumstances of the
In Cuartero v. Court of Appeals, this Court held that the grant of the
case. The subsequent service of summons does not confer a
provisional remedy of attachment involves three stages: first, the
retroactive acquisition of jurisdiction
court issues the order granting the application; second, the writ of
attachment issues pursuant to the order granting the writ; and third,
B. PRELIMINARY INJUNCTION (RULE 58)
the writ is implemented. For the initial two stages, it is not necessary
that jurisdiction over the person of the defendant be first obtained.
However, once the implementation of the writ commences, the court 1. Definition and Classes
must have acquired jurisdiction over the defendant, for without such
Definition: A preliminary injunction is an order granted at any stage of
jurisdiction, the court has no power and authority to act in any
an action or proceeding prior to the judgment or final order, requiring
manner against the defendant. Any order issuing from the Court will
a party or a court, agency or a person to either refrain (prohibitory)
not bind the defendant.
from or to perform (mandatory) a particular act or acts during the
At the time the trial court issued the writ of attachment on November pendency of the action.
15, 2002, it can validly to do so since the motion for its issuance can
Classes:
be filed “at the commencement of the action or at any time before
entry of judgment.” However, at the time the writ was implemented, Requisites for the issuance of preliminary injunction
the trial court has not acquired jurisdiction over the persons of the
respondent since no summons was yet served upon them. The (a) The invasion of the right is material and substantial;
proper officer should have previously or simultaneously with the

12 Aicka Singson
Notes in Remedial Law Review II
Provisional Remedies, Special Civil Actions, Special Proceedings and Evidence
School Year 2015-2016 REMEDIAL LAW REVIEW II - ATTY. BRONDIAL

(b) The right of a complainant is clear and unmistakable; If you are granted a 20-day TRO, that includes the 72 hours. If you
are granted at first the 72-hour TRO, if it is extended, the extension
(c) There is an urgent and permanent necessity for the writ to is 17 days only. The court can never grant beyond 20 days of TRO. It
prevent serious damage. will be grave abuse of discretion on the part of the court to grant
more than 20-day TRO. We are talking about MTC and RTC.
General rule: There can be no injunctive relief or preliminary
injunction without notice and hearing. The writ of preliminary But in the case of the CA, the TRO is good for 60 days.
injunction can only be issued after due notice and hearing. Don’t
confuse the writ of preliminary injunction with the precedent writs In the case of the SC, there is no time limit. If you get a TRO in the
which is the temporary restraining order. The TRO is precedent or SC, that amounts to getting injunction before the SC.
preparatory to that because you cannot get the writ of preliminary
injunction without notice and hearing. Section 2. Who may grant preliminary injunction. — A preliminary
injunction may be granted by the court where the action or
Exception: But under administrative matter 07-7-12 which took effect proceeding is pending. If the action or proceeding is pending in the
December 2007, you can now get TROs without hearing. In other Court of Appeals or in the Supreme Court, it may be issued by said
words, it can be granted ex parte. court or any member thereof.

TRO is a source of judicial corruption. This is aggravated by the fact Prohibitory injunction – to maintain the status quo. Example: If
that this is not ex parte. MERALCO gave A a notice of disconnection, A should file a
prohibitory injunction because he wants to maintain the status quo.
There are 2 kinds of TRO
Mandatory injunction – sought to return to the status quo.
• 72 hours – begins or takes effect upon issuance Example: If there is brown-out in A’s residence because MERALCO
cut his electric power, A should file a mandatory injunction because
Why 72 hours? 72 hours is equivalent to 3 days so even if it is issued
he wants light again.
on a Friday, you have up until Monday within which the court may
decide whether to extend it or not because the 72-hour TRO is 2. Grounds (Section 3), TRO
granted by the court with the intent of extending that because these
used to be granted due to very urgent situations. Sec. 3. Grounds for issuance of preliminary injunction.

• 20 days – takes effect upon notice (upon receipt of the copy A preliminary injunction may be granted when it is established:
of the writ by the respondent)
(a) That the applicant is entitled to the relief demanded, and the
This used to be granted only after a summary hearing. Presently, the whole or part of such relief consists in restraining the commission or
summary hearing is no longer needed. The 20-day TRO can be continuance of the act or acts complained of, or in requiring the
granted ex parte. There is no distinction whether it is a 72-hour TRO performance of an act or acts, either for a limited period or
or a 20-day TRO. perpetually;

13 Aicka Singson
Notes in Remedial Law Review II
Provisional Remedies, Special Civil Actions, Special Proceedings and Evidence
School Year 2015-2016 REMEDIAL LAW REVIEW II - ATTY. BRONDIAL

(b) That the commission, continuance or non-performance of the act day period, the court must order said party to show cause why the
or acts complained of during the litigation would probably work injunction should not be granted, determine within the same period
injustice to the applicant; or whether or not the preliminary injunction shall be granted, and
accordingly issue the corresponding order.
(c) That a party, court, agency or a person is doing, threatening, or is
attempting to do, or is procuring or suffering to be done, some act or If the matter is of extreme urgency and the applicant will suffer grave
acts probably in violation of the rights of the applicant respecting the injustice and irreparable injury, the executive judge of a multiple-sala
subject of the action or proceeding, and tending to render the court or the presiding judge of a single sala court may issue ex parte
judgment ineffectual. a temporary restraining order effective for only seventy-two (72)
hours. Within such period, the judge shall conduct a summary
TRO hearing to determine whether the temporary restraining order shall
be extended to 20 days. The 72 hours shall be included in the
Temporary restraining order (TRO) is issued is an order to maintain maximum 20 day period
the status quo between and among the parties until the
determination of the prayer for a writ of preliminary injunction. The 3. Requirements
status quo is the last, actual, peaceable and uncontested situation
which precedes a controversy. Sec. 4. Verified application and bond for preliminary injunction or
temporary restraining order.
The judge may issue a TRO with a limited life of 20 days from date of
issue. If before the expiration of the 20 day period, the application for A preliminary injunction or temporary restraining order may be
preliminary injunction is denied, the TRO would be deemed granted only when:
automatically vacated. If no action is taken by the judge within the 20
(a) The application in the action or proceeding is verified, and shows
day period, the TRO would automatically expire on the 20th day by
facts entitling the applicant to the relief demanded; and
the sheer force of law, no judicial declaration to that effect being
necessary. (b) Unless exempted by the court, the applicant files with the court
Duration of TRO where the action or proceeding is pending, a bond executed to the
party or person enjoined, in an amount to be fixed by the court, to the
The lifetime of a TRO is 20 days, which is non-extendible effect that the applicant will pay to such party or person all damages
which he may sustain by reason of the injunction or temporary
(AM 02-02-07-SC). restraining order if the court should finally decide that the applicant
was not entitled thereto. Upon approval of the requisite bond, a writ
If it is shown that the applicant would suffer great or irreparable injury of preliminary injunction shall be issued.
before the application for the writ of injunction can be heard, the
court may issue a temporary restraining order (TRP) ex parte which (c) When an application for a writ of preliminary injunction or a
shall be effective for a period not exceeding twenty (20) days from temporary restraining order is included in a complaint or any initiatory
service on the party sought to be enjoined. Within the said twenty- pleading, the case, if filed in a multiple-sala court, shall be raffled

14 Aicka Singson
Notes in Remedial Law Review II
Provisional Remedies, Special Civil Actions, Special Proceedings and Evidence
School Year 2015-2016 REMEDIAL LAW REVIEW II - ATTY. BRONDIAL

only after notice to and in the presence of the adverse party or the However, and subject to the provisions of the preceding sections, if
person to be enjoined. In any event, such notice shall be preceded, the matter is of extreme urgency and the applicant will suffer grave
or contemporaneously accompanied, by service of summons, injustice and irreparable injury, the executive judge of a multiple-sala
together with a copy of the complaint or initiatory pleading and the court or the presiding judge of a single sala court may issue ex parte
applicant’s affidavit and bond, upon the adverse party in the a temporary restraining order effective for only seventy-two (72)
Philippines. hours from issuance but he shall immediately comply with the
provisions of the next preceding section as to service of summons
However, where the summons could not be served personally or by and the documents to be served therewith. Thereafter, within the
substituted service despite diligent efforts, or the adverse party is a aforesaid seventy-two (72) hours, the judge before whom the case is
resident of the Philippines temporarily absent therefrom or is a pending shall conduct a summary hearing to determine whether the
nonresident thereof, the requirement of prior or contemporaneous temporary restraining order shall be extended until the application for
service of summons shall not apply. preliminary injunction can be heard. In no case shall the total period
of effectivity of the temporary restraining order exceed twenty (20)
(d) The application for a temporary restraining order shall thereafter
days, including the original seventy-two hours provided herein.
be acted upon only after all parties are heard in a summary hearing
which shall be conducted within twenty-four (24) hours after the In the event that the application for preliminary injunction is denied or
sheriff’s return of service and/or the records are received by the not resolved within the said period, the temporary restraining order is
branch selected by raffle and to which the records shall be deemed, automatically vacated. The effectivity of a temporary
transmitted immediately. restraining order is not extendible without need of any judicial
declaration to that effect and no court shall have authority to extend
Section 5. Preliminary injunction not granted without notice;
or renew the same on the same ground for which it was issued.
exception. — No preliminary injunction shall be granted without
hearing and prior notice to the party or person sought to be enjoined. However, if issued by the Court of Appeals or a member thereof, the
If it shall appear from facts shown by affidavits or by the verified temporary restraining order shall be effective for sixty (60) days from
application that great or irreparable injury would result to the service on the party or person sought to be enjoined. A restraining,
applicant before the matter can be heard on notice, the court to order issued by the Supreme Court or a member thereof shall be
which the application for preliminary injunction was made, may issue effective until further orders.
a temporary restraining order to be effective only for a period of
twenty (20) days from service on the party or person sought to be Section 6. Grounds for objection to, or for motion of dissolution of,
enjoined, except as herein provided. Within the said twenty-day injunction or restraining order. — The application for injunction or
period, the court must order said party or person to show cause, at a restraining order may be denied, upon a showing of its insufficiency.
specified time and place, why the injunction should not be granted, The injunction or restraining order may also be denied, or, if granted,
determine within the same period whether or not the preliminary may be dissolved, on other grounds upon affidavits of the party or
injunction shall be granted, and accordingly issue the corresponding person enjoined, which may be opposed by the applicant also by
order. (Bar Matter No. 803, 17 February 1998) affidavits. It may further be denied, or if granted, may be dissolved, if
it appears after hearing that although the applicant is entitled to the

15 Aicka Singson
Notes in Remedial Law Review II
Provisional Remedies, Special Civil Actions, Special Proceedings and Evidence
School Year 2015-2016 REMEDIAL LAW REVIEW II - ATTY. BRONDIAL

injunction or restraining order, the issuance or continuance thereof, 4) A bond must be posted, unless otherwise exempted by the court.
as the case may be, would cause irreparable damage to the party or
person enjoined while the applicant can be fully compensated for 5) The threatened injury must be incapable of pecuniary estimation.
such damages as he may suffer, and the former files a bond in an
4. Damages
amount fixed by the court conditioned that he will pay all damages
which the applicant may suffer by the denial or the dissolution of the Sec. 8. Judgment to include damages against party and sureties.
injunction or restraining order. If it appears that the extent of the
preliminary injunction or restraining order granted is too great, it may At the trial, the amount of damages to be awarded to either party,
be modified. upon the bond of the adverse party, shall be claimed, ascertained,
and awarded under the same procedure prescribed in section 20 of
Section 7. Service of copies of bonds; effect of disapproval of same. Rule 57.
— The party filing a bond in accordance with the provisions of this
Rule shall forthwith serve a copy of such bond on the other party, Section 9. When final injunction granted. — If after the trial of the
who may except to the sufficiency of the bond, or of the surety or action it appears that the applicant is entitled to have the act or acts
sureties thereon. If the applicant's bond is found to be insufficient in complained of permanently enjoined the court shall grant a final
amount, or if the surety or sureties thereon fail to justify, and a bond injunction perpetually restraining the party or person enjoined from
sufficient in amount with sufficient sureties approved after the commission or continuance of the act or acts of confirming the
justification is not filed forthwith the injunction shall be dissolved. If preliminary mandatory injunction.
the bond of the adverse party is found to be insufficient in amount, or
the surety or sureties thereon fail to justify a bond sufficient in
amount with sufficient sureties approved after justification is not filed CASES:
forthwith, the injunction shall be granted or restored, as the case may a. Idolor vs. CA (351 SCRA)
be.
FACTS:
Notes:
Teresita Idolor executed in favor of private respondent Gumersindo
1) There must be a verified petition, De Guzman a Deed of Real Estate Mortgage with right of extra-
judicial foreclosure upon failure to redeem the mortgage.
2) The application must establish that he has a right of relief or a
right to be protected and that the act against which the injunction is Upon the failure of the petitioner to settle her mortgage, respondents
sought violates such right, went to the Barangay which resulted into a ―Kasunduang Pag-
aayos‖ which noted that the petitioner shall pay within 90 days and
3) The applicant must establish that there is a need to restrain the her failure would warrant the foreclosure of the property with the right
commission of the continuance of the acts complained of and if not to repurchase within one year without interest.
enjoined would work injustice to him,

16 Aicka Singson
Notes in Remedial Law Review II
Provisional Remedies, Special Civil Actions, Special Proceedings and Evidence
School Year 2015-2016 REMEDIAL LAW REVIEW II - ATTY. BRONDIAL

Petitioner failed to comply with her undertaking; thus respondent extinguishment of the obligation, here the original obligation was not
Gumersindo De Guzman filed an extra judicial foreclosure of the real extinguished.
estate mortgage. The property was sold in a public auction to
respondent Gumersindo and the Certificate of Sale was registered. b. Gustilo vs Real (353 SCRA)

After more than a year, petitioner filed with the Regional Trial Court FACTS: Complainant avers that he was a candidate for punong
of Quezon City, Branch 220, a complaint for annulment of Sheriff’s barangay of Barangay Punta Mesa, Manapla, Negros Occidental in
Certificate of Sale with prayer for the issuance of a temporary the May 12, 1997 elections. His lone opponent was Weddy C. Libo-
restraining order (TRO) and a writ of preliminary injunction.Trial court on, then the incumbent punong barangay and the representative of
subsequently issued the TRO and the writ. the Association of Barangay Captains (ABC) to the Sangguniang
Bayan of Manapla and the Sangguniang Panlalawigan of Negros
CA anulled the writ Occidental. Both complainant and Libo-on garnered 819 votes during
the elections, resulting in a tie. The breaking of the tie by the Board
ISSUES: of Canvassers was in complainant's favor and he was proclaimed
duly elected punong barangay.
1. Whether or not the Petitioner has proprietary rights to the writ
of preliminary injunction His opponent filed an election protest case before the MCTC. Libo-
on sought the recounting of ballots in two precincts, preliminary
HELD:
prohibitory injunction, and damages.
Injunction is a preservative remedy aimed at protecting substantive
Respondent ordered the issuance of summons to the parties and set
rights and interests. Before an injunction can be issued, it is essential
the hearing on June 6, 1997. However, Libo-on filed a motion to
that the following requisites be present:
advance the hearing to May 29 and 30, 1997 which was granted.
1. There must be a right in esse or the existence of a right to be Complainant avers that he was not furnished a copy of this Order
protected; dated May 28, 1997.

2. The act against which the injunction is to be directed is a violation On May 29, 1997, respondent judge issued a TRO and annulled the
of such right. proclamation of complainant as the duly elected punong barangay .
Complainant declares that no copy of this Order dated May 29, 1997
Petitioner had one year redemption period from the registration of was served on him. That same day, however, he was able to secure
the sheriff’s sale to redeem the property but she failed to exercise copies of the orders of respondent from the COMELEC Registrar
this right. Hence, the right no longer exists. and the DILG. Moreover, it was only in the afternoon of May 29,
1997 that complainant received a copy of Libo-on's petition in Civil
There was no novation that was brought by the Kasunduan, since it Case and respondent's Order.
is essentially the same agreement as the first, only that the
conditions were changed a little. Novation requires the

17 Aicka Singson
Notes in Remedial Law Review II
Provisional Remedies, Special Civil Actions, Special Proceedings and Evidence
School Year 2015-2016 REMEDIAL LAW REVIEW II - ATTY. BRONDIAL

On May 30, 1997, complainant took his oath of office as punong But since the COMELEC ignored Libo-on's petition for correction of
barangay. That same day, he also filed a petition for certiorari before erroneous tabulation and Libo-on had no other remedy under the
the RTC. law, he was constrained to annul complainant's proclamation, which
from the very beginning was illegal. He justified his action by our
On June 5, 1997, the RTC lifted the TRO issued by respondent and rulings in Bince, Jr. v. COMELEC and Tatlonghari v. COMELEC,
declared as null and void the order nullifying complainant's which held that a faulty tabulation cannot be the basis of a valid
proclamation as duly elected punong barangay. proclamation.

Believing that respondent could not decide on the Civil Case ISSUE: WON injunctive writ were properly issued
impartially, complainant moved for his inhibition.
HELD: NO. The OCA found that respondent's errors were not honest
Respondent denied complainant's motion for inhibition and after mistakes in the performance of his duties. Rather, his actions
hearing Libo-on's motion for permanent injunction, issued a second showed a bias in favor of Libo-on and "evinced a pattern to prevent
TRO "to maintain the status quo between the contending parties." the complainant from assuming office as the duly elected punong
barangay despite his having been proclaimed as such by the Board
Complainant argues that by issuing the second TRO, respondent
of Canvassers."
reversed the order of the RTC of Silay City. He also claims that by
preventing him from assuming office, he was excluded by the DILG Supreme Court Administrative Circular No. 20-95 provides:
from participating in the election of the Liga ng Mga Barangay on
June 14, 1997. 2. The application for a TRO shall be acted upon only after all parties
are heard in a summary hearing conducted within twenty-four (24)
In his Comment, respondent denied the allegations. He claimed that hours after the records are transmitted to the branch selected by
when Libo-on filed his motion to advance the hearing of the prayer raffle. The records shall be transmitted immediately after raffle.
for injunction, complainant was served a copy by registered mail as
shown by the registry receipts attached to said motion. Considering xxx
the urgency of the matter and since there was substantial
compliance with due process, he issued the Order of May 28, 1997 4. With the exception of the provisions which necessarily involve
which cancelled the hearing set for June 6, 1997 and advanced it to multiple-sala stations, these rules shall apply to single-sala stations
May 29 and 30, 1997. especially with regard to immediate notice to all parties of all
applications for TRO.
Furthermore, Libo-on and his counsel appeared but complainant did
not, despite due notice. The hearing then proceeded, with Libo-on The foregoing clearly show that whenever an application for a TRO
presenting his evidence. As a result, he issued the TRO prayed for is filed, the court may act on the application only after all parties have
and annulled complainant's proclamation. Respondent admits that been notified and heard in a summary hearing. In other words, a
the Order of May 29, 1997, particularly the annulment of summary hearing may not be dispensed with. In the instant case,
complainant's proclamation, was outside the jurisdiction of his court. respondent admits that he issued the injunctive writ sought on May
29, 1997 after receiving the applicant's evidence ex parte. His failure

18 Aicka Singson
Notes in Remedial Law Review II
Provisional Remedies, Special Civil Actions, Special Proceedings and Evidence
School Year 2015-2016 REMEDIAL LAW REVIEW II - ATTY. BRONDIAL

to abide by Administrative Circular No. 20-95 in issuing the first TRO case, complainant had been duly proclaimed as the winning
is grave abuse of authority, misconduct, and conduct prejudicial to candidate for punong barangay. He had taken his oath of office.
the proper administration of justice. Unless his election was annulled, he was entitled to all the rights of
said office. We do not see how the complainant's exercise of such
Worse, he compounded the infraction by annulling complainant's rights would cause an irreparable injury or violate the right of the
proclamation as the duly elected punong barangay of Punta Mesa, losing candidate so as to justify the issuance of a temporary
Manapla and prohibiting him from assuming office. Respondent restraining order "to maintain the status quo." We see no reason to
admits that his court was not vested with the power or jurisdiction to disagree with the finding of the OCA that the evident purpose of the
annul the proclamation, but seeks to justify his action on the ground second TRO was to prevent complainant from participating in the
that the proclamation was void ab initio. In so doing, respondent election of the Liga ng mga Barangay. Respondent must be held
wantonly usurped a power exclusively vested by law in the liable for violating Rule 3.02 of the Code of Judicial Conduct which
COMELEC. Rule 3.01 of the Code of Judicial Conduct provides that provides that, "In every case, a judge shall endeavor diligently to
a "judge shall be faithful to the law and maintain professional ascertain the facts and the applicable law unswayed by partisan
competence." By annulling complainant's proclamation as the duly interests, public opinion, or fear of criticism."
elected punong barangay, despite being aware of the fact that his
court had no power to do so, not only is respondent guilty of grave' c. Lagrosas vs. Bristol-Myers (565 SCRA)
abuse of authority, he also manifests unfaithfulness to a basic legal
rule as well as injudicious conduct. Michael J. Lagrosas was employed by Bristol-Myers Squibb (Phil.),
Inc./Mead Johnson Phil. from January 6, 1997 until March 23, 2000
Note that the RTC of Silay City corrected respondent's errors by as Territory Manager in its Medical Sales Force Division.
declaring null and void his Order dated May 29, 1997. Nonetheless,
he compounded his previous errors of judgment by proceeding to Bristol-Myers dismissed Lagrosas effective immediately. Lagrosas
hear Libo-on's motion for permanent injunction and issuing a second then filed a complaint for illegal dismissal, non-payment of vacation
TRO on June 11, 1997 on the ground that "extreme urgency" and and sick leave benefits, 13th month pay, attorney’s fees, damages
"grave injustice and irreparable injury will arise" if no injunctive and fair market value of his Team Share Stock Option Grant. Labor
remedy were granted. Arbiter Renaldo O. Hernandez rendered a Decision in NLRC
declaring the dismissal illegal.
Before an injunctive writ can be issued, it is essential that the
following requisites be present: (1) there must be aright in esse or On appeal, the (NLRC) set aside the Decision of Labor Arbiter
the existence of a right to be protected; and (2) the act against which Hernandez in its Decision. The NLRC issued a Resolution reversing
injunction to be directed is a violation of such right. The onus its earlier ruling. It ratiocinated that the incident was not work-related
probandi is on movant to show that there exists a right to be since it occurred only after the district meeting of territory managers.
protected, which is directly threatened by the act sought to be It emphasized that for a serious misconduct to merit dismissal, it
enjoined. Further, there must be a showing that the invasion of the must be connected with the employee’s work.
right is material and substantial and that there is an urgent and
paramount necessity for the writ to prevent a serious damage. In this

19 Aicka Singson
Notes in Remedial Law Review II
Provisional Remedies, Special Civil Actions, Special Proceedings and Evidence
School Year 2015-2016 REMEDIAL LAW REVIEW II - ATTY. BRONDIAL

Bristol-Myers filed a motion for reconsideration which the NLRC which was posted by the petitioners is ordered DISCHARGED and
denied. Later, Labor Arbiter Hernandez issued a writ of execution. RELEASED to the petitioners.
Notices of garnishment were then served upon the Philippine British
Assurance Co., Inc. for the supersedeas bond posted by Bristol- The appellate court held that upon the expiration of the TRO, the
Myers and the Bank of the Philippine Islands for the balance of the cash bond intended for it also expired. Thus, the discharge and
judgment award. release of the cash bond for the expired TRO is proper. But the
appellate court disallowed the discharge of the injunction cash bond
Bristol-Myers moved to quash the writ of execution contending that it since the writ of preliminary injunction was issued pendente lite.
timely filed a petition for certiorari with the Court of Appeals. The Since there is a pending appeal with the Supreme Court, the
appellate court gave due course to Bristol-Myers’ petition and issued Decision dated January 28, 2005 is not yet final and executory.
a temporary restraining order (TRO) enjoining the enforcement of the
writ of execution and notices of garnishment. Upon the expiration of ISSUE: Did the Court of Appeals err in disallowing the
the TRO, the appellate court issued a writ of preliminary injunction. discharge and release of the injunction cash bond?

Bristol-Myers then moved to discharge and release the TRO cash YES. It is settled that the purpose of a preliminary injunction is to
bond. It argued that since it has posted an injunction cash bond, the prevent threatened or continuous irremediable injury to some of the
TRO cash bond should be legally discharged and released. parties before their claims can be thoroughly studied and
adjudicated. Its sole aim is to preserve the status quo until the merits
The appellate court considered the misconduct as having been of the case can be heard fully.
committed in connection with Lagrosas’ duty as Territory Manager
since it occurred immediately after the district meeting of territory A preliminary injunction may be granted only when, among other
managers. It also held that the gravity and seriousness of the things, the applicant, not explicitly exempted, files with the court
misconduct cannot be denied. where the action or proceeding is pending, a bond executed to the
party or person enjoined, in an amount to be fixed by the court, to the
Lagrosas filed a motion for reconsideration which the appellate court effect that the applicant will pay such party or person all damages
denied. which he may sustain by reason of the injunction or temporary
restraining order if the court should finally decide that the applicant
In the meantime, Bristol-Myers moved to release the TRO cash bond was not entitled thereto. Upon approval of the requisite bond, a writ
and injunction cash bond in view of the Decision dated January 28, of preliminary injunction shall be issued.
2005. On August 12, 2005, the appellate court denied the motion as
premature since the decision is not yet final and executory due to The injunction bond is intended as a security for damages in case it
Lagrosas’ appeal to this Court. is finally decided that the injunction ought not to have been granted.
Its principal purpose is to protect the enjoined party against loss or
Bristol-Myers filed a motion for reconsideration CA decided that the damage by reason of the injunction, and the bond is usually
temporary restraining order cash bond WORTH (P600,000.00) conditioned accordingly.

20 Aicka Singson
Notes in Remedial Law Review II
Provisional Remedies, Special Civil Actions, Special Proceedings and Evidence
School Year 2015-2016 REMEDIAL LAW REVIEW II - ATTY. BRONDIAL

In this case, the Court of Appeals issued the writ of preliminary (petitioner parents) sent a letter to the University President urging
injunction to enjoin the implementation of the writ of execution and him not to implement the agreement. According to petitioner
notices of garnishment "pending final resolution of this case or parents, the Principal, without convening the COSD, decided to
unless the [w]rit is sooner lifted by the Court." The appellate court order the immediate transfer of petitioner students.
disposed of the case by granting Bristol-Myers’ petition and
reinstating the Decision of the NLRC which dismissed the complaint Petitioner parents also wrote a letter to Mrs. Ida B. Endonila, School
for dismissal. It also ordered the discharge of the TRO cash bond Division Superintendent, DepEd, Iloilo City, seeking her intervention
and injunction cash bond. Thus, both conditions of the writ of and prayed that petitioner students be allowed to take the home
preliminary injunction were satisfied. study program instead of transferring to another school. The DepEd
asked the University to comment on the letter. The University
Notably, the appellate court ruled that Lagrosas had no right to the replied and attached the minutes of the 28 November 2002
monetary awards granted by the labor arbiter and the NLRC, and meeting.
that the implementation of the writ of execution and notices of Petitioners filed a complaint for injunction and damages with the
garnishment was properly enjoined. This in effect amounted to a RTC. Petitioners assailed the Principal’s decision to order the
finding that Lagrosas did not sustain any damage by reason of the immediate transfer of petitioner students as a violation of their right
injunction. To reiterate, the injunction bond is intended to protect to due process because the COSD was not convened.
Lagrosas against loss or damage by reason of the injunction only.
Contrary to Lagrosas’ claim, it is not a security for the judgment The trial court issued a writ of preliminary injunction and directed
award by the labor arbiter. respondents to admit petitioner students during the pendency of the
case.
Considering the foregoing, we hold that the appellate court erred in
Respondents filed an MR and asked for the dissolution of the writ –
disallowing the discharge and release of the injunction cash bond.
DENIED. Respondents complied but with reservations.
d. Jenosa vs. Delariarte (630 SCRA)
FACTS: Petitioner students, were caught engaging in hazing Respondents filed a motion to dismiss. Respondents alleged that the
outside the school premises. The hazing incident was entered into trial court had no jurisdiction over the subject matter of the case and
that petitioners were guilty of forum shopping. The trial court denied
the blotter of the Iloilo City Police.
respondents’ motion. Respondents filed a motion for reconsideration.
Thereafter, dialogues and consultations were conducted among the
school authorities, the apprehended students and their parents. Petitioners wrote the DepEd and asked that it direct the University to
During the 28 November 2002 meeting, the parties agreed that, release the report cards and other credentials of petitioner students.
The DepEd sent a letter to the University advising it to release
instead of the possibility of being charged and found guilty of
petitioner students’ report cards and other credentials if there was no
hazing, the students who participated in the hazing incident as
valid reason to withhold the same. On The DepEd sent another letter
initiators, including petitioner students, would just transfer to another
school, while those who participated as neophytes would be to the University to follow-up petitioners’ request. The University
suspended for one month. The parents of petitioner students

21 Aicka Singson
Notes in Remedial Law Review II
Provisional Remedies, Special Civil Actions, Special Proceedings and Evidence
School Year 2015-2016 REMEDIAL LAW REVIEW II - ATTY. BRONDIAL

replied that it could not release petitioner students’ report cards due to injunction, and injunction is the strong arm of equity, petitioners must
their pending disciplinary case with the COSD. come to court with clean hands. In University of the Philippines v.
Hon. Catungal, Jr., a case involving student misconduct, this Court
Petitioners filed another complaint for mandatory injunction praying for ruled:
the release of petitioner students’ report cards and other credentials.
The trial court consolidated the two cases. Since injunction is the strong arm of equity, he who must apply for it
must come with equity or with clean hands. This is so because
The trial court issued a writ of preliminary injunction and directed the among the maxims of equity are (1) he who seeks equity must do
University to release petitioner students’ report cards and other equity, and (2) he who comes into equity must come with clean
credentials. Respondents filed a motion for reconsideration. hands. The latter is a frequently stated maxim which is also
Respondents alleged that they could not comply with the writ because expressed in the principle that he who has done inequity shall not
of the on-going disciplinary case against petitioner students. have equity. It signifies that a litigant may be denied relief by a court
of equity on the ground that his conduct has been inequitable, unfair
The COSD met with petitioners for a preliminary conference on the and dishonest, or fraudulent, or deceitful as to the controversy in
hazing incident. The University, through the COSD, issued its report issue.
finding petitioner students guilty of hazing. The COSD also
Here, petitioners, having reneged on their agreement without any
recommended the exclusion of petitioner students from its rolls.
justifiable reason, come to court with unclean hands. This Court may
The trial court issued an Order denying both motions for deny a litigant relief if his conduct has been inequitable, unfair and
reconsideration. dishonest as to the controversy in issue.

ISSUE: WON writ of injunction should be issued Since petitioners have come to court with inequitable and unfair
conduct, we deny them relief.
HELD: NO. In this case, we rule that the Principal had the authority
to order the immediate transfer of petitioner students because of the e. Solid Builders vs. China Bank (695 SCRA)
28 November 2002 agreement. Petitioner parents affixed their
FACTS: During the period from 1992 to 1996, China Banking
signatures to the minutes of the 28 November 2002 meeting and
Corporation (CBC) granted several loans to Solid Builders, Inc.
signified their conformity to transfer their children to another school.
(SBI), which amounted to P139,999,234.34, exclusive of interests
Petitioners Socorro Canto and Nelia Duro even wrote a letter to
and other charges. To secure the loans, Medina Foods Industries,
inform the University that they would transfer their children to another
school and requested for the pertinent papers needed for the Inc. (MFII) executed in CBC’s favor several surety agreements and
contracts of real estate mortgage over parcels of land.
transfer. In turn, the University did not anymore convene the COSD.
The University agreed that it would no longer conduct disciplinary In a letter addressed to CBC, SBI requested the restructuring of its
proceedings and instead issue the transfer credentials of petitioner
loans, a reduction of interests and penalties and the implementation
students. Then petitioners reneged on their agreement without any
of a dacion en pago of the New Cubao Central property. CBC sent
justifiable reason. Since petitioners’ present complaint is one for

22 Aicka Singson
Notes in Remedial Law Review II
Provisional Remedies, Special Civil Actions, Special Proceedings and Evidence
School Year 2015-2016 REMEDIAL LAW REVIEW II - ATTY. BRONDIAL

SBI a letter dated stating that the loans had been completely preserve the status quo ante, upon the applicant’s showing of two
restructured effective March 1, 1999 in the amount of important requisite conditions, namely: (1) the right to be protected
P218,540,646.00. On the aspect of interests and charges, CBC exists prima facie, and (2) the acts sought to be enjoined are
suggested the updating of the obligation to avoid paying interests violative of that right. It must be proven that the violation sought to be
and charges. prevented would cause an irreparable injury.

Subsequently, in a letter dated September 18, 2000, CBC demanded There is no clear right that warrants the extraordinary protection of
SBI to settle its outstanding account within ten days from receipt an injunctive writ has been shown by SBI and MFII to exist in their
thereof. Claiming that the interests, penalties and charges imposed favor. Here, SBI and MFII basically claim a right to have their
by CBC were iniquitous and unconscionable and to enjoin CBC from mortgaged properties shielded from foreclosure by CBC on the
initiating foreclosure proceedings, SBI and MFII filed a Complaint "To ground that the interest rate and penalty charges imposed by CBC
Compel Execution of Contract and for Performance and Damages, on the loans availed of by SBI are iniquitous and unconscionable. As
With Prayer for Writ of Preliminary Injunction and Ex-Parte debtor-mortgagors, however, SBI and MFII do not have a right to
Temporary Restraining Order" in the RTC of Pasig City. prevent the creditor-mortgagee CBC from foreclosing on the
mortgaged properties simply on the basis of alleged "usurious,
The trial court granted the application of SBI and MFII for the exorbitant and confiscatory rate of interest
issuance of a writ of preliminary injunction for they were able to
sufficiently comply with the requisites for the issuance of an Neither has there been a showing of irreparable injury. An injury is
injunctive writ. considered irreparable if it is of such constant and frequent
recurrence that no fair or reasonable redress can be had therefor in
CBC sought reconsideration but the trial court denied it. a court of law, or where there is no standard by which their amount
Subsequently, CBC filed a "Motion to Dissolve Injunction Order" but can be measured with reasonable accuracy, that is, it is not
this was denied. Aggrieved, CBC filed a Petition for Certiorari in the susceptible of mathematical computation. The provisional remedy of
Court of Appeals. The Court of Appeals granted the petition of CBC, preliminary injunction may only be resorted to when there is a
set aside the Orders of RTC and dissolved the injunctive writ issued pressing necessity to avoid injurious consequences which cannot be
by the RTC of Pasig City. remedied under any standard of compensation.

SBI and MFII filed a motion for reconsideration but it was denied by
the Court of Appeals in a Resolution dated September 18, 2007. f. Plaza vs. Lustiva, (718 SCRA) March 5, 2014

ISSUE: WON the SBI and MFII failed to satisfy the requisites for FACTS: On August 28, 1997, the CA ruled that among the Plaza
the issuance of a writ of preliminary injunction siblings, namely: Aureliano, Emiliana, Vidal, Marciano, and Barbara,
Barbara was the owner of the subject agricultural land. The decision
HELD: Yes. A writ of preliminary injunction is an extraordinary event
became final and executory and Barbara's successors, respondents
which must be granted only in the face of actual and existing
have continued occupying the property.
substantial rights. A writ of preliminary injunction is issued to

23 Aicka Singson
Notes in Remedial Law Review II
Provisional Remedies, Special Civil Actions, Special Proceedings and Evidence
School Year 2015-2016 REMEDIAL LAW REVIEW II - ATTY. BRONDIAL

The petitioners, filed a Complaint for Injunction, Damages, Attorney’s Through a petition for review on certiorari under Rule 65, the
Fees with Prayer for the Issuance of the Writ of Preliminary petitioners challenged the RTC’s order before the CA.
Injunction and/or TRO against the respondents and the City
Government of Butuan. They prayed that the respondents be The CA affirmed the RTC’s ruling, found the petitioners guilty of
enjoined from unlawfully and illegally threatening to take possession forum shopping, dismissed the case, and referred the case to the
of the subject property. According to the petitioners, they acquired Court and to the IBP for investigation and institution of the
the land from Virginia Tuazon in 1997; Tuazon was the sole bidder appropriate administrative action. The CA, after legal analysis,
and winner in a tax delinquency sale conducted by the City of Butuan similarly concluded that for being disqualified to bid under Section 89
on December 27, 1996. of the LGC of 1991, Tuazon never obtained ownership over the
property; much less transmit any proprietary rights to the petitioners.
In their answer, the respondents pointed out that they were never Clearly, the petitioners failed to establish any clear and unmistakable
delinquent in paying the land taxes and were in fact not aware that right enforceable by the injunctive relief.
their property had been offered for public auction. Moreover, Tuazon,
ISSUE: WON petitioners are entitled to WPI; and WON prov rem
being a government employee, was disqualified to bid in the public
may be issued despite dismissal of the main action
auction, as stated in Section 89 of the LGC of 1991. As Tuazon’s
participation in the sale was void, she could have not transferred HELD: NO. The petitioners failed to show clear and unmistakable
ownership to the petitioners. Equally important, the petitioners rights to be protected by the writ; the present action has been
merely falsified the property tax declaration by inserting the name of rendered moot and academic by the dismissal of the main action
the petitioners’ father, making him appear as a co-owner of the
auctioned land. Armed with the falsified tax declaration, the As the lower courts correctly found, Tuazon had no ownership to
petitioners, as heirs of their father, fraudulently redeemed the land confer to the petitioners despite the latter’s reimbursement of
from Tuazon. Nonetheless, there was nothing to redeem as the land Tuazon’s purchase expenses. Because they were never owners of
was not sold. For these irregularities, the petitioners had no right to the property, the petitioners failed to establish entitlement to the writ
the Writ of Preliminary Injunction and/or Temporary Restraining of preliminary injunction. "To be entitled to an injunctive writ, the right
Order prayed for against them. to be protected and the violation against that right must be shown. A
writ of preliminary injunction may be issued only upon clear showing
The RTC reconsidered its earlier order, denied the prayer for a Writ of an actual existing right to be protected during the pendency of the
of Preliminary Injunction, and ordered that the possession and principal action. When the complainant’s right or title is doubtful or
occupation of the land be returned to the respondents. The RTC disputed, he does not have a clear legal right and, therefore, the
found that the auction sale was tainted with irregularity as the bidder issuance of injunctive relief is not proper."
was a government employee disqualified in accordance with Section
89 of the LGC of 1991. The petitioners are not buyers in good faith Likewise, upon the dismissal of the main case by the RTC on August
either. On the contrary, they were in bad faith for having falsified the 8, 2013, the question of issuance of the writ of preliminary injunction
tax declaration they redeemed the property with. has become moot and academic. In Arevalo v. Planters
Development Bank, the Court ruled that a case becomes moot and

24 Aicka Singson
Notes in Remedial Law Review II
Provisional Remedies, Special Civil Actions, Special Proceedings and Evidence
School Year 2015-2016 REMEDIAL LAW REVIEW II - ATTY. BRONDIAL

academic when there is no more issue between the parties or object insufficient to discharge the mortgage debt, or that the parties have
that can be served in deciding the merits of the case. Upon the so stipulated in the contract of mortgage;
dismissal of the main action, the question of the non-issuance of a
writ of preliminary injunction automatically died with it. A writ of (c) After judgment, to preserve the property during the pendency of
preliminary injunction is a provisional remedy; it is auxiliary, an an appeal, or to dispose of it according to the judgment, or to aid
adjunct of, and subject to the determination of the main action. It is execution when the execution has been returned unsatisfied or the
deemed lifted upon the dismissal of the main case, any appeal judgment obligor refuses to apply his property in satisfaction of the
therefrom notwithstanding. judgment, or otherwise to carry the judgment into effect;

C. RECEIVERSHIP RULE 59 (d) Whenever in other cases it appears that the appointment of a
A receiver is a person appointed by the court in behalf of the parties receiver is the most convenient and feasible means of preserving,
to an action for the purpose of preserving the property involved in the administering, or disposing of the property in litigation.
suit and to protect the rights of the parties under the direction of the
During the pendency of an appeal, the appellate court may allow an
court. He is a real party in interest but he cannot file a case without
application for the appointment of a receiver to be filed in and
the consent of the receivership court.
decided by the court of origin and the receiver appointed to be
subject to the control of said court.
1. When Writ may Issue (Section 1)
Notes: (Section 1 is also the grounds)
Section 1. Appointment of receiver.
1) Verified application;
Upon a verified application, one or more receivers of the property
subject of the action or proceeding may be appointed by the court 2) Appointed by the court where the action is pending, or by the CA
where the action is pending, or by the Court of Appeals or by the or by the SC, or a member thereof;
Supreme Court, or a member thereof, in the following cases:
• During the pendency of an appeal, the appellate court may
(a) When it appears from the verified application, and such other allow an application for the appointment of a receiver to be
proof as the court may require, that the party applying for the filed in and decided by the court of origin and the receiver
appointment of a receiver has an interest in the property or fund appointed to be subject to the control of said court.
which is the subject of the action or proceeding, and that such
property or fund is in danger of being lost, removed, or materially 3) Applicant’s bond conditioned on paying the adverse party all
injured unless a receiver be appointed to administer and preserve it; damages he may sustain by the appointment of the receiver in case
the appointment is without sufficient cause;
(b) When it appears in an action by the mortgagee for the foreclosure
of a mortgage that the property is in danger of being wasted or 4) Receiver takes his oath and files his bond.
dissipated or materially injured, and that its value is probably

25 Aicka Singson
Notes in Remedial Law Review II
Provisional Remedies, Special Civil Actions, Special Proceedings and Evidence
School Year 2015-2016 REMEDIAL LAW REVIEW II - ATTY. BRONDIAL

Can the mortgagee apply for the appointment of the receiver? Notes:
Yes. On what grounds? There are only 3 grounds for a mortgagee to
apply for the appointment of a receiver. Rule 59, Section 1(b): When a) Before issuing the order appointing a receiver the court shall
it appears in an action by the mortgagee for the foreclosure of a require the applicant to file a bond executed to the party
mortgage that the property is in danger of being wasted or dissipated against whom the application is presented, in an amount to
or materially injured, and that its value is probably insufficient to be fixed by the court, to the effect that the applicant will pay
discharge the mortgage debt, or that the parties have so stipulated in such party all damages he may sustain by reason of the
the contract of mortgage. appointment of such receiver in case the applicant shall
have procured such appointment without sufficient cause;
A mortgage is a security. Why would an appointment of receiver be and
necessary when there is already a security? Isn’t it that one of the b) The court may, in its discretion, at any time after the
requirements under Rule 57 is that when there is already security, appointment, require an additional bond as further security
there is really no need for another security? When the value is for such damages
probably insufficient to discharge the mortgage debt.
Notwithstanding that there is security, the security is insufficient. TWO (2) KINDS OF BONDS

Where do you file the application for receiver? With the court where 1) Applicant’s Bond (for appointment of receiver) – To pay the
the principal action is pending because a provisional remedy cannot damages the adverse party may sustain by reason of appointment of
exist without the principal action. Where the principal action is, the receiver; and
provisional remedy goes with it. The basic rule in provisional remedy
2) Receiver’s Bond (of the appointed receiver, aside from oath) – To
is that it cannot exist on its own. It has to be attached, contingent or
answer for receiver’s faithful discharge of his duties.
dependent on a principal action.
3) Counter Bond
2. REQUIREMENTS (Section 2)
Grounds for Discharge of a Receiver:
Sec. 2. Bond on appointment of receiver.
1. Filing of bond executed to the applicant;
Before issuing the order appointing a receiver the court shall require
the applicant to file a bond executed to the party against whom the 2. His appointment was obtained without sufficient cause (Section 3)
application is presented, in an amount to be fixed by the court, to the
effect that the applicant will pay such party all damages he may
sustain by reason of the appointment of such receiver in case the 3. POWER OF RECEIVER (Section 6)
applicant shall have procured such appointment without sufficient Sec. 6. General powers of receiver.
cause; and the court may, in its discretion, at any time after the
appointment, require an additional bond as further security for such Subject to the control of the court in which the action or proceeding
damages. is pending, a receiver shall have the power to bring and defend, in

26 Aicka Singson
Notes in Remedial Law Review II
Provisional Remedies, Special Civil Actions, Special Proceedings and Evidence
School Year 2015-2016 REMEDIAL LAW REVIEW II - ATTY. BRONDIAL

such capacity, actions in his own name; to take and keep He may be punished for contempt and shall be liable to the receiver
possession of the property in controversy; to receive rents; to collect for the money or the value of the property and other things so
debts due to himself as receiver or to the fund, property, estate, refused or neglected to be surrendered, together with damages that
person, or corporation of which he is the receiver; to compound for may have been sustained by the party entitled thereto as a
and compromise the same; to make transfers; to pay outstanding consequence of such refusal or neglect. (Sec. 7)
debts; to divide the money and other property that shall remain
among the persons legally entitled to receive the same; and Section 6 speaks of the rights and duties of an appointed
generally to do such acts respecting the property as the court may receiver. It is necessary that before a receiver is appointed, he
authorize. However, funds in the hands of a receiver may be must post a bond. This is a requisite for the appointment of a
invested only by order of the court upon the written consent of all receiver. If he is appointed as a receiver, does he have to post
the parties to the action. another bond?
The applicant is different from the receiver. The appointed receiver
No action may be filed by or against a receiver without leave of the is not necessarily the applicant. When one applies for the
court which appointed him. appointment of a receiver, it does not follow that he will be
appointed as the receiver. When the applicant applies for the
Notes: appointment of a receiver, he is required to post a bond. If the
1) To bring and defend, in such capacity, actions in his own name applicant himself is appointed as the receiver, should he post
2) To take and keep possession of the property in controversy another bond (because the receiver is also required to post a
3) To receive rents bond)?
4) To collect debts due to himself as receiver or to the fund,
property, estate, person, or corporation of which he is the receiver In Rule 59, there are 2 bonds posted: one for the applicant and the
5) To compound for and compromise the same other for the receiver. If the applicant himself is appointed by the
6) To make transfer court as the receiver, should the applicant post another bond?
7) To pay outstanding debts
8) To divide the money and other property that shall remain among What is the applicant’s bond for? Why is an applicant required to
the persons legally entitled to receive the same post a bond? To answer for damages that may be caused by the
9) To do such acts respecting the property as the court may applicant. Example of a damage that an application will entail:
authorize.
10) However, funds in the hands of a receiver may be invested only Koruga vs Arcenas:
by order of the court upon the written consent of all the parties to
the action. No action may be filed by or against a receiver without What are the duties and responsibilities of a receiver?
leave of the court which appointed him. The receiver is in possession of the properties under receivership.
Can he invest them? A receiver is not necessarily an individual
Remedy against a party who refuses or neglects to deliver the person. It can be a corporation, law firm, or a partnership.
property subject of receivership

27 Aicka Singson
Notes in Remedial Law Review II
Provisional Remedies, Special Civil Actions, Special Proceedings and Evidence
School Year 2015-2016 REMEDIAL LAW REVIEW II - ATTY. BRONDIAL

4. TERMINATION AND COMPENSATION – Section 8 appointment of a receiver? Even during the executory portion of the
Sec. 8. Termination of receivership; compensation of receiver. judgment. Of all the provisional remedies, the appointment of a
receiver has the longest timeframe. Why? The very purpose of the
Whenever the court, motu proprio or on motion of either party, shall appointment of the receiver is for the preservation, administration,
determine that the necessity for a receiver no longer exists, it shall, and disposition of property. It has the longest timeframe because it
after due notice to all interested parties and hearing, settle the involves the disposition of property.
accounts of the receiver, direct the delivery of the funds and other
property in his possession to the person adjudged to be entitled to CASES:
receive them, and order the discharge of the receiver from further
duty as such. The court shall allow the receiver such reasonable a. Larrobis Jr. Vs. Phil Veterans Bank (440 SCRA)
compensation as the circumstances of the case warrant, to be
taxed as costs against the defeated party, or apportioned, as justice Petitioner spouses contracted a monetary loan with respondent
requires. Philippine Veterans Bank in the amount of P135,000.00, evidenced
by a promissory note, due and demandable on February 27, 1981,
and secured by a Real Estate Mortgage executed on their lot
Notes:
together with the improvements thereon.
Whenever the court, motu proprio or on motion of either party, shall
determine that the necessity for a receiver no longer exists, it shall, The respondent bank went bankrupt and was placed under
after due notice to all interested parties and hearing, settle the receivership/liquidation by the Central Bank from April 25, 1985 until
accounts of the receiver, direct the delivery of the funds and other August 1992.
property in his possession to the person adjudged to be entitled to
receive them, and order the discharge of the receiver from further The bank, through Francisco Go, sent the spouses a demand letter
duty as such. for "accounts receivable in the total amount of P6,345.00 as of
The court shall allow the receiver such reasonable compensation as August 15, 1984,"4 which pertains to the insurance premiums
the circumstances of the case warrant, to be taxed as costs against advanced by respondent bank over the mortgaged property of
the defeated party, or apportioned, as justice requires. petitioners.

Receivership shall also be terminated when: On August 23, 1995, more than fourteen years from the time the loan
a) Its continuance is not justified by the facts and circumstances of became due and demandable, respondent bank filed a petition for
the case; or extrajudicial foreclosure of mortgage of petitioners’ property. On
b) Court is convinced that the powers are abused. October 18, 1995, the property was sold in a public auction by Sheriff
Arthur Cabigon with Philippine Veterans Bank as the lone bidder.
Aguilar vs Manila Banking Corp
Petitioners filed a complaint with the RTC to declare the extra-judicial
foreclosure and the subsequent sale to respondent bank null and
Of all the provisional remedies, which has the shortest
void.
timeframe? Replevin. Until when may one apply for the

28 Aicka Singson
Notes in Remedial Law Review II
Provisional Remedies, Special Civil Actions, Special Proceedings and Evidence
School Year 2015-2016 REMEDIAL LAW REVIEW II - ATTY. BRONDIAL

In the pre-trial conference, the parties agreed to limit the issue to be such as to render it impossible for a party to fulfill his obligation in
whether or not the period within which the bank was placed under a normal manner.
receivership and liquidation was a fortuitous event which suspended
the running of the ten-year prescriptive period in bringing actions. Respondent’s claims that because of a fortuitous event, it was not
able to exercise its right to foreclose the mortgage on petitioners’
RTC rendered its decision, the fallo of which reads: WHEREFORE, property; and that since it was banned from pursuing its business
premises considered judgment is hereby rendered dismissing the and was placed under receivership from April 25, 1985 until August
complaint for lack of merit. Likewise the compulsory counterclaim of 1992, it could not foreclose the mortgage on petitioners’ property
defendant is dismissed for being unmeritorious. within such period since foreclosure is embraced in the phrase
"doing business," are without merit.
It reasoned that:
While it is true that foreclosure falls within the broad definition of
…defendant bank was placed under receivership by the Central "doing business," that is:
Bank from April 1985 until 1992. The defendant bank was given
authority by the Central Bank to operate as a private commercial …a continuity of commercial dealings and arrangements and
bank and became fully operational only on August 3, 1992. From contemplates to that extent, the performance of acts or words or the
April 1985 until July 1992, defendant bank was restrained from doing exercise of some of the functions normally incident to and in
its business. progressive prosecution of the purpose and object of its organization.

Petitioners filed a motion for reconsideration which the RTC denied it should not be considered included, however, in the acts prohibited
on August 25, 1998.11 Thus, the present petition for review where whenever banks are "prohibited from doing business" during
petitioners claim that the RTC erred: receivership and liquidation proceedings.

IN RULING THAT THE PERIOD WITHIN WHICH RESPONDENT This we made clear in Banco Filipino Savings & Mortgage Bank vs.
BANK WAS PUT UNDER RECEIVERSHIP AND LIQUIDATION Monetary Board, Central Bank of the Philippines where we explained
WAS A FORTUITOUS EVENT THAT INTERRUPTED THE that:
RUNNING OF THE PRESCRIPTIVE PERIOD.
Section 29 of the Republic Act No. 265, as amended known as the
Issue: Whether or not the period within which the respondent Central Bank Act, provides that when a bank is forbidden to do
bank was placed under receivership and liquidation business in the Philippines and placed under receivership, the
proceedings may be considered a fortuitous event which person designated as receiver shall immediately take charge of the
interrupted the running of the prescriptive period in bringing bank’s assets and liabilities, as expeditiously as possible, collect and
actions; gather all the assets and administer the same for the benefit of its
creditors, and represent the bank personally or through counsel as
HELD: No. One characteristic of a fortuitous event, in a legal sense he may retain in all actions or proceedings for or against the
and consequently in relations to contract, is that its occurrence must institution, exercising all the powers necessary for these purposes

29 Aicka Singson
Notes in Remedial Law Review II
Provisional Remedies, Special Civil Actions, Special Proceedings and Evidence
School Year 2015-2016 REMEDIAL LAW REVIEW II - ATTY. BRONDIAL

including, but not limited to, bringing and foreclosing mortgages in bank is obliged to collect pre-existing debts due to the bank, and in
the name of the bank. connection therewith, to foreclose mortgages securing such debts.

This is consistent with the purpose of receivership proceedings, i.e., It is true that we also held in said case that the period during which
to receive collectibles and preserve the assets of the bank in the bank was placed under receivership was deemed fuerza mayor
substitution of its former management, and prevent the dissipation of which validly interrupted the prescriptive period. This is being
its assets to the detriment of the creditors of the bank. invoked by the respondent and was used as basis by the trial court in
its decision. Contrary to the position of the respondent and court a
When a bank is declared insolvent and placed under receivership, quo however, such ruling does not find application in the case at bar.
the Central Bank, through the Monetary Board, determines whether
to proceed with the liquidation or reorganization of the financially A close scrutiny of the Provident case, shows that the Court arrived
distressed bank. A receiver, who concurrently represents the bank, at said conclusion, which is an exception to the general rule, due to
then takes control and possession of its assets for the benefit of the the peculiar circumstances of Provident Savings Bank at the time. In
bank’s creditors. A liquidator meanwhile assumes the role of the said case, we stated that:
receiver upon the determination by the Monetary Board that the bank
can no longer resume business. His task is to dispose of all the Having arrived at the conclusion that a foreclosure is part of a bank’s
assets of the bank and effect partial payments of the bank’s business activity which could not have been pursued by the receiver
obligations in accordance with legal priority. In both receivership and then because of the circumstances discussed in the Central Bank
liquidation proceedings, the bank retains its juridical personality case, we are thus convinced that the prescriptive period was legally
notwithstanding the closure of its business and may even be sued as interrupted by fuerza mayor in 1972 on account of the prohibition
its corporate existence is assumed by the receiver or liquidator. The imposed by the Monetary Board against petitioner from transacting
receiver or liquidator meanwhile acts not only for the benefit of the business, until the directive of the Board was nullified in 1981.
bank, but for its creditors as well.
Further examination of the Central Bank case reveals that the
In Provident Savings Bank vs. Court of Appeals, we further stated circumstances of Provident Savings Bank at the time were peculiar
that: because after the Monetary Board issued MB Resolution No. 1766
on September 15, 1972, prohibiting it from doing business in the
When a bank is prohibited from continuing to do business by the Philippines, the bank’s majority stockholders immediately went to the
Central Bank and a receiver is appointed for such bank, that bank Court of First Instance of Manila, which prompted the trial court to
would not be able to do new business, i.e., to grant new loans or to issue its judgment dated February 20, 1974, declaring null and void
accept new deposits. However, the receiver of the bank is in fact the resolution and ordering the Central Bank to desist from
obliged to collect debts owing to the bank, which debts form part of liquidating Provident. The decision was appealed to and affirmed by
the assets of the bank. The receiver must assemble the assets and this Court in 1981. Thus, the Superintendent of Banks, which was
pay the obligation of the bank under receivership, and take steps to instructed to take charge of the assets of the bank in the name of the
prevent dissipation of such assets. Accordingly, the receiver of the Monetary Board, had no power to act as a receiver of the bank and
carry out the obligations specified in Sec. 29 of the Central Bank Act.

30 Aicka Singson
Notes in Remedial Law Review II
Provisional Remedies, Special Civil Actions, Special Proceedings and Evidence
School Year 2015-2016 REMEDIAL LAW REVIEW II - ATTY. BRONDIAL

In this case, it is not disputed that Philippine Veterans Bank was Hence, the period within which respondent bank was placed under
placed under receivership by the Monetary Board of the Central receivership and liquidation proceedings does not constitute a
Bank by virtue of Resolution No. 364 on April 25, 1985, pursuant to fortuitous event which interrupted the prescriptive period in bringing
Section 29 of the Central Bank Act on insolvency of banks. actions.

Unlike Provident Savings Bank, there was no legal prohibition b. EVELINA G. CHAVEZ and AIDA CHAVEZ-DELES vs. CA and
imposed upon herein respondent to deter its receiver and liquidator ATTY. FIDELA Y. VARGAS G.R. No. 174356
from performing their obligations under the law. Thus, the ruling laid January 20, 2010
down in the Provident case cannot apply in the case at bar.
This case is about the propriety of the CA, which hears the case on
There is also no truth to respondent’s claim that it could not continue appeal, placing the property in dispute under receivership upon a
doing business from the period of April 1985 to August 1992, the claim that the defendant has been remiss in making an accounting
time it was under receivership. As correctly pointed out by petitioner, to the plaintiff of the fruits of such property.
respondent was even able to send petitioners a demand letter,
through Francisco Go, on August 23, 1985 for "accounts receivable FACTS: Respondent Fidela Vargas owned a five-hectare mixed
in the total amount of P6,345.00 as of August 15, 1984" for the coconut land and rice fields in Sorsogon. Petitioner Evelina Chavez
insurance premiums advanced by respondent bank over the had been staying in a remote portion of the land with her family,
mortgaged property of petitioners. How it could send a demand letter planting coconut seedlings on the land and supervising the harvest
on unpaid insurance premiums and not foreclose the mortgage of coconut and palay. Fidela and Evelina agreed to divide the gross
during the time it was "prohibited from doing business" was not sales of all products from the land between themselves. Since
adequately explained by respondent. Fidela was busy with her law practice, Evelina undertook to hold in
trust for Fidela her half of the profits.
Settled is the principle that a bank is bound by the acts, or failure to
act of its receiver.34 As we held in Philippine Veterans Bank vs.
But respondent Fidela claimed that Evelina had failed to remit her
NLRC, a labor case which also involved respondent bank,
share of the profits and, despite demand to turn over the
… all the acts of the receiver and liquidator pertain to petitioner, both administration of the property to Fidela, had refused to do so.
having assumed petitioner’s corporate existence. Petitioner cannot Consequently, Fidela filed a complaint against Evelina and her
disclaim liability by arguing that the non-payment of MOLINA’s just daughter, Aida C. Deles, who was assisting her mother, for
wages was committed by the liquidators during the liquidation period. recovery of possession, rent, and damages with prayer for the
immediate appointment of a receiver before the RTC. In their
However, the bank may go after the receiver who is liable to it for answer, Evelina and Aida claimed that the RTC did not have
any culpable or negligent failure to collect the assets of such bank jurisdiction over the subject matter of the case since it actually
and to safeguard its assets.37 involved an agrarian dispute.

31 Aicka Singson
Notes in Remedial Law Review II
Provisional Remedies, Special Civil Actions, Special Proceedings and Evidence
School Year 2015-2016 REMEDIAL LAW REVIEW II - ATTY. BRONDIAL

After hearing, the RTC dismissed the complaint for lack of require such protection or preservation, the remedy is not
jurisdiction based on Fidela’s admission that Evelina and Aida were receivership.
tenants who helped plant coconut seedlings on the land and
supervised the harvest of coconut and palay. As tenants, the Here Fidela’s main gripe is that Evelina and Aida deprived her of
defendants also shared in the gross sales of the harvest. The court her share of the land’s produce. She does not claim that the land or
threw out Fidela’s claim that, since Evelina and her family received its productive capacity would disappear or be wasted if not
the land already planted with fruit-bearing trees, they could not be entrusted to a receiver. Nor does Fidela claim that the land has
regarded as tenants. Cultivation, said the court, included the been materially injured, necessitating its protection and
tending and caring of the trees. The court also regarded as relevant preservation. Because receivership is a harsh remedy that can be
Fidela’s pending application for a five-hectare retention and granted only in extreme situations, Fidela must prove a clear right to
Evelina’s pending protest relative to her three-hectare beneficiary its issuance. But she has not. Indeed, in none of the other cases
share. she filed against Evelina and Aida has that remedy been granted
her.
Dissatisfied, Fidela appealed to the CA. She also filed with that
court a motion for the appointment of a receiver. CA granted the c. Koruga vs Arcenas (590 SCRA)
motion and ordained receivership of the land, noting that there
appeared to be a need to preserve the property and its fruits in light Before this Court are two petitions that originated from a Complaint
of Fidela’s allegation that Evelina and Aida failed to account for her filed by Ana Maria A. Koruga (Koruga) before the Regional Trial
share of such fruits. Court (RTC) of Makati City against the Board of Directors of Banco
Filipino and the Members of the Monetary Board of the Bangko
Parenthetically, Fidela also filed three estafa cases with the RTC of Sentral ng Pilipinas (BSP) for violation of the Corporation Code, for
Olongapo City and a complaint for dispossession with the DARAB inspection of records of a corporation by a stockholder, for
against Evelina and Aida. In all these cases, Fidela asked for the receivership, and for the creation of a management committee.
immediate appointment of a receiver for the property.
Koruga is a minority stockholder of Banco Filipino Savings and
Mortgage Bank. On August 20, 2003, she filed a complaint before
ISSUE: WON the CA erred in granting respondent Fidela’s
the Makati RTC which was raffled to Branch 138, presided over by
application for receivership.
Judge Sixto Marella, Jr. Koruga’s complaint alleged:
HELD: YES. In any event, we hold that the CA erred in granting 10. 1 Violation of Sections 31 to 34 of the Corporation Code ("Code")
receivership over the property in dispute in this case. For one thing, which prohibit self-dealing and conflicts of interest of directors and
a petition for receivership under Section 1(b), Rule 59 of the Rules officers.
of Civil Procedure requires that the property or fund subject of the
action is in danger of being lost, removed, or materially injured, Arcenas, et al. filed their Answer raising, among others, the trial
necessitating its protection or preservation. Its object is the court’s lack of jurisdiction to take cognizance of the case. They also
prevention of imminent danger to the property. If the action does not filed a Manifestation and Motion seeking the dismissal of the case on

32 Aicka Singson
Notes in Remedial Law Review II
Provisional Remedies, Special Civil Actions, Special Proceedings and Evidence
School Year 2015-2016 REMEDIAL LAW REVIEW II - ATTY. BRONDIAL

the following grounds: (a) lack of jurisdiction over the subject matter; Code; and prayed for Receivership and Creation of a Management
(b) lack of jurisdiction over the persons of the defendants; (c) forum- Committee, pursuant to Rule 59 of the Rules of Civil Procedure, the
shopping; and (d) for being a nuisance/harassment suit. But RTC Securities Regulation Code, the Interim Rules of Procedure
denied the motion. This prompted Arcenas, et al. to file before the Governing Intra-Corporate Controversies, the General Banking Law
CA a Petition for Certiorari and Prohibition under Rule 65 of the of 2000, and the New Central Bank Act. She accused the directors
Rules of Court with a prayer for the issuance of a writ of preliminary and officers of Banco Filipino of engaging in unsafe, unsound, and
injunction and a temporary retraining order (TRO). On February 9, fraudulent banking practices, more particularly, acts that violate the
2005, the CA issued a 60-day TRO enjoining Judge Marella from prohibition on self-dealing.
conducting further proceedings in the case.
From the foregoing disquisition, there is no doubt that the RTC has
RTC issued a Notice of Pre-trial setting the case for pre-trial on June no jurisdiction to hear and decide a suit that seeks to place Banco
2 and 9, 2005. Arcenas, et al. filed a Manifestation and Motion before Filipino under receivership.
the CA, reiterating their application for a writ of preliminary injunction.
Koruga herself recognizes the BSP’s power over the allegedly
Dissatisfied, Koruga filed this Petition for Certiorari under Rule 65 of unlawful acts of Banco Filipino’s directors. The records of this case
the Rules of Court. Koruga alleged that the CA effectively gave due bear out that Koruga, through her legal counsel, wrote the Monetary
course to Arcenas, et al.’s petition when it issued a writ of preliminary Board on April 21, 2003 to bring to its attention the acts she had
injunction without factual or legal basis. She prayed that this Court enumerated in her complaint before the RTC. The letter reads in
restrain the CA from implementing the writ of preliminary injunction part:
and, after due proceedings, make the injunction against the assailed
CA Resolution permanent.12 Banco Filipino and the current members of its Board of Directors
should be placed under investigation for violations of banking laws,
ISSUE: WON RTC HAS JURISDICTION OVER THE CASE the commission of irregularities, and for conducting business in an
unsafe or unsound manner. They should likewise be placed under
NO. The Court finds that the CA erred in upholding the jurisdiction of, preventive suspension by virtue of the powers granted to the
and remanding the case to, the RTC. Monetary Board under Section 37 of the Central Bank Act. These
blatant violations of banking laws should not go by without penalty.
The resolution of these petitions rests mainly on the determination of
They have put Banco Filipino, its depositors and stockholders, and
one fundamental issue: Which body has jurisdiction over the Koruga
the entire banking system (sic) in jeopardy.
Complaint, the RTC or the BSP?
xxxx
We hold that it is the BSP that has jurisdiction over the case.
We urge you to look into the matter in your capacity as regulators.
Koruga’s Complaint charged defendants with violation of Sections 31
Our clients, a minority stockholders, (sic) and many depositors of
to 34 of the Corporation Code, prohibiting self-dealing and conflict of
Banco Filipino are prejudiced by a failure to regulate, and taxpayers
interest of directors and officers; invoked her right to inspect the
corporation’s records under Sections 74 and 75 of the Corporation

33 Aicka Singson
Notes in Remedial Law Review II
Provisional Remedies, Special Civil Actions, Special Proceedings and Evidence
School Year 2015-2016 REMEDIAL LAW REVIEW II - ATTY. BRONDIAL

are prejudiced by accommodations granted by the BSP to Banco The petition for certiorari may only be filed by the stockholders of
Filipino35 record representing the majority of the capital stock within ten (10)
days from receipt by the board of directors of the institution of the
In a letter dated May 6, 2003, BSP Supervision and Examination order directing receivership, liquidation or conservatorship.
Department III Director Candon B. Guerrero referred Koruga’s letter
to Arcenas for comment.36 On June 6, 2003, Banco Filipino’s then All the foregoing discussion yields the inevitable conclusion that the
Executive Vice President and Corporate Secretary Francisco A. CA erred in upholding the jurisdiction of, and remanding the case to,
Rivera submitted the bank’s comments essentially arguing that the RTC. Given that the RTC does not have jurisdiction over the
Koruga’s accusations lacked legal and factual bases.37 subject matter of the case, its refusal to dismiss the case on that
ground amounted to grave abuse of discretion.
On the other hand, the BSP, in its Answer before the RTC, said that
it had been looking into Banco Filipino’s activities. An October 2002 SO ORDERED.
Report of Examination (ROE) prepared by the Supervision and
Examination Department (SED) noted certain dacion payments, out- D. Replevin Rule 60
of-the-ordinary expenses, among other dealings. On July 24, 2003, It is a proceeding by which the owner or one who has a general or
the Monetary Board passed Resolution No. 1034 furnishing Banco special property in the thing taken or detained seeks to recover
Filipino a copy of the ROE with instructions for the bank to file its possession in specie, the recovery of damages being only incidental.
comment or explanation within 30 to 90 days under threat of being Replevin may be a main action or a provisional remedy.
fined or of being subjected to other remedial actions. The ROE, the As a principal action its ultimate goal is to recover personal property
BSP said, covers substantially the same matters raised in Koruga’s capable of manual delivery wrongfully detained by a person. Used in
complaint. At the time of the filing of Koruga’s complaint on August this sense, it is a suit in itself.
20, 2003, the period for Banco Filipino to submit its explanation had It is a provisional remedy in the nature of possessory action and the
not yet expired.38 applicant who seeks immediate possession of the property involved
need not be the holder of the legal title thereto. It is sufficient that he
Thus, the court’s jurisdiction could only have been invoked after the is entitled to possession thereof.
Monetary Board had taken action on the matter and only on the
ground that the action taken was in excess of jurisdiction or with 1. When Writ may Issue (Section 1)
such grave abuse of discretion as to amount to lack or excess of
jurisdiction. Section 1. Application.

Finally, there is one other reason why Koruga’s complaint before the A party praying for the recovery of possession of personal property
RTC cannot prosper. Given her own admission – and the same is may, at the commencement of the action or at any time before
likewise supported by evidence – that she is merely a minority answer, apply for an order for the delivery of such property to him, in
stockholder of Banco Filipino, she would not have the standing to the manner hereinafter provided.
question the Monetary Board’s action. Section 30 of the New Central
Bank Act provides:

34 Aicka Singson
Notes in Remedial Law Review II
Provisional Remedies, Special Civil Actions, Special Proceedings and Evidence
School Year 2015-2016 REMEDIAL LAW REVIEW II - ATTY. BRONDIAL

Notes: hearing, the defendant will be there and the defendant can say he is
a) The provisional remedy of replevin can only be applied for before the owner of the property. The court can no longer issue the writ.
answer. That will make the writ illusory. So how can now the court issue the
writ? It cannot be issued ex parte either. The Rule provides that the
b) A party praying for the recovery of possession of personal writ of replevin may only be issued before answer because at that
property may, at the commencement of the action or at any time point of time, the court is only dealing with the applicant. But once
before answer, apply for an order for the delivery of such property to you call on the defendant or you notify the defendant for the hearing
him. and he is there, you can no longer issue the writ. There is an
insufficient explanation of whether the writ of replevin may be issued.
While in receivership, you have until or even after execution in the
course of the disposal or disposition of the property within which to Preliminary attachment may be issued ex parte but it can never be
apply for the appointment of a receiver. But in application for writ of implemented. You have to consider the implementation. It requires
replevin, it may be only done until before answer (answer should be prior or contemporaneous service of summons nonetheless. This is
filed within 15 days from receipt of summons). If you are applying for what is meant by notice in the writ of replevin. So that in the writ of
writ of replevin, it should be before the answer. Why? replevin, the applicant must act very fast. Upon application, notice,
let it issue because we are dealing with personal property which can
The grounds for the application for a writ of replevin is such that the
be relocated or transferred and you have to run after that personal
applicant is entitled to the possession of the subject matter and in
property.
that he is the owner of the subject matter of the writ. You file
because you are either the owner of the property or you are entitled The bond is double the amount of the personal property. If I am the
to the possession of the property. applicant for the immediate recovery and possession of a car worth
P1,200,000, I will be required to post a bond at P2,400,000. Isn’t that
If answer has been filed to the complaint, what does that answer
foolish? I might as well buy a new car rather than posting a bond of
lead to? Issues are joined. If I say that I should have possession of
P2,400,000. The car that I sold is P1,200,000 with chattel mortgage
the property because I’m entitled to its possession because I am the
and here comes an applicant for a writ of replevin posting twice the
owner of the property, the court has yet no knowledge as to who is
value of the property. What is the rationale of the law? You are not
really the owner or entitled to the possession of the property. But
depositing P2,400,000 cash. This is just a bond, unlike in preliminary
once the answer is already filed and issues are joined, the allegation
attachment where you can file a cash deposit or bond. It would be
in the answer will say “I am the owner of the property.” There is now
foolish to post a cash bond. It is only surety bond. In surety bonds,
an issue: who is the owner of the property? The court has to decide
you don’t have to give a cash of the total amount. You only pay the
that. The writ of replevin can no longer be issued. That is why it can
premium and the prevailing market price is minimum of 3% and
only be issued before answer.
maximum of 10%. So it is worth applying for a writ of replevin. You
Because it is only before the answer that the writ of replevin may be are posting an P240,000 maximum and you are covering a car worth
issued by the court, can the writ therefore be granted ex parte? If it P1,200,000 and you can sell this again. This is common among car
can be issued ex parte or if it requires notice and hearing, at the dealers.

35 Aicka Singson
Notes in Remedial Law Review II
Provisional Remedies, Special Civil Actions, Special Proceedings and Evidence
School Year 2015-2016 REMEDIAL LAW REVIEW II - ATTY. BRONDIAL

2. Requirements (Section 2) object, he may, at any time before the delivery of the property to the
applicant, require the return thereof, by filing with the court where the action
Sec. 2. Affidavit and bond. is pending a bond executed to the applicant, in double the value of the
The applicant must show by his own affidavit or that of some other person property as stated in the applicant's affidavit for the delivery thereof to the
who personally knows the facts: applicant, if such delivery be adjudged, and for the payment of such sum, to
(a) That the applicant is the owner of the property claimed, particularly him as may be recovered against the adverse party, and by serving a copy
describing it, or is entitled to the possession thereof; of such bond on the applicant.
(b) That the property is wrongfully detained by the adverse party, alleging Section 6. Disposition of property by sheriff. — If within five (5) days after the
the cause of detention thereof according to the best of his knowledge, taking of the property by the sheriff, the adverse party does not object to the
information, and belief; sufficiency of the bond, or of the surety or sureties thereon; or if the adverse
(c) That the property has not been distrained or taken for a tax assessment party so objects and the court affirms its approval of the applicant's bond or
or a fine pursuant to law, or seized under a writ of execution or preliminary approves a new bond, or if the adverse party requires the return of the
attachment, or otherwise placed under custodia legis, or if so seized, that it property but his bond is objected to and found insufficient and he does not
is exempt from such seizure or custody; and forthwith file an approved bond, the property shall be delivered to the
(d) The actual market value of the property. applicant. If for any reason the property is not delivered to the applicant, the
The applicant must also give a bond, executed to the adverse party in sheriff must return it to the adverse party.
double the value of the property as stated in the affidavit aforementioned, for Notes:
the return of the property to the adverse party if such return be adjudged,
and for the payment to the adverse party of such sum as he may recover The applicant must show by his own affidavit or that of some other
from the applicant in the action. person who personally knows the facts:
Section 3. Order. — Upon the filing of such affidavit and approval of the
bond, the court shall issue an order and the corresponding writ of replevin, 1) A party praying for the provisional remedy must file an application
describing the personal property alleged to be wrongfully detained and for a writ of replevin. His application must be filed at the
requiring the sheriff forthwith to take such property into his custody. commencement of the action or at any time before the defendant
answers, and must contain an affidavit particularly describing the
Section 4. Duty of the sheriff. — Upon receiving such order, the sheriff must
property to which he entitled of possession.
serve a copy thereof on the adverse party, together with a copy of the
application, affidavit and bond, and must forthwith take the property, if it be
2) The affidavit must state that the property is wrongfully detained by
in the possession of the adverse party, or his agent, and retain it in his
the adverse party, alleging therein the cause of the detention. It must
custody. If the property or any part thereof be concealed in a building or
enclosure, the sheriff must demand its delivery, and if it be not delivered, he also state that the property has not been destrained or taken for tax
must cause the building or enclosure to be broken open and take the assessment or a fine pursuant to law, or seized under a writ of
property into his possession. After the sheriff has take possession of the execution or preliminary attachment, or otherwise placed in custodia
property as herein provided, he must keep it in a secure place and shall be legis. If it has been seized, then the affidavit must state that it is
responsible for its delivery to the party entitled thereto upon receiving his exempt from such seizure or custody.
fees and necessary expenses for taking and keeping the same.
Section 5. Return of property. — If the adverse party objects to the 3) The affidavit must state the actual market value of the property;
sufficiency of the applicant's bond, or of the surety or sureties thereon, he and
cannot immediately require the return of the property, but if he does not so

36 Aicka Singson
Notes in Remedial Law Review II
Provisional Remedies, Special Civil Actions, Special Proceedings and Evidence
School Year 2015-2016 REMEDIAL LAW REVIEW II - ATTY. BRONDIAL

4) The applicant must give a bond, executed to the adverse party adverse party (to the person against whom the writ of replevin was
and double the value of the property. issued).

Hao vs Andres: What was the subject matter of the writ here? 22 After the 5-day period, the sheriff must deliver the property to the
motor vehicles. Andres, the sheriff, with the writ of replevin, was able applicant.
to take into custody 9 cars. He placed them in a warehouse, the
owner of which was related to the applicant. The issue is as to the 3. Third Party Claim (Section 7)
duty of the sheriff. What did the sheriff Andres do? He did not wait for Sec. 7. Proceedings where property claimed by third person.
5 days, which is mandatory in the Rules. He immediately gave it to
If the property taken is claimed by any person other than the party against
the applicants. His excuse was the cars were lost (he allowed the
whom the writ of replevin had been issued or his agent, and such person
applicant to make a duplicate of the keys which happened during the makes an affidavit of his title thereto, or right to the possession thereof,
5-day period). The penalty for Andres (not dismissal) was a fine and stating the grounds therefor, and serves such affidavit upon the sheriff while
suspension. the latter has possession of the property and a copy thereof upon the
applicant, the sheriff shall not be bound to keep the property under replevin
When the sheriff takes custody of the subject of replevin, for how or deliver it to the applicant unless the applicant or his agent, on demand of
long should he keep the subject matter? 5 days. said sheriff, shall file a bond approved by the court to indemnify the third-
party claimant in a sum not less than the value of the property under replevin
What is the purpose of keeping it for 5 days from the time he as provided in section 2 hereof. In case of disagreement as to such value,
took custody of the property? the court shall determine the same. No claim for damages for the taking or
keeping of the property may be enforced against the bond unless the action
(1) To allow the adverse party to object to the sufficiency of the therefor is filed within one hundred twenty (120) days from the date of the
applicant’s bond; filing of the bond.

What happens when the adverse party questions the sufficiency of The sheriff shall not be liable for damages, for the taking or keeping of such
the bond? If the bond supposed to be posted is P2,400,000 and what property, to any such third-party claimant if such bond shall be filed. Nothing
was posted was only P1,200,000, the sheriff will allow the applicant herein contained shall prevent such claimant or any third person from
vindicating his claim to the property, or prevent the applicant from claiming
to pay for the insufficiency. If the applicant fails to comply, the sheriff
damages against a third-party claimant who filed a frivolous or plainly
should deliver the property mumble mumble fucking mumble open
spurious claim, in the same or a separate action.
your fucking mumbling mouth (to the adverse party siguro I don’t
understand the person reciting/mumbling). When the writ of replevin is issued in favor of the Republic of the Philippines,
or any officer duly representing it, the filing of such bond shall not be
(2) for the other party to be able to file a redelivery bond, required, and in case the sheriff is sued for damages as a result of the
equivalent to a counterbond in preliminary attachment, which is twice replevin, he shall be represented by the Solicitor General, and if held liable
the value of the property. therefor, the actual damages adjudged by the court shall be paid by the
National Treasurer out of the funds to be appropriated for the purpose.
Upon posting of the redelivery bond, the writ is automatically
discharged. Without the writ, the property should be returned to the

37 Aicka Singson
Notes in Remedial Law Review II
Provisional Remedies, Special Civil Actions, Special Proceedings and Evidence
School Year 2015-2016 REMEDIAL LAW REVIEW II - ATTY. BRONDIAL

What is the procedure here? The third party claimant is a non-party attachment was issued or his surety or sureties, before the judgment
to the action. Third party claims are formulated in the same way as in of the appellate court becomes executory. The appellate court may
Section 14, Rule 57 and Section 16, Rule 39. The third party allow the application to be heard and decided by the trial court.
claimant would execute an affidavit of third party claim, gives that to
the sheriff who is in possession of the property, and the sheriff is Nothing herein contained shall prevent the party against whom the
supposed to give or to surrender the property subject of the writ of attachment was issued from recovering in the same action the
replevin (under Rule 57, subject of attachment; under Rule 39, damages awarded to him from any property of the attaching party
subject of execution) to the third party claimant, unless the applicant not exempt from execution should the bond or deposit given by the
under Rule 60 posts another bond. It is not the third party claimant latter be insufficient or fail to fully satisfy the award.
that posts the bond. Rather, it is the applicant in Rule 57. He has
Section 8. Return of papers. — The sheriff must file the order, with
already posted twice the value of the property. Here again, when
his proceedings indorsed, thereon, with the court within ten (10) days
there is a third party claim, he has to post another bond equivalent to
after taking the property mentioned therein.
the value of the property. In effect, when there is a third party
claimant, the applicant posts a bond equivalent to thrice the value of
4. Judgment and Damages (Sections 9 and 10)
the property.
Sec. 9. Judgment.
What would be the remedy of the applicant in the event that the third
party claim is frivolous or baseless or unwarranted? Section 20, After trial of the issues, the court shall determine who has the right of
which is cross-referred in all the provisional remedies except for possession to and the value of the property and shall render judgment in the
support pendente lite. It is not only in the same action but even in a alternative for the delivery thereof to the party entitled to the same, or for its
separate action. value in case delivery cannot be made, and also for such damages as either
party may prove, with costs.
Rule 57, Section 20. Claim for damages on account of improper,
irregular or excessive attachment. — An application for damages on Sec. 10. Judgment to include recovery against sureties.
account of improper, irregular or excessive attachment must be filed The amount, if any, to be awarded to any party upon any bond filed in
before the trial or before appeal is perfected or before the judgment accordance with the provisions of this Rule, shall be claimed, ascertained,
becomes executory, with due notice to the attaching party and his and granted under the same procedure as prescribed in section 20 of Rule
surety or sureties setting forth the facts showing his right to damages 57.
and the amount thereof. Such damages may be awarded only after
proper hearing and shall be included in the judgment on the main Smart Communications vs Astorga: Astorga was employed in Smart
case. Communications. She was transferred to another division but she
declined it. Actually, there was an employee evaluation. She did not
If the judgment of the appellate court be favorable to the party get a good grade. So this was her excuse. She was dismissed. She
against whom the attachment was issued he must claim damages filed a case of illegal dismissal before the NLRC. As an employee,
sustained during the pendency of the appeal by filing an application Astorga was entitled to a car under an installment plan deducted
in the appellate court, with notice to the party in whose favor the from her salary. After her dismissal, Smart notified her that she has

38 Aicka Singson
Notes in Remedial Law Review II
Provisional Remedies, Special Civil Actions, Special Proceedings and Evidence
School Year 2015-2016 REMEDIAL LAW REVIEW II - ATTY. BRONDIAL

to return the car. Astorga said she will just continue paying from her P133,824.00 payable in monthly installments. To secure payment,
own pocket. Smart refused because it was a company car. Because petitioner executed a chattel mortgage over the subject motor vehicle
Astorga still didn’t return the car, Smart filed an action for recovery of in favor of Fiesta Motor Sales Corp. On September 28, 1983, Fiesta
the car with prayer for immediate recovery of possession (replevin) Motor Sales assigned the promissory note and chattel mortgage to
with the RTC of Makati. RTC denied the MTD. CA, reversing the private respondent FCP Credit Corporation. The complaint further
RTC, said the RTC has no jurisdiction because the case is clearly a alleged that petitioner failed to pay part of the installment which fell
case of employer-employee relationship. Astorga would not have due on July 28, 1984 as well as three (3) consecutive installment
had the car had it not been for the fact that she was employed with which fell due on August 28, September 28, and October 28, 1984.
Smart. This was therefore a result of her employment. Consequently, private respondent FCP Credit Corporation
demanded from petitioner payment of the entire outstanding balance
SC: CA was reversed. An employer-employee relationship may be of the obligation amounting to P106,154.48 with accrued interest and
converted to a debtor-creditor relationship. There is no more to surrender the vehicle which petitioner was allegedly detaining.
employer-employee relationship. It is already debtor-creditor
relationship so the RTC has jurisdiction. After trial, the lower court dismissed private respondent's complaint.

Professor: I agree with the disposition of the CA. The trial court ruled that private respondent FCP had no reason to
file the present action since petitioner already paid the installments
Does the NLRC have jurisdiction over replevin? Even if the case is for the months of July to November 1984, which are the sole bases
pending before the NLRC, can the NLRC issue a writ? Let’s look at of the complaint. The lower court declared that private respondent
provisional remedies in general. In criminal procedure, it is very was not entitled to the writ of replevin, and was liable to petitioner for
clear. Provisional remedies apply in criminal cases (Rule 127). actual damages under the replevin bond it filed.

Navarro vs Escobido: There is no need for a demand for the Ruling on petitioner's counterclaim, the trial court stated that there
issuance of the writ of replevin. was no legal or factual basis for the writ of replevin and that its
enforcement by the sheriff was "highly irregular, and unlawful, done,
CASES as it was, under shades of extortion, threats and force." The trial
court ordered private respondent to pay moral damages; exemplary
1. Orosa vs. CA (329 SCRA) damages and attorney's fees. Private respondent was also ordered
to return to petitioner the 1983 Ford Laser 1.5 Sedan, or its
FACTS: Private respondent FCP Credit Corporation filed a complaint equivalent, in kind or value in cash, as of date of judgment and to
for replevin and damages in the RTC of Manila against petitioner pay the costs of the suit.
Orosa and one John Doe to recover possession of a 1983 Ford
Laser 1.5 Sedan. The complaint alleged that on September 28, On June 7, 1988, a "Supplemental Decision" was rendered by the
1983, petitioner purchased the subject motor vehicle on installment trial court ordering private respondent's surety, Stronghold Insurance
from Fiesta Motor Sales Corporation. He executed and delivered to Co., Inc. to jointly and severally [with private respondent].
Fiesta Motor Sales Corp. a promissory note in the sum of

39 Aicka Singson
Notes in Remedial Law Review II
Provisional Remedies, Special Civil Actions, Special Proceedings and Evidence
School Year 2015-2016 REMEDIAL LAW REVIEW II - ATTY. BRONDIAL

The surety company filed with the CA a petition for certiorari to annul was the main decision, while the subject of the petition in CA-G.R.
the Order of the trial court denying its motion for partial SP No. 14938 was the Supplemental Decision.
reconsideration, as well as the Supplemental Decision – DENIED.
On the other hand, private respondent appealed the decision of the We agree with the CA that:
RTC Manila to the CA – decision of RTC was PARTIALLY
The decisions of the Court of Appeals in CA-G.R. SP No. 14938 and
AFFIRMED. Hence, this petition for review.
the Supreme Court in G.R. No. 84979 did not pass on the merits of
ISSUE: WON the CA (8th Division) acted without or in excess of this case. It merely ruled on the issues of whether the surety,
jurisdiction when reversed a final decision of a co-equal Stronghold Insurance, Co., Inc., can be held jointly and solidarily
division of the CA (Special First Division), and which was liable with plaintiff-appellant and whether execution pending appeal
sustained by the Hon. Supreme Court in a final decision which is proper under the facts and circumstances of this case.
cases have the same causes of actions, same set of facts, the Consequently, this Court is not estopped from reviewing the
same parties and the same relief. conclusions reached by the court a quo.

HELD: NO. Jurisdiction is simply the power or authority to hear a We also agree with the Court of Appeals that the trial court erred
case. The appellate jurisdiction of the CA to review decisions and when it ordered private respondent to return the subject car or its
orders of lower courts is conferred by BP Blg. 129. More importantly, equivalent considering that petitioner had not yet fully paid the
petitioner cannot now assail the Court of Appeals' jurisdiction after purchase price. Verily, to sustain the trial court's decision would
having actively participated in the appeal and after praying for amount to unjust enrichment. The Court of Appeals was correct
affirmative relief. when it instead ordered private respondent to return, not the car
itself, but only the amount equivalent to the fourteen installments
Neither can petitioner argue that res judicata bars the determination actually paid with interest.
of the present case. The two cases involve different subject matters,
parties and seek different reliefs. 2. Smart Communications vs. Astorga (542 SCRA)
SMART launched an organizational realignment to achieve more
The petition docketed as CA-G.R. SP No. 14938 was for certiorari efficient operations. SMART entered into a joint venture agreement
with injunction, brought by Stronghold Insurance Company, Inc. with NTT of Japan, and formed SMART-NTT Multimedia,
alleging that there was grave abuse of discretion when the trial court Incorporated (SNMI). Since SNMI was formed to do the sales and
adjudged it liable for damages without due process, in violation of marketing work, SMART abolished the CSMG/FSD, Astorga’s
Rule 60, Section 10 in relation to Rule 57, Section 20, of the Rules of division.
Court. The surety also questioned the propriety of the writ of Despite the abolition of the CSMG/FSD, Astorga continued reporting
execution issued by the trial court pending appeal. for work. But on March 3, 1998, SMART issued a memorandum
advising Astorga of the termination of her employment on ground of
On the other hand, CA-G.R. CV No. 25929 was filed by petitioner redundancy, effective April 3, 1998. Astorga received it on March 16,
Orosa under Rule 45 of the Revised Rules of Court raising alleged 1998.
errors of law on the part of the trial court. The subject of the appeal

40 Aicka Singson
Notes in Remedial Law Review II
Provisional Remedies, Special Civil Actions, Special Proceedings and Evidence
School Year 2015-2016 REMEDIAL LAW REVIEW II - ATTY. BRONDIAL

The termination of her employment prompted Astorga to file a Astorga filed a motion for reconsideration, but the RTC denied.
Complaint for illegal dismissal. She claimed that abolishing CSMG Astorga elevated the denial of her motion via certiorari to the CA,
and, consequently, terminating her employment was illegal for it which, reversed the RTC ruling. Granting the petition and,
violated her right to security of tenure. consequently, dismissing the replevin case, the CA held that the
SMART responded that there was valid termination. It argued that case is intertwined with Astorga’s complaint for illegal dismissal;
Astorga was dismissed by reason of redundancy, which is an thus, it is the labor tribunal that has rightful jurisdiction over the
authorized cause for termination of employment, and the dismissal complaint. SMART’s motion for reconsideration having been denied,
was effected in accordance with the requirements of the Labor Code. it elevated the case to this Court, now docketed as G.R. No. 148132.
In the meantime, on May 18, 1998, SMART sent a letter to Astorga ISSUE: WON THE RTC HAS JURISDICTION OVER THE
demanding that she pay the current market value of the Honda Civic REPLEVIN CASE
Sedan which was given to her under the company’s car plan YES. The Court shall first deal with the propriety of dismissing the
program, or to surrender the same to the company for proper replevin case filed with the RTC of Makati City allegedly for lack of
disposition. Astorga, however, failed and refused to do either, thus jurisdiction, which is the issue raised in G.R. No. 148132.
prompting SMART to file a suit for replevin with the Regional Trial Replevin is an action whereby the owner or person entitled to
Court of Makati (RTC). repossession of goods or chattels may recover those goods or
Astorga moved to dismiss the complaint on grounds of (i) lack of chattels from one who has wrongfully distrained or taken, or who
jurisdiction; (ii) failure to state a cause of action; (iii) litis pendentia; wrongfully detains such goods or chattels. It is designed to permit
and (iv) forum-shopping. Astorga posited that the regular courts have one having right to possession to recover property in specie from
no jurisdiction over the complaint because the subject thereof one who has wrongfully taken or detained the property. The term
pertains to a benefit arising from an employment contract; hence, may refer either to the action itself, for the recovery of personalty, or
jurisdiction over the same is vested in the labor tribunal and not in to the provisional remedy traditionally associated with it, by which
regular courts. Pending resolution of Astorga’s motion to dismiss the possession of the property may be obtained by the plaintiff and
replevin case, the Labor Arbiter rendered a Decision, declaring retained during the pendency of the action.
Astorga’s dismissal from employment illegal. That the action commenced by SMART against Astorga in the RTC
Subsequently, the RTC issued an Order denying Astorga’s motion to of Makati City was one for replevin hardly admits of doubt.
dismiss the replevin case. In reversing the RTC ruling and consequently dismissing the case for
As correctly pointed out, this case is to enforce a right of possession lack of jurisdiction, the CA made the following disquisition, viz.:
over a company car assigned to the defendant under a car plan [I]t is plain to see that the vehicle was issued to [Astorga] by [Smart]
privilege arrangement. The car is registered in the name of the as part of the employment package. We doubt that [SMART] would
plaintiff. Recovery thereof via replevin suit is allowed by Rule 60 of extend [to Astorga] the same car plan privilege were it not for her
the 1997 Rules of Civil Procedure, which is undoubtedly within the employment as district sales manager of the company. Furthermore,
jurisdiction of the Regional Trial Court. there is no civil contract for a loan between [Astorga] and [Smart].
In the Complaint, plaintiff claims to be the owner of the company car Consequently, We find that the car plan privilege is a benefit arising
and despite demand, defendant refused to return said car. This is out of employer-employee relationship. Thus, the claim for such falls
clearly sufficient statement of plaintiff’s cause of action.

41 Aicka Singson
Notes in Remedial Law Review II
Provisional Remedies, Special Civil Actions, Special Proceedings and Evidence
School Year 2015-2016 REMEDIAL LAW REVIEW II - ATTY. BRONDIAL

squarely within the original and exclusive jurisdiction of the labor In thus ruling, this Court is not sanctioning split jurisdiction but
arbiters and the NLRC. defining avenues of jurisdiction as laid down by pertinent laws.
We do not agree. Contrary to the CA’s ratiocination, the RTC The CA, therefore, committed reversible error when it overturned the
rightfully assumed jurisdiction over the suit and acted well within its RTC ruling and ordered the dismissal of the replevin case for lack of
discretion in denying Astorga’s motion to dismiss. SMART’s demand jurisdiction.
for payment of the market value of the car or, in the alternative, the 3. Hao vs Andres (444 SCRA)
surrender of the car, is not a labor, but a civil, dispute. It involves the
relationship of debtor and creditor rather than employee-employer Before us is an administrative complaint for gross neglect of duty,
relations.33 As such, the dispute falls within the jurisdiction of the grave abuse of authority (oppression) and violation of Republic Act
regular courts. No. 3019 filed by complainant Kenneth Hao against respondent Abe
In Basaya, Jr. v. Militante,34 this Court, in upholding the jurisdiction C. Andres, Sheriff IV of the RTC of Davao City, Branch 16.
of the RTC over the replevin suit, explained:
FACTS: Complainant Hao is one of the defendants in a civil case for
Replevin is a possessory action, the gist of which is the right of
replevin entitled "Zenaida Silver v. Hao" pending before the RTC of
possession in the plaintiff. The primary relief sought therein is the
Davao City, Branch 16.
return of the property in specie wrongfully detained by another
person. It is an ordinary statutory proceeding to adjudicate rights to On October 17, 2005, Judge Fuentes issued an Order of Seizure
the title or possession of personal property. The question of whether against 22 motor vehicles allegedly owned by the complainant. On
or not a party has the right of possession over the property involved the strength of the said order, Andres was able to seize two of the
and if so, whether or not the adverse party has wrongfully taken and subject motor vehicles on October 17, 2005; four on October 18,
detained said property as to require its return to plaintiff, is outside 2005, and another three on October 19, 2005, or a total of 9 motor
the pale of competence of a labor tribunal and beyond the field of vehicles.
specialization of Labor Arbiters.
xxxx In his Affidavit-Complaint against Andres before the OCA, Hao
The labor dispute involved is not intertwined with the issue in the alleged that Andres gave undue advantage to Zenaida Silver in the
Replevin Case. The respective issues raised in each forum can be implementation of the order and that Andres seized the nine motor
resolved independently on the other. In fact in 18 November 1986, vehicles in an oppressive manner. Hao also averred that Andres was
the NLRC in the case before it had issued an Injunctive Writ accompanied by unidentified armed personnel on board a military
enjoining the petitioners from blocking the free ingress and egress to vehicle which was excessive since there were no resistance from
the Vessel and ordering the petitioners to disembark and vacate. them. Hao also discovered that the compound where the seized
That aspect of the controversy is properly settled under the Labor motor vehicles were placed is actually owned by Silver.
Code. So also with petitioners’ right to picket. But the determination
of the question of who has the better right to take possession of the In view of the approval of the complainant’s counter-replevin bond,
Vessel and whether petitioners can deprive the Charterer, as the Judge Carpio ordered Andres to immediately cease and desist from
legal possessor of the Vessel, of that right to possess in addressed further implementing the order of seizure, and to return the seized
to the competence of Civil Courts. motor vehicles including its accessories to their lawful owners.

42 Aicka Singson
Notes in Remedial Law Review II
Provisional Remedies, Special Civil Actions, Special Proceedings and Evidence
School Year 2015-2016 REMEDIAL LAW REVIEW II - ATTY. BRONDIAL

However, 8 of the 9 seized motor vehicles were reported missing. In vehicle; and (4) Andres admitted that prior to the implementation of
his report, Andres stated that he was shocked to find that the motor the writ of seizure, he consulted Silver and Atty. Macadangdang
vehicles were already missing when he inspected it. He narrated that regarding the implementation of the writ and was accompanied by
on October 21, 2005, PO3 Rodrigo Despe, one of the policemen the latter in the course of the implementation. Judge Fuentes
guarding the subject motor vehicles, reported to him that a certain observed that the motor vehicles were speedily seized without strictly
"Nonoy" entered the compound and caused the duplication of the observing fairness and regularity in its implementation.
vehicles’ keys. But Andres claimed the motor vehicles were still
intact when he inspected it on October 21, 2005. Anent the safekeeping of the seized motor vehicles, Judge Fuentes
pointed out several instances where Andres lacked due diligence to
Subsequently, Hao reported that three of the carnapped vehicles wit: (1) the seized motor vehicles were placed in a compound
were recovered by the police. He then accused Andres of conspiring surrounded by an insufficiently locked see-through fence; (2) three
and conniving with Atty. Oswaldo Macadangdang (Silver’s counsel) motor vehicles were left outside the compound; (3) Andres turned
and the policemen in the carnapping of the motor vehicles. Hao also over the key of the gate to the policemen guarding the motor
accused Andres of concealing the depository receipts from them and vehicles; (4) Andres does not even know the full name of the owner
pointed out that the depository receipts show that Silver and Atty. of the compound, who was merely known to him as "Gloria"; (5)
Macadangdang were the ones who chose the policemen who will except for PO3 Despe and SPO4 Nelson Salcedo, the identities of
guard the motor vehicles. the other policemen tapped to guard the compound were unknown to
Andres; (6) Andres also admitted that he only stayed at least one
In his Comment, Andres vehemently denied violating Rep. Act No. hour each day from October 19-21, 2005 during his visits to the
3019 and committing gross neglect of duty. compound; and (7) even after it was reported to him that a certain
"Nonoy" entered the compound and duplicated the keys of the motor
After the OCA recommended that the matter be investigated, we
vehicles, he did not exert his best effort to look for that "Nonoy" and
referred the case to Executive Judge Fuentes for investigation,
to confiscate the duplicated keys.
report and recommendation.
Judge Fuentes also observed that Andres appeared to be more or
In his Investigation Report, Judge Fuentes found Andres guilty of
less accommodating to Silver and her counsel but hostile and
serious negligence in the custody of the 9 motor vehicles. He
uncooperative to the complainant. He pointed out that Andres
recommended that Andres be suspended from office.
depended solely on Silver in the selection of the policemen who
Judge Fuentes found numerous irregularities in the implementation would guard the seized motor vehicles. He added that even the
of the writ of replevin/order of seizure, to wit: (1) at the time of the depository receipts were not turned over to the defendants/third-
implementation of the writ, Andres knew that the vehicles to be party claimants in the replevin case but were in fact concealed from
seized were not in the names of any of the parties to the case; (2) them. Andres also gave inconsistent testimonies as to whether he
one vehicle was taken without the knowledge of its owner, a certain has in his possession the depository receipts.
Junard Escudero; (3) Andres allowed Atty. Macadangdang to get a
keymaster to duplicate the vehicles’ keys in order to take one motor

43 Aicka Singson
Notes in Remedial Law Review II
Provisional Remedies, Special Civil Actions, Special Proceedings and Evidence
School Year 2015-2016 REMEDIAL LAW REVIEW II - ATTY. BRONDIAL

The OCA disagreed with the observations of Judge Fuentes. It any reason the property is not delivered to the applicant, the sheriff
recommended that Andres be held liable only for simple neglect of must return it to the adverse party.
duty and be suspended for one (1) month and one (1) day.
First, the rules provide that property seized under a writ of replevin is
ISSUE: WON sheriff Andres is guilty of gross neglect of duty not to be delivered immediately to the plaintiff. In accordance with
and grave abuse of authority the said rules, Andres should have waited no less than five days in
order to give the complainant an opportunity to object to the
HELD: YES. Being an officer of the court, Andres must be aware that sufficiency of the bond or of the surety or sureties thereon, or require
there are well-defined steps provided in the Rules of Court regarding the return of the seized motor vehicles by filing a counter-bond. This,
the proper implementation of a writ of replevin and/or an order of he failed to do.
seizure. The Rules, likewise, is explicit on the duty of the sheriff in its
implementation. To recapitulate what should be common knowledge Records show that Andres took possession of two of the subject
to sheriffs, the pertinent provisions of Rule 60, of the Rules of Court motor vehicles on October 17, 2005, four on October 18, 2005, and
are quoted hereunder: another three on October 19, 2005. Simultaneously, as evidenced by
the depository receipts, on October 18, 2005, Silver received from
SEC. 4. Duty of the sheriff.–Upon receiving such order, the sheriff Andres six of the seized motor vehicles, and three more motor
must serve a copy thereof on the adverse party, together with a copy vehicles on October 19, 2005. Consequently, there is no question
of the application, affidavit and bond, and must forthwith take the that Silver was already in possession of the nine seized vehicles
property, if it be in the possession of the adverse party, or his agent, immediately after seizure, or no more than three days after the taking
and retain it in his custody. If the property or any part thereof be of the vehicles. Thus, Andres committed a clear violation of Section
concealed in a building or enclosure, the sheriff must demand its 6, Rule 60 of the Rules of Court with regard to the proper disposal of
delivery, and if it be not delivered, he must cause the building or the property.
enclosure to be broken open and take the property into his
possession. After the sheriff has taken possession of the property as It matters not that Silver was in possession of the seized vehicles
herein provided, he must keep it in a secure place and shall be merely for safekeeping as stated in the depository receipts. The rule
responsible for its delivery to the party entitled thereto upon receiving is clear that the property seized should not be immediately delivered
his fees and necessary expenses for taking and keeping the same. to the plaintiff, and the sheriff must retain custody of the seized
property for at least five days. Hence, the act of Andres in delivering
SEC. 6. Disposition of property by sheriff.–If within five (5) days after the seized vehicles immediately after seizure to Silver for whatever
the taking of the property by the sheriff, the adverse party does not purpose, without observing the five-day requirement finds no legal
object to the sufficiency of the bond, or of the surety or sureties justification.
thereon; or if the adverse party so objects and the court affirms its
approval of the applicant’s bond or approves a new bond, or if the Second, it must be stressed that from the moment an order of
adverse party requires the return of the property but his bond is delivery in replevin is executed by taking possession of the property
objected to and found insufficient and he does not forthwith file an specified therein, such property is in custodia legis. As legal
approved bond, the property shall be delivered to the applicant. If for custodian, it is Andres’ duty to safekeep the seized motor vehicles.

44 Aicka Singson
Notes in Remedial Law Review II
Provisional Remedies, Special Civil Actions, Special Proceedings and Evidence
School Year 2015-2016 REMEDIAL LAW REVIEW II - ATTY. BRONDIAL

Hence, when he passed his duty to safeguard the motor vehicles to then represented by its Manager, the aforementioned GLENN O.
Silver, he committed a clear neglect of duty. GO, and defendant ROGER NAVARRO xxx; That plaintiff hereby
respectfully applies for an order of the Honorable Court for the
Third, we are appalled that even after PO3 Despe reported the immediate delivery of the above-described motor vehicle from
unauthorized duplication of the vehicles’ keys, Andres failed to take defendants unto plaintiff pending the final determination of this case
extra precautionary measures to ensure the safety of the vehicles. It on the merits and, for that purpose, there is attached hereto an
is obvious that the vehicles were put at risk by the unauthorized affidavit duly executed and bond double the value of the personal
duplication of the keys of the vehicles. Neither did he immediately property subject matter hereof to answer for damages and costs
report the incident to the police or to the court. Indeed, Andres’ which defendants may suffer in the event that the order for replevin
failure to take the necessary precaution and proper monitoring of the prayed for may be found out to having not been properly issued.
vehicles to ensure its safety constitutes plain negligence. The RTC issued writs of replevin for both cases; as a result, the
Sheriff seized the two vehicles and delivered them to the possession
Fourth, despite the cease and desist order, Andres failed to return
of Karen Go.
the motor vehicles to their lawful owners. Instead of returning the
In his Answers, Navarro alleged as a special affirmative defense that
motor vehicles immediately as directed, he opted to write Silver and
the two complaints stated no cause of action, since Karen Go was
demand that she put up an indemnity bond to secure the third-party
not a party to the Lease Agreements with Option to Purchase
claims. Consequently, due to his delay, the eventual loss of the
(collectively, the lease agreements) – the actionable documents on
motor vehicles rendered the order to return the seized vehicles
which the complaints were based. RTC dismissed the case on the
ineffectual to the prejudice of the complaining owners.
ground that the complaints did not state a cause of action.
Fifth, as found by the OCA, we agree that Andres also disregarded In response to the motion for reconsideration Karen Go filed dated
the provisions of Rule 141 of the Rules of Court with regard to May 26, 2000, the RTC issued another order setting aside the order
payment of expenses. of dismissal. Acting on the presumption that Glenn Go’s leasing
business is a conjugal property, the RTC held that Karen Go had
4. Navarro vs Escobido (606 SCRA) sufficient interest in his leasing business to file the action against
Respondent Karen T. Go filed two complaints, before the RTC for Navarro.
replevin and/or sum of money with damages against Navarro. In When the RTC denied Navarro’s motion for reconsideration on
these complaints, Karen Go prayed that the RTC issue writs of March 7, 2001, Navarro filed a petition for certiorari with the CA
replevin for the seizure of two (2) motor vehicles in Navarro’s which was denied.
possession. On appeal on the SC, Navarro states that even assuming the
Stating that plaintiff KAREN T. GO is doing business under the trade complaints stated a cause of action against him, Navarro maintains
name KARGO ENTERPRISES, business of buying and selling motor that the complaints were premature because no prior demand was
vehicles, including hauling trucks and other heavy equipment and made on him to comply with the provisions of the lease agreements
said defendant leased [from] plaintiff a certain motor vehicle before the complaints for replevin were filed. Lastly, Navarro posits
evidenced by a LEASE AGREEMENT WITH OPTION TO that since the two writs of replevin were issued based on flawed
PURCHASE entered into by and between KARGO ENTERPRISES,

45 Aicka Singson
Notes in Remedial Law Review II
Provisional Remedies, Special Civil Actions, Special Proceedings and Evidence
School Year 2015-2016 REMEDIAL LAW REVIEW II - ATTY. BRONDIAL

complaints, the vehicles were illegally seized from his possession More importantly, Navarro is no longer in the position to claim that a
and should be returned to him immediately. prior demand is necessary, as he has already admitted in his
Answers that he had received the letters that Karen Go sent him,
HELD: DEMAND IS NOT NEEDED. In arguing that prior demand is demanding that he either pay his unpaid obligations or return the
required before an action for a writ of replevin is filed, Navarro leased motor vehicles. Navarro’s position that a demand is
apparently likens a replevin action to an unlawful detainer. necessary and has not been made is therefore totally unmeritorious.

For a writ of replevin to issue, all that the applicant must do is to file
5. Agner vs. BPI Family Savings Bank (697 SCRA)
an affidavit and bond, pursuant to Section 2, Rule 60 of the Rules,
which states: FACTS: Petitioners spouses Agner executed a Promissory Note with
Chattel Mortgage in favor of Citimotors, Inc.
Sec. 2. Affidavit and bond.
Citimotors, Inc. assigned all its rights, title and interests in the
The applicant must show by his own affidavit or that of some other
Promissory Note with Chattel Mortgage to ABN AMRO Savings
person who personally knows the facts:
Bank, Inc. (ABN AMRO), which, likewise assigned the same to
(a) That the applicant is the owner of the property claimed, respondent BPI Family Savings Bank, Inc.
particularly describing it, or is entitled to the possession thereof;
For failure to pay four successive installments respondent, through
(b) That the property is wrongfully detained by the adverse party,
counsel, sent to petitioners a demand letter declaring the entire
alleging the cause of detention thereof according to the best of his
obligation as due and demandable and requiring to pay
knowledge, information, and belief;
Php576,664.04, or surrender the mortgaged vehicle immediately
(c) That the property has not been distrained or taken for a tax
upon receiving the letter. As the demand was left unheeded,
assessment or a fine pursuant to law, or seized under a writ of
respondent filed an action for Replevin and Damages before the
execution or preliminary attachment, or otherwise placed under
Manila RTC.
custodia legis, or if so seized, that it is exempt from such seizure or
custody; and A writ of replevin was issued. Despite this, the subject vehicle was
(d) The actual market value of the property. not seized. Trial on the merits ensued. The Manila RTC ruled for the
The applicant must also give a bond, executed to the adverse party respondent and ordered petitioners to jointly and severally pay the
in double the value of the property as stated in the affidavit amount of Php576,664.04 plus interest at the rate of 72% per annum
aforementioned, for the return of the property to the adverse party if and the costs of suit.
such return be adjudged, and for the payment to the adverse party of
such sum as he may recover from the applicant in the action. Petitioners appealed the decision to the CA, but the CA affirmed the
We see nothing in these provisions which requires the applicant to lower court’s decision and, subsequently, denied the MR; hence, this
make a prior demand on the possessor of the property before he can petition.
file an action for a writ of replevin. Thus, prior demand is not a
condition precedent to an action for a writ of replevin.

46 Aicka Singson
Notes in Remedial Law Review II
Provisional Remedies, Special Civil Actions, Special Proceedings and Evidence
School Year 2015-2016 REMEDIAL LAW REVIEW II - ATTY. BRONDIAL

ISSUE: WON respondent’s remedy of resorting to both actions 2. Commencement, Hearing, Order (Sections 2, 3 and 4)
of replevin and collection of sum of money is contrary to the
provision of Article 1484 of the Civil Code and the Elisco Tool Sec. 2. Comment.
Manufacturing Corporation v. CA ruling.
A copy of the application and all supporting documents shall be served upon
the adverse party, who shall have five (5) days to comment thereon unless a
HELD: NO. There is no violation of Article 1484 of the Civil Code and
different period is fixed by the court upon his motion. The comment shall be
the Court’s decision in Elisco Tool Manufacturing Corporation v.
verified and shall be accompanied by affidavits, depositions or other
Court of Appeals. authentic documents in support thereof.

The remedies provided for in Art. 1484 are alternative, not Sec. 3. Hearing.
cumulative. The exercise of one bars the exercise of the others. This
limitation applies to contracts purporting to be leases of personal After the comment is filed, or after the expiration of the period for its filing,
property with option to buy by virtue of Art. 1485. the application shall be set for hearing not more than three (3) days
thereafter. The facts in issue shall be proved in the same manner as is
Compared with Elisco, the vehicle subject matter of this case was provided for evidence on motions.
never recovered and delivered to respondent despite the issuance of
Sec. 4. Order.
a writ of replevin. As there was no seizure that transpired, it cannot
be said that petitioners were deprived of the use and enjoyment of The court shall determine provisionally the pertinent facts, and shall render
the mortgaged vehicle or that respondent pursued, commenced or such orders as justice and equity may require, having due regard to the
concluded its actual foreclosure. The trial court, therefore, rightfully probable outcome of the case and such other circumstances as may aid in
granted the alternative prayer for sum of money, which is equivalent the proper resolution of the question involved. If the application is granted,
to the remedy of "exacting fulfillment of the obligation." Certainly, the court shall fix the amount of money to be provisionally paid or such other
forms of support as should be provided, taking into account the necessities
there is no double recovery or unjust enrichment to speak of.
of the applicant and the resources or means of the adverse party, and the
terms of payment or mode for providing the support. If the application is
E. SUPPORT PENDE LITE (Rule 61) denied, the principal case shall be tried and decided as early as possible.

1. Application for Support Pende Lite (Sec 1) Ms. X, an unwed mother, files a case against Mr. Y, whom she
Section 1. Application. claims to have sired a child. How should Ms. X go about it?
At the commencement of the proper action or proceeding, or at any time
prior to the judgment or final order, a verified application for support Ms. X should file with the Family Court where the plaintiff or
pendente lite may be filed by any party stating the grounds for the claim and defendant resides (because this is a personal action) an action for
the financial conditions of both parties, and accompanied by affidavits, support with prayer for support pendente lite. After the filing, Ms. X
depositions or other authentic documents in support thereof. should serve the adverse party with a copy of the application and Mr.
Y shall have five days to comment. This is a comment to the
provisional remedy, not to the principal action. In actual practice,
summons is immediately served and answer is filed. In that answer,

47 Aicka Singson
Notes in Remedial Law Review II
Provisional Remedies, Special Civil Actions, Special Proceedings and Evidence
School Year 2015-2016 REMEDIAL LAW REVIEW II - ATTY. BRONDIAL

you have the comment on the provisional remedy of support subject of writ of execution. So that is peculiar. You don’t find that in
pendente lite. After the 5-day period, with or without the comment, other cases.
there should be a hearing regarding the application for support
pendente lite within the 3-day period. After such hearing, the court Because of the writ of execution issued by the court, Mr. Y starts
should determine the financial status of both parties; the need as to giving P10,000/month while the principal case of support is being
the applicant and the capability as to the adverse party. In support heard by the court. Suppose the case lasted for 10 months before it
and support pendente lite, there are only 2 issues that must be was decided by the Family Court. Mr. Y has already given Ms. X an
resolved: (1) the capacity or capability of adverse party; and (2) amount of P100,000. The Decision says Ms. X is not entitled to
necessity or need of the applicant. The court determines this support. Just because you were granted support pendente lite
provisionally. Then, it may either grant or deny the application for doesn’t mean that you will get the support. The principal action is
support pendente lite. separate and distinct from the provisional remedy. What happens to
the P100,000 that Mr. Y has been given to Ms. X?
3. Enforcement of Order (Section 5)
The amount should be given back to Mr. Y.
Sec. 5. Enforcement of order.
What may be the reason why the court did not grant her support?
If the adverse party fails to comply with an order granting support Maybe paternity was not established.
pendente lite, the court shall, motu proprio or upon motion, issue an
Section 6. Support in criminal cases. — In criminal actions where the
order of execution against him, without prejudice to his liability for
civil liability includes support for the offspring as a consequence of
contempt.
the crime and the civil aspect thereof has not been waived, reserved
When the person ordered to give support pendente lite refuses or and instituted prior to its filing, the accused may be ordered to
fails to do so, any third person who furnished that support to the provide support pendente lite to the child born to the offended party
applicant may, after due notice and hearing in the same case, obtain allegedly because of the crime. The application therefor may be filed
a writ of execution to enforce his right of reimbursement against the successively by the offended party, her parents, grandparents or
person ordered to provide such support. guardian and the State in the corresponding criminal case during its
pendency, in accordance with the procedure established under this
Ms. X was granted by the court support pendente lite in the amount Rule.
of P10,000/month. How should Ms. X get the money if Mr. Y does
4. RESTITUTION (Section 7)
not voluntarily give, notwithstanding the order of support pendente
lite? Sec. 7. Restitution.
Ms. X should file a writ of execution. This is a characteristic peculiar When the judgment or final order of the court finds that the person
to support pendente lite because an order granting support pendente who has been providing support pendente lite is not liable therefor, it
lite is not a final order; it is an interlocutory order. But this time, it is shall order the recipient thereof to return to the former the amounts
already paid with legal interest from the dates of actual payment,

48 Aicka Singson
Notes in Remedial Law Review II
Provisional Remedies, Special Civil Actions, Special Proceedings and Evidence
School Year 2015-2016 REMEDIAL LAW REVIEW II - ATTY. BRONDIAL

without prejudice to the right of the recipient to obtain reimbursement alleging that the defendant Manuel de Asis (the petitioner here) is the
in a separate action from the person legally obliged to give the father of subject minor Glen Camil Andres de Asis, and the former
support. Should the recipient fail to reimburse said amounts, the refused and/or failed to provide for the maintenance of the latter,
person who provided through same may likewise seek despite repeated demands.
reimbursement thereof in a separate action from the person legally In his Answer, petitioner denied his paternity of the said minor and
obliged to give such support. theorized that he cannot therefore be required to provide support for
him.
Both the plaintiff and the defendant agreed to move for the dismissal
of the case.
What if Mr. Y becomes incapacitated? Can that be a valid ground? Another Complaint for maintenance and support was brought against
There are 2 remedies available under the Rules: (1) reimbursement; Manuel A. de Asis, this time in the name of Glen Camil Andres de
and (2) restitution. Reimbursement always entails a third party. An Asis, represented by her legal guardian/mother, Vircel D. Andres.
example is if instead of Mr. Y giving the P10,000/month, it was the Petitioner moved to dismiss the Complaint on the ground of res
father of Mr. Y who was giving the money, then there can be a judicata, alleging that Civil Case C-16107 is barred by the prior
reimbursement. Restitution is only between the 2 parties. If the court judgment.
pronounces that Ms. X is not entitled to support, then Ms. X must In the Order dated November 25, 1993 denying subject motion to
restitute the P100,000 which has already been given by Mr. Y. dismiss, the trial court ruled that res judicata is inapplicable in an
action for support for the reason that renunciation or waiver of future
Is that reasonable? Here is a person who goes to court and asks for support is prohibited by law. Petitioner's motion for reconsideration of
support and now that the court ruled otherwise, she is asked to the said Order met the same fate. It was likewise denied.
restitute. It is justified because a person is asked to give support Petitioner filed with the Court of Appeals a Petition for Certiorari. But
when he has no obligation to. This is a recognition of the fact that on June 7, 1996, the Court of Appeals found that the said Petition
people in general is becoming. A person is not static, a person devoid of merit and dismissed the same.
progresses. Perhaps the court realizes that at the time she was Undaunted, petitioner found his way to this court via the present
asking for support, she was just working a small videoke bar. She petition, posing the question whether or not the public respondent
was really in need of support. But now that she is asked to restitute, acted with grave abuse of discretion amounting to lack or excess of
the court perhaps realize that she is already working at the Pegasus. jurisdiction in upholding the denial of the motion to dismiss by the
She is more capable. There is a recognition of the fact that one trial court, and holding that an action for support cannot be barred by
improves. res judicata.
The right to receive support can neither be renounced nor
CASES: transmitted to a third person. Article 301 of the Civil Code, the law in
a. De Asis vs, CA (303 SCRA) point, reads:
Vircel D. Andres, (the herein private respondent) in her capacity as Art. 301. The right to receive support cannot be renounced, nor can it
the legal guardian of the minor, Glen Camil Andres de Asis, brought be transmitted to a third person. Neither can it be compensated with
an action for maintenance and support against Manuel de Asis, what the recipient owes the obligor. . . .

49 Aicka Singson
Notes in Remedial Law Review II
Provisional Remedies, Special Civil Actions, Special Proceedings and Evidence
School Year 2015-2016 REMEDIAL LAW REVIEW II - ATTY. BRONDIAL

Furthermore, future support cannot be the subject of a compromise. existence or absence. It cannot be left to the will or agreement of the
Art. 2035, ibid, provides, that: parties.
No compromise upon the following questions shall be valid: The new Civil Code provides that the allowance for support is
(1) The civil status of persons; provisional because the amount may be increased or decreased
(2) The validity of a marriage or legal separation; depending upon the means of the giver and the needs of the
(3) Any ground for legal separation recipient (Art. 297); and that the right to receive support cannot be
(4) Future support; renounced nor can it be transmitted to a third person neither can it
(5) The jurisdiction of courts; be compensated with what the recipient owes the obligator (Art
(6) Future legitime. .301). Furthermore, the right to support can not be waived or
The raison d' etre behind the proscription against renunciation, transferred to third parties and future support cannot be the subject
transmission and/or compromise of the right to support is stated, of compromise (Art. 2035; Coral v. Gallego, 38 O.G. 3135, cited in IV
thus: Civil Code by Padilla, p. 648; 1956 Ed.). This being true, it is
The right to support being founded upon the need of the recipient to indisputable that the present action for support can be brought,
maintain his existence, he is not entitled to renounce or transfer the notwithstanding the fact the previous case filed against the same
right for this would mean sanctioning the voluntary giving up of life defendant was dismissed. And it also appearing that the dismissal of
itself. The right to life cannot be renounce; hence, support which is Civil Case No. 3553, was not an adjudication upon the merits, as
the means to attain the former, cannot be renounced. heretofore shown, the right of herein plaintiff-appellant to reiterate
xxx xxx xxx her suit for support and acknowledgment is available, as her needs
To allow renunciation or transmission or compensation of the family arise. Once the needs of plaintiff arise, she has the right to bring an
right of a person to support is virtually to allow either suicide or the action for support, for it is only then that her cause for action is
conversion of the recipient to a public burden. This is contrary to accrues.
public policy.
In the case at bar, respondent minor's mother, who was the plaintiff b. People vs. Manahan (315 SCRA)
in the first case, manifested that she was withdrawing the case as it FACTS: MANUEL MANAHAN was found guilty of rape and
seemed futile to claim support from petitioner who denied his sentenced to death by the court a quo. He was also ordered to
paternity over the child. Since the right to claim for support is indemnify the victim P50,000.00 as moral damages, pay the costs,
predicated on the existence of filiation between the minor child and and acknowledge and support the offspring of his indiscretion.
the putative parent, petitioner would like us to believe that such
manifestation admitting the futility of claiming support from him puts Complainant Teresita Tibigar, 16 years old, worked at the Espiritu
the issue to rest and bars any and all future complaint for support. Canteen in Dagupan City. As a stay-in waitress she slept at the
It is true that in order to claim support, filiation and/or paternity must second floor of the canteen. Manuel Manahan is the brother-in-law
first be shown between the claimant and the parent. However, of Josefina Espiritu, owner of the canteen. His wife Primadonna is
paternity and filiation or the lack of the same is a relationship that the sister of Josefina Espiritu.
must be judicially established and it is for the court to declare its

50 Aicka Singson
Notes in Remedial Law Review II
Provisional Remedies, Special Civil Actions, Special Proceedings and Evidence
School Year 2015-2016 REMEDIAL LAW REVIEW II - ATTY. BRONDIAL

On 5 January 1995, at about 2:00am, Teresita who was asleep was under review is accordingly deleted. In any case, we sustain that
suddenly awakened when she felt someone beside her. Upon part ordering the accused to support the child as it is in accordance
opening her eyes she saw accused Manuel Manahan as he with law.
immediately placed himself on top of her. He succeeded in having
carnal knowledge of her. c. Lim vs. Lim (604 SCRA)
Respondent Cheryl S. Lim (Cheryl) married Edward Lim (Edward),
Within the month Teresita left the canteen and returned home to her son of petitioners. They have three children, respondents Lester
parents in Mangaldan, Pangasinan. The sexual encounter resulted Edward, Candice Grace and Mariano III. They resided at the house
in her pregnancy. When her parents discovered it and learned of her of petitioners in Forbes Park, together with Edward’s ailing
story, they brought her to the hospital where she was examined by grandmother, Chua Giak and her husband Mariano Lim (Mariano).
Dr. Casimero Bacugan. Thereafter, with the assistance of her Edward’s family business, which provided him with a monthly salary
mother, Teresita filed a criminal complaint accusing Manuel of P6,000, shouldered the family expenses. Cheryl had no steady
Manahan of rape. Meanwhile, she gave birth to a healthy baby girl source of income.
and christened her Melanie Tibigar. Cheryl abandoned the Forbes Park residence, bringing the children
with her, after a violent confrontation with Edward whom she caught
The accused was arrested in connection with the case filed by with the in-house midwife of Chua Giak in what the trial court
Teresita but was later released. We fail to discern from the records described “a very compromising situation.”
the reason for his release. But he was again arrested and detained Cheryl, for herself and her children, sued petitioners, Edward,
at the Dagupan City Jail. Chua Giak and Mariano (defendants) in the Regional Trial Court of
Makati City, Branch 140 (trial court) for support. The trial court
ISSUE: WON persons guilty of rape may be compelled to ordered Edward to provide monthly support of P6,000 pendente lite.
acknowledge the child and give support The trial court rendered judgment ordering Edward and petitioners to
“jointly” provide P40,000 monthly support to respondents, with
HELD: On the matter of acknowledgment and support of the child, a Edward shouldering P6,000 and petitioners the balance of P34,000
correction of the view of the court a quo is in order. Article 345 of subject to Chua Giak’s subsidiary liability.
The Revised Penal Code provides that persons guilty of rape shall The defendants sought reconsideration, questioning their liability.
also be sentenced to "acknowledge the offspring, unless the law The trial court, while denying reconsideration, clarified that
should prevent him from doing so," and "in every case to support the petitioners and Chua Giak were held jointly liable with Edward
offspring." In the case before us, compulsory acknowledgment of the because of the latter’s “inability x x x to give sufficient support x x x.”
child Melanie Tibigar is not proper there being a legal impediment in Petitioners appealed to the Court of Appeals assailing, among
doing so as it appears that the accused is a married man. As others, their liability to support respondents. Petitioners argued that
pronounced by this Court in People v. Guerrero, "the rule is that if the while Edward’s income is insufficient, the law itself sanctions its
rapist is a married man, he cannot be compelled to recognize the effects by providing that legal support should be “in keeping with the
offspring of the crime, should there be any, as his child, whether financial capacity of the family”
legitimate or illegitimate." Consequently, that portion of the judgment

51 Aicka Singson
Notes in Remedial Law Review II
Provisional Remedies, Special Civil Actions, Special Proceedings and Evidence
School Year 2015-2016 REMEDIAL LAW REVIEW II - ATTY. BRONDIAL

The Court of Appeals affirmed the trial court. On the issue material to liable. Relying on provisions[11] found in Title IX of the Civil Code, as
this appeal, that is, whether there is basis to hold petitioners, as amended, on Parental Authority, petitioners theorize that their liability
Edward’s parents, liable with him to support respondents, the Court is activated only upon default of parental authority, conceivably either
of Appeals held: by its termination or suspension during the children’s minority.
The law on support under Article 195 of the Family Because at the time respondents sued for support, Cheryl and
Code is clear on this matter. Parents and their legitimate children Edward exercised parental authority over their children, petitioners
are obliged to mutually support one another and this obligation submit that the obligation to support the latter’s offspring ends with
extends down to the legitimate grandchildren and great them.
grandchildren. Thus, although the obligation to provide support arising from parental
In connection with this provision, Article 200 authority ends upon the emancipation of the child, the same
paragraph (3) of the Family Code clearly provides that should the obligation arising from spousal and general familial ties ideally lasts
person obliged to give support does not have sufficient means to during the obligee's lifetime. we observed in another case raising the
satisfy all claims, the other persons enumerated in Article 199 in its ancillary issue of an ascendant’s obligation to give support in light of
order shall provide the necessary support. This is because the the father’s sufficient means:
closer the relationship of the relatives, the stronger the tie that binds Professor Pineda is of the view that grandchildren
them. Thus, the obligation to support is imposed first upon the cannot demand support directly from their grandparents if they have
shoulders of the closer relatives and only in their default is the parents (ascendants of nearest degree) who are capable of
obligation moved to the next nearer relatives and so on.[8] supporting them. This is so because we have to follow the order of
Petitioners sought reconsideration but the Court of Appeals support under Art. 199. We agree with this view.
denied their motion in the Resolution dated 12 April 2004.
Hence, this petition. xxxx
There is no showing that private respondent is without
The issue is whether petitioners are concurrently liable with means to support his son; neither is there any evidence to prove that
Edward to provide support to respondents. petitioner, as the paternal grandmother, was willing to voluntarily
provide for her grandson's legal support. x x x[18] (Emphasis
YES. However, SC modify the appealed judgment by limiting supplied; internal citations omitted)
petitioners’ liability to the amount of monthly support needed by
respondents Lester Edward, Candice Grace and Mariano III only. Here, there is no question that Cheryl is unable to discharge
her obligation to provide sufficient legal support to her children, then
Petitioners Liable to Provide Support all school-bound. It is also undisputed that the amount of support
but only to their Grandchildren Edward is able to give to respondents, P6,000 a month, is insufficient
By statutory and jurisprudential mandate, the liability of ascendants to meet respondents’ basic needs. This inability of Edward and
to provide legal support to their descendants is beyond cavil. Cheryl to sufficiently provide for their children shifts a portion of their
Petitioners themselves admit as much – they limit their petition to the obligation to the ascendants in the nearest degree, both in the
narrow question of when their liability is triggered, not if they are paternal (petitioners) and maternal lines, following the ordering in

52 Aicka Singson
Notes in Remedial Law Review II
Provisional Remedies, Special Civil Actions, Special Proceedings and Evidence
School Year 2015-2016 REMEDIAL LAW REVIEW II - ATTY. BRONDIAL

Article 199. To hold otherwise, and thus subscribe to petitioners’ During the pendency of the case, the RTC, on the respondent’s
theory, is to sanction the anomalous scenario of tolerating extreme motion, granted a P2,000.00 monthly child support.
material deprivation of children because of parental inability to give The RTC dismissed the complaint for insufficiency of evidence
adequate support even if ascendants one degree removed are more proving Gliffze’s filiation. It ordered the respondent to return the
than able to fill the void. amount of support pendente lite erroneously awarded, and to pay P
10,000.00 as attorney’s fees.
d. Gotardo vs. Buling (678 SCRA) The respondent appealed the RTC ruling to the CA.
The CA departed from the RTC's appreciation of the respondent’s
FACTS: Respondent Buling filed a complaint with the RTC of testimony, concluding that the latter merely made an honest mistake
Maasin, Southern Leyte, Branch 25, for compulsory recognition and in her understanding of the questions of the petitioner’s counsel. The
support pendente lite, claiming that the petitioner is the father of her CA consequently set aside the RTC decision and ordered the
child Gliffze. petitioner to recognize his minor son Gliffze. It also reinstated the
In his answer, the petitioner denied the imputed paternity of Gliffze. RTC order granting a P 2,000.00 monthly child support.
For the parties’ failure to amicably settle the dispute, the RTC
terminated the pre-trial proceedings. Trial on the merits ensued. ISSUE: Whether the CA committed a reversible error when it set
The respondent testified for herself and presented Rodulfo Lopez as aside the RTC’s findings and ordered the petitioner to recognize
witness. Evidence for the respondent showed that she met the and provide legal support to his minor son Gliffze.
petitioner at the PCIB Southern Leyte branch where she had been
hired as a casual employee, while the petitioner worked as HELD: NO. We have recognized that "filiation proceedings are
accounting supervisor. The petitioner started intimate sexual usually filed not just to adjudicate paternity but also to secure a legal
relations with the respondent. Eventually, respondent found out that right associated with paternity, such as citizenship, support (as in this
she was pregnant. When told of the pregnancy, the petitioner was case) or inheritance. In paternity cases, the burden of proof is on the
happy and made plans to marry the respondent. Subsequently, person who alleges that the putative father is the biological father of
however, the petitioner backed out of the wedding plans. the child."
The respondent responded by filing a complaint with the MTC for One can prove filiation, either legitimate or illegitimate, through the
damages against the petitioner for breach of promise to marry. Later, record of birth appearing in the civil register or a final judgment, an
however, the petitioner and the respondent amicably settled the admission of filiation in a public document or a private handwritten
case. instrument and signed by the parent concerned, or the open and
The respondent gave birth to their son Gliffze. When the petitioner continuous possession of the status of a legitimate or illegitimate
did not show up and failed to provide support to Gliffze, the child, or any other means allowed by the Rules of Court and special
respondent sent him a letter demanding recognition of and support laws. We have held that such other proof of one's filiation may be a
for their child. When the petitioner did not answer the demand, the "baptismal certificate, a judicial admission, a family bible in which his
respondent filed her complaint for compulsory recognition and name has been entered, common reputation respecting his pedigree,
support pendente lite. admission by silence, the testimonies of witnesses, and other kinds
of proof admissible under Rule 130 of the Rules of Court."

53 Aicka Singson
Notes in Remedial Law Review II
Provisional Remedies, Special Civil Actions, Special Proceedings and Evidence
School Year 2015-2016 REMEDIAL LAW REVIEW II - ATTY. BRONDIAL

In Herrera v. Alba, we stressed that there are four significant


procedural aspects of a traditional paternity action that parties have
to face: a prima facie case, affirmative defenses, presumption of
legitimacy, and physical resemblance between the putative father
and the child. We explained that a prima facie case exists if a woman
declares — supported by corroborative proof — that she had sexual
relations with the putative father; at this point, the burden of evidence
shifts to the putative father. We explained further that the two
affirmative defenses available to the putative father are: (1)
incapability of sexual relations with the mother due to either physical
absence or impotency, or (2) that the mother had sexual relations
with other men at the time of conception.
In this case, the respondent established a prima facie case that the
petitioner is the putative father of Gliffze through testimony that she
had been sexually involved only with one man, the petitioner, at the
time of her conception. Rodulfo corroborated her testimony that the
petitioner and the respondent had intimate relationship.
On the other hand, the petitioner did not deny that he had sexual
encounters with the respondent, only that it occurred on a much later
date than the respondent asserted, such that it was physically
impossible for the respondent to have been three (3) months
pregnant already in September 1994 when he was informed of the
pregnancy.
Since filiation is beyond question, support follows as a matter of
obligation; a parent is obliged to support his child, whether legitimate
or illegitimate. Support consists of everything indispensable for
sustenance, dwelling, clothing, medical attendance, education and
transportation, in keeping with the financial capacity of the family.
Thus, the amount of support is variable and, for this reason, no final
judgment on the amount of support is made as the amount shall be
in proportion to the resources or means of the giver and the
necessities of the recipient. It may be reduced or increased
proportionately according to the reduction or increase of the
necessities of the recipient and the resources or means of the
person obliged to support.

54 Aicka Singson
Notes in Remedial Law Review II
Provisional Remedies, Special Civil Actions, Special Proceedings and Evidence
School Year 2015-2016 REMEDIAL LAW REVIEW II - ATTY. BRONDIAL

II. SPECIAL CIVIL ACTIONS (Rule 62-71) Interpleader vs intervention (Rule 19)

What is so special about special civil actions? There are certain Interpleader Intervention
Rules governing these special civil actions.
Special civil action Ordinary action
What certain Rules govern them that you do not find in ordinary
actions? Summons is not required in some special civil actions There is no pending action There is a pending action (it is an
(certiorari, prohibition, mandamus under Rule 65). auxiliary action)

If there is no summons, how does the court acquire jurisdiction over The plaintiff has no interest One of the requisites is the
the person of the respondent in a petition for certiorari, prohibition or intervenor must have an interest
mandamus? The court issues an order for the respondent to over the subject matter
comment. It acquires jurisdiction upon receipt of the respondent of a
No leave of court because Always requires leave of court
copy of the order requiring him to file a comment. it is a principal action

A. Interpleader (Rule 62)


1. What is an action in Interpleader (Section 1)
A special civil remedy whereby a party who has property in his
Section 1. When interpleader proper.
possession or an obligation to perform, either wholly or partially, but
Whenever conflicting claims upon the same subject matter are or
who claims no interest in the subject, or whose interest, in whole or
may be made against a person who claims no interest whatever in
in part, is not disputed by others, goes to court and asks that
the subject matter, or an interest which in whole or in part is not
conflicting claimants to the property or obligation be required to
disputed by the claimants, he may bring an action against the
litigate among themselves in order to determine finally who is entitled
conflicting claimants to compel them to interplead and litigate their
to the same.
several claims among themselves.
Purpose: To protect a person not against double liability but against
double vexation in respect to one’s liability. 2. Requisites (Section 1)
Notes:
i. There must be two or more claimants with adverse or conflicting
interests to a property in the custody or possession of the plaintiff;
ii. The plaintiff in an action for interpleader has no claim upon the
subject matter of the adverse claims or if he has an interest at all,
such interest is not disputed by the claimants;
iii. The subject matter of the adverse claims must be one and the
same; and
iv. The parties impleaded must make effective claims.

55 Aicka Singson
Notes in Remedial Law Review II
Provisional Remedies, Special Civil Actions, Special Proceedings and Evidence
School Year 2015-2016 REMEDIAL LAW REVIEW II - ATTY. BRONDIAL

If I am BDO, I am a debtor to the depositors. There is Account No. 3. Procedure (Section 2 to 7)


10170065-200. I deposit in that bank account. There are 3 claimants
over that deposit. A says give me the deposit of P1,000,000 of X Sec. 2. Order.
Upon the filing of the complaint, the court shall issue an order requiring the
because X is my father and he died a month ago. B says he is the conflicting claimants to interplead with one another. If the interests of justice
legatee of his father, X, and he has a last will and testament to prove so require, the court may direct in such order that the subject matter be paid
it. C says he has been living with X for the past 7 years and X gave it or delivered to the court.
to him. BDO has no interest over that. The bank cannot decide to Sec. 3. Summons.
Summons shall be served upon the conflicting claimants, together with a
whom it should give the deposit. The bank must file an action for copy of the complaint and order.
interpleader. Sec. 4. Motion to dismiss.
Within the time for filing an answer, each claimant may file a motion to
The subject matter of an interpleader will determine the jurisdiction of dismiss on the ground of impropriety of the interpleader action or on other
appropriate grounds specified in Rule 16. The period to file the answer shall
the court. The subject matter of an interpleader may either be real
be tolled and if the motion is denied, the movant may file his answer within
property, personal property or the performance of an obligation. As the remaining period, but which shall not be less than five (5) days in any
to real and personal property, the value of the subject matter event, reckoned from notice of denial.
determines jurisdiction. Sec. 5. Answer and other pleadings.
Each claimant shall file his answer setting forth his claim within fifteen (15)
days from service of the summons upon him, serving a copy thereof upon
A found a wallet containing 3 100-dollar bills. X, Y and Z are claiming each of the other conflicting claimants who may file their reply thereto as
ownership over the wallet. X says the serial number of the bills. Y provided by these Rules. If any claimant fails to plead within the time herein
says the wallet has a picture of his girlfriend. Z says his driver’s fixed, the court may, on motion, declare him in default and thereafter render
license is in the wallet. A should file an interpleader with MTC ($300 judgment barring him from any claim in respect to the subject matter.
The parties in an interpleader action may file counterclaims, cross-claims,
bills plus $100 wallet = $400). third-party complaints and responsive pleadings thereto, as provided by
these Rules.
If it is a parcel of land worth more than P300,000, RTC would have Sec. 6. Determination.
jurisdiction. After the pleadings of the conflicting claimants have been filed, and pre-trial
has been conducted in accordance with the Rules, the court shall proceed to
Example where the subject matter is the performance of an determine their respective rights and adjudicate their several claims.
Sec. 7. Docket and other lawful fees, costs and litigation expenses as liens.
obligation: A musical artist is contracted to perform a concert. A is The docket and other lawful fees paid by the party who filed a complaint
claiming the band is supposed to perform at Araneta Coliseum while under this Rule, as well as the costs and litigation expenses, shall constitute
B is claiming that the band is supposed to perform at MOA Arena. a lien or charge upon the subject matter of the action, unless the court shall
order otherwise.
Who interpleads? The band. Which court has jurisdiction over that?
Notes:
RTC because it is incapable of pecuniary estimation.

Who has no interest in the subject matter? In the wallet example, A Effect if there are no conflicting claims
has no interest. In intervention, he must have interest. The complaint for interpleader may be dismissed for lack of Cause of
Action

56 Aicka Singson
Notes in Remedial Law Review II
Provisional Remedies, Special Civil Actions, Special Proceedings and Evidence
School Year 2015-2016 REMEDIAL LAW REVIEW II - ATTY. BRONDIAL

 The court only determines who has a better right among the certificate 201, or to issue another membership fee certificate to the
conflicting claimants defendant Lee, without violating its articles of incorporation and by-
laws; and that the membership fee certificate 201-serial no. 1199
CASES held by the defendant Tan and the membership fee certificate 201-
a. Wack wack vs. Won (70 SCRA serial No. 1478 issued to the defendant Lee proceed from the same
membership fee certificate 201, originally issued in the name of
The Corporation prayed that (a) an order be issued requiring Lee "Swan, Culbertson and Fritz".
and Tan to interplead and litigate their conflicting claims; and (b)
judgment. be rendered, after hearing, declaring who of the two is the For its second cause of action. it alleged that the membership fee
lawful owner of membership fee certificate 201, and ordering the certificate 201-serial no. 1478 issued by the deputy clerk of court of
surrender and cancellation of membership fee certificate 201-serial court is null and void because issued in violation of its by-laws, which
no. 1478 issued in the name of Lee. require the surrender and cancellation of the outstanding
membership fee certificate 201 before issuance may be made to the
Petitioner Wack Wack Golf & Country Club, Inc., filed a case for transferee of a new certificate duly signed by its president and
interpleader and allege, for its first cause of action, that the secretary, aside from the fact that the decision of the CFI of Manila in
defendant Lee claims ownership of its membership fee certificate civil case 26044 is not binding upon the defendant Tan, holder of
201, by virtue of the decision rendered in civil case 26044 of the CFI membership fee certificate 201-serial no. 1199; that Tan is made a
of Manila and also by virtue of membership fee certificate 201-serial party because of his refusal to join it in this action or bring a separate
no. 1478 issued by Ponciano B. Jacinto, deputy clerk of court of the action to protect his rights despite the fact that he has a legal and
said CFI of Manila, for and in behalf of the president and the beneficial interest in the subject matter of this litigation; and that he is
secretary of the Corporation and of the People's Bank & Trust made a part so that complete relief may be accorded herein.
Company as transfer agent of the said Corporation, pursuant to the
order; that the defendant Tan, on the other hand, claims to be lawful Defendants, in separate motions, moved to dismiss the complaint
owner of its aforesaid membership fee certificate 201 by virtue of upon the grounds of res judicata, failure of the complaint to state a
membership fee certificate 201-serial no. 1199 issued to him cause of action, and bar by prescription. These motions were duly
pursuant to an assignment made in his favor by "Swan, Culbertson opposed by the Corporation. The trial court dismissed the complaint,
and Fritz," the original owner and holder of membership fee with costs against the Corporation.
certificate 201; that under its articles of incorporation and by-laws the
In this appeal, Corporation contends that the court a quo erred (1) in
Corporation is authorized to issue a maximum of 400 membership
finding that its complaint do not constitute a valid ground for an
fee certificates to persons duly elected or admitted to proprietary
action of interpleader, and in holding that "the principal motive for the
membership, all of which have been issued as early as December
present action is to reopen the Manila Case and collaterally attack
1939; that it claims no interest whatsoever in the said membership
the decision of the said Court"; (2) in finding that the decision in civil
fee certificate 201; that it has no means of determining who of the
case 26044 of the CFI of Manila constitutes res judicata and bars its
two defendants is the lawful owner thereof; that it is without power to
present action; and (3) in dismissing its action instead of compelling
issue two separate certificates for the same membership fee

57 Aicka Singson
Notes in Remedial Law Review II
Provisional Remedies, Special Civil Actions, Special Proceedings and Evidence
School Year 2015-2016 REMEDIAL LAW REVIEW II - ATTY. BRONDIAL

the appellees to interplead and litigate between themselves their who also claimed the same membership fee certificate. Yet it did not
respective claims. interplead Tan. It preferred to proceed with the litigation (civil case
26044) and to defend itself therein. As a matter of fact, final
On the other hand, the appellees argue that the trial court properly judgment was rendered against it and said judgment has already
dismissed the complaint, because, having the effect of reopening been executed. It is not therefore too late for it to invoke the remedy
civil case 26044, the present action is barred by res judicata. of interpleader.

WON there was a timely and proper filing of Interpleader It has been held that a stakeholder's action of interpleader is too late
when filed after judgment has been rendered against him in favor of
NO. The action of interpleader, under section 120 of the Code of
one of the contending claimants, especially where he had notice of
Civil Procedure, is a remedy whereby a person who has personal
the conflicting claims prior to the rendition of the judgment and
property in his possession, or an obligation to render wholly or
neglected the opportunity to implead the adverse claimants in the
partially, without claiming any right to either, comes to court and asks
suit where judgment was entered. This must be so, because once
that the persons who claim the said personal property or who
judgment is obtained against him by one claimant he becomes liable
consider themselves entitled to demand compliance with the
to the latter.
obligation, be required to litigate among themselves in order to
determine finally who is entitled to tone or the one thing. The remedy The Corporation has not shown any justifiable reason why it did not
is afforded to protect a person not against double liability but against file an application for interpleader in civil case 26044 to compel the
double vexation in respect of one liability. appellees herein to litigate between themselves their conflicting
claims of ownership. It was only after adverse final judgment was
There is no question that the subject matter of the present
rendered against it that the remedy of interpleader was invoked by it.
controversy, i.e., the membership fee certificate 201, is proper for an
By then it was too late, because to he entitled to this remedy the
interpleader suit. What is here disputed is the propriety and
applicant must be able to show that lie has not been made
timeliness of the remedy in the light of the facts and circumstances
independently liable to any of the claimants. And since the
obtaining.
Corporation is already liable to Lee under a final judgment, the
A stakeholde should file an action of interpleader within a reasonable present interpleader suit is clearly improper and unavailing.
time after a dispute has arisen without waiting to be sued by either of
It is the general rule that before a person will be deemed to be in a
the contending claimants. Otherwise, he may be barred by laches or
position to ask for an order of intrepleader, he must be prepared to
undue delay. But where he acts with reasonable diligence in view of
show, among other prerequisites, that he has not become
the environmental circumstances, the remedy is not barred.
independently liable to any of the claimants.
The Corporation in the case aware of the conflicting claims of the
It is also the general rule that a bill of interpleader comes too late
appellees with respect to the membership fee certificate 201 long
when it is filed after judgment has been rendered in favor of one of
before it filed the present interpleader suit. It had been recognizing
the claimants of the fund, this being especially true when the holder
Tan as the lawful owner thereof. The Corporation was sued by Lee
of the funds had notice of the conflicting claims prior to the rendition

58 Aicka Singson
Notes in Remedial Law Review II
Provisional Remedies, Special Civil Actions, Special Proceedings and Evidence
School Year 2015-2016 REMEDIAL LAW REVIEW II - ATTY. BRONDIAL

of the judgment and had an opportunity to implead the adverse b. Eternal Gardens vs IAC (165 SCRA)
claimants in the suit in which the judgment was rendered.
FACTS: Petitioner Eternal Gardens and private respondent
Indeed, if a stakeholder defends a suit filed by one of the adverse MISSION executed a Land Development Agreement whereby the
claimants and allows said suit to proceed to final judgment against former undertook to introduce and construct at its own expense and
him, he cannot later on have that part of the litigation repeated in an responsibility necessary improvements on the property owned by
interpleader suit. In the case at hand, the Corporation allowed civil private respondent into a memorial park to be subdivided into and
case 26044 to proceed to final judgment. And it offered no sold as memorial plot lots, at a stipulated area and price per lot. Out
satisfactory explanation for its failure to implead Tan in the same of the proceeds from the sale, private respondent is entitled to
litigation. In this factual situation, it is clear that this interpleader suit receive 40% of the net gross collection from the project to be
cannot prosper because it was filed much too late. remitted monthly by petitioner to private respondent through a
designated depositary trustee bank. All went well until Maysilo Estate
To now permit the Corporation to bring Lee to court after the latter's asserted its claim of ownership over the parcel of land in question.
successful establishment of his rights in civil case 26044 to the Confronted with such conflicting claims, petitioner as plaintiff filed a
membership fee certificate 201, is to increase instead of to diminish complaint for interpleader against private respondent MISSION and
the number of suits, which is one of the purposes of an action of Maysilo Estate.
interpleader, with the possibility that the latter would lose the benefits
of the favorable judgment. This cannot be done because having An order was issued by the presiding judge requiring defendants to
elected to take its chances of success in said civil case 26044, with interplead. MISSION filed a motion to dismiss for lack of cause of
full knowledge of all the fact, the Corporation must submit to the action but also presented an answer. The MTD was denied.
consequences of defeat.
Earlier on November 21, 1982, private respondent presented a
It is well settled, both by reasons and authority, that one who asks motion for the placing on judicial deposit the amounts due and
the interposition of a court of equity to compel others, claiming unpaid from petitioner. Acting on such motion, the trial court denied
property in his hands, to interplead, must do so before putting them judicial deposit.
to the test of trials at law. The remedy by interpleader is afforded to
protect the party from the annoyance and hazard of two or more MISSION, herein private respondent filed a petition for certiorari with
actions touching the same property or demand; but one who, with the then IAC praying that the Orders of the RTC be set aside and
knowledge of all the facts, neglects to avail himself of the relief, or that an order be issued to deposit in court or in a depositor trustee
elects to take the chances for success in the actions at law, ought to bank of any and all payments, plus interest thereon, due the private
submit to the consequences of defeat. To permit an unsuccessful respondent MISSION under the Land Development Agreement, said
defendant to compel the successful plaintiffs to interplead, is to amounts deposited to be paid to whomever may be found later to be
increase instead of to diminish the number of suits; to put upon the entitled thereto, with costs.
shoulders of others the burden which he asks may be taken from his
The IAC dismissed the petition. Eternal Gardens moved for a
own. ....'
reconsideration – DENIED.

59 Aicka Singson
Notes in Remedial Law Review II
Provisional Remedies, Special Civil Actions, Special Proceedings and Evidence
School Year 2015-2016 REMEDIAL LAW REVIEW II - ATTY. BRONDIAL

ISSUE: WON CA abused its discretion amounting to lack of respondent was constrained to refer the matter to its lawyer who, in
jurisdiction in dismissing the petition turn, made a final demand on petitioners for the payment of the
accrued because petitioners still refused to comply, a complaint for
HELD: Under the circumstances, there appears to be no plausible ejectment was filed by private respondent through its representative,
reason for petitioner's objections to the deposit of the amounts in Ms. Bautista.
litigation after having asked for the assistance of the lower court by
filing a complaint for interpleader where the deposit of aforesaid Petitioners admitted their failure to pay the stipulated rent for the
amounts is not only required by the nature of the action but is a leased premises but claimed that such refusal was justified because
contractual obligation of the petitioner under the Land Development of the internal squabble in respondent company as to the person
Program. authorized to receive payment. MeTC rendered a Decision
dismissing the complaint for ejectment but the RTC reversed said
As correctly observed by the CA, the essence of an interpleader, decision. Aggrieved, petitioners elevated the matter to the Court of
aside from the disavowal of interest in the property in litigation on the Appeals in a petition for review on certiorari. CA affirmed the RTC.
part of the petitioner, is the deposit of the property or funds in
controversy with the court. it is a rule founded on justice and equity: WON the petitioner’s refusal to pay was justified
"that the plaintiff may not continue to benefit from the property or
funds in litigation during the pendency of the suit at the expense of NO. What was, instead, clearly established by the evidence was
whoever will ultimately be decided as entitled thereto." petitioners’ non-payment of rentals because ostensibly they did not
know to whom payment should be made. However, this did not
The case at bar was elevated to the CA on certiorari with prohibitory justify their failure to pay, because if such were the case, they were
and mandatory injunction. Said appellate court found that more than not without any remedy. They should have availed of the provisions
Php20M are involved; so that on interest alone for savings or time of the Civil Code of the Philippines on the consignation of payment
deposit would be considerable, now accruing in favor of the Eternal and of the Rules of Court on interpleader.
Gardens. Finding that such is violative of the very essence of the
complaint for interpleader as it clearly runs against the interest of Moreover, Section 1, Rule 62 of the Rules of Court provides
justice in this case, the CA cannot be faulted for finding that the
Section 1. When interpleader proper. – Whenever conflicting
lower court committed a grave abuse of discretion which requires
claims upon the same subject matter are or may be made against a
correction by the requirement that a deposit of said amounts should
person who claims no interest whatever in the subject matter, or an
be made to a bank approved by the Court.
interest which in whole or in part is not disputed by the claimants, he
c. Pasricha vs. Don Luis Dizon Realty (548 SCRA) may bring an action against the conflicting claimants to compel them
to interplead and litigate their several claims among themselves.
Respondent Don Luis Dison Realty, Inc. and petitioners entered into
a Contract of Lease. Petitioners religiously paid the monthly rentals Otherwise stated, an action for interpleader is proper when the
until May 1992. After that, despite repeated demands, petitioners lessee does not know to whom payment of rentals should be made
continuously refused to pay the stipulated rent. Consequently, due to conflicting claims on the property (or on the right to

60 Aicka Singson
Notes in Remedial Law Review II
Provisional Remedies, Special Civil Actions, Special Proceedings and Evidence
School Year 2015-2016 REMEDIAL LAW REVIEW II - ATTY. BRONDIAL

collect).[60] The remedy is afforded not to protect a person against On April 20, 1994, the BOC sold the remaining four (4) CB bills to
double liability but to protect him against double vexation in respect Capital One Equities Corporation which transferred them to All-Asia
of one liability.[61] Capital and Trust Corporation (All Asia). On September 30, 1994, All
Asia further transferred the four CB bills back to the RCBC.
Notably, instead of availing of the above remedies, petitioners opted
to refrain from making payments. On November 16, 1994, the RCBC sold back to All Asia one of these
4 CB bills. When the BSP refused to release the amount of this CB
d. Bank of Commerce vs. Planters Development Bank (681 bill on maturity, the BOC purchased from All Asia this lone CB bill.
SCRA)
As the registered owner of the remaining three CB bills, the RCBC
FACTS: I. First set of CB bills Central bank bills sold them to IVI Capital and Insular Savings Bank. Again, when the
BSP refused to release the amount of this CB bill on maturity, the
The RCBC was the registered owner of seven CB bills with a total RCBC paid back its transferees, reacquired these three CB bills and
face value of P 70 million, issued on January 2, 1994 and would sold them to the BOC – ultimately, the BOC acquired these three CB
mature on January 2, 1995.2 As evidenced by a "Detached bills.
Assignment", the RCBC sold these CB bills to the BOC. As
evidenced by another "Detached Assignment" of even date, the All in all, the BOC acquired the first set of seven CB bills.
BOC, in turn, sold these CB bills to the PDB. The BOC delivered the
Detached Assignments to the PDB. II. Second set of CB bills

On April 15, 1994 (April 15 transaction), the PDB, in turn, sold to the On April 19, 1994, the RCBC, as registered owner, (i) sold two CB
BOC Treasury Bills worth P 70 million, with maturity date of June 29, bills with a total face value of P 20 million to the PDB and (ii)
1994, as evidenced by a Trading Order and a Confirmation of Sale. delivered to the PDB the corresponding Detached Assignment.
However, instead of delivering the Treasury Bills, the PDB delivered
On even date, the PDB delivered to Bancap the two CB bills (April 19
the seven CB bills to the BOC, as evidenced by a PDB Security
transaction). In turn, Bancap sold the CB bills to Al-Amanah Islamic
Delivery Receipt, bearing a "note: ** substitution in lieu of 06-29-94"
Investment Bank of the Philippines, which in turn sold it to the BOC.
– referring to the Treasury Bills. Nevertheless, the PDB retained
possession of the Detached Assignments. It is basically the nature of PDB’s move against the transfer of the first and second sets of CB
this April 15 transaction that the PDB and the BOC cannot agree on. bills
The transfer of the first set of seven CB bills On June 30, 1994, upon learning of the transfers involving the CB
bills, the PDB informed20 the Officer-in-Charge of the BSP’s
On April 20, 1994, according to the BOC, it "sold back" to the PDB
Government Securities Department, Lagrimas Nuqui, of the PDB’s
three of the seven CB bills. In turn, the PDB transferred these three
claim over these CB bills, based on the Detached Assignments in its
CB bills to Bancapital Development Corporation (Bancap). On April
possession. The PDB requested the BSP to record its claim in the
25, 1994, the BOC bought the three CB bills from Bancap – so,
BSP’s books, explaining that its non-possession of the CB bills is "on
ultimately, the BOC reacquired these three CB bills.

61 Aicka Singson
Notes in Remedial Law Review II
Provisional Remedies, Special Civil Actions, Special Proceedings and Evidence
School Year 2015-2016 REMEDIAL LAW REVIEW II - ATTY. BRONDIAL

account of imperfect negotiations thereof and/or subsequent setoff or After the petitions were filed, the BOC acquired/reacquired all the
transfer." nine CB bills – the first and second sets of CB bills (collectively,
subject CB bills).
Nuqui denied the request, invoking Section 8 of CB Circular No. 28
which requires the presentation of the bond before a registered bond Defenses of the BSP and of the BOC
may be transferred on the books of the BSP.
The BOC filed its Answer, praying for the dismissal of the petition. It
The PDB clarified to Nuqui that it was not "asking for the transfer of argued that the PDB has no cause of action against it since the PDB
the CB Bill, rather it intends to put the BSP on formal notice that is no longer the owner of the CB bills.
whoever is in possession of said bills is not a holder in due course,"
and, therefore the BSP should not make payment upon the On the other hand, the BSP countered that the PDB cannot invoke
presentation of the CB bills on maturity. Nuqui responded that the Section 10 (d) 4 of CB Circular No. 28 because this section applies
BSP was "not in a position at that point in time to determine who is only to an "owner" and a "person presenting the bond," of which the
and who is not the holder in due course since it is not privy to all acts PDB is neither. The PDB has not presented to the BSP any
and time involving the transfers or negotiation" of the CB bills. assignment of the subject CB bills, duly recorded in the BSP’s books,
in its favor to clothe it with the status of an "owner."
The PDB also asked BSP Deputy Governor Edgardo Zialcita that (i)
a notation in the BSP’s books be made against the transfer,
exchange, or payment of the bonds and the payment of interest
Alternatively, the BSP asked that an interpleader suit be allowed
thereon; and (ii) the presenter of the bonds upon maturity be
between and among the claimants to the subject CB bills on the
required to submit proof as a holder in due course (of the first set of
position that while it is able and willing to pay the subject CB bills’
CB bills).
face value, it is duty bound to ensure that payment is made to the
Nuqui again denied the request, reiterating the BSP’s previous rightful owner.
stand.
The RTC granted the BSP’s motion to interplead and, accordingly,
In light of these BSP responses and the impending maturity of the required the BOC to amend its Answer and for the conflicting
CB bills, the PDB filed with the RTC two separate petitions for claimants to comment thereon. The BOC filed its Amended
Mandamus, Prohibition and Injunction with prayer for Preliminary Consolidated Answer with Compulsory Counterclaim, reiterating its
Injunction and TRO against Nuqui, the BSP and the RCBC. earlier arguments asserting ownership over the subject CB bills.

The RTC temporarily enjoined Nuqui and the BSP from paying the In the alternative, the BOC added that even assuming that there was
face value of the CB bills on maturity. The PDB filed an Amended no effective transfer of the nine CB bills ultimately to the BOC, the
Petition, additionally impleading the BOC and All Asia. The cases PDB remains obligated to deliver to the BOC, as buyer in the April 15
were consolidated. The RTC granted the PDB’s application for a writ transaction and ultimate successor-in-interest of the buyer (Bancap)
of preliminary prohibitory injunction. in the April 19 transaction, either the original subjects of the sales or

62 Aicka Singson
Notes in Remedial Law Review II
Provisional Remedies, Special Civil Actions, Special Proceedings and Evidence
School Year 2015-2016 REMEDIAL LAW REVIEW II - ATTY. BRONDIAL

the value thereof, plus whatever income that may have been earned Without emasculating its jurisdiction, the RTC could have properly
during the pendency of the case. dismissed the PDB’s petition but on the ground that mandamus does
not lie against the BSP; but even this correct alternative is no longer
The PDB filed an Omnibus Motion, questioning the RTC’s jurisdiction plausible since the BSP, as a respondent below, already properly
over the BOC’s "additional counterclaims." The PDB argues that its brought before the RTC the remaining conflicting claims over the
petitions pray for the BSP (not the RTC) to determine who among subject CB bills by way of a counterclaim/crossclaim for interpleader.
the conflicting claimants to the CB bills stands in the position of the Section 1, Rule 62 of the Rules of Court provides when an
bona fide holder for value. The RTC cannot entertain the BOC’s interpleader is proper:
counterclaim, regardless of its nature, because it is the BSP which
has jurisdiction to determine who is entitled to receive the proceeds SECTION 1. When interpleader proper. – Whenever conflicting
of the CB bills. claims upon the same subject matter are or may be made against a
person who claims no interest whatever in the subject matter, or an
The BOC opposed the PDB’s Omnibus Motion. The PDB filed its interest which in whole or in part is not disputed by the claimants, he
Reply. may bring an action against the conflicting claimants to compel them
to interplead and litigate their several claims among themselves.
The RTC dismissed the PDB’s petition, the BOC’s counterclaim and
the BSP’s counter-complaint/cross-claim for interpleader, holding The remedy of an action of interpleader is designed to protect a
that under CB Circular No. 28, it has no jurisdiction (i) over the person against double vexation in respect of a single liability. It
BOC’s "counterclaims" and (ii) to resolve the issue of ownership of requires, as an indispensable requisite, that conflicting claims upon
the CB bills. With the denial of their separate motions for the same subject matter are or may be made against the stakeholder
Reconsideration, the BOC and the BSP separately filed the present (the possessor of the subject matter) who claims no interest
petitions for review on certiorari. whatever in the subject matter or an interest which in whole or in part
is not disputed by the claimants.
ISSUE: WON an interpleader suit may be initiated through
petitioner’s answer Through this remedy, the stakeholder can join all competing
claimants in a single proceeding to determine conflicting claims
HELD: YES. Of the three possible options available to the RTC, the
without exposing the stakeholder to the possibility of having to pay
adoption of either of these two would lead the trial court into serious
more than once on a single liability.
legal error: first, if it granted the PDB’s petition, its decision would
have to be set aside on appeal because the BSP has no jurisdiction When the court orders that the claimants litigate among themselves,
as previously discussed; and second when it dismissed the PDB’s in reality a new action arises, where the claims of the interpleaders
petitions and the BOC’s counterclaims on the ground that it lacks themselves are brought to the fore, the stakeholder as plaintiff is
jurisdiction, the trial court seriously erred because precisely, the relegated merely to the role of initiating the suit. In short, the remedy
resolution of the conflicting claims over the CB bills falls within its of interpleader, when proper, merely provides an avenue for the
general jurisdiction. conflicting claims on the same subject matter to be threshed out in
an action.

63 Aicka Singson
Notes in Remedial Law Review II
Provisional Remedies, Special Civil Actions, Special Proceedings and Evidence
School Year 2015-2016 REMEDIAL LAW REVIEW II - ATTY. BRONDIAL

This is precisely what the RTC did by granting the BSP’s motion to considered that insofar as the BSP is concerned, the PDB does not
interplead. The PDB itself "agreed that the various claimants should possess any right to have its claim recorded in the BSP’s books;
now interplead." Thus, the PDB and the BOC subsequently entered consequently, the PDB cannot properly be considered even as a
into two separate escrow agreements, covering the CB bills, and potential claimant to the proceeds of the CB bills upon maturity.
submitted them to the RTC for approval. Thus, the interpleader was only an alternative position, made only in
the BSP’s Answer.
In granting the BSP’s motion, the RTC acted on the correct premise
that it has jurisdiction to resolve the parties’ conflicting claims over The remedy of interpleader, as a special civil action, is primarily
the CB bills - consistent with the rules and the parties’ conduct - and governed by the specific provisions in Rule 62 of the Rules of Court
accordingly required the BOC to amend its answer and for the PDB and secondarily by the provisions applicable to ordinary civil
to comment thereon. Suddenly, however, the PDB made an about- actions.136 Indeed, Rule 62 does not expressly authorize the filing of
face and questioned the jurisdiction of the RTC. Swayed by the a complaint-in-interpleader as part of, although separate and
PDB’s argument, the RTC dismissed even the PDB’s petition - which independent from, the answer. Similarly, Section 5, Rule 6, in
means that it did not actually compel the BSP to resolve the BOC’s relation to Section 1, Rule 9 of the Rules of Court does not include a
and the PDB’s claims. complaint-in-interpleader as a claim, a form of defense, or as an
objection that a defendant may be allowed to put up in his answer or
Without the motion to interplead and the order granting it, the RTC in a motion to dismiss. This does not mean, however, that the BSP’s
could only dismiss the PDB’s petition since it is the RTC which has "counter-complaint/cross-claim for interpleader" runs counter to
jurisdiction to resolve the parties’ conflicting claims – not the BSP. general procedures.
Given that the motion to interplead has been actually filed, the RTC
could not have really granted the relief originally sought in the PDB’s Apart from a pleading, the rules allow a party to seek an affirmative
petition since the RTC’s order granting the BSP’s motion to relief from the court through the procedural device of a motion. While
interplead - to which the PDB in fact acquiesced into - effectively captioned "Answer with counter complaint/cross-claim for
resulted in the dismissal of the PDB’s petition. This is not altered by interpleader," the RTC understood this as in the nature of a motion,
the fact that the PDB additionally prayed in its petition for damages, seeking relief which essentially consists in an order for the conflicting
attorney’s fees and costs of suit "against the public respondents" claimants to litigate with each other so that "payment is made to the
because the grant of the order to interplead effectively sustained the rightful or legitimate owner" of the subject CB bills.
propriety of the BSP’s resort to this procedural device.
The rules define a "civil action" as "one by which a party sues
Interpleader another for the enforcement or protection of a right, or the prevention
or redress of a wrong." Interpleader may be considered as a
1. as a special civil action stakeholder’s remedy to prevent a wrong, that is, from making
payment to one not entitled to it, thereby rendering itself vulnerable
What is quite unique in this case is that the BSP did not initiate the
to lawsuit/s from those legally entitled to payment.
interpleader suit through an original complaint but through its
Answer. This circumstance becomes understandable if it is

64 Aicka Singson
Notes in Remedial Law Review II
Provisional Remedies, Special Civil Actions, Special Proceedings and Evidence
School Year 2015-2016 REMEDIAL LAW REVIEW II - ATTY. BRONDIAL

Interpleader is a civil action made special by the existence of validity arising, and for a declaration of his rights or duties,
particular rules to govern the uniqueness of its application and thereunder.
operation. Under Section 2, Rule 6 of the Rules of Court, governing
ordinary civil actions, a party’s claim is asserted "in a complaint, An action for the reformation of an instrument, to quiet title to real
counterclaim, cross-claim, third (fourth, etc.)-party complaint, or property or remove clouds therefrom, or to consolidate ownership
complaint-in-intervention." In an interpleader suit, however, a claim is under Article 1607 of the Civil Code, may be brought under this Rule.
not required to be contained in any of these pleadings but in the
answer-(of the conflicting claimants)-in-interpleader. This claim is Notes:
different from the counter-claim (or cross-claim, third party-
An action for declaratory relief is brought to secure an authoritative
complaint) which is separately allowed under Section 5, par. 2 of
statement of the rights and obligations of the parties under a contract
Rule 62.
or a statute for their guidance in the enforcement or compliance with
the same. Thus, the purpose is to seek for a judicial interpretation of
B. Declaratory Relief and Similar Remedies (Rule 63) an instrument or for a judicial declaration of a person’s rights under a
statute and not to ask for affirmative reliefs like injunction, damages
A special civil action brought by a person interested under a deed,
or any other relief beyond the purpose of the petition as declared
will, contract or other written instrument, or whose rights are affected
under the Rules.
by a statute, executive order, or regulation, ordinance or any other
governmental regulation, before breach or violation thereof, asking The subject matter in a petition for declaratory relief is any of the
the court to determine any question of a construction or validity following:
arising, and for a declaration of his rights or duties thereunder.
a) Deed;
Purpose: To relieve the litigants of the common law rule that no
declaration of rights may be judicially adjudged unless a right has b) Will;
been violated and for the violation of which relief may be granted.
c) Contract or other written instrument;
It is commenced by a petition.
d) Statute;
1. Nature; Kinds (Section 1)
e) Executive order or regulation;
Section 1. Who may file petition.
f) Ordinance; or
Any person interested under a deed, will, contract or other written
instrument, whose rights are affected by a statute, executive order or g) Any other governmental regulation.
regulation, ordinance, or any other governmental regulation may,
The petition for declaratory relief is filed before the occurrence of any
before breach or violation thereof, bring an action in the appropriate
breach or violation of the deed, contract, statute, ordinance or
Regional Trial Court to determine any question of construction or
executive order or regulation. It will not prosper when brought after a

65 Aicka Singson
Notes in Remedial Law Review II
Provisional Remedies, Special Civil Actions, Special Proceedings and Evidence
School Year 2015-2016 REMEDIAL LAW REVIEW II - ATTY. BRONDIAL

contract or a statute has already been breached or violated. If there cause of action? This is the only kind of action that has no cause of
has already been a breach, the appropriate ordinary civil action and action.
not declaratory relief should be filed.
When one files an action for declaratory relief, what does he pray
4 requirements for an action of declaratory relief proper to be for? Declaration of his rights.
filed:
People vs Orbecido: A and B went to the U.S. to get a divorce, which
(1) there must be justiciable controversy was granted. A went to the Philippines and filed a declaratory relief
asking what his rights are under this status. B is already married to
(2) the controversy must be between parties of adverse interest someone else, but here in the Philippines, A is still married to her.
What are my rights? Will the divorce judgment affect me? My wife is
(3) it must be ripe for judicial determination
divorced from me but I am not divorced from her. I cannot remarry
There must be an actual controversy, threat or expectancy. There is here but she is already married. SC: He was asking for the
no breach so there is no cause of action. In fact, even in the course declaration of his rights under the Philippine law in that particular
of the proceeding (meaning you have already filed a declaratory status. E.O. No. 227 was applied here. Although we do not recognize
relief), if a breach occurs, you convert it into an ordinary action. divorce, it would be an incongruous situation where your wife is not
married to you but you are still married to her. SC said your rights
The City of Manila passes an ordinance: no smoking throughout include your right to remarry.
Manila. This will take effect December 25, 2014. Can you file a
declaratory relief? Yes, to determine what are your rights. There can To these 4 requirements were added 2 more under the case of
be no breach here because the law is not effective yet. But after Almeda vs Bathala Marketing Ind.
Christmas, you can no longer file an action for declaratory relief. The
(5) there is no other plain, speedy, and adequate remedy in the
presumption is that once the law takes effect, there are already
ordinary course of law
violators.
(6) must be the last resort or the last recourse
What should you do now? Convert it into an ordinary action. File an
action questioning the constitutionality or legality of the ordinance.

Is a will a contract? No. If it were, then it would not have been added 2. PARTIES (Section 2)
anymore because contract is already included in the enumeration.
Sec. 2. Parties.
(4) it must be before breach
All persons who have or claim any interest which would be affected
What is the cause of action in declaratory relief? Cause of action is by the declaration shall be made parties; and no declaration shall,
the act or omission violating the right of another. If the requirement except as otherwise provided in these Rules, prejudice the rights of
for declaratory relief is there must be no breach, how can there be a persons not parties to the action.

66 Aicka Singson
Notes in Remedial Law Review II
Provisional Remedies, Special Civil Actions, Special Proceedings and Evidence
School Year 2015-2016 REMEDIAL LAW REVIEW II - ATTY. BRONDIAL

Notes: • A refusal of the court to declare rights or construe an


instrument is actually the functional equivalent of the
WHO MAY FILE dismissal of the petition.
1) Any person interested under a deed, will, contract or other written On the other hand, the court does not have the discretion to refuse to
instrument or whose rights are affected by a statute, executive order act with respect to actions described as similar remedies. Thus, in an
or regulation, ordinance or other governmental regulation may before action for reformation of an instrument, to quiet or to consolidate
breach or violation thereof, bring an action in the RTC to determine ownership, the court cannot refuse to render a judgment.
any question of construction or validity arising and for a declaration
of his rights or duties, thereunder. 3. Conversion to Ordinary Action (Section 6)
2) Those who may sue under the contract should be those with
Sec. 6. Conversion into ordinary action.
interest under the contract like the parties, the assignees and the
If before the final termination of the case, a breach or violation of an
heirs as required by substantive law.
instrument or a statute, executive order or regulation, ordinance, or
3) If it be a statute, executive order, regulation or ordinance, the any other governmental regulation should take place, the action may
petitioner is one whose rights are affected by the same. The other thereupon be converted into an ordinary action, and the parties shall
parties are all persons who have or claim any interest which would be allowed to file such pleadings as may be necessary or proper.
be affected by the declaration. The rights of person not made parties Notes:
to the action do not stand to be prejudiced by the declaration.
It may be converted into an ordinary action if the final termination of
Grounds for the court to refuse to exercise declaratory relief; a case, breach or violation of an instrument, or a statute, executive
order, or regulation ordinance or other governmental regulation
a) A decision would not terminate the uncertainty or controversy should take place.
which gave rise to the action; or
Ordinary civil action – plaintiff alleges that his right has been violated
b) The declaration or construction is not necessary and proper under
by the defendant; judgment rendered is coercive in character; a writ
the circumstances as when the instrument or the statute has already
of execution may be executed against the defeated party.
been breached.

In declaratory relief, the court is given the discretion to act or not to Special civil action of declaratory relief – an impending violation is
act on the petition. It may therefore choose not to construe the sufficient to file a declaratory relief; no execution may be issued; the
instrument sought to be construed or could refrain from declaring the court merely makes a declaration.
rights of the petitioner under the deed or the law.
PROCEEDINGS CONSIDERED AS SIMILAR REMEDIES

Similar remedies are:


(a) Action for reformation of an instrument;

67 Aicka Singson
Notes in Remedial Law Review II
Provisional Remedies, Special Civil Actions, Special Proceedings and Evidence
School Year 2015-2016 REMEDIAL LAW REVIEW II - ATTY. BRONDIAL

(b) Action for quieting of title; and 1) Simple donation inter vivos wherein no condition is imposed;
(c) Action to consolidate ownership (Art. 1607, Civil Code).
2) Wills; or
A. REFORMATION OF AN INSTRUMENT
3) When the agreement is void (Art. 1666, CC).
It is not an action brought to reform a contract but to reform the
instrument evidencing the contract. It presupposes that there is B. CONSOLIDATION OF OWNERSHIP
nothing wrong with the contract itself because there is a meeting of
minds between the parties. The concept of consolidation of ownership under Art. 1607, Civil
Code, has its origin in the substantive provisions of the law on sales.
The contract is to be reformed because despite the meeting of minds Under the law, a contract of sale may be extinguished either by legal
of the parties as to the object and cause of the contract, the redemption (Art. 1619) or conventional redemption (Art. 1601).
instrument which is supposed to embody the agreement of the
parties does not reflect their true agreement by reason of mistake, Legal redemption (retracto legal) is a statutory mandated redemption
inequitable conduct or accident. The action is brought so the true of a property previously sold. For instance, a co-owner of a property
intention of the parties may be expressed in the instrument (Art. may exercise the right of redemption in case the shares of all the
1359, CC). other co-owners or any of them are sold to a third person (Art. 1620).
The owners of adjoining lands shall have the right of redemption
The instrument may be reformed if it does not express the true when a piece of rural land with a size of one hectare or less is
intention of the parties because of lack of skill of the person drafting alienated (Art. 1621).
the instrument (Art. 1363, CC).
Conventional redemption (pacto de retro) sale is one that is not
If the parties agree upon the mortgage or pledge of property, but the mandated by the statute but one which takes place because of the
instrument states that the property is sold absolutely or with a right of stipulation of the parties to the sale. The period of redemption may
repurchase, reformation of the instrument is proper (Art. 1365, CC). be fixed by the parties in which case the period cannot exceed ten
(10) years from the date of the contract. In the absence of any
Where the consent of a party to a contract has been procured by agreement, the redemption period shall be four (4) years from the
fraud, inequitable conduct or accident, and an instrument was date of the contract (Art. 1606).
executed by the parties in accordance with the contract, what is
defective is the contract itself because of vitiation of consent. When the redemption is not made within the period agreed upon, in
case the subject matter of the sale is a real property, Art. 1607
The remedy is not to bring an action for reformation of the instrument provides that the consolidation of ownership in the vendee shall not
but to file an action for annulment of the contract (Art. 1359, CC). be recorded in the Registry of Property without a judicial order, after
the vendor has been duly heard.
Reformation of the instrument cannot be brought to reform any of the
following: The action brought to consolidate ownership is not for the purpose of
consolidating the ownership of the property in the person of the

68 Aicka Singson
Notes in Remedial Law Review II
Provisional Remedies, Special Civil Actions, Special Proceedings and Evidence
School Year 2015-2016 REMEDIAL LAW REVIEW II - ATTY. BRONDIAL

vendee or buyer but for the registration of the property. The lapse of CASES:
the redemption period without the seller a retro exercising his right of
redemption consolidates ownership or title upon the person of the a. Almeda vs. Bathala Marketing Inc (542 SCRA)
vendee by operation of law. Art. 1607 requires the filing of the
petition to consolidate ownership because the law precludes the Sometime in May 1997, respondent Bathala Marketing Industries,
registration of the consolidated title without judicial order. Inc., as lessee, renewed its Contract of Lease with Ponciano L.
Almeda (Ponciano), as lessor, husband of petitioner Eufemia and
C. QUIETING OF TITLE TO REAL PROPERTY father of petitioner Romel Almeda. Under the said contract,
Ponciano agreed to lease a portion of the Almeda Compound, for a
This action is brought to remove a cloud on title to rea lproperty or monthly rental of P1,107,348.69, for a term of four (4) years from
any interest therein. The action contemplates a situation where the May 1, 1997 unless sooner terminated as provided in the contract.
instrument or a record is apparently valid or effective but is in truth
and in fact invalid, ineffective, voidable or unenforceable, and may During the effectivity of the contract, Ponciano died. Thereafter,
be prejudicial to said title to real property. respondent dealt with petitioners. Respondent received a letter from
petitioners informing the former that its monthly rental should be
It may also be brought as a preventive remedy to prevent a cloud increased by 73% pursuant to condition No. 7 of the contract and
from being cast upon title to real property or any interest therein (Art. Article 1250 of the Civil Code. Respondent opposed petitioners’
476). demand and insisted that there was no extraordinary inflation to
warrant the application of Article 1250 in light of the pronouncement
The plaintiff need not be in possession of the real property before he
of this Court in various cases.
may bring the action as long as he can show that he has a legal or
an equitable title to the property which is the subject matter of the Respondent Bathala instituted an action for declaratory relief for
action (Art. 477). purposes of determining the correct interpretation of condition Nos. 6
and 7 of the lease contract to prevent damage and prejudice.
Instances where Declaratory Relief is not Available
Petitioners in turn filed an action for ejectment, rescission and
1. To obtain Judicial declaration of citizenship;
damages against respondent for failure of the latter to vacate the
2. To Establish legitimate filiation and determine hereditary rights; premises after the demand.

3. The subject of the action is court decision; Petitioners later moved for the dismissal of the declaratory relief case
for being an improper remedy considering that respondent was
already in breach of the obligation and that the case would not end
the litigation and settle the rights of the parties. The trial court,
however, was not persuaded, and consequently, denied the motion.
The RTC ruled in favor of respondent Bathala and against
petitioners.

69 Aicka Singson
Notes in Remedial Law Review II
Provisional Remedies, Special Civil Actions, Special Proceedings and Evidence
School Year 2015-2016 REMEDIAL LAW REVIEW II - ATTY. BRONDIAL

The court ordered the restitution by the petitioner to the amounts not available through other means or other forms of action or
paid, notwithstanding the well-established rule that in an action for proceeding.
declaratory relief, other than a declaration of rights and obligations,
affirmative reliefs are not sought by or awarded to the parties. It is beyond cavil that the foregoing requisites are present in the
instant case, except that petitioners insist that respondent was
Petitioners elevated the aforesaid case to the Court of Appeals already in breach of the contract when the petition was filed.
which affirmed the RTC decision.
After petitioners demanded payment of adjusted rentals and in
The appellate court agreed with the conclusions of law and the the months that followed, respondent complied with the terms and
application of the decisional rules on the matter made by the RTC. conditions set forth in their contract of lease by paying the rentals
However, it found that the trial court exceeded its jurisdiction in stipulated therein. Respondent religiously fulfilled its obligations to
granting affirmative relief to the respondent, particularly the petitioners even during the pendency of the present suit. There is no
restitution of its excess payment. showing that respondent committed an act constituting a breach of
the subject contract of lease. Thus, respondent is not barred from
Issue: Whether the action for declaratory relief is proper instituting before the trial court the petition for declaratory relief.

YES. Declaratory relief is defined as an action by any person Petitioners claim that the instant petition is not proper because
interested in a deed, will, contract or other written instrument, a separate action for rescission, ejectment and damages had been
executive order or resolution, to determine any question of commenced before another court; thus, the construction of the
construction or validity arising from the instrument, executive order or subject contractual provisions should be ventilated in the same
regulation, or statute, and for a declaration of his rights and duties forum. SC is not convinced.
thereunder. The only issue that may be raised in such a petition is
the question of construction or validity of provisions in an instrument It is true that in Panganiban v. Pilipinas Shell Petroleum
or statute. Corollary is the general rule that such an action must be Corporation we held that the petition for declaratory relief should be
justified, as no other adequate relief or remedy is available under the dismissed in view of the pendency of a separate action for unlawful
circumstances. detainer. However, we cannot apply the same ruling to the instant
case. In Panganiban, the unlawful detainer case had already been
Decisional law enumerates the requisites of an action for declaratory resolved by the trial court before the dismissal of the declaratory
relief, as follows: 1) the subject matter of the controversy must be a relief case; and it was petitioner in that case who insisted that the
deed, will, contract or other written instrument, statute, executive action for declaratory relief be preferred over the action for unlawful
order or regulation, or ordinance; 2) the terms of said documents and detainer. Conversely, in the case at bench, the trial court had not yet
the validity thereof are doubtful and require judicial construction; 3) resolved the rescission/ejectment case during the pendency of the
there must have been no breach of the documents in question; 4) declaratory relief petition. In fact, the trial court, where the rescission
there must be an actual justiciable controversy or the “ripening case was on appeal, itself initiated the suspension of the
seeds” of one between persons whose interests are adverse; 5) the proceedings pending the resolution of the action for declaratory
issue must be ripe for judicial determination; and 6) adequate relief is relief.

70 Aicka Singson
Notes in Remedial Law Review II
Provisional Remedies, Special Civil Actions, Special Proceedings and Evidence
School Year 2015-2016 REMEDIAL LAW REVIEW II - ATTY. BRONDIAL

We are not unmindful of the doctrine enunciated in Teodoro, ISSUE: WON declaratory relief was proper
Jr. v. Miraso,] where the declaratory relief action was dismissed
because the issue therein could be threshed out in the unlawful At the outset, we note that the petition for authority to remarry filed
detainer suit. Yet, again, in that case, there was already a breach of before the trial court actually constituted a petition for declaratory
contract at the time of the filing of the declaratory relief petition. This relief.
dissimilar factual milieu proscribes the Court from applying Teodoro
HELD: YES. The requisites of a petition for declaratory relief are: (1)
to the instant case.
there must be a justiciable controversy; (2) the controversy must be
Given all these attendant circumstances, the Court is disposed between persons whose interests are adverse; (3) that the party
to entertain the instant declaratory relief action instead of dismissing seeking the relief has a legal interest in the controversy; and (4) that
it, notwithstanding the pendency of the ejectment/rescission case the issue is ripe for judicial determination.
before the trial court. The resolution of the present petition would
This case concerns the applicability of Paragraph 2 of Article 26 to a
write finis to the parties’ dispute, as it would settle once and for all
marriage between two Filipino citizens where one later acquired alien
the question of the proper interpretation of the two contractual
citizenship, obtained a divorce decree, and remarried while in the
stipulations subject of this controversy.
U.S.A. The interests of the parties are also adverse, as petitioner
b. Republic vs. Orbecido (472 SCRA) representing the State asserts its duty to protect the institution of
marriage while respondent, a private citizen, insists on a declaration
FACTS: On May 24, 1981, Cipriano Orbecido III married Lady Myros of his capacity to remarry. Respondent, praying for relief, has legal
M. Villanueva at the United Church of Christ in the Philippines in interest in the controversy. The issue raised is also ripe for judicial
Lam-an, Ozamis City. Their marriage was blessed with a son and a determination inasmuch as when respondent remarries, litigation
daughter. ensues and puts into question the validity of his second marriage.

In 1986, Cipriano’s wife left for the United States bringing along their c. Malana vs. Toppa (600 SCRA)
son Kristoffer. A few years later, Cipriano discovered that his wife
had been naturalized as an American citizen. Petitioners filed before the RTC their Complaint for Reivindicacion,
Quieting of Title, and Damages against respondents. Petitioners
Sometime in 2000, Cipriano learned from his son that his wife had alleged in their Complaint that they are the owners of a parcel of land
obtained a divorce decree and then married a certain Innocent situated in Tuguegarao City. Petitioners inherited the subject
Stanley. property from Anastacio Danao (Anastacio), who died intestate.
During the lifetime of Anastacio, he had allowed Consuelo Pauig
Cipriano thereafter filed with the trial court a petition for authority to (Consuelo), who was married to Joaquin Boncad, to build on and
remarry invoking Paragraph 2 of Article 26 of the Family Code. No occupy the southern portion of the subject property. Anastacio and
opposition was filed. Finding merit in the petition, the court granted Consuelo agreed that the latter would vacate the said land at any
the same. The Republic, herein petitioner, through the Office of the time that Anastacio and his heirs might need it.
Solicitor General (OSG), sought reconsideration but it was denied.

71 Aicka Singson
Notes in Remedial Law Review II
Provisional Remedies, Special Civil Actions, Special Proceedings and Evidence
School Year 2015-2016 REMEDIAL LAW REVIEW II - ATTY. BRONDIAL

Petitioners claimed that respondents, Consuelo’s family members, Petitioners filed another pleading, simply designated as Motion, in
continued to occupy the subject property even after her death, which they prayed that the RTC Orders dated 4 May 2007 and 30
already building their residences thereon using permanent materials. May 2007, dismissing their Complaint, be set aside. They reiterated
Petitioners also learned that respondents were claiming ownership their earlier argument that Section 1, Rule 63 of the Rules of Court
over the subject property. Petitioners demanded that respondents states that an action to quiet title falls under the exclusive jurisdiction
vacate the same. Respondents, however, refused to heed of the RTC. They also contended that there was no obstacle to their
petitioners’ demand. joining the two causes of action, i.e., quieting of title and
reivindicacion, in a single Complaint, citing Rumarate v. Hernandez.
Petitioners referred their land dispute with respondents to the And even if the two causes of action could not be joined, petitioners
Lupong Tagapamayapa of Barangay for conciliation. During the maintained that the misjoinder of said causes of action was not a
conciliation proceedings, respondents asserted that they owned the ground for the dismissal of their Complaint.
subject property and presented documents ostensibly supporting
their claim of ownership.

Thus, petitioners were compelled to file before the RTC a Complaint The RTC issued an Order denying petitioners’ Motion. It clarified
to remove such cloud from their title. RTC dismissed thr petitioners’ that their Complaint was dismissed, not on the ground of misjoinder
Complaint on the ground of lack of jurisdiction. of causes of action, but for lack of jurisdiction.

Petitioners filed a Motion for Reconsideration of the aforementioned The RTC differentiated between the first and the second paragraphs
RTC Order dismissing their Complaint. They argued that their of Section 1, Rule 63 of the Rules of Court. The first paragraph
principal cause of action was for quieting of title; the accion refers to an action for declaratory relief, which should be brought
reivindicacion was included merely to enable them to seek complete before the RTC. The second paragraph, however, refers to a
relief from respondents. Petitioner’s Complaint should not have different set of remedies, which includes an action to quiet title to real
been dismissed, since Section 1, Rule 63 of the Rules of Court property. The second paragraph must be read in relation to Republic
states that an action to quiet title falls under the jurisdiction of the Act No. 7691, which vests the MTC with jurisdiction over real actions,
RTC. where the assessed value of the real property involved does not
exceed P50,000.00 in Metro Manila and P20,000.00 in all other
RTC denied petitioners’ Motion for Reconsideration. It reasoned that places.
an action to quiet title is a real action. Pursuant to Republic Act No.
7691, it is the Municipal Trial Court (MTC) that exercises exclusive Hence, it has no jurisdiction over the action.
jurisdiction over real actions where the assessed value of real
property does not exceed P20,000.00. Since the assessed value of Whether the RTC committed grave abuse of discretion in
subject property per Tax Declaration No, 02-48386 was P410.00, the dismissing petitioners’ Complaint for lack of jurisdiction.
real action involving the same was outside the jurisdiction of the
NO. Petitions for declaratory relief are governed by Rule 63 of the
RTC.
Rules of Court. The RTC correctly made a distinction between the

72 Aicka Singson
Notes in Remedial Law Review II
Provisional Remedies, Special Civil Actions, Special Proceedings and Evidence
School Year 2015-2016 REMEDIAL LAW REVIEW II - ATTY. BRONDIAL

first and the second paragraphs of Section 1, Rule 63 of the Rules of To determine which court has jurisdiction over the actions identified
Court. in the second paragraph of Section 1, Rule 63 of the Rules of Court,
said provision must be read together with those of the Judiciary
The first paragraph of Section 1, Rule 63 of the Rules of Court, Reorganization Act of 1980, as amended.
describes the general circumstances in which a person may file a
petition for declaratory relief, to wit: It is important to note that Section 1, Rule 63 of the Rules of Court
does not categorically require that an action to quiet title be filed
Any person interested under a deed, will, contract or other written before the RTC. It repeatedly uses the word “may” – that an action
instrument, or whose rights are affected by a statute, executive order for quieting of title “may be brought under [the] Rule” on petitions for
or regulation, ordinance, or any other governmental regulation may, declaratory relief, and a person desiring to file a petition for
before breach or violation thereof, bring an action in the appropriate declaratory relief “may x x x bring an action in the appropriate
Regional Trial Court to determine any question of construction or Regional Trial Court.” The use of the word “may” in a statute
validity arising, and for a declaration of his rights or duties, denotes that the provision is merely permissive and indicates a mere
thereunder. possibility, an opportunity or an option.

As the afore-quoted provision states, a petition for declaratory relief In contrast, the mandatory provision of the Judiciary Reorganization
under the first paragraph of Section 1, Rule 63 may be brought Act of 1980, as amended, uses the word “shall” and explicitly
before the appropriate RTC. requires the MTC to exercise exclusive original jurisdiction over all
civil actions which involve title to or possession of real property
Section 1, Rule 63 of the Rules of Court further provides in its
where the assessed value does not exceed P20,000.00, thus:
second paragraph that:
Section 33. Jurisdiction of Metropolitan Trial Courts, Municipal Trial
An action for the reformation of an instrument, to quiet title to real
Courts and Municipal Circuit Trial Courts in Civil Cases.—
property or remove clouds therefrom, or to consolidate ownership
Metropolitan Trial Courts, Municipal Trial Courts and Municipal
under Article 1607 of the Civil Code, may be brought under this Rule.
Circuit Trial Courts shall exercise:
The second paragraph of Section 1, Rule 63 of the Rules of Court
(3) Exclusive original jurisdiction in all civil actions which involve title
specifically refers to (1) an action for the reformation of an
to, possession of, real property, or any interest therein where the
instrument, recognized under Articles 1359 to 1369 of the Civil Code;
assessed value of the property or interest therein does not exceed
(2) an action to quiet title, authorized by Articles 476 to 481 of the
Twenty thousand pesos (P20,000.00) or, in civil actions in Metro
Civil Code; and (3) an action to consolidate ownership required by
Manila, where such assessed value does not exceeds Fifty thousand
Article 1607 of the Civil Code in a sale with a right to repurchase.
pesos (P50,000.00) exclusive of interest, damages of whatever kind,
These three remedies are considered similar to declaratory relief
attorney’s fees, litigation expenses and costs: x x x
because they also result in the adjudication of the legal rights of the
litigants, often without the need of execution to carry the judgment As found by the RTC, the assessed value of the subject property as
into effect. stated in Tax Declaration is only P410.00; therefore, petitioners’

73 Aicka Singson
Notes in Remedial Law Review II
Provisional Remedies, Special Civil Actions, Special Proceedings and Evidence
School Year 2015-2016 REMEDIAL LAW REVIEW II - ATTY. BRONDIAL

Complaint involving title to and possession of the said property is The present action stemmed from the unexpected departure of
within the exclusive original jurisdiction of the MTC, not the RTC. former Chief Justice Renato C. Corona and the nomination of
petitioner, as his potential successor. In his initiatory pleading,
Furthermore, an action for declaratory relief presupposes that there petitioner asked the Court to determine 1] whether the first
has been no actual breach of the instruments involved or of rights paragraph of Section 8, Article VIII of the 1987 Constitution allows
arising thereunder. Since the purpose of an action for declaratory more than one (1) member of Congress to sit in the JBC; and 2] if
relief is to secure an authoritative statement of the rights and the practice of having two (2) representatives from each House of
obligations of the parties under a statute, deed, or contract for their Congress with one (1) vote each is sanctioned by the Constitution.
guidance in the enforcement thereof, or compliance therewith, and
not to settle issues arising from an alleged breach thereof, it may be The Court handed down the assailed subject decision, disposing the
entertained only before the breach or violation of the statute, deed, same in the following manner:
or contract to which it refers. A petition for declaratory relief gives a
practical remedy for ending controversies that have not reached the WHEREFORE, the petition is GRANTED. The current numerical
state where another relief is immediately available; and supplies the composition of the Judicial and Bar Council is declared
need for a form of action that will set controversies at rest before UNCONSTITUTIONAL. The Judicial and Bar Council is hereby
they lead to a repudiation of obligations, an invasion of rights, and a enjoined to reconstitute itself so that only one (1) member of
commission of wrongs. Congress will sit as a representative in its proceedings, in
accordance with Section 8(1),f Article VIII of the 1987 Constitution.
Where the law or contract has already been contravened prior to the
filing of an action for declaratory relief, the courts can no longer This disposition is immediately executory.
assume jurisdiction over the action. In other words, a court has no
Respondents’ motion for reconsideration and with due regard to
more jurisdiction over an action for declaratory relief if its subject has
Senate Resolution Nos. 111,3 112,4 113,5 and 114, the Court set
already been infringed or transgressed before the institution of the
the subject motion for oral arguments. The Court discussed the
action.
merits of the arguments and agreed, in the meantime, to suspend
Petitioners averred in the Complaint that they had already been the effects of the second paragraph of the dispositive portion of the
deprived of the possession of their property, the proper remedy for July 17, 2012 Decision which decreed that it was immediately
them is the filing of an accion publiciana or an accion reivindicatoria, executory.
not a case for declaratory relief. An accion publiciana is a suit for the
Through the subject motion, respondents pray that the Court
recovery of possession, filed one year after the occurrence of the
reconsider its decision and dismiss the petition.
cause of action or from the unlawful withholding of possession of the
realty. An accion reivindicatoria is a suit that has for its object one’s Ruling of the Court
recovery of possession over the real property as owner.
The Constitution evinces the direct action of the Filipino people by
d. Chavez vs. JBC (676 SCRA) which the fundamental powers of government are established,
limited and defined and by which those powers are distributed

74 Aicka Singson
Notes in Remedial Law Review II
Provisional Remedies, Special Civil Actions, Special Proceedings and Evidence
School Year 2015-2016 REMEDIAL LAW REVIEW II - ATTY. BRONDIAL

among the several departments for their safe and useful exercise for statute prior to a determination of unconstitutionality is an operative
the benefit of the body politic. The Framers reposed their wisdom fact and may have consequences which cannot always be ignored.
and vision on one suprema lex to be the ultimate expression of the The past cannot always be erased by a new judicial declaration. The
principles and the framework upon which government and society doctrine is applicable when a declaration of unconstitutionality will
were to operate. Thus, in the interpretation of the constitutional impose an undue burden on those who have relied on the invalid
provisions, the Court firmly relies on the basic postulate that the law. Thus, it was applied to a criminal case when a declaration of
Framers mean what they say. The language used in the Constitution unconstitutionality would put the accused in double jeopardy or
must be taken to have been deliberately chosen for a definite would put in limbo the acts done by a municipality in reliance upon a
purpose. Every word employed in the Constitution must be law creating it.
interpreted to exude its deliberate intent which must be maintained
inviolate against disobedience and defiance. What the Constitution Under the circumstances, the Court finds the exception applicable in
clearly says, according to its text, compels acceptance and bars this case and holds that notwithstanding its finding of
modification even by the branch tasked to interpret it. unconstitutionality in the current composition of the JBC, all its prior
official actions are nonetheless valid.
For this reason, the Court cannot accede to the argument of plain
oversight in order to justify constitutional construction. As stated in Considering that the Court is duty bound to protect the Constitution
the July 17, 2012 Decision, in opting to use the singular letter "a" to which was ratified by the direct action of the Filipino people, it cannot
describe "representative of Congress," the Filipino people through correct what respondents perceive as a mistake in its mandate.
the Framers intended that Congress be entitled to only one (1) seat Neither can the Court, in the exercise of its power to interpret the
in the JBC. Had the intention been otherwise, the Constitution could spirit of the Constitution, read into the law something that is contrary
have, in no uncertain terms, so provided, as can be read in its other to its express provisions and justify the same as correcting a
provisions. perceived inadvertence. To do so would otherwise sanction the
Court action of making amendment to the Constitution through a
It would not be amiss to point out, however, that as a general rule, an judicial pronouncement.
unconstitutional act is not a law; it confers no rights; it imposes no
duties; it affords no protection; it creates no office; it is inoperative as Stated differently, the Court has no power to add another member by
if it has not been passed at all. This rule, however, is not absolute. judicial construction.
Under the doctrine of operative facts, actions previous to the
The call for judicial activism fails to stir the sensibilities of the Court
declaration of unconstitutionality are legally recognized. They are not
tasked to guard the Constitution against usurpation. The Court
nullified. This is essential in the interest of fair play. To reiterate the
remains steadfast in confining its powers in the sphere granted by
doctrine enunciated in Planters Products, Inc. v. Fertiphil
the Constitution itself. Judicial activism should never be allowed to
Corporation:
become judicial exuberance. In cases like this, no amount of
The doctrine of operative fact, as an exception to the general rule, practical logic or convenience can convince the Court to perform
only applies as a matter of equity and fair play. It nullifies the effects either an excision or an insertion that will change the manifest intent
of an unconstitutional law by recognizing that the existence of a of the Framers. To broaden the scope of congressional

75 Aicka Singson
Notes in Remedial Law Review II
Provisional Remedies, Special Civil Actions, Special Proceedings and Evidence
School Year 2015-2016 REMEDIAL LAW REVIEW II - ATTY. BRONDIAL

representation in the JBC is tantamount to the inclusion of a subject declared null and void and of no effect; that petitioners be ordered to
matter which was not included in the provision as enacted. True to its respect and recognize Juanito’s title over the lot; and that moral and
constitutional mandate, the Court cannot craft and tailor exemplary damages, attorney’s fees, and litigation expenses be
constitutional provisions in order to accommodate all of situations no awarded to him.
matter how ideal or reasonable the proposed solution may sound. To
the exercise of this intrusion, the Court declines. In their Answer with Counterclaim, petitioners asserted mainly
that the sale to Juanito is null and void absent the marital
e. Sabitsana vs. Muertegui (703 SCRA) consent of Garcia’s wife, Soledad Corto (Soledad); that they
acquired the property in good faith and for value; and that the
On September 2, 1981, Alberto Garcia (Garcia) executed an Complaint is barred by prescription and laches. They likewise
unnotarized Deed of Sale in favor of respondent Juanito Muertegui insisted that the Regional Trial Court (RTC) did not have
(Juanito) parcel of unregistered land (the lot) located in Leyte del jurisdiction over the case, which involved title to or interest in a
Norte covered by Tax Declaration issued in 1985 in Garcia’s name. parcel of land the assessed value of which is merely P1,230.00.

Juanito’s father Domingo Muertegui, Sr. (Domingo Sr.) and brother RTC ruled that the Deed of Absolute Sale in the name of Atty.
Domingo Jr. took actual possession of the lot and planted thereon Clemencio C. Sabitsana, Jr. are VOID and of no legal effect.
coconut and ipil-ipil trees. They also paid the real property taxes on
the lot for the years 1980 up to 1998. The Provincial Assessor and the Municipal Assessor of Naval
are directed to cancel Tax Declaration as void and done in bad
On October 17, 1991, Garcia sold the lot to the Muertegui family faith.
lawyer, petitioner Atty. Clemencio C. Sabitsana, Jr. (Atty. Sabitsana),
through a notarized deed of absolute sale. Petitioners filed a Motion for Reconsideration but the trial court
denied the same.
When Domingo Sr. passed away, his heirs applied for registration
and coverage of the lot under the Public Land Act or Commonwealth Petitioners appealed to the CA and the CA denied the appeal
Act No. 141. Atty. Sabitsana opposed the application, claiming that and affirmed the trial court’s Decision in toto. The CA added that
he was the true owner of the lot. He asked that the application for the fact that the Deed of Sale in favor of Juanito was not
registration be held in abeyance until the issue of conflicting notarized could not affect its validity. As against the notarized
ownership has been resolved. deed of sale in favor of petitioners, the CA held that the sale in
favor of Juanito still prevails.
Juanito, through his attorney-in-fact Domingo Jr., filed Civil Case for
quieting of title and preliminary injunction, against herein petitioners THE COURT OF APPEALS ERRED IN NOT HOLDING THAT
Atty. Sabitsana and his wife, Rosario, claiming that they bought the THE REGIONAL TRIAL COURT DID NOT HAVE
lot in bad faith and are exercising acts of possession and ownership JURISDICTION OVER THE CASE IN VIEW OF THE FACT
over the same, which acts thus constitute a cloud over his title. The THAT THE ASSESSED VALUE OF THE SUBJECT LAND WAS
Complaint prayed, among others, that the Sabitsana Deed of Sale be

76 Aicka Singson
Notes in Remedial Law Review II
Provisional Remedies, Special Civil Actions, Special Proceedings and Evidence
School Year 2015-2016 REMEDIAL LAW REVIEW II - ATTY. BRONDIAL

ONLY P1,230.00 (AND STATED MARKET VALUE OF ONLY In Relation to Southern Hemisphere case 632
P3,450.00). SCRA 10/5/2010
Petitioners assert that the RTC did not have jurisdiction over the FACTS: On July 17, 2007, private respondents filed a Petition for
case. They argue that since the assessed value of the lot was a declaratory relief before the RTC, assailing the constitutionality of the
mere P1,230.00, jurisdiction over the case lies with the first level following sections of RA 9372:
courts. Petitioners thus conclude that the Decision in Civil Case
No. B-1097 is null and void for lack of jurisdiction. (a) Section 3,7 for being void for vagueness;

Respondent, on the other hand, counters that a suit for quieting (b) Section 7,9 for violating the right to privacy of communication and
of title is one whose subject matter is incapable of pecuniary due process and the privileged nature of priest-penitent
estimation, and thus falls within the jurisdiction of the RTC. relationships;

The Petition must be denied. (c)Section 18,11 for violating due process, the prohibition against ex
post facto laws or bills of attainder, the Universal Declaration of
The Regional Trial Court has jurisdiction over the suit for quieting Human Rights, and the International Covenant on Civil and Political
of title. Rights, as well as for contradicting Article 12512 of the Revised
Penal Code, as amended;
On the question of jurisdiction, it is clear under the Rules that an
action for quieting of title may be instituted in the RTCs, (d) Section 26,14 for violating the right to travel;and
regardless of the assessed value of the real property in dispute.
Under Rule 63 of the Rules of Court an action to quiet title to real (e) Section 27,16 for violating the prohibition against unreasonable
property or remove clouds therefrom may be brought in the searches and seizures.
appropriate RTC.
Petitioners moved to suspend the proceedings, the said motion was
It must be remembered that the suit for quieting of title was granted.
prompted by petitioners’ August 24, 1998 letter-opposition to
respondent’s application for registration. Thus, in order to On October 5, 2010, the Court promulgated its Decision in the
prevent a cloud from being cast upon his application for a title, Southern Hemisphere cases and thereby dismissed the SC petitions.
respondent filed Civil Case No. B-1097 to obtain a declaration of
Petitioners filed the subject motion to dismiss, contending that
his rights. In this sense, the action is one for declaratory relief,
private respondents failed to satisfy the requisites for declaratory
which properly falls within the jurisdiction of the RTC pursuant to
relief. Likewise, they averred that the constitutionality of RA 9372
Rule 63 of the Rules.
had already been upheld by the Court in the Southern Hemisphere
f. Republic vs. Roque (706 SCRA) cases.

In their Comment/Opposition, private respondents countered that:

77 Aicka Singson
Notes in Remedial Law Review II
Provisional Remedies, Special Civil Actions, Special Proceedings and Evidence
School Year 2015-2016 REMEDIAL LAW REVIEW II - ATTY. BRONDIAL

(a) the Court did not resolve the issue of RA 9372’s constitutionality fifth , the issue must be ripe for judicial determination; and
in Southern Hemisphere as the SC petitions were dismissed
based purely on technical grounds; and sixth , adequate relief is not available through other means or other
forms of action or proceeding.
(b) the requisites for declaratory relief were met.
The Court observes that while the first, second, and third
RTC issued an Order which denied the subject motion to dismiss, requirements appear to exist in this case, the fourth, fifth, and sixth
finding that the Court did not pass upon the constitutionality of RA requirements, however, remain wanting.
9372 and that private respondents’ petition for declaratory relief was
properly filed. As to the fourth requisite, there is serious doubt that an actual
justiciable controversy or the "ripening seeds" of one exists in this
ISSUES: Whether or not private respondents failed to satisfy the
case.
requirements for declaratory relief and that the Court had
Pertinently, a justiciable controversy refers to an existing case or
already sustained with finality the constitutionality of RA 9372
controversy that is appropriate or ripe for judicial determination, not
HELD: YES. It is clear that the Court, in Southern Hemisphere, did one that is conjectural or merely anticipatory. The concept describes
not make any definitive ruling on the constitutionality of RA 9372. a state of facts indicating imminent and inevitable litigation provided
The certiorari petitions in those consolidated cases were dismissed that the issue is not settled and stabilized by tranquilizing
based solely on procedural grounds, namely: (a) the remedy of declaration.
certiorari was improper; (b) petitioners therein lack locus standi; and
(c) petitioners therein failed to present an actual case or controversy. A perusal of private respondents’ petition for declaratory relief would
Therefore, there was no grave abuse of discretion show that they have failed to demonstrate how they are left to
sustain or are in immediate danger to sustain some direct injury as a
Requisites for an action for declaratory relief: result of the enforcement of the assailed provisions of RA 9372.

first , the subject matter of the controversy must be a deed, will, Not far removed from the factual milieu in the Southern Hemisphere
contract or other written instrument, statute, executive order or cases, private respondents only assert general interests as citizens,
regulation, or ordinance; and taxpayers and infractions which the government could
prospectively commit if the enforcement of the said law would remain
second , the terms of said documents and the validity thereof are
untrammeled.
doubtful and require judicial construction;

third , there must have been no breach of the documents in question; As their petition would disclose, private respondents’ fear of
prosecution was solely based on remarks of certain government
fourth , there must be an actual justiciable controversy or the officials which were addressed to the general public. They, however,
"ripening seeds" of one between persons whose interests are failed to show how these remarks tended towards any prosecutorial
adverse;

78 Aicka Singson
Notes in Remedial Law Review II
Provisional Remedies, Special Civil Actions, Special Proceedings and Evidence
School Year 2015-2016 REMEDIAL LAW REVIEW II - ATTY. BRONDIAL

or governmental action geared towards the implementation of RA C. Review of Judgments and Final Orders of the Comelec
9372 against them. and COA (Rule 64)
In other words, there was no particular, real or imminent threat to What is the distinctive nature of this action? Neypes will not
any of them. As held in Southern Hemisphere, apply (fresh period rule). Refer to Section 3 – “within the remaining
Without any justiciable controversy, the petitions have become period, but which shall not be less than 5 days in any event,
pleas for declaratory relief, over which the Court has no original reckoned from notice of denial.” The period is a 30-day period. If you
jurisdiction. Then again, declaratory actions characterized by file it on the 5th day of the 30-day period, you still have a balance of
"double contingency," where both the activity the petitioners 25 days. If you file on the 29th day, the balance is 5 days. You count
intend to undertake and the anticipated reaction to it of a public the balance from the time that you received a copy of the order
official are merely theorized, lie beyond judicial review for lack of denying the motion for reconsideration.
ripeness.
Which court has jurisdiction? SC. What are the other tribunals?
As to the fifth requisite for an action for declaratory relief, neither can
Sandiganbayan, CTA en banc, CA.
it be inferred that the controversy at hand is ripe for adjudication
since the possibility of abuse, based on the above-discussed
What kind of petition is applicable under Rule 64? Certiorari. With
allegations in private respondents’ petition, remain highly-speculative
only one difference which is? The period of filing which is 30 days
and merely theorized. It is well-settled that a question is ripe for
under Rule 64, while in Rule 65 it is 60 days.
adjudication when the act being challenged has had a direct adverse
effect on the individual challenging it. This private respondents failed
If you do not apply the fresh period rule in Rule 64, do you apply that
to demonstrate in the case at bar.
in Rule 65? It will not find application in Rule 65 because in Rule 65,
Sixth requisite, the Court finds it irrelevant to proceed with a the 60-day period is counted from the denial of the motion for
discussion on the availability of adequate reliefs since no impending reconsideration. As a general rule, a motion for reconsideration is
threat or injury to the private respondents exists in the first place. mandatory. There are about 15 exceptions.

Section 1. Scope. — This Rule shall govern the review of judgments and
All told, in view of the absence of the fourth and fifth requisites for an
final orders or resolutions of the Commission on Elections and the
action for declaratory relief, as well as the irrelevance of the sixth Commission on Audit.
requisite, private respondents’ petition for declaratory relief should
have been dismissed. Thus, by giving due course to the same, it Section 2. Mode of review. — A judgment or final order or resolution of the
cannot be gainsaid that the RTC gravely abused its discretion. Commission on Elections and the Commission on Audit may be brought by
the aggrieved party to the Supreme Court on certiorari under Rule 65,
except as hereinafter provided.

Section 3. Time to file petition. — The petition shall be filed within thirty (30)
days from notice of the judgment or final order or resolution sought to be
reviewed. The filing of a motion for new trial or reconsideration of said

79 Aicka Singson
Notes in Remedial Law Review II
Provisional Remedies, Special Civil Actions, Special Proceedings and Evidence
School Year 2015-2016 REMEDIAL LAW REVIEW II - ATTY. BRONDIAL

judgment or final order or resolution, if allowed under the procedural rules of Section 6. Order to comment. — If the Supreme Court finds the petition
the Commission concerned, shall interrupt the period herein fixed. If the sufficient in form and substance, it shall order the respondents to file their
motion is denied, the aggrieved party may file the petition within the comments on the petition within ten (10) days from notice thereof; otherwise,
remaining period, but which shall not be less than five (5) days in any event, the Court may dismiss the petition outright. The Court may also dismiss the
reckoned from notice of denial. petition if it was filed manifestly for delay or the questions raised are too
unsubstantial to warrant further proceedings.
Section 4. Docket and other lawful fees. — Upon the filing of the petition, the
petitioner shall pay to the clerk of court the docket and other lawful fees and Section 7. Comments of respondents. — The comments of the respondents
deposit the amount of P500.00 for costs. shall be filed in eighteen (18) legible copies. The original shall be
accompanied by certified true copies of such material portions of the record
Section 5. Form and contents of petition. — The petition shall be verified and as are referred to therein together with other supporting papers. The
filed in eighteen (18) legible copies. The petition shall name the aggrieved requisite number of copies of the comments shall contain plain copies of all
party as petitioner and shall join as respondents the Commission concerned documents attached to the original and a copy thereof shall be served on the
and the person or persons interested in sustaining the judgment, final order petitioner.
or resolution a quo. The petition shall state the facts with certainty, present
clearly the issues involved, set forth the grounds and brief arguments relied No other pleading may be filed by any party unless required or allowed by
upon for review, and pray for judgment annulling or modifying the questioned the Court.
judgment, final order or resolution. Findings of fact of the Commission
supported by substantial evidence shall be final and non-reviewable. Section 8. Effect of filing. — The filing of a petition for certiorari shall not stay
the execution of the judgment or final order or resolution sought to be
The petition shall be accompanied by a clearly legible duplicate original or reviewed, unless the Supreme Court shall direct otherwise upon such terms
certified true copy of the judgment, final order or resolution subject thereof, as it may deem just.
together with certified true copies of such material portions of the record as
are referred to therein and other documents relevant and pertinent thereto. Section 9. Submission for decision. — Unless the Court sets the case for
The requisite number of copies of the petition shall contain plain copies of all oral argument, or requires the parties to submit memoranda, the case shall
documents attached to the original copy of said petition. be deemed submitted for decision upon the filing of the comments on the
petition, or of such other pleadings or papers as may be required or allowed,
The petition shall state the specific material dates showing that it was filed or the expiration of the period to do so.
within the period fixed herein, and shall contain a sworn certification against
forum shopping as provided in the third paragraph of section 3, Rule 46.
CASE:
The petition shall further be accompanied by proof of service of a copy
thereof on the Commission concerned and on the adverse party, and of the ALLIANCE FOR NATIONALISM AND DEMOCRACY (ANAD) vs.
timely payment of docket and other lawful fees. COMMISSION ON ELECTIONS G.R. No. 206987
September 10, 2013
The failure of petitioner to comply with any of the foregoing requirements
shall be sufficient ground for the dismissal of the petition. Before the Court is a Petition for Certiorari with Urgent Prayer for the
Issuance of a TRO and Writ of Mandamus, seeking to compel the
COMELEC to canvass the votes cast for petitioner Alliance for

80 Aicka Singson
Notes in Remedial Law Review II
Provisional Remedies, Special Civil Actions, Special Proceedings and Evidence
School Year 2015-2016 REMEDIAL LAW REVIEW II - ATTY. BRONDIAL

Nationalism and Democracy (ANAD) in the recently held 2013 Party- "Grave abuse of discretion," under Rule 65, has a specific meaning.
List Elections. It is the arbitrary or despotic exercise of power due to passion,
prejudice or personal hostility; or the whimsical, arbitrary, or
On 7 November 2012, the COMELEC En Banc promulgated a capricious exercise of power that amounts to an evasion or a refusal
Resolution canceling petitioner’s Certificate of Registration and/or to perform a positive duty enjoined by law or to act at all in
Accreditation. contemplation of law. For an act to be struck down as having been
done with grave abuse of discretion, the abuse of discretion must be
ANAD went before this Court challenging the above-mentioned
patent and gross.
resolution.
ANAD was already given the opportunity to prove its qualifications
In the assailed Resolution dated 11 May 2013, the COMELEC
during the summary hearing of 23 August 2012, during which ANAD
affirmed the cancellation of petitioner’s Certificate of Registration
submitted documents and other pieces of evidence to establish said
and/or Accreditation and disqualified it from participating in the 2013
qualifications. In re-evaluating ANAD’s qualifications in accordance
Elections. The COMELEC held that while ANAD can be classified as
with the parameters laid down in Atong Paglaum, Inc. v. COMELEC ,
a sectoral party lacking in well-defined political constituencies, its
the COMELEC need not have called another summary hearing. The
disqualification still subsists for violation of election laws and
Comelec could, as in fact it did, readily resort to documents and
regulations, particularly for its failure to submit at least five nominees,
other pieces of evidence previously submitted by petitioners in re-
and for its failure to submit its Statement of Contributions and
appraising ANAD’s qualifications. After all, it can be presumed that
Expenditures for the 2007 Elections.
the qualifications, or lack thereof, which were established during the
Hence, the present petition. summary hearing of 23 August2012 continued until election day and
even thereafter.
ISSUE: WON the COMELEC gravely abused its discretion in
promulgating the assailed Resolution without the benefit of a As to ANAD’s averment that the COMELEC erred in finding that it
summary evidentiary hearing mandated by the due process violated election laws and regulations, we hold that the COMELEC,
clause being a specialized agency tasked with the supervision of elections
all over the country, its factual findings, conclusions, rulings and
HELD: NO decisions rendered on matters falling within its competence shall not
be interfered with by this Court in the absence of grave abuse of
The only question that may be raised in a petition for certiorari under discretion or any jurisdictional infirmity or error of law.
Section 2, Rule 64 of the Rules of Court is whether or not the
COMELEC acted with grave abuse of discretion amounting to lack or As found by the COMELEC, ANAD, for unknown reasons, submitted
excess of jurisdiction. For a petition for certiorari to prosper, there only three nominees instead of five, in violation of Sec. 8 of R.A. No.
must be a clear showing of caprice and arbitrariness in the exercise 7941. Such factual finding of the COMELEC was based on the
of discretion. Certificate of Nomination presented and marked by petitioner during
the 22 and 23 August 2012summary hearings.

81 Aicka Singson
Notes in Remedial Law Review II
Provisional Remedies, Special Civil Actions, Special Proceedings and Evidence
School Year 2015-2016 REMEDIAL LAW REVIEW II - ATTY. BRONDIAL

We herein take the opportunity to reiterate the well-established Board: Board of Regulatory Energy, Board of Regulatory
principle that the rule that factual findings of administrative bodies Commission
will not be disturbed by the courts of justice except when there is
absolutely no evidence or no substantial evidence in support of such Officer: Chairman of SEC
findings should be applied with greater force when it concerns the
Corporation: as an entity but only the members of the Board
COMELEC, as the framers of the Constitution intended to place the
COMELEC – created and explicitly made independent by the Person: in prohibition, it may be against a private individual in
Constitution itself – on a level higher than statutory administrative prohibition. In mandamus, it is against an officer acting in his official
organs. The COMELEC has broad powers to ascertain the true capacity.
results of the election by means available to it. For the attainment of
that end, it is not strictly bound by the rules of evidence. Lack of jurisdiction: Jurisdiction over the issues cannot be a ground
because this is waivable. Jurisdiction refers to jurisdiction over the
As empowered by law, the COMELEC may motu proprio cancel, subject matter or over the person.
after due notice and hearing, the registration of any party-list
organization if it violates or fails to comply with laws, rules or Example: A files a case against B for a sum of money in the amount
regulations relating to elections. Thus, we find no grave abuse of of P500,000 before the Metropolitan Trial Court. The court can
discretion on the part of the COMELEC when it issued the assailed dismiss that. Suppose judgment is rendered. Is your remedy
Resolution dated 11 May 2013. certiorari? No, the conditions must be taken into consideration: there
must be no appeal, or any plain, speedy, and adequate remedy in
In any event, the official tally results of the COMELEC show that the ordinary course of law. Settled is the rule that certiorari and
ANAD garnered 200,972 votes. As such, even if petitioner is appeal are mutually exclusive. One cannot be a substitute for the
declared qualified and the votes cast for it are canvassed, statistics other.
show that it will still fail to qualify for a seat in the House of
Representatives Excess of jurisdiction: When the court, tribunal, officer oversteps its
authority.
D. CERTIORARI, PROHIBITION, MANDAMUS (Rule 65)
Example 1: An MTC judge is handling a case for sum of money. It is
Petitioner is any aggrieved party. Who is the aggrieved party? within his jurisdiction. He renders a judgment awarding more than
According to Rule 3: natural persons, juridical persons, or entities the jurisdictional amount under his jurisdiction, say P450,000. That is
authorized by law. How are you aggrieved? By the actuation of the clearly in excess of jurisdiction.
respondent. In certiorari, you limit that to the tribunal, board or
officer. Example 2: An MTC judge who renders a judgment in seduction
imposing a penalty of 20 years exceeds his jurisdiction.
Tribunal: RTC, MTC
Grave abuse of discretion amounting to lack of jurisdiction: Common
ground for certiorari. Capricious, Whimsical, Arbitrary, Despotic.

82 Aicka Singson
Notes in Remedial Law Review II
Provisional Remedies, Special Civil Actions, Special Proceedings and Evidence
School Year 2015-2016 REMEDIAL LAW REVIEW II - ATTY. BRONDIAL

Whimsical – no basis, no ground, unwarranted Certiorari is a remedy for the correction of errors of jurisdiction, not
errors of judgment. It is an original and independent action that was
Abritrary – unfounded not part of the trial that had resulted in the rendition of the judgment
or order complained of. Since the issue is jurisdiction, an original
Despotic – based on emotions
action for certiorari may be directed against an interlocutory order of
Discretionary: You have more than one choice. the lower court prior to an appeal from the judgment.

Ministerial: You don’t have any other choice. Where the error is not one of jurisdiction, but of law or fact which is a
mistake of judgment, the proper remedy should be appeal. Hence, if
Example: Writ of execution is ministerial on the part of the judge. there was no question of jurisdiction involved in the decision and
what was being questioned was merely the findings in the decision of
Why should you file a motion if it is ministerial? The motion is filed to whether or not the practice of the other party constitutes a violation
establish whether the judgment is already executory because it may of the agreement, the matter is a proper subject of appeal, not
be executory on the part of one party but not executory on the part of certiorari.
another party.
Filing of petition for certiorari does not interrupt the course of the
1. Certiorari (Section 1) principal action nor the running of the reglementary periods involved
in the proceeding, unless an application for a restraining order or a
Section 1. Petition for certiorari. writ of preliminary injunction to the appellate court is granted. Neither
When any tribunal, board or officer exercising judicial or quasi- does it interrupt the reglementary period for the filing of an answer
judicial functions has acted without or in excess of its or his nor the course of the case where there is no writ of injunction.
jurisdiction, or with grave abuse of discretion amounting to lack or
excess of jurisdiction, and there is no appeal, or any plain, speedy, In a summary proceeding, petitions for certiorari, prohibition or
and adequate remedy in the ordinary course of law, a person mandamus against an interlocutory order of the court are not
aggrieved thereby may file a verified petition in the proper court, allowed. Certiorari is not and cannot be made a substitute for an
alleging the facts with certainty and praying that judgment be appeal where the latter remedy is available but was lost through fault
rendered annulling or modifying the proceedings of such tribunal, or negligence. The remedy to obtain a reversal of judgment on the
board or officer, and granting such incidental reliefs as law and merits is appeal. This holds true even if the error ascribed to the
justice may require. lower court is its lack of jurisdiction over the subject matter, or the
The petition shall be accompanied by a certified true copy of the exercise of power in excess thereof, or grave abuse of discretion.
judgment, order or resolution subject thereof, copies of all pleadings The existence and availability of the right to appeal prohibits the
and documents relevant and pertinent thereto, and a sworn resort to certiorari because one of the requirements for certiorari is
certification of non-forum shopping as provided in the third paragraph that there is no appeal.
of section 3, Rule 46.
NOTES:

83 Aicka Singson
Notes in Remedial Law Review II
Provisional Remedies, Special Civil Actions, Special Proceedings and Evidence
School Year 2015-2016 REMEDIAL LAW REVIEW II - ATTY. BRONDIAL

Exceptions to the rule that certiorari is not available when the period such tribunal, board or officer, and granting such incidental
for appeal has lapsed and certiorari may still be invoked are the reliefs as law and justice may require.
following:
The petition shall be accompanied by a certified true copy of the
1) Appeal was lost without the appellant’s negligence; judgment, order or resolution subject thereof, copies of all
pleadings and documents relevant and pertinent thereto, and a
2) When public welfare and the advancement of public policy sworn certification of non-forum shopping.
dictates;
Requisites:
3) When the broader interest of justice so requires;
i. There must be a controversy;
4) When the writs issued are null and void; and ii. Respondent is exercising judicial or quasi-judicial
functions;
5) When the questioned order amounts to an oppressive exercise of
iii. Respondents acted without or in excess of its jurisdiction
judicial authority.
or with grave abuse of discretion amounting to lack of
a. GROUNDS jurisdiction;
iv. There must be no appeal or other plain, speedy and
Certiorari is an extraordinary writ ANNULLING OR MODIFYING adequate remedy.
the proceedings of a tribunal, board or officer exercising judicial
or quasi-judicial functions when such tribunal, board or officer Note: It shall be filed not later than 60 days from notice of the
has acted without or in excess of its or his jurisdiction, or with judgment, order or resolution. In case a timely MR or NT is filed, the
grave abuse of discretion amounting to lack or excess of petition shall be filed not later than 60 days from the denial of such
jurisdiction, there being no appeal or any other plain, speedy and motions.
adequate remedy in the ordinary course of law. (Sec. 1, Rule
65). CASES:

NOTE: The tribunal, board or officer has acted without, or in a. Ampil vs. Ombudsman (703 SCRA)
excess of jurisdiction or with abuse of discretion amounting to
Challenged in the petition for certiorari is the Resolution of the
lack or excess or jurisdiction.
Ombudsman in OMB-C-C-07-0444-J, dismissing the criminal complaint
b. REQUIREMENTS filed by Ampil against respondents and the Order denying Ampil’s
motion for reconsideration thereof. Ampil’s complaint charged
A person aggrieved thereby may file a verified petition in the respondents with Falsification of Public Documents under the Revised
proper court, alleging the facts with certainty and praying that Penal Code and violation of The Anti-Graft and Corrupt Practices Act,
judgment be rendered annulling or modifying the proceedings of as amended.

84 Aicka Singson
Notes in Remedial Law Review II
Provisional Remedies, Special Civil Actions, Special Proceedings and Evidence
School Year 2015-2016 REMEDIAL LAW REVIEW II - ATTY. BRONDIAL

The appeal by certiorari, on the other hand, assails the Decision of the Plainly, the Ombudsman has "full discretion," based on the attendant
Court of Appeals in CA G.R. SP No. 113171, which affirmed the Order facts and circumstances, to determine the existence of probable cause
dated 13 July 2009 of the Ombudsman in OMB-C-A-07-0474-J on the or the lack thereof. On this score, we have consistently hewed to the
administrative aspect of the mentioned criminal complaint for policy of non-interference with the Ombudsman’s exercise of its
Falsification and violation of Republic Act No. 3019 against the constitutionally mandated powers. The Ombudsman’s finding to
Registrar of Deeds, respondent Espenesin. Initially, the Ombudsman proceed or desist in the prosecution of a criminal case can only be
issued a Decision dated 30 April 2008, finding Espenesin guilty of assailed through certiorari proceedings before this Court on the ground
Simple Misconduct and meting on Espenesin the penalty of one (1) that such determination is tainted with grave abuse of discretion which
month suspension. On motion for reconsideration of Ampil, the contemplates an abuse so grave and so patent equivalent to lack or
Ombudsman favored Espenesin’s arguments in his Opposition, and excess of jurisdiction.
recalled the one-month suspension the Ombudsman had imposed on
the latter. However, on several occasions, we have interfered with the
Ombudsman’s discretion in determining probable cause:
Accordingly, the Supreme Court find grave abuse of discretion in the
Ombudsman’s incomplete disposition of Ampil’s complaint. (a) To afford protection to the constitutional rights of the accused;
(b) When necessary for the orderly administration of justice or to
That the Ombudsman is a constitutional officer duty bound to avoid oppression or multiplicity of actions;
"investigate on its own, or on complaint by any person, any act or (c) When there is a prejudicial question which is sub judice;
omission of any public official, employee, office or agency, when such (d) When the acts of the officer are without or in excess of authority;
act or omission appears to be illegal, unjust, improper, or inefficient" (e) Where the prosecution is under an invalid law, ordinance or
brooks no objection. The Ombudsman’s conduct of preliminary regulation;
investigation is both power and duty. Thus, the Ombudsman and his (f) When double jeopardy is clearly apparent;
Deputies, are constitutionalized as protectors of the people, who "shall (g) Where the court has no jurisdiction over the offense;
act promptly on complaints filed in any form or manner against public (h) Where it is a case of persecution rather than prosecution;
officials or employees of the government x x x, and shall, x x x notify the (i) Where the charges are manifestly false and motivated by the lust
complainants of the action taken and the result thereof." for vengeance.
While we agree with the Ombudsman’s disquisition that there is no
The raison d'être for its creation and endowment of broad investigative probable cause to indict respondents for Falsification of Public
authority is to insulate the Office of the Ombudsman from the long Documents under Article 171(6) of the Revised Penal Code, we are
tentacles of officialdom that are able to penetrate judges’ and fiscals’ puzzled why the Ombudsman completely glossed over Ampil’s
offices, and others involved in the prosecution of erring public officials, charge that respondents committed prohibited acts listed in Sections
and through the execution of official pressure and influence, quash, 3(a) and (e) of Republic Act No. 3019. Nowhere in the Resolution or
delay, or dismiss investigations into malfeasances and misfeasances in the Order denying reconsideration thereof did the Ombudsman
committed by public officers. tackle and resolve the issue of whether respondents violated the
particular provisions of Republic Act No. 3019.

85 Aicka Singson
Notes in Remedial Law Review II
Provisional Remedies, Special Civil Actions, Special Proceedings and Evidence
School Year 2015-2016 REMEDIAL LAW REVIEW II - ATTY. BRONDIAL

Curiously, the Ombudsman docketed Ampil’s complaint-affidavit as represented her unpaid water bills for the period June 1, 2002 to
one "for: Falsification of Public Documents and Violation of Sections September 30, 2005.
3(a) and (e) of Republic Act No. 3019, as amended."24 The
Ombudsman even prefaced the Resolution, thus: "this has reference Petitioner claimed that it was duly authorized to supply water to and
to the complaint filed by Oscar Ampil on 17 September 2007 against collect payment therefor from the homeowners of Regent Pearl
respondents, for Falsification of Public Documents and Violation of Subdivision, one of whom is respondent. Respondent only paid the
Sections 3, paragraphs (a) and (e) of Republic Act No. 3019, amount of P5,468.38, thus, leaving a balance of P23,111.71 which
otherwise known as the Anti-Graft and Corrupt Practices Act, as was left unpaid despite petitioner’s repeated demands.
amended."
In the interim, petitioner disconnected respondent’s water line for not
We are aware that the calibration of evidence to assess whether a paying the adjusted water charges.
prima facie graft case exists against respondents is a question of
fact. We have consistently held that the Supreme Court is not a trier
of facts, more so in the consideration of the extraordinary writ of The MTCC Ruling
certiorari where neither questions of fact nor law are entertained, but
only questions of lack or excess of jurisdiction or grave abuse of The MTCC rendered a Decision holding that since petitioner was
discretion. In this case, however, certiorari will lie, given that the issued a CPC by the National Water Resources Board (NWRB) only
Ombudsman made no finding at all on respondents possible liability on August 7, 2003, then, it can only charge respondent the agreed
for violation of Sections 3(a) and (e) of Republic Act No. 3019. flat rate of P75.00 per month prior thereto or the sum of P1,050.00
for the period June 1, 2002 to August 7, 2003. Thus, given that
We hasten to reiterate that we are only dealing herein with the respondent had made total payments equivalent to P1,685.99 for the
preliminary investigation aspect of this case. We do not adjudge same period, she should be considered to have fully paid petitioner.
respondents’ guilt or the lack thereof. Thus, it ordered respondent to pay petitioner the balance thereof,
equivalent to P1,200.00 with legal interest at the rate of 6% per
annum from date of receipt of the extrajudicial demand on October
b. A.L Ang Network Inc. vs Mondejar (714 SCRA) 14, 2010 until fully paid.
This is a direct recourse to the Court from the Decision and Order of Aggrieved, petitioner filed a petition for certiorari under Rule 65 of the
the RTC which dismissed, on the ground of improper remedy, Rules of Court before the RTC, ascribing grave abuse of discretion
petitioner A.L. Ang Network, Inc.'s (petitioner) petition for certiorari on the part of the MTCC.
from the Decision of the MTCC in a small claims case for sum of
money against respondent Emma Mondejar (respondent). The RTC Ruling

FACTS: Petitioner filed a complaint for sum of money under the The RTC issued a Decision dismissing the petition for certiorari,
Rule of Procedure for Small Claims Cases before the MTCC, finding that the said petition was only filed to circumvent the non-
seeking to collect from respondent the amount of P23,111.71 which appealable nature of small claims cases as provided under Section

86 Aicka Singson
Notes in Remedial Law Review II
Provisional Remedies, Special Civil Actions, Special Proceedings and Evidence
School Year 2015-2016 REMEDIAL LAW REVIEW II - ATTY. BRONDIAL

23 of the Rule of Procedure on Small Claims Cases. To this end, the appeal or any other plain, speedy and adequate remedy in the
RTC ruled that it cannot supplant the decision of the MTCC with ordinary course of law."
another decision directing respondent to pay petitioner a bigger sum
than that which has been awarded. In Jaca v. Davao Lumber Co., the Court ruled:

Petitioner moved for reconsideration but was denied. Hence, the Although Section 1, Rule 65 of the Rules of Court provides that the
instant petition. special civil action of certiorari may only be invoked when "there is
no appeal, nor any plain, speedy and adequate remedy in the course
ISSUE: WON the RTC erred in dismissing petitioner’s recourse of law," this rule is not without exception. The availability of the
under Rule 65 of the Rules of Court assailing the propriety of ordinary course of appeal does not constitute sufficient ground to
the MTCC Decision in the subject small claims case. prevent a party from making use of the extraordinary remedy of
certiorari where appeal is not an adequate remedy or equally
HELD: YES beneficial, speedy and sufficient. It is the inadequacy – not the mere
absence – of all other legal remedies and the danger of failure of
Section 23 of the Rule of Procedure for Small Claims Cases states
justice without the writ that usually determines the propriety of
that:
certiorari.
SEC. 23. Decision. — After the hearing, the court shall render its
The Court finds that petitioner filed the said petition before the proper
decision on the same day, based on the facts established by the
forum (i.e., the RTC). To be sure, the Court, the CA and the RTC
evidence (Form 13-SCC). The decision shall immediately be entered
have concurrent jurisdiction to issue a writ of certiorari. Such
by the Clerk of Court in the court docket for civil cases and a copy
concurrence of jurisdiction, however, does not give a party unbridled
thereof forthwith served on the parties. The decision shall be final
freedom to choose the venue of his action lest he ran afoul of the
and unappealable.
doctrine of hierarchy of courts. Instead, a becoming regard for
Considering the final nature of a small claims case decision under judicial hierarchy dictates that petitions for the issuance of writs of
the above-stated rule, the remedy of appeal is not allowed, and the certiorari against first level courts should be filed with the Regional
prevailing party may, thus, immediately move for its execution. Trial Court, and those against the latter, with the CA, before resort
Nevertheless, the proscription on appeals in small claims cases, may be had before the Court. This procedure is also in consonance
similar to other proceedings where appeal is not an available with Section 4, Rule 65 of the Rules of Court.
remedy, does not preclude the aggrieved party from filing a petition
Hence, considering that small claims cases are exclusively within the
for certiorari under Rule 65 of the Rules of Court. This general rule
jurisdiction of the MeTC, MTCC, MTC, and MCTC, certiorari petitions
has been enunciated in the case of Okada v. Security Pacific
assailing its dispositions should be filed before their corresponding
Assurance Corporation, wherein it was held that:
RTC. This petitioner complied with when it instituted its petition for
In a long line of cases, the Court has consistently ruled that "the certiorari before the RTC which, as previously mentioned, has
extraordinary writ of certiorari is always available where there is no jurisdiction over the same. In fine, the RTC erred in dismissing the
said petition on the ground that it was an improper remedy, and, as

87 Aicka Singson
Notes in Remedial Law Review II
Provisional Remedies, Special Civil Actions, Special Proceedings and Evidence
School Year 2015-2016 REMEDIAL LAW REVIEW II - ATTY. BRONDIAL

such, RTC Case No. 11-13833 must be reinstated and remanded


thereto for its proper disposition. Petitioner filed a petition for certiorari under Rule 65 of the 1997
Rules of Civil Procedure, as amended, before the CA, averring that
c. Maglalang vs. PAGCOR (712 SCRA) there is no evidence, much less factual and legal basis to support the
finding of guilt against him. Moreover, petitioner ascribed grave
Before this Court is a petition for review on certiorari under Rule 45 abuse of discretion amounting to lack or excess of jurisdiction to the
of the 1997 Rules of Civil Procedure, as amended, seeking the acts of PAGCOR in adjudging him guilty of the charge, in failing to
reversal of the Resolution dated September 30, 2009 issued by the observe the proper procedure in the rendition of its decision and in
Court of Appeals (CA) in CA-G.R. SP No. 110048, which outrightly imposing the harsh penalty of a 30-day suspension. Justifying his
dismissed the petition for certiorari filed by herein petitioner Mark recourse to the CA, petitioner explained that he did not appeal to the
Jerome S. Maglalang (petitioner). Also assailed is the appellate Civil Service Commission (CSC) because the penalty imposed on
court’s Resolution dated November 26, 2009 which denied him was only a 30-day suspension which is not within the CSC’s
petitioner’s motion for reconsideration. appellate jurisdiction. He also claimed that discourtesy in the
performance of official duties is classified as a light offense which is
Petitioner was a teller at the Casino Filipino in Angeles City, which punishable only by reprimand.
was operated by respondent Philippine Amusement and Gaming
Corporation (PAGCOR). CA outrightly dismissed the petition for certiorari for being premature
Petitioner received a Memorandum issued by the casino’s Branch as petitioner failed to exhaust administrative remedies before
Manager informing him that he was being charged with Discourtesy seeking recourse from the CA. Petitioner filed his Motion for
towards a casino customer and directing him to explain within 72 Reconsideration which the CA denied in the assailed Resolution.
hours upon receipt of the memorandum why he should not be Hence, this petition.
sanctioned or dismissed. Incompliance therewith, petitioner Petitioner claims that the CA clearly overlooked the applicable laws
submitted a letter-explanation. and jurisprudence that provide that when the penalty involved in an
administrative case is suspension for not more than 30 days, the
Petitioner received another Memorandum stating that the Board of CSC has no appellate jurisdiction over the said administrative case.
Directors of PAGCOR found him guilty of Discourtesy towards a Petitioner asserts that his case, involving a 30-day suspension
casino customer and imposed on him a 30-day suspension for this penalty, is not appealable to the CSC. Thus, he submits that his case
first offense. Aggrieved, petitioner filed a Motion for Reconsideration was properly brought before the CA via a petition for certiorari.
seeking a reversal of the board’s decision a Motion for Production,
praying that he be furnished with copies of documents relative to the Issue:
case including the recommendation of the investigating committee Was the CA correct in outrightly dismissing the petition for
and the Decision/Resolution of the Board supposedly containing the certiorari filed before it on the ground of non-exhaustion of
latter’s factual findings. In a letter-reply one Atty. Carlos R. Bautista, administrative remedies
Jr. who did not indicate his authority therein to represent PAGCOR,
denied the said motion.

88 Aicka Singson
Notes in Remedial Law Review II
Provisional Remedies, Special Civil Actions, Special Proceedings and Evidence
School Year 2015-2016 REMEDIAL LAW REVIEW II - ATTY. BRONDIAL

NO. Our ruling in Public Hearing Committee of the Laguna Lake indicating the urgency of judicial intervention, and unreasonable
Development Authority v. SM Prime Holdings, Inc. on the doctrine of delay would greatly prejudice the complainant; (12) where no
exhaustion of administrative remedies is instructive, to wit: administrative review is provided by law; (13) where the rule of
Under the doctrine of exhaustion of administrative remedies, before qualified political agency applies and (14) where the issue of non-
a party is allowed to seek the intervention of the court, he or she exhaustion of administrative remedies has been rendered moot.29
should have availed himself or herself of all the means of The case before us falls squarely under exception number 12 since
administrative processes afforded him or her. Hence, if resort to a the law per se provides no administrative review for administrative
remedy within the administrative machinery can still be made by cases whereby an employee like petitioner is covered by Civil
giving the administrative officer concerned every opportunity to Service law, rules and regulations and penalized with a suspension
decide on a matter that comes within his or her jurisdiction, then for not more than 30 days.
such remedy should be exhausted first before the court’s judicial
power can be sought. The premature invocation of the intervention of In sum, there being no appeal or any plain, speedy, and adequate
the court is fatal to one’s cause of action. The doctrine of exhaustion remedy in the ordinary course of law in view of petitioner’s allegation
of administrative remedies is based on practical and legal reasons. that PAGCOR has acted without or in excess of jurisdiction, or with
The availment of administrative remedy entails lesser expenses and grave abuse of discretion amounting to lack or excess of jurisdiction,
provides for a speedier disposition of controversies. Furthermore, the the CA’s outright dismissal of the petition for certiorarion the basis of
courts of justice, for reasons of comity and convenience, will shy non-exhaustion of administrative remedies is bereft of any legal
away from a dispute until the system of administrative redress has standing and should therefore be set aside.
been completed and complied with, so as to give the administrative
agency concerned every opportunity to correct its error and dispose Finally, as a rule, a petition for certiorari under Rule 65 is valid only
of the case. when the question involved is an error of jurisdiction, or when there
However, the doctrine of exhaustion of administrative remedies is not is grave abuse of discretion amounting to lack or excess of
absolute as it admits of the following exceptions: jurisdiction on the part of the court or tribunals exercising quasi-
(1) when there is a violation of due process; (2) when the issue judicial functions. Hence, courts exercising certiorari jurisdiction
involved is purely a legal question; (3) when the administrative action should refrain from reviewing factual assessments of the respondent
is patently illegal amounting to lack or excess of jurisdiction; (4) when court or agency. Occasionally, however, they are constrained to
there is estoppel on the part of the administrative agency concerned; wade into factual matters when the evidence on record does not
(5) when there is irreparable injury; (6) when the respondent is a support those factual findings; or when too much is concluded,
department secretary whose acts as an alter ego of the President inferred or deduced from the bare or incomplete facts appearing on
bears the implied and assumed approval of the latter; (7) when to record. Considering the circumstances and since this Court is not a
require exhaustion of administrative remedies would be trier of facts, remand of this case to the CA for its judicious resolution
unreasonable; (8) when it would amount to a nullification of a claim; is in order.
(9) when the subject matter is a private land in land case
proceedings; (10) when the rule does not provide a plain, speedy d. People vs. Castaneda (712 SCRA)
and adequate remedy, and (11) when there are circumstances

89 Aicka Singson
Notes in Remedial Law Review II
Provisional Remedies, Special Civil Actions, Special Proceedings and Evidence
School Year 2015-2016 REMEDIAL LAW REVIEW II - ATTY. BRONDIAL

This is a petition for certiorari under Rule 65 of the Rules of Court Hence, this petition for certiorari, ascribing grave abuse of discretion
seeking to review the March 26, 20131 and May 15, 20132 on the part of the CTA.
Resolutions of the CTA ordering the dismissal of the case against
the private respondents for violation of Section 36023 in relation to ISSUE: WON CTA gravely abused its discretion;
Sections 2503 and 2530 (f) (i) and 1, (3) (4) and (5) of the Tariff and
WON petition was timely filed
Customs Code of the Philippines, as amended, on the ground of
insufficiency of evidence. HELD: NO
FACTS: Private respondents were charged before the CTA for At the outset, it should be noted that the petition was filed beyond the
violation of Tariff and Customs Code of the Philippines. reglementary period for the filing thereof under Rule 65. The petition
itself stated that a copy of the May 15, 2013 Resolution was received
Garcia and VestidasJr.pleaded "Not Guilty" to the charge.
by the BOC two (2) days after its promulgation, or on May 17, 2013.
Thereafter, a preliminary conference was held followed by the pre-
Nonetheless, the RATS was only alerted by the developments in the
trial. Both the prosecution and the defense agreed to adopt the joint
case on July 24, 2013, when Atty. Danilo M. Campos Jr. (Atty.
stipulations of facts and issues entered in the course of the
Campos) received the July 15, 2013 Resolution of the CTA ordering
preliminary conference.
the entry of judgment in the case, considering that no appeal was
Thereafter, trial ensued. taken by any of the parties. According to Atty. Campos, it was only
on that occasion when he discovered the May 15, 2013 Resolution of
Subsequent to the presentation of witnesses, the prosecution filed its the CTA. Thus, it was prayed that the petition be given due course
Formal Offer of Evidence. despite its late filing.

Garcia and Vestidas, Jr. filed their Omnibus Motion to File Demurrer Section 4, Rule 65 of the 1997 Rules of Civil Procedure is explicit in
to Evidence with Leave of Court to Cancel Hearing, which was stating that certiorari should be instituted within a period of 60 days
granted by the CTA. Thereafter, they filed the Demurrer to Evidence, from notice of the judgment, order or resolution sought to be
claiming that the prosecution failed to prove their guilt beyond assailed. The 60-day period is inextendible to avoid any
reasonable doubt. unreasonable delay that would violate the constitutional rights of
parties to a speedy disposition of their case. While there are
Despite opposition, the CTA dismissed the case against Garcia and recognized exceptions to such strict observance, there should be an
Vestidas Jr. for failure of the prosecution to establish their guilt effort on the part of the party invoking liberality to advance a
beyond reasonable doubt. reasonable or meritorious explanation for his/her failure to comply
with the rules.
The prosecution filed its motion for reconsideration, but it was
denied, stressing, among others, that to grant it would place the In the case at bench, no convincing justification for the belated filing
accused in double jeopardy. of the petition was advanced to warrant the relaxation of the Rules.
Notably, the records show that the petition was filed only on August

90 Aicka Singson
Notes in Remedial Law Review II
Provisional Remedies, Special Civil Actions, Special Proceedings and Evidence
School Year 2015-2016 REMEDIAL LAW REVIEW II - ATTY. BRONDIAL

12, 2013, or almost a month late from the due date which fell on July evasion of a positive duty or a virtual refusal to perform a duty
16, 2013. To excuse this grave procedural lapse will not only be enjoined by law, or to act at all in contemplation of law, as where the
unfair to the other party, but it will also sanction a seeming power is exercised in an arbitrary and despotic manner by reason of
rudimentary attempt to circumvent standing rules of procedure. The passion and hostility. Here, the subject resolutions of the CTA have
late filing of the petition was borne out of the petitioner’s failure to been issued in accordance with the rules on evidence and existing
monitor incoming court processes that needed to be addressed by jurisprudence.
the office. Clearly, this is an admission of inefficiency, if not lack of
zeal, on the part of an office tasked to effectively curb smuggling e. UP Board of Regents vs. Ligot-Taylon (227 SCRA)
activities which rob the government of millions of revenue every year. In an effort to make the University of the Philippines (U.P.) truly the
university of the people, the U.P. administration conceptualized and
The display of patent violations of even the elementary rules leads implemented the socialized scheme of tuition fee payments through
the Court to suspect that the case against Garcia and Vestidas Jr. the Socialized Tuition Fee and Assistance Program (STFAP),
was doomed by design from the start. The failure to present the popularly known as the "Iskolar ng Bayan" program.
certified true copies of documentary evidence; the failure to Applicants are required to accomplish a questionnaire where, among
competently and properly identify the misdeclared goods; the failure others, they state the amount and source of the annual income of the
to identify the accused in court; and, worse, the failure to file this family, their real and personal properties and special circumstances
petition on time challenging a judgment of acquittal, are tell-tale signs from which the University may evaluate their financial status and
of a reluctant and subdued attitude in pursuing the case. This stance need on the basis of which they are categorized into brackets.
taken by the lawyers in government service rouses the Court’s From the early stages of its implementation, measures were adopted
vigilance against inefficiency in the administration of justice. to safeguard the integrity of the program. One such precautionary
measure was the inclusion as one of the punishable acts under
In any case, even if the Court decides to suspend the rules and Section 2 (a) of the Rules and Regulations on Student Conduct and
permit this recourse, the end result would remain the same. While a Discipline of the University the deliberate falsification or
judgment of acquittal in a criminal case may be assailed in a petition suppression/withholding of any material information required in the
for certiorari under Rule 65 of the Rules of Court, it must be shown application form.
that there was grave abuse of discretion amounting to lack or excess To further insure the integrity of the program, a random sampling
of jurisdiction or a denial of due process. In this case, a perusal of scheme of verification of data indicated in a student's application
the challenged resolutions of the CTA does not disclose any form is undertaken. Among those who applied for STFAP benefits for
indication of grave abuse of discretion on its part or denial of due School Year 1989-90 was Ramon P. Nadal, a student enrolled in the
process. The records are replete with indicators that the petitioner College of Law. The school conducted a home investigation. The
actively participated during the trial and, in fact, presented its offer of report for the investigation found discrepancies between the report
evidence and opposed the demurrer. and Nadal's application form. Forthwith, she and Bella M. Villanueva,
head of the Office of Scholarships and Student Services, presented
Grave abuse of discretion is defined as capricious or whimsical
the matter to the Diliman Committee on Scholarships and Financial
exercise of judgment as is equivalent to lack of jurisdiction. The
Assistance.
abuse of discretion must be patent and gross as to amount to an

91 Aicka Singson
Notes in Remedial Law Review II
Provisional Remedies, Special Civil Actions, Special Proceedings and Evidence
School Year 2015-2016 REMEDIAL LAW REVIEW II - ATTY. BRONDIAL

Nadal was required "to pay back the equivalent amount of full school immediately, plus reimbursement of all benefits received from the
fees" with "interest based on current commercial rates." Failure to STFAP, with legal interest." The BOR also decided against giving
settle his account would mean the suspension of his registration Nadal, a certification of good moral character.
privileges and the withholding of clearance and transcript of records. Nadal filed with the Regional Trial Court of Quezon City a petition for
He was also warned that his case might be referred to the Student mandamus with preliminary injunction and prayer for a temporary
Disciplinary Tribunal for further investigation. 3 restraining order. RTC issued a writ of preliminary injunction be
Noting further discrepancies between Nadal's application form and issued restraining the respondents, their officers, agent(s),
the certification, the U.P. charged Nadal before the Student representatives, and all persons acting in their behalf, from further
Disciplinary Tribunal (SDT) acts of willfully withholding information is proceeding with SDT Case No. 91-026, and from suspending
tantamount to acts of dishonesty in relation to his studies, in violation petitioner, upon the latter's filing a bond in the amount of P3,000.00.
of paragraph (a), Section 2, of the Rules and Regulations on Student Dispensing with the filing of a motion for reconsideration, the
Conduct and Discipline. petitioners filed the instant petition for certiorari and prohibition with
After hearing, the SDT rendered a decision in SDT Case No. 91-026 prayer for the issuance of an injunction or temporary restraining
exculpating Nadal of the charge of deliberately withholding in his order, raising whether or not the respondent judge gravely
STFAP application form information that he was maintaining a abused her discretion in issuing the May 29, 1993 writ of
Toyota Corolla car, but finding him guilty of "wilfully and deliberately preliminary injunction thereby preventing the BOR from
withholding information about the income of his mother, who is living implementing the suspension penalty it had imposed on Nadal.
abroad, in support of the studies of his brothers Antonio and YES. The Court finds that the lower court gravely abused its
Federico, which is tantamount to acts of dishonesty in relation to his discretion in issuing the writ of preliminary injunction of May 29,
studies in violation of paragraph [a], Section 2 of the Rules [now 1993. The issuance of the said writ was based on the lower court's
covered by paragraph (i), Section 2 of the Rules, as amended 25 finding that the implementation of the disciplinary sanction of
June 1992]." As such, the SDT imposed upon Nadal the penalty of suspension on Nadal "would work injustice to the petitioner as it
expulsion from the University and required him to reimburse all would delay him in finishing his course, and consequently, in getting
STFAP benefits he had received but if he does not voluntarily make a decent and good paying job." Sadly, such a ruling considers only
reimbursement, it shall be "effected by the University thru outside the situation of Nadal without taking into account the circumstances
legal action." 8 clearly of his own making, which led him into such a predicament.
The SDT decision was thereafter automatically elevated to the More importantly, it has completely disregarded the overriding issue
Executive Committee of U.P. Diliman for review pursuant to Sec. 20 of academic freedom which provides more than ample justification
of the U.P. Rules on Student Conduct and Discipline. Committee, for the imposition of a disciplinary sanction upon an erring student of
affirmed the decision of the SDT; whereupon, Nadal appealed to the an institution of higher learning.
Board of Regents (BOR). BOR affirmed the decision of the SDT but From the foregoing arguments, it is clear that the lower court should
because "the Board was willing to grant a degree of compassion to have restrained itself from assuming jurisdiction over the petition filed
the appellant in view of the alleged status and predicament of the by Nadal. Mandamus is never issued in doubtful cases, a showing of
mother as an immigrant 'TNT' in the United States," the penalty was a clear and certain right on the part of the petitioner being required. It
modified "from Expulsion to One Year- Suspension, effective

92 Aicka Singson
Notes in Remedial Law Review II
Provisional Remedies, Special Civil Actions, Special Proceedings and Evidence
School Year 2015-2016 REMEDIAL LAW REVIEW II - ATTY. BRONDIAL

is of no avail against an official or government agency whose duty members of the Malacanang Homeowners Association, Inc., the
requires the exercise of discretion or judgment. present bona fide occupants thereof."
Hence, by issuing the writ of preliminary injunction, the lower court
dared to tread upon legally forbidden grounds. For, by virtue of the On September 14, 1973-a year almost to the day after the
writ, the University's exercise of academic freedom was peremptorily declaration of martial law Mr. Ferdinand Marcos, then president of
curtailed. Moreover, the door was flung wide open for Nadal to do the country, invoking his emergency powers, issued PD No. 293 with
exactly what the decision of the BOR prohibited him from doing and immediate effect. The decree invalidated inter alia the title of the
that is, to violate the suspension order by enrolling for the first Tuasons' vendor, Carmel, which had earlier purchased from the
semester of 1993-1994. It must have been with consternation that Government the land it had subsequently subdivided into several lots
the University officials helplessly watching him complete his for sale to the public (the Tuasons being among the buyers). The
academic requirements for taking the Bar. In the event that he be land bought by Carmel was part of the Tala Estate (one of the so-
allowed to continue with his studies he would, in effect render moot called "Friar Lands").
and academic the disciplinary sanction of suspension legally
Said Presidential Decree No. 293 made the finding that Carmel had
imposed upon him by the BOR's final decision of March 29, 1993.
failed to complete payment of the price.
What is to prevent other aspirants for STFAP scholarships from
misleading the University authorities by misrepresenting certain facts Upon this adjudgment, Mr. Marcos invalidated the titles of Carmel
or as in instant case, withholding vital information and stating Farms, Inc. and all those derived therefrom, and declared as
downright falsehoods, in their application forms with impunity? Not aforestated "the members of the Malacanang Homeowners
only would this undermine the authority of the U.P. to discipline its Association, Inc. the present bona fide occupants" of the lots which,
students who violated the rules and regulations of the institution but, in consequence, thereby became open to them for "disposition and
more importantly, subvert the very concept and lofty intent to give sale.
financial assistance to poor but deserving students through the
STFAP which, incidentally, has not ceased refining and modifying its On the strength of this presidential decree, the Register of Deeds of
operations. Caloocan City caused the inscription on the Tuasons' title.
f. Tuazon vs. RD of Caloocan (157 SCRA)
The Tuason Spouses thereupon filed with this Court a petition for
Farms, Inc. (Carmel) a piece of land in the latter's subdivision in certiorari assailing the Marcos decree as an arbitrary measure which
Barrio Makatipo, Caloocan City. In virtue of this sale, Carmel's deprived them of their property in favor of a selected group, in
Torrens title (No. 64007) over the lot was cancelled and a new one violation not only of the constitutional provisions on due process and
(No. 8314) issued in the name of the Tuasons. The Tuasons took eminent domain but also of the provisions of the Land Registration
possession of their property. Act on the indefeasibility of Torrens titles; and they prayed that the
Register of Deeds be directed to cancel the derogatory inscription on
Some eight (8) years thereafter, their land and the other lots in the their title and restore its efficacy, or in the alternative, that they be
subdivision had been "declared open for disposition and sale to the compensated for the loss from the Assurance Fund.

93 Aicka Singson
Notes in Remedial Law Review II
Provisional Remedies, Special Civil Actions, Special Proceedings and Evidence
School Year 2015-2016 REMEDIAL LAW REVIEW II - ATTY. BRONDIAL

ISSUE: WON certiorari applies against Marcos who, through his These acts may thus be properly struck down by the writ of certiorari,
P.D, nullified a sale because done by an officer in the performance of what in essence is
a judicial function, if it be shown that the acts were done without or in
HELD: YES. excess of jurisdiction, or with grave abuse of discretion. Since Mr.
Marcos was never vested with judicial power, such power, as
It is true that the extraodinary writ of certiorari may properly issue to
everyone knows, being vested in the Supreme Court and such
nullify only judicial or quasi-judicial acts, unlike the writ of prohibition
inferior courts as may be established by law — the judicial acts done
which may be directed against acts either judicial or ministerial.
by him were in the circumstances indisputably perpetrated without
Section 1, Rule 65 of the Rules of Court deals with the writ of
jurisdiction. The acts were completely alien to his office as chief
certiorari in relation to "any tribunal, board or officer exercising
executive, and utterly beyond the permissible scope of the legislative
judicial functions, while Section 2 of the same Rule treats of the writ
power that he had assumed as head of the martial law regime.
of prohibition in relation to "proceedings of any tribunal, corporation,
board, or person ... exercising functions judicial or ministerial." But Moreover, he had assumed to exercise power — i.e. determined the
the petition will be shown upon analysis to be in reality directed relevant facts and applied the law thereto without a trial at which all
against an unlawful exercise of judicial power. interested parties were accorded the opportunity to adduce evidence
to furnish the basis for a determination of the facts material to the
The decree reveals that Mr. Marcos exercised an obviously judicial
controversy. He made the finding ostensibly on the basis of "the
function. He made a determination of facts, and applied the law to
records of the Bureau of Lands." The adjudication was patently and
those facts, declaring what the legal rights of the parties were in the
grossly violative of the right to due process to which the petitioners
premises. These acts essentially constitute a judicial function, or an
are entitled in virtue of the Constitution. Mr. Marcos, in other words,
exercise of jurisdiction — which is the power and authority to hear or
not only arrogated unto himself a power never granted to him by the
try and decide or determine a cause. He adjudged it to be an
Constitution or the laws but had in addition exercised it
established fact that neither the original purchasers nor their
unconstitutionally.
subsequent transferees have made full payment of all installments of
the purchase money and interest on the lots claimed by Carmel In any event, this Court has it in its power to treat the petition for
Farms, Inc., including those on which the dwellings of the members certiorari as one for prohibition if the averments of the former
of the Association of homeowners stand." And applying the law to sufficiently made out a case for the latter. 13 Considered in this wise,
that situation, he made the adjudication that "title to said land has it will also appear that an executive officer had acted without
remained with the Government, and the land now occupied by the jurisdiction — exercised judicial power not granted to him by the
members of said association has never ceased to form part of the Constitution or the laws — and had furthermore performed the act in
property of the Republic of the Philippines," and that 'any and all acts violation of the constitutional rights of the parties thereby affected.
affecting said land and purporting to segregate it from the said The Court will grant such relief as may be proper and efficacious in
property of the Republic were null and void ab initio as against the the premises even if not specifically sought or set out in the prayer of
law and public policy. the appropriate pleading, the permissible relief being determined
after all not by the prayer but by the basic averments of the parties'
pleadings.

94 Aicka Singson
Notes in Remedial Law Review II
Provisional Remedies, Special Civil Actions, Special Proceedings and Evidence
School Year 2015-2016 REMEDIAL LAW REVIEW II - ATTY. BRONDIAL

Separate Opinions pleadings and documents relevant and pertinent thereto, and a
sworn certification of non-forum shopping as provided in the third
TEEHANKEE, C.J., concurring: paragraph of section 3, Rule 46.
1. Grounds
These arbitrary, capricious and oppressive decrees, tailored to suit
the deposed President's every wish and whim, were the product of
The tribunal, corporation, board or person must have acted
unrestrained power, as the deposed President took over the entire
without or in excess of jurisdiction or with grave abuse of
government with the imposition of martial law in September, 1972. discretion amounting to lack of jurisdiction;
FELICIANO, J., concurring:
There is no appeal or any plain, speedy and adequate remedy in
the ordinary course of law.
Presidential Decree No. 293 is constitutionally offensive for still
another reason: it constitutes a bill of attainder, prohibited not only 2. Requirements
under the 1935 and 1987 Constitutions but also under the 1973
Constitution. Bills of attainder are an ancient instrument of tyranny.
A person aggrieved thereby may file a verified petition in the
proper court, alleging the facts with certainty and praying that
2. PROHIBITION (Section 2)
judgment be rendered commanding the respondent to desist from
It is a remedy to prevent inferior courts, corporations, boards or
further proceedings in the action or matter specified therein, or
persons from usurping or exercising a jurisdiction or power which
otherwise granting such incidental reliefs as law and justice may
they have not been vested by law
require.
Sec. 2. Petition for prohibition. The petition shall likewise be accompanied by a certified true
copy of the judgment, order or resolution subject thereof, copies
When the proceedings of any tribunal, corporation, board, officer or of all pleadings and documents relevant and pertinent
person, whether exercising judicial, quasi-judicial or ministerial thereto, and a sworn certification of non-forum shopping.
functions, are without or in excess of its or his jurisdiction, or with
grave abuse of discretion amounting to lack or excess of jurisdiction,
and there is no appeal or any other plain, speedy, and adequate
Requisites:
remedy in the ordinary course of law, a person aggrieved thereby
i. There must be a controversy;
may file a verified petition in the proper court, alleging the facts with
ii. Respondent is exercising judicial or quasi-judicial functions or
certainty and praying that judgment be rendered commanding the
ministerial duty;
respondent to desist from further proceedings in the action or matter
iii. Respondents acted without or in excess of its jurisdiction or with
specified therein, or otherwise granting such incidental reliefs as law
grave abuse of discretion amounting to lack of jurisdiction;
and justice may require.
iv. There must be no appeal or other plain, speedy and adequate
The petition shall likewise be accompanied by a certified true copy of remedy.
the judgment, order or resolution subject thereof, copies of all

95 Aicka Singson
Notes in Remedial Law Review II
Provisional Remedies, Special Civil Actions, Special Proceedings and Evidence
School Year 2015-2016 REMEDIAL LAW REVIEW II - ATTY. BRONDIAL

Function: advance findings noted during the said examination. Board (MB)
It is a preventive remedy. Its function is to restrain the doing of some issued Resolution No. 1255, placing ECBI under Prompt Corrective
act about to be done. It is not intended to provide a remedy for acts Action (PCA) framework because of the following serious findings and
already accomplished. If the thing be already done, the writ of supervisory concerns noted during the general examination: 1]
prohibition cannot undo it. negative capital of 14.674 million and capital adequacy ratio of
negative 18.42%; 2] CAMEL (Capital Asset Management Earnings
Note: Prohibition and not mandamus is the correct remedy where a Liquidity) composite rating of "2" with a Management component rating
motion to dismiss is wrongfully denied. of "1"; and 3] serious supervisory concerns particularly on activities
deemed unsafe or unsound. Vivas claimed that the BSP took the
a. Vivas vs. Monetary Board of BSP (703 SCRA) above courses of action due to the joint influence exerted by a certain
hostile shareholder and a former BSP examiner.
This is a petition for prohibition with prayer for the issuance of a status
quo ante order or writ of preliminary injunction ordering the Vivas moved for a reconsideration on the grounds of non-observance
respondents to desist from closing EuroCredit Community Bank, of due process and arbitrariness. The ISD II, on several instances, had
Incorporated (ECBI) and from pursuing the receivership thereof. The invited the BOD of ECBI to discuss matters pertaining to the placement
petition likewise prays that the management and operation of ECBI be of the bank under PCA framework and other supervisory concerns
restored to its Board of Directors (BOD) and its officers. before making the appropriate recommendations to the MB. The
proposed meeting, however, did not materialize due to postponements
Record shows that the corporate life of RBFI expired on May 31, 2005 sought by Vivas.
Notwithstanding, petitioner Alfeo D. Vivas (Vivas) and his principals
acquired the controlling interest in RBFI sometime in January 2006. At The MB, on the other hand, posited that ECBI unjustly refused to allow
the initiative of Vivas and the new management team, an internal audit the BSP examiners from examining and inspecting its books and
was conducted on RBFI and results thereof highlighted the dismal records, in violation of Sections 25 and 34 of R.A. No. 7653. BSP
operation of the rural bank. In view of those findings, certain measures informed ECBI that it was already due for another annual examination
calculated to revitalize the bank were allegedly introduced. Bangko and that the pendency of its appeal before the MB would not prevent
Sentral ng Pilipinas (BSP) issued the Certificate of Authority extending the BSP from conducting another one.
the corporate life of RBFI for another fifty (50) years. The BSP also
approved the change of its corporate name to EuroCredit Community In view of ECBI’s refusal to comply with the required examination, the
Bank, Incorporated. MB issued Resolution imposing monetary penalty/fine on ECBI, and
referred the matter to the Office of the Special Investigation (OSI) for
Pursuant to The New Central Bank Act, the Integrated Supervision the filing of appropriate legal action. The BSP also wrote a letter,
Department II (ISD II) of the BSP conducted a general examination on advising ECBI to comply with MB Resolution No. 771, which
ECBI. Shortly after the completion of the general examination, an exit essentially required the bank to follow its directives. ISD II reiterated its
conference was held at the BSP during which the BSP officials and demand upon the ECBI BOD to allow the BSP examiners to conduct a
examiners apprised Vivas, the Chairman and President of ECBI, as general examination.
well as the other bank officers and members of its BOD, of the

96 Aicka Singson
Notes in Remedial Law Review II
Provisional Remedies, Special Civil Actions, Special Proceedings and Evidence
School Year 2015-2016 REMEDIAL LAW REVIEW II - ATTY. BRONDIAL

MB issued Resolution approving the issuance of a cease and desist The actions of the Monetary Board taken under this section or under
order against ECBI, which enjoined it from pursuing certain acts and Section 29 of this Act shall be final and executory, and may not be
transactions that were considered unsafe or unsound banking restrained or set aside by the court except on petition for certiorari on
practices, and from doing such other acts or transactions constituting the ground that the action taken was in excess of jurisdiction or with
fraud or might result in the dissipation of its assets. such grave abuse of discretion as to amount to lack or excess of
jurisdiction. The petition for certiorari may only be filed by the
OSI filed with the Department of Justice (DOJ) a complaint for Estafa stockholders of record representing the majority of the capital stock
Through Falsification of Commercial Documents against certain within ten (10) days from receipt by the board of directors of the
officials and employees of ECBI. Meanwhile, the MB issued Resolution institution of the order directing receivership, liquidation or
No. 1164, denying the appeal of ECBI from Resolution No. 1255 which conservatorship.
placed it under PCA framework. Later, the BSP officials and examiners
met with the representatives of ECBI, including Vivas, and discussed Prohibition is already unavailing.
their findings.
Granting that a petition for prohibition is allowed, it is already an
MB issued Resolution No. 276 placing ECBI under receivership in ineffective remedy under the circumstances obtaining. Prohibition or a
accordance with the recommendation of the ISD II which prohibits the "writ of prohibition" is that process by which a superior court prevents
Eurocredit Bank from doing business in the Philippines and to place its inferior courts, tribunals, officers, or persons from usurping or
assets and affairs under receivership and designating the Philippine exercising a jurisdiction with which they have not been vested by law,
Deposit Insurance Corporation as Receiver of the bank. and confines them to the exercise of those powers legally conferred.
Its office is to restrain subordinate courts, tribunals or persons from
Vivas filed this petition for prohibition before this Court, ascribing grave exercising jurisdiction over matters not within its cognizance or
abuse of discretion to the MB for prohibiting ECBI from continuing its exceeding its jurisdiction in matters of which it has cognizance. In our
banking business and for placing it under receivership. jurisdiction, the rule on prohibition is enshrined in Section 2, Rule 65 of
the Rules on Civil Procedure, to wit:
Issue: WON the petition for prohibition is proper
Sec. 2. Petition for prohibition - When the proceedings of any tribunal,
NO. Vivas Availed of the Wrong Remedy
corporation, board, officer or person, whether exercising judicial, quasi-
The MB issued Resolution No. 276, dated March 4, 2010, in the judicial or ministerial functions, are without or in excess of its or his
exercise of its power under R.A. No. 7653. Under Section 30 thereof, jurisdiction, or with grave abuse of discretion amounting to lack or
any act of the MB placing a bank under conservatorship, receivership excess of jurisdiction, and there is no appeal or any other plain,
or liquidation may not be restrained or set aside except on a petition for speedy, and adequate remedy in the ordinary course of law, a person
certiorari. Pertinent portions of R.A. 7653 read: aggrieved thereby may file a verified petition in the proper court,
alleging the facts with certainty and praying that the judgment be
Section 30. – rendered commanding the respondent to desist from further
proceedings in the action or matter specified therein, or otherwise
granting such incidental reliefs as the law and justice require.

97 Aicka Singson
Notes in Remedial Law Review II
Provisional Remedies, Special Civil Actions, Special Proceedings and Evidence
School Year 2015-2016 REMEDIAL LAW REVIEW II - ATTY. BRONDIAL

x x x x. petitioner Corales renewed the appointment of petitioner Dr.


Angeles. But, on these times, the Sangguniang Bayan per
Indeed, prohibition is a preventive remedy seeking that a judgment be Resolution No. 2001-0787 dated 12 July 2001 and 26 subsequent
rendered which would direct the defendant to desist from continuing Resolutions, disapproved petitioner Dr. Angeles’ appointment on
with the commission of an act perceived to be illegal. As a rule, the the ground of nepotism, as well as the latter’s purported unfitness
proper function of a writ of prohibition is to prevent the doing of an act and unsatisfactory performance. Even so, petitioner Dr. Angeles
which is about to be done. It is not intended to provide a remedy for continued to discharge the functions and duties of a Municipal
acts already accomplished. Administrator for which he received an annual salary of
P210,012.00.
Though couched in imprecise terms, this petition for prohibition
apparently seeks to prevent the acts of closing of ECBI and placing it Following an audit on various local disbursements, Andal, the
under receivership. Resolution No. 276, however, had already been Provincial State Auditor of Laguna, issued an Audit Observation
issued by the MB and the closure of ECBI and its placement under Memorandum (AOM) addressed to petitioner Corales who was
receivership by the PDIC were already accomplished. Apparently, the asked to comment/reply.
remedy of prohibition is no longer appropriate. Settled is the rule that
prohibition does not lie to restrain an act that is already a fait accompli. Instead of submitting his comment/reply thereon, petitioner Corales,
together with petitioner Dr. Angeles, opted to file a Petition for
The petition shall be filed in the Supreme Court or, if it relates to the Prohibition and Mandamus against Andal and the then members of
acts or omissions of a lower court or of a corporation, board, officer or the Sangguniang Bayan before the RTC of San Pablo City, Laguna.
person, in the Regional Trial Court exercising jurisdiction over the Petitioners sought, by way of prohibition, to require the Office of the
territorial area as defined by the Supreme Court. It may also be filed in Provincial Auditor, through Andal, to recall its AOM and to eventually
the Court of Appeals whether or not the same is in aid of its appellate desist from collecting reimbursement from petitioner Corales for the
jurisdiction, or in the Sandiganbayan if it is in aid of its appellate salaries paid to and received by petitioner Dr. Angeles for the latter’s
jurisdiction. If it involves the acts or omissions of a quasi-judicial services as Municipal Administrator. Petitioners similarly sought, by
agency, unless otherwise provided by law or these Rules, the petition way of mandamus, to compel the then members of the Sangguniang
shall be filed in and cognizable only by the Court of Appeals. Bayan, as a collegial body, to recall its Resolutions denying
confirmation to petitioner Dr. Angeles’ appointment as Municipal
b. Corales vs. Republic (703 SCRA)
Administrator and in their stead to confirm the validity and legitimacy
FACTS: Petitioner Corales was the duly elected Municipal Mayor
of such appointment.
of Nagcarlan, Laguna for three (3) consecutive terms, 1998, 2001
and 2004 elections. In his first term as local chief executive, In its turn, the OSG, on Andal’s behalf, who was impleaded in his
petitioner Corales appointed petitioner Dr. Angeles to the position official capacity, filed a Motion to Dismiss petitioners’ Petition for
of Municipal Administrator, whose appointment was unanimously Prohibition and Mandamus grounded on lack of cause of action,
approved by the Sangguniang Bayan of Nagcarlan, Laguna prematurity and non-exhaustion of administrative remedies.
(Sangguniang Bayan) per Resolution No. 98-646 dated 22 July
1998. During his second and third terms as municipal mayor,

98 Aicka Singson
Notes in Remedial Law Review II
Provisional Remedies, Special Civil Actions, Special Proceedings and Evidence
School Year 2015-2016 REMEDIAL LAW REVIEW II - ATTY. BRONDIAL

The trial court denied the said Motion to Dismiss on the ground that As previously stated, petitioners’ action for prohibition was
Andal was merely a nominal party. The subsequent MR was also premature. The audit investigative process was still in its initial
denied. phase. There was yet no Notice of Disallowance issued. And, even
granting that the AOM issued to petitioner Corales is already
Respondent Republic, as represented by COA, as represented by equivalent to an order, decision or resolution of the Auditor or that
Andal, consequently filed a Petition for Certiorari with the CA such AOM is already tantamount to a directive for petitioner Corales
ascribing grave abuse of discretion amounting to lack or excess of to reimburse the salaries paid to petitioner Dr. Angeles, still, the
jurisdiction on the part of the trial court in rendering the Orders as it action for prohibition is premature since there are still many
unjustly denied respondent’s right to actively prosecute the case administrative remedies available to petitioners to contest the said
through a mere declaration that it was a nominal party despite a AOM. Section 1, Rule V of the 1997 Revised Rules of Procedure of
clear showing that the Petition for Prohibition referred to the the COA, provides: “an aggrieved party may appeal from an order or
respondent as a real party in interest. decision or ruling rendered by the Auditor embodied in a report,
memorandum, letter, notice of disallowances and charges,
The CA rendered its now assailed Decision granting respondent’s
Certificate of Settlement and Balances, to the Director who has
Petition for Certiorari, thereby annulling and setting aside the RTC
jurisdiction over the agency under audit.” From the final order or
Orders and, accordingly, dismissing petitioners’ Petition for
decision of the Director, an aggrieved party may appeal to the
Prohibition with the court a quo.
Commission proper. It is the decision or resolution of the
Petitioners moved for its reconsideration but it was denied for lack of Commission proper which can be appealed to this Court.
merit.
Clearly, petitioners have all the remedies available to them at the
ISSUE: WON CA erred in dismissing petitioner’s suit for administrative level but they failed to exhaust the same and instead,
prohibition immediately sought judicial intervention. Otherwise stated, the
auditing process has just begun but the petitioners already thwarted
HELD: NO the same by immediately filing a Petition for Prohibition.

Prohibition, being a preventive remedy to seek a judgment ordering Moreover, courts have accorded respect for the specialized ability of
the defendant to desist from continuing with the commission of an other agencies of government to deal with the issues within their
act perceived to be illegal, may only be resorted to when there is “no respective specializations prior to any court intervention. The Court
appeal or any other plain, speedy, and adequate remedy in the has reasoned thus:
ordinary course of law.”
We have consistently declared that the doctrine of exhaustion of
In this case, petitioners insist that it is no longer necessary to administrative remedies is a cornerstone of our judicial system. The
exhaust administrative remedies considering that there is no appeal thrust of the rule is that courts must allow administrative agencies to
or any other plain, speedy and appropriate remedial measure to carry out their functions and discharge their responsibilities within the
assail the imposition under the AOM aside from an action for specialized areas of their respective competence. The rationale for
prohibition. this doctrine is obvious. It entails lesser expenses and provides for

99 Aicka Singson
Notes in Remedial Law Review II
Provisional Remedies, Special Civil Actions, Special Proceedings and Evidence
School Year 2015-2016 REMEDIAL LAW REVIEW II - ATTY. BRONDIAL

the speedier resolution of controversies. Comity and convenience injunction, preliminary or final, to enjoin or restrain a criminal
also impel courts of justice to shy away from a dispute until the prosecution; (3) public respondent Montero was duly authorized by
system of administrative redress has been completed. the Secretary of Justice to conduct the preliminary investigation and,
if the evidence so warranted, to file the corresponding informations
c. Tan vs. CA (524 SCRA) relative to the subject criminal cases; (4) petitioners failed to prove
that public respondents acted with grave abuse of discretion; and (5)
(King) charged Roderick Lim-Go, Lucy Go, Nelson Go, John Doe
petitioners’ claims contesting the public prosecutor’s finding of
and Peter Doe with violation of Batas Pambansa Bilang 22 (B.P. 22)
probable cause are matters of defense that should be threshed out
and Estafa involving two checks.
during the trial of the criminal cases and not through the
All the accused, except Roderick Go, submitted their counter- extraordinary remedy of prohibition.
affidavits. In their Joint Counter-Affidavit, petitioners allege that only
After their motion for reconsideration was denied, petitioners
Roderick Go could be held liable for the bouncing checks
interposed the instant petition raising nine issues revolving around
considering that he alone issued the same. Thus petitioners claim
the factual and legal bases of the finding of probable cause for estafa
that the criminal cases filed against them were an afterthought and
against them as well as the authority of public respondent Montero to
prayed that the same be dismissed.
file the subject criminal cases with the trial court
On November 11, 2002, five informations for estafa under Article
At the outset, it must be stressed that petitioners are asking us to
315, 2(a) of the Revised Penal Code were filed against Roderick L.
review the Decision of the Court of Appeals which dismissed their
Go, Grace Tan-Go, Go Tong Go, Lucy Go, Nelson Go, and herein
petition for prohibition.
petitioners. From the above-quoted adverse Resolution of public
respondent Montero, only Roderick Go and Grace Tan-Go Issue: Whether resort to the extraordinary remedy of
separately appealed to the Secretary of Justice. prohibition was proper

Petitioners filed a Petition for Prohibition and Injunction with NO. Basic is the rule that the writ of prohibition is an extraordinary
Preliminary Injunction and Prayer for Temporary Restraining Order remedy to prevent the unlawful and oppressive exercise of legal
before the Court of Appeals. They sought to restrain the trial court authority and to provide for a fair and orderly administration of
from proceeding with the subject criminal cases against them and justice. It is available only when there is no appeal or any plain,
prayed that the same be dismissed. speedy and adequate remedy in the ordinary course of law, and
when the proceedings are done without or in excess of jurisdiction or
Court of Appeals issued the assailed Decision dismissing the petition
with grave abuse of discretion. The petitioner must allege in his
for lack of merit. It found that (1) petitioners failed to avail
petition and establish facts to show that any other existing remedy is
themselves of other plain, speedy and adequate remedies to
not speedy or adequate. A remedy is plain, speedy and adequate if it
challenge the public prosecutor’s finding of probable cause; (2) the
will promptly relieve the petitioner from the injurious effects of that
petition failed to establish that it falls under any of the exceptions to
judgment and the acts of the tribunal or inferior court. Further, the
the general rule that the court will not issue writs of prohibition or
writ will not lie to correct errors of judgment but only errors of

100 Aicka Singson


Notes in Remedial Law Review II
Provisional Remedies, Special Civil Actions, Special Proceedings and Evidence
School Year 2015-2016 REMEDIAL LAW REVIEW II - ATTY. BRONDIAL

jurisdiction. As long as the tribunal acts within its jurisdiction, any found, among others, probable cause against them for estafa. Only
alleged errors committed in the exercise of its discretion will amount co-accused Grace Tan-Go and Roderick Go separately and timely
to nothing more than mere errors of judgment which are correctible appealed to the Secretary of Justice. Then Secretary of Justice
by a timely appeal. In determining whether a tribunal acted in grave Simeon A. Datumanong subsequently issued a Resolution dated
abuse of discretion, mere abuse of discretion is not enough. There December 23, 2003 granting Grace Tan-Go’s petition for review.
must be grave abuse of discretion as where the tribunal exercised its The aforesaid Resolution was, likewise, favorable to petitioners’
power in an arbitrary or despotic manner, by reason of passion or cause and ordered, among others, the withdrawal of the informations
personal hostility, and it must be so patent or gross as would amount for estafa against them.
to an evasion, or virtual refusal to perform the duty enjoined, or to act
in contemplation of law. 3. MANDAMUS (Section 3)
Sec. 3. Petition for mandamus.
In the case at bar, petitioners contend that there was no appeal or When any tribunal, corporation, board, officer or person unlawfully
other plain, speedy or adequate remedy available in the ordinary neglects the performance of an act which the law specifically enjoins
course of law because they were prevented by the trial court from as a duty resulting from an office, trust, or station, or unlawfully
appealing public respondent Montero’s Joint Resolution dated excludes another from the use and enjoyment of a right or office to
November 8, 2002 which found, among others, probable cause for which such other is entitled, and there is no other plain, speedy and
estafa against them. adequate remedy in the ordinary course of law, the person aggrieved
thereby may file a verified petition in the proper court, alleging the
In fine, the arguments raised in their petition for prohibition facts with certainty and praying that judgment be rendered
ineluctably shows that petitioners are principally questioning the commanding the respondent, immediately or at some other time to
factual and legal bases of the finding of probable cause against be specified by the court, to do the act required to be done to protect
them. This is but a veiled attempt to litigate issues which should the rights of the petitioner, and to pay the damages sustained by the
have been timely appealed to the Secretary of Justice via a petition petitioner by reason of the wrongful acts of the respondent.
for review. However, petitioners, through their own fault, failed to The petition shall also contain a sworn certification of non-forum
avail themselves of this remedy. Countless times we have ruled that shopping as provided in the third paragraph of section 3, Rule 46.
the extraordinary remedy of certiorari or prohibition is not a substitute
Notes:
for a lost appeal. This case is no different.

There is another equally important reason why the instant petition Mandamus is a writ issued in the name of the state, to an inferior
should be denied outright. After the Court of Appeals issued the tribunal, corporation, board or person, commanding the performance
assailed Decision dated November 24, 2003 which dismissed of an act which the law enjoins as a duty resulting from an office,
petitioners’ petition for prohibition, several supervening events took trust, or station.
place.
It is commenced by a verified petition accompanied by a sworn
As earlier noted, petitioners failed to appeal from the Joint Resolution certification of non-forum shopping.
dated November 8, 2002 issued by public respondent Montero which

101 Aicka Singson


Notes in Remedial Law Review II
Provisional Remedies, Special Civil Actions, Special Proceedings and Evidence
School Year 2015-2016 REMEDIAL LAW REVIEW II - ATTY. BRONDIAL

a. Grounds GR: Mandamus will not issue when administrative remedies are still
available.
Any tribunal, corporation, board, officer or person unlawfully neglects
the performance of an act which the law specifically enjoins as a duty XPN:
resulting from an office, trust or station; or
 If the party is in estoppel.
When any tribunal, corporation, board, officer or person unlawfully  Only questions of law are raised.
excludes another from the use and enjoyment of a right or office to
which the other is entitled. May mandamus be used to compel a discretionary duty?

b. Requisites GR: Mandamus is only applicable to a ministerial duty. However, a


mandamus can be used to the extent of requiring the performance of
 There must be a clear legal right or duty; a discretionary duty to act but not to require performance of such
 The act to be performed must be within the powers of the duty in a particular manner.
respondent to perform;
 The respondent must be exercising a ministerial duty; XPN:
 The duty or act to be performed must be existing (a correlative
 IF there has been grave abuse of discretion;
right will be denied if not performed by the respondent); and
 Manifest injustice;
 There is no appeal or other plain, speedy and adequate
 Palpable excess of Authority
remedy in the ordinary course of law.
CASES:
c. Procedure, Parties and Effects a. Hipos Sr. vs. Bay (581 SCRA)
The person aggrieved thereby may file a verified petition in the FACTS: Two Informations for the crime of rape and one Information
proper court, alleging the facts with certainty and praying that for the crime of acts of lasciviousness were filed against petitioners
judgment be rendered commanding the respondent, immediately or Darryl Hipos, Jaycee Corsiño, Arthur Villaruel and two others before
at some other time to be specified by the court, to do the act required RTC of Quezon City, acting as a Family Court, presided by
to be done to protect the rights of the petitioner, and to pay the respondent Judge Bay.
damages sustained by the petitioner by reason of the wrongful acts
of the respondent. The petition shall also contain a sworn Private complainants AAA and BBB filed a Motion for Reinvestigation
certification of non-forum shopping. asking Judge Bay to order the City Prosecutor of Quezon City to
study if the proper Informations had been filed against petitioners
Note: and their co-accused. Judge Bay granted the Motion and ordered a
reinvestigation of the cases.
Will mandamus issue despite the availability of administrative
remedies?

102 Aicka Singson


Notes in Remedial Law Review II
Provisional Remedies, Special Civil Actions, Special Proceedings and Evidence
School Year 2015-2016 REMEDIAL LAW REVIEW II - ATTY. BRONDIAL

Petitioners filed their Joint Memorandum to Dismiss the Cases station; or when the respondent excludes another from the use and
before the City Prosecutor. They claimed that there was no probable enjoyment of a right or office to which the latter is entitled, and there
cause to hold them liable for the crimes charged. is no other plain, speedy and adequate remedy in the ordinary
course of law.
The Office of the City Prosecutor issued a Resolution on the
reinvestigation affirming the Informations filed against petitioners and As an extraordinary writ, the remedy of mandamus lies only to
their co-accused. The Resolution was signed by Assistant City compel an officer to perform a ministerial duty, not a discretionary
Prosecutor Raniel S. Cruz and approved by City Prosecutor Claro A. one; mandamus will not issue to control the exercise of discretion by
Arellano. a public officer where the law imposes upon him the duty to exercise
his judgment in reference to any manner in which he is required to
2nd Assistant City Prosecutor Lamberto C. de Vera, treating the act, because it is his judgment that is to be exercised and not that of
Joint Memorandum to Dismiss the Case as an appeal of the 10 the court.
August 2004 Resolution, reversed the Resolution dated 10 August
2004, holding that there was lack of probable cause. On the same In the case at bar, the act which petitioners pray that we compel the
date, the City Prosecutor filed a Motion to Withdraw Informations trial court to do is to grant the Office of the City Prosecutor’s Motion
before Judge Bay. for Withdrawal of Informations against petitioners. In effect,
petitioners seek to curb Judge Bay’s exercise of judicial discretion.
Judge Bay denied the Motion to Withdraw Informations in an Order
of even date. There is indeed an exception to the rule that matters involving
judgment and discretion are beyond the reach of a writ of
Without moving for a reconsideration of the above assailed Order, mandamus, for such writ may be issued to compel action in those
petitioners filed the present Petition for Mandamus. matters, when refused. However, mandamus is never available to
direct the exercise of judgment or discretion in a particular way or the
ISSUE: CAN THE HON. SUPREME COURT COMPEL
retraction or reversal of an action already taken in the exercise of
RESPONDENT JUDGE BAY TO DISMISS THE CASE THROUGH
either. In other words, while a judge refusing to act on a Motion to
A WRIT OF MANDAMUS BY VIRTUE OF THE RESOLUTION OF
Withdraw Informations can be compelled by mandamus to act on the
THE OFFICE OF THE CITY PROSECUTOR OF QUEZON CITY
same, he cannot be compelled to act in a certain way, i.e., to grant or
FINDING NO PROBABLE CAUSE AGAINST THE ACCUSED AND
deny such Motion. In the case at bar, Judge Bay did not refuse to act
SUBSEQUENTLY FILING A MOTION TO WITHDRAW
on the Motion to Withdraw Informations; he had already acted on it
INFORMATION?
by denying the same. Accordingly, mandamus is not available
HELD: Mandamus is an extraordinary writ commanding a tribunal, anymore. If petitioners believed that Judge Bay committed grave
corporation, board, officer or person, immediately or at some other abuse of discretion in the issuance of such Order denying the Motion
specified time, to do the act required to be done, when the to Withdraw Informations, the proper remedy of petitioners should
respondent unlawfully neglects the performance of an act which the have been to file a Petition for Certiorari against the assailed Order
law specifically enjoins as a duty resulting from an office, trust, or of Judge Bay.

103 Aicka Singson


Notes in Remedial Law Review II
Provisional Remedies, Special Civil Actions, Special Proceedings and Evidence
School Year 2015-2016 REMEDIAL LAW REVIEW II - ATTY. BRONDIAL

In the case at bar, the Petition for Mandamus is directed not against issuance of absorption orders to the 45 PC constables included in
the prosecution, but against the trial court, seeking to compel the trial the initial batch of those covered by the PNP Board Resolutions.8
court to grant the Motion to Withdraw Informations by the City Petitioner Sanchez is in number 45 of that list.9
Prosecutor’s Office. The prosecution has already filed a case against
petitioners. Recently, in Santos v. Orda, Jr.,we reiterated the doctrine As no absorption order had yet been issued by the Chief of the PNP,
we established in the leading case of Crespo v. Mogul, that once a the constables in the list requested the assistance of the Secretary of
criminal complaint or an information is filed in court, any disposition the (DILG). Office of the Secretary of the DILG sent a memorandum
or dismissal of the case or acquittal or conviction of the accused to respondent Roberto T. Lastimoso, then the Chief of the PNP,
rests within the jurisdiction, competence, and discretion of the trial endorsing the constables’ entreaties and requesting for a feedback
court. thereon.

WHEREFORE, the instant Petition for Mandamus is DISMISSED. Without any response from the Chief of the PNP, and their pleas for
Let the records of this case be remanded to the Regional Trial Court the issuance of the absorption orders still unacted upon, petitioners
of Quezon City for the resumption of the proceedings therein. The instituted a petition for mandamus docketed as Civil Case No. Q-98-
Regional Trial Court is directed to act on the case with dispatch. 35659 in the Regional Trial Court (RTC) of Quezon City.

Atty. Procopio S. Beltran, Jr. is ORDERED to SHOW CAUSE why he During the pendency of the said petition, NAPOLCOM issued
should not be disciplined as a member of the Bar for his disquieting Resolution No. 99-061 on April 19, 1999 recalling the earlier
conduct as herein discussed. Resolution No. 98-105. In other words, the 126 named in the list
were still to be interviewed and their applications to be deliberated
SO ORDERED upon by the PNP Special Committee.

b. Sanchez vs. Lastimosa (534 SCRA) However, the RTC rendered its Decision in the mandamus case
declaring as void ab initio NAPOLCOM Resolution No. 99-061 and
Petitioner Sanchez, a constable in the Philippine Constabulary (PC), ruling in favor of the petitioners.
was discharged from the service for allegedly losing his service
firearm. Petitioner Meteoro, also a constable, was likewise On appeal, the CA, in the assailed and reversed the ruling of the
discharged from the service in 1990 for being absent without leave. trial court and ruled that a writ of mandamus could not be issued
On appeal, they were both cleared of all charges. They then applied because petitioners had not established with distinct clarity their right
for reinstatement but their applications were not acted upon even up to be absorbed into the PNP. The CA disposed of the appeal as
to the integration of the PC into the Philippine National Police (PNP). follows:

NAPOLCOM issued Resolution No. 98-105 affirming and confirming Aggrieved, petitioners brought the case before us via a petition for
the absorption into the PNP. petitioners Sanchez and Meteoro. review on certiorari, raising

NAPOLCOM Commissioner Rogelio A. Pureza issued a


Memorandum to then Chief of the PNP Santiago Alino for the

104 Aicka Singson


Notes in Remedial Law Review II
Provisional Remedies, Special Civil Actions, Special Proceedings and Evidence
School Year 2015-2016 REMEDIAL LAW REVIEW II - ATTY. BRONDIAL

WHETHER OR NOT PETITIONERS HAVE A CAUSE OF ACTION integration of the PC into the PNP in 1990 when R.A. No. 6975 was
FOR MANDAMUS TO COMPEL THE RESPONDENT TO ABSORB enacted. Thus, we no longer speak of the reinstatement of the
THE PETITIONERS IN THE PHILIPPINE NATIONAL POLICE. petitioners to the service because the Philippine Constabulary no
longer exists, but of their employment in the PNP which is, as we
No. We have repeatedly stressed in our prior decisions that the held in Gloria v. De Guzman, technically an issuance of a new
remedy of mandamus is employed only to compel the performance, appointment. The power to appoint is essentially discretionary to be
when refused, of a ministerial duty, but not to require anyone to fulfill performed by the officer in which it is vested according to his best
a discretionary one. The issuance of the writ is simply a command to lights, the only condition being that the appointee should possess the
exercise a power already possessed and to perform a duty already qualifications required by law. Consequently, it cannot be the subject
imposed. In Manila International Airport Authority v. Rivera Village of an application for a writ of mandamus.
Lessee Homeowners Association, Inc., we emphasized, through the
erudite and eloquent ponencia of Justice Dante O. Tinga, that the Furthermore, the petitioners do not have a clear legal right over the
writ can be issued only when the applicant’s legal right to the issuance of the absorption orders.1âwphi1 They cannot claim the
performance of a particular act sought to be compelled is clear and right to be issued an appointment based on the NAPOLCOM
complete, one which is indubitably granted by law or is inferable as a issuances, specifically Resolution Nos. 98-037 and 98-105. Suffice it
matter of law, thus: to state that R.A. No. 6975 clearly provides that the power to appoint
PNP personnel with the rank of "Police Officer I" to "Senior Police
In order that a writ of mandamus may aptly issue, it is essential that, Officer IV" to which petitioners may be appointed is vested in the
on the one hand, petitioner has a clear legal right to the claim that is PNP regional director or in the Chief of the PNP as the case may be,
sought and that, on the other hand, respondent has an imperative and not in the NAPOLCOM.
duty to perform that which is demanded of him. Mandamus will not
issue to enforce a right, or to compel compliance with a duty, which Even if, for the sake of argument, petitioners can derive a right from
is questionable or over which a substantial doubt exists. The NAPOLCOM Resolution Nos. 98-037 and 98-105, still their right
principal function of the writ of mandamus is to command and to collapses and their mandamus petition becomes moot with the
expedite, not to inquire and to adjudicate. Thus, it is neither the office issuance by NAPOLCOM of Resolution No. 99-061 recalling the
nor the aim of the writ to secure a legal right but to implement that approval of their absorption. The trial court should then have
which is already established. Unless the right to relief sought is immediately dismissed the mandamus petition when the OSG
unclouded, mandamus will not issue. submitted a copy of Resolution No. 99-061 because well-settled is
the rule that courts will not resolve a moot question.
Viewed in light of the said guideposts, the PNP Chief’s issuance of
the orders for the absorption of herein petitioners in the police force c. Social Justice Society vs. Atienza (517 SCRA)
is not compellable by a writ of mandamus precisely because the
same does not involve a performance of a ministerial duty. Let it be FACTS: On November 20, 2001, the Sangguniang Panlungsod of
noted that petitioners were discharged from the PC service, Manila enacted Ordinance No. 8027. Respondent mayor approved
subsequently cleared of the charges against them, applied for the ordinance on November 28, 2001. It became effective on
reinstatement but their applications were not acted upon until the December 28, 2001, after its publication.

105 Aicka Singson


Notes in Remedial Law Review II
Provisional Remedies, Special Civil Actions, Special Proceedings and Evidence
School Year 2015-2016 REMEDIAL LAW REVIEW II - ATTY. BRONDIAL

Ordinance No. 8027 was enacted pursuant to the police power ISSUE: Whether respondent may be compelled to enforce
delegated to LGUs, a principle described as the power inherent in a Ordinance No. 8027 and order the removal of the Pandacan
government to enact laws, within constitutional limits, to promote the Terminals by way of mandamus
order, safety, health, morals and general welfare of the society.
HELD: YES. Under Rule 65, Section 3 of the Rules of Court, a
Ordinance No. 8027 reclassified the area from industrial to petition for mandamus may be filed when any tribunal, corporation,
commercial and directed the owners and operators of businesses board, officer or person unlawfully neglects the performance of an
disallowed under Section 1 to cease and desist from operating their act which the law specifically enjoins as a duty resulting from an
businesses within six months from the date of effectivity of the office, trust or station. Mandamus is an extraordinary writ that is
ordinance. Among the businesses situated in the area are the so- employed to compel the performance, when refused, of a ministerial
called "Pandacan Terminals" of the oil companies Caltex duty that is already imposed on the respondent and there is no other
(Philippines), Inc., Petron Corporation and Pilipinas Shell Petroleum plain, speedy and adequate remedy in the ordinary course of law.
Corporation. The petitioner should have a well-defined, clear and certain legal
right to the performance of the act and it must be the clear and
However, on June 26, 2002, the City of Manila and the Department imperative duty of respondent to do the act required to be done.
of Energy (DOE) entered into a memorandum of understanding
(MOU) with the oil companies in which they agreed that "the scaling Mandamus will not issue to enforce a right, or to compel compliance
down of the Pandacan Terminals was the most viable and with a duty, which is questionable or over which a substantial doubt
practicable option." exists. The principal function of the writ of mandamus is to command
and to expedite, not to inquire and to adjudicate; thus, it is neither the
The Sangguniang Panlungsod ratified the MOU in Resolution No. 97. office nor the aim of the writ to secure a legal right but to implement
In the same resolution, the Sanggunian declared that the MOU was that which is already established. Unless the right to the relief sought
effective only for a period of six months starting July 25, 2002. is unclouded, mandamus will not issue.
Thereafter, on January 30, 2003, the Sanggunian adopted
Resolution No. 13 extending the validity of Resolution No. 97 to April To support the assertion that petitioners have a clear legal right to
30, 2003 and authorizing Mayor Atienza to issue special business the enforcement of the ordinance, petitioner SJS states that it is a
permits to the oil companies. Resolution No. 13, s. 2003 also called political party registered with the Commission on Elections and has
for a reassessment of the ordinance. its offices in Manila. It claims to have many members who are
residents of Manila. The other petitioners, Cabigao and Tumbokon,
Meanwhile, petitioners filed this original action for mandamus on are allegedly residents of Manila.
December 4, 2002 praying that Mayor Atienza be compelled to
enforce Ordinance No. 8027 and order the immediate removal of the We need not belabour this point. We have ruled in previous cases
terminals of the oil companies. that when a mandamus proceeding concerns a public right and its
object is to compel a public duty, the people who are interested in
the execution of the laws are regarded as the real parties in interest
and they need not show any specific interest. Besides, as residents

106 Aicka Singson


Notes in Remedial Law Review II
Provisional Remedies, Special Civil Actions, Special Proceedings and Evidence
School Year 2015-2016 REMEDIAL LAW REVIEW II - ATTY. BRONDIAL

of Manila, petitioners have a direct interest in the enforcement of the 2010. Petitioner filed the suit in his capacities as “taxpayer,
city’s ordinances. Respondent never questioned the right of concerned citizen, a member of the Philippine Bar and law book
petitioners to institute this proceeding. author.. He impleaded both the COA and the MECO.

On the other hand, the Local Government Code imposes upon Both MECO and CoA prayed for the dismissal of the mandamus
respondent the duty, as city mayor, to "enforce all laws and petition on substantial and procedural grounds.
ordinances relative to the governance of the city." One of these is
Ordinance No. 8027. As the chief executive of the city, he has the Issue: WON the Mandamus should be dismissed
duty to enforce Ordinance No. 8027 as long as it has not been
Held:
repealed by the Sanggunian or annulled by the courts. He has no
other choice. It is his ministerial duty to do so. In Dimaporo v. Mitra, Mootness of Petition
Jr., we stated the reason for this:
The first preliminary issue relates to the alleged mootness of the
These officers cannot refuse to perform their duty on the ground of instant mandamus petition, occasioned by the COA’s issuance of
an alleged invalidity of the statute imposing the duty. The reason for Office Order No. 2011–698. The COA claims that by issuing Office
this is obvious. It might seriously hinder the transaction of public Order No. 2011–698, it had already conceded its jurisdiction over the
business if these officers were to be permitted in all cases to accounts of the MECO and so fulfilled the objective of the instant
question the constitutionality of statutes and ordinances imposing petition.58 The COA thus urges that the instant petition be dismissed
duties upon them and which have not judicially been declared for being moot and academic.59
unconstitutional. Officers of the government from the highest to the
lowest are creatures of the law and are bound to obey it. We decline to dismiss the mandamus petition on the ground of
mootness.
d. Funa vs. Manila Economic and Cultural Office (715
SCRA) A case is deemed moot and academic when, by reason of the
occurrence of a supervening event, it ceases to present any
Facts: On 23 August 2010, petitioner sent a letter to the COA justiciable controversy.60 Since they lack an actual controversy
requesting for a “copy of the latest financial and audit report” of the otherwise cognizable by courts, moot cases are, as a rule,
MECO invoking, for that purpose, his “constitutional right to dismissible.61
information on matters of public concern.” Assistant Commissioner
Naranjo revealed that the MECO was “not among the agencies The rule that requires dismissal of moot cases, however, is not
audited by any of the three Clusters of the Corporate Government absolute. It is subject to exceptions. In David v. Macapagal–
Sector. Arroyo,62 this Court comprehensively captured these exceptions
scattered throughout our jurisprudence:
Taking the 25 August 2010 memorandum as an admission that the
COA had never audited and examined the accounts of the MECO, The “moot and academic” principle is not a magical formula that can
the petitioner filed the instant petition for mandamus on 8 September automatically dissuade the courts in resolving a case. Courts will

107 Aicka Singson


Notes in Remedial Law Review II
Provisional Remedies, Special Civil Actions, Special Proceedings and Evidence
School Year 2015-2016 REMEDIAL LAW REVIEW II - ATTY. BRONDIAL

decide cases, otherwise moot and academic, if: first, there is a grave not—has a direct bearing on the country’s commitment to the One
violation of the Constitution;63second, the exceptional character of China policy of the PROC.67
the situation and the paramount public interest is involved;64third,
when constitutional issue raised requires formulation of controlling An allegation as serious as a violation of a constitutional or legal
principles to guide the bench, the bar, and the public;65 and fourth, duty, coupled with the pressing public interest in the resolution of all
the case is capable of repetition yet evading review.66 related issues, prompts this Court to pursue a definitive ruling
thereon, if not for the proper guidance of the government or agency
In this case, We find that the issuance by the COA of Office Order concerned, then for the formulation of controlling principles for the
No. 2011–698 indeed qualifies as a supervening event that education of the bench, bar and the public in general.68 For this
effectively renders moot and academic the main prayer of the instant purpose, the Court invokes its symbolic function.69
mandamus petition. A writ of mandamus to compel the COA to audit
the accounts of the MECO would certainly be a mere superfluity, If the foregoing reasons are not enough to convince, We still add
when the former had already obliged itself to do the same. another:

Be that as it may, this Court refrains from dismissing outright the Assuming that the allegations of neglect on the part of the COA were
petition. We believe that the mandamus petition was able to craft true, Office Order No. 2011–698 does not offer the strongest
substantial issues presupposing the commission of a grave violation certainty that they would not be replicated in the future. In the first
of the Constitution and involving paramount public interest, which place, Office Order No. 2011–698 did not state any legal justification
need to be resolved nonetheless: as to why, after decades of not auditing the accounts of the MECO,
the COA suddenly decided to do so. Neither does it state any
First. The petition makes a serious allegation that the COA had been determination regarding the true status of the MECO. The
remiss in its constitutional or legal duty to audit and examine the justifications provided by the COA, in fact, only appears in the
accounts of an otherwise auditable entity in the MECO. memorandum70 it submitted to this Court for purposes of this case.

Second. There is paramount public interest in the resolution of the Thus, the inclusion of the MECO in Office Order No. 2011–698
issue concerning the failure of the COA to audit the accounts of the appears to be entirely dependent upon the judgment of the
MECO. The propriety or impropriety of such a refusal is incumbent chairperson of the COA; susceptible of being undone,
determinative of whether the COA was able to faithfully fulfill its with or without reason, by her or even her successor. Hence, the
constitutional role as the guardian of the public treasury, in which any case now before this Court is dangerously capable of being repeated
citizen has an interest. yet evading review.

Third. There is also paramount public interest in the resolution of the Verily, this Court should not dismiss the mandamus petition on the
issue regarding the legal status of the MECO; a novelty insofar as ground of mootness.
our jurisprudence is concerned. We find that the status of the
MECO—whether it may be considered as a government agency or Standing of Petitioner

108 Aicka Singson


Notes in Remedial Law Review II
Provisional Remedies, Special Civil Actions, Special Proceedings and Evidence
School Year 2015-2016 REMEDIAL LAW REVIEW II - ATTY. BRONDIAL

The second preliminary issue is concerned with the standing of the (5) for legislators, there must be a claim that the official action
petitioner to file the instant mandamus petition. The COA claims that complained of infringes upon their prerogatives as legislators.
petitioner has none, for the latter was not able to concretely establish
that he had been aggrieved or prejudiced by its failure to audit the We rule that the instant petition raises issues of transcendental
accounts of the MECO.71 importance, involved as they are with the performance of a
constitutional duty, allegedly neglected, by the COA. Hence, We hold
Related to the issue of lack of standing is the MECO’s contention that the petitioner, as a concerned citizen, has the requisite legal
that petitioner has no cause of action to file the instant mandamus standing to file the instant mandamus petition.
petition. The MECO faults petitioner for not making any demand for it
to submit to an audit by the COA or for the COA to perform such an To be sure, petitioner does not need to make any prior demand on
audit, prior to filing the instant petition.72 the MECO or the COA in order to maintain the instant petition. The
duty of the COA sought to be compelled by mandamus, emanates
We sustain petitioner’s standing, as a concerned citizen, to file the from the Constitution and law, which explicitly require, or “demand,”
instant petition. that it perform the said duty. To the mind of this Court, petitioner
already established his cause of action against the COA when he
The rules regarding legal standing in bringing public suits, or locus alleged that the COA had neglected its duty in violation of the
standi, are already well–defined in our case law. Again, We cite Constitution and the law.
David, which summarizes jurisprudence on this point:73
Principle of Hierarchy of Courts
By way of summary, the following rules may be culled from the cases
decided by this Court. Taxpayers, voters, concerned citizens, and The last preliminary issue is concerned with the petition’s non–
legislators may be accorded standing to sue, provided that the observance of the principle of hierarchy of courts. The COA assails
following requirements are met: the filing of the instant mandamus petition directly with this Court,
when such petition could have very well been presented, at the first
(1) the cases involve constitutional issues; instance, before the Court of Appeals or any Regional Trial Court.74
The COA claims that the petitioner was not able to provide
(2) for taxpayers, there must be a claim of illegal disbursement of
compelling reasons to justify a direct resort to the Supreme Court.75
public funds or that the tax measure is unconstitutional;
In view of the transcendental importance of the issues raised in the
(3) for voters, there must be a showing of obvious interest in the
mandamus petition, as earlier mentioned, this Court waives this last
validity of the election law in question;
procedural issue in favor of a resolution on the merits.
(4) for concerned citizens, there must be a showing that the issues
raised are of transcendental importance which must be settled early;
and

109 Aicka Singson


Notes in Remedial Law Review II
Provisional Remedies, Special Civil Actions, Special Proceedings and Evidence
School Year 2015-2016 REMEDIAL LAW REVIEW II - ATTY. BRONDIAL

DIFFERENCES BETWEEN CERTIORARI, PROHIBITION AND MANDAMUS

CERTIORARI PROHIBITION MANDAMUS

Certiorari is an extraordinary writ Prohibition is an extraordinary writ Mandamus is an extraordinary writ


ANNULLING OR MODIFYING the COMMANDING a tribunal, commanding a tribunal, corporation, board or
proceedings of a tribunal, board or officer corporation, board or person, whether person, to do an act REQUIRED to be
exercising judicial or quasi- judicial exercising judicial, quasi-judicial or done:
functions when such tribunal, board or officer ministerial functions, TO DESIST from a) When he unlawfully neglects the
has acted without or in excess of its or his further proceedings when said proceedings performance of an act which the law
jurisdiction, or with grave abuse of are without or in excess of its jurisdiction, specifically enjoins as a duty, and there is
discretion amounting to lack or excess or with abuse of its discretion, there being no other plain, speedy and adequate
of jurisdiction, there being no appeal or any no appeal or any other plain, speedy and remedy in the ordinary course of law; or
other plain, speedy and adequate remedy in adequate remedy in the ordinary course of b) When one unlawfully excludes another
the ordinary course of law (Sec. 1, Rule 65). law (Sec. 2, Rule 65). from the use and enjoyment of a
right or office to which the other is entitled
(Sec. 3, Rule 65).
Directed against a person exercising to Directed against a person exercising Directed against a person exercising ministerial
judicial or quasi-judicial functions judicial or quasi-judicial functions, or duties
ministerial functions
The tribunal, board or officer has acted The tribunal, corporation, board or person It must be the duty of the defendant to perform
without, or in excess of jurisdiction or must have acted without or in excess of the act, which is ministerial and not
with abuse of discretion amounting to lack jurisdiction or with grave abuse of discretionary, because the same is mandated
or excess or jurisdiction discretion amounting to lack of jurisdiction; by law.

There is no appeal or any plain, speedy There is no appeal or any plain, speedy The defendant unlawfully neglects the
and adequate remedy in the ordinary and adequate remedy in the ordinary performance of the duty enjoined by law;
course of law. course of law.
Object is to correct Object is to prevent Object is to compel
Purpose is to annul or modify the Purpose is to stop the proceedings Purpose is to compel performance of the act
proceedings required and to collect damages
Person or entity must have acted Person or entity must have acted Person must have neglected a
without or in excess of jurisdiction, or with without or in excess of jurisdiction, or with ministerial duty or excluded another from a
grave abuse of discretion grave abuse of discretion right or office

110 Aicka Singson


Notes in Remedial Law Review II
Provisional Remedies, Special Civil Actions, Special Proceedings and Evidence
School Year 2015-2016 REMEDIAL LAW REVIEW II - ATTY. BRONDIAL

A person aggrieved thereby may file a A person aggrieved thereby may file a The person aggrieved thereby may file a verified
verified petition in the proper court, alleging verified petition in the proper court, petition in the proper court, alleging the facts
the facts with certainty and praying that alleging the facts with certainty and with certainty and praying that judgment be
judgment be rendered annulling or praying that judgment be rendered rendered commanding the respondent,
modifying the proceedings of such commanding the respondent to desist from immediately or at some other time to be
tribunal, board or officer, and granting such further proceedings in the action or matter specified by the court, to do the act required to be
incidental reliefs as law and justice may specified therein, or otherwise granting done to protect the rights of the petitioner, and to
require. The petition shall be accompanied by such incidental reliefs as law and justice may pay the
a certified true copy of the judgment, order or require. damages sustained by the petitioner by reason of the
resolution subject thereof, copies of all The petition shall likewise be accompanied by wrongful acts of the respondent. The petition shall
pleadings and documents relevant and a certified true copy of the judgment, order or also contain a sworn certification of nonforum
pertinent thereto, and a sworn certification of resolution subject thereof, copies of all shopping.
non-forum shopping. pleadings and documents relevant and
pertinent thereto, and a sworn certification of
non-forum shopping.

111 Aicka Singson


Notes in Remedial Law Review II
Provisional Remedies, Special Civil Actions, Special Proceedings and Evidence
School Year 2015-2016 REMEDIAL LAW REVIEW II - ATTY. BRONDIAL

PROHIBITION INJUNCTION

Always the main action May be the main action or just a provisional remedy
Directed against a court, a tribunal exercising judicial Directed against a party
or quasi-judicial functions
Ground must be the court acted without or in excess Does not involve a question of jurisdiction
of jurisdiction

PROHIBITION MANDAMUS

To prevent an act by a respondent To compel an act desired


May be directed against entities exercising judicial May be directed against judicial and non-judicial entities
or quasi-judicial, or ministerial functions
Extends to discretionary functions Extends only to ministerial functions

MANDAMUS QUO WARRANTO

Clarifies legal duties, not legal titles Clarifies who has legal title to the office, or franchise
Respondent, without claiming any right to the Respondent usurps the office
office, excludes the petitioner

 Mandamus can be issued to perform an act but not to approve a certain request

INJUNCTIVE RELIEF
The court in which the petition is filed may issue orders expediting the proceedings, and it may also grant a temporary restraining order or a writ of
preliminary injunction for the preservation of the rights of the parties pending such proceedings. The petition shall not interrupt the course of the
principal case unless a temporary restraining order or a writ of preliminary injunction has been issued against the public respondent from further
proceeding in the case. The public respondent shall proceed with the principal case within ten (10) days from the filing of a petition for certiorari with
a higher court or tribunal, absent a Temporary Restraining Order (TRO) or a Writ of Preliminary Injunction, or upon its expiration. Failure of the public
respondent to proceed with the principal case may be a ground for an administrative charge (AM 07-7- 12-SC, Dec. 12, 2007).

112 Aicka Singson


Notes in Remedial Law Review II
Provisional Remedies, Special Civil Actions, Special Proceedings and Evidence
School Year 2015-2016 REMEDIAL LAW REVIEW II - ATTY. BRONDIAL

CERTIORARI AS A MODE OF APPEAL CERTIORARI AS A SPECIAL CIVIL ACTION


(RULE 45) (RULE 65)
Called petition for review on certiorari, is a mode A special civil action that is an original action and not a mode
of appeal, which is but a continuation of the of appeal, and not a part of the appellate process but an
appellate process over the original case; independent action.
Seeks to review final judgments or final orders; May be directed against an interlocutory order of the court
or where not appeal or plain or speedy remedy available
in the ordinary course of law
Raises only questions of law; Raises questions of jurisdiction because a tribunal, board
or officer exercising judicial or quasi-judicial functions has
acted without jurisdiction or in excess of jurisdiction or with NOTE: The remedies of
grave abuse of discretion amounting to lack of jurisdiction; appeal and certiorari are
Filed within 15 days from notice of judgment or final Filed not later than 60 days from notice of judgment, order mutually exclusive and not
order appealed from, or of the denial of or resolution sought to be assailed and in case a alternative or successive. The
petitioner‘s motion for reconsideration or new trial; motion for reconsideration or new trial is timely filed, whether
antithetic character of appeal
such motion is required or not, the 60 day period is counted
from notice of denial of said motion; and certiorari has been
Extension of 30 days may be granted for Extension no longer allowed; generally recognized and
justifiable reasons observed save only on those
Does not require a prior motion for reconsideration; Motion for Reconsideration is a condition precedent, subject rare instances when appeal is
to exceptions satisfactorily shown to be an
Stays the judgment appealed from; Does not stay the judgment or order subject of the petition inadequate remedy. Thus, a
unless enjoined or restrained; petitioner must show valid
Parties are the original parties with the The tribunal, board, officer exercising judicial or quasi-
reasons why the issues raised
appealing party as the petitioner and the adverse judicial functions is impleaded as respondent
party as the respondent without impleading the in his petition for certiorari
lower court or its judge; could not have been raised on
Filed with only the Supreme Court May be filed with the Supreme Court, Court of appeal.
Appeals, Sandiganbayan, or Regional Trial Court
SC may deny the decision motu propio on the
ground that the appeal is without merit, or is
prosecuted manifestly for delay, or that the
questions raised therein are too unsubstantial to
require consideration.

113 Aicka Singson


Notes in Remedial Law Review II
Provisional Remedies, Special Civil Actions, Special Proceedings and Evidence
School Year 2015-2016 REMEDIAL LAW REVIEW II - ATTY. BRONDIAL

PROHIBITION MANDAMUS INJUNCTION

Prohibition is an extraordinary writ Mandamus is an extraordinary writ Main action for injunction seeks to
commanding a tribunal, commanding a tribunal, corporation, enjoin the defendant from the
corporation, board or person, board or person, to do an act required commission or continuance of a
whether exercising judicial, quasi- to be done: (a) When he unlawfully specific act, or to compel a particular
judicial or ministerial functions, to neglects the performance of an act act in violation of the rights of the
desist from further proceedings which the law specifically enjoins as a applicant. Preliminary injunction is a
when said proceedings are without duty, and there is no other plain, provisional remedy to preserve the
or in excess of its jurisdiction, or speedy and adequate remedy in the status quo and prevent future wrongs
with abuse of its discretion, there ordinary course of law; or (b) When in order to preserve and protect
being no appeal or any other plain, one unlawfully excludes another from certain interests or rights during the
speedy and adequate remedy in the the use and enjoyment of a right or pendency of an action.
ordinary course of law office to which the other is entitled
(Sec. 2, Rule 65). (Sec. 3, Rule 65).

Special civil action Special civil action Ordinary civil action


To prevent an encroachment, To compel the performance of a For the defendant either to refrain
excess, usurpation or assumption of ministerial and legal duty; from an act or to perform not
jurisdiction; necessarily a legal and ministerial
duty;
May be directed against entities May be directed against judicial and Directed against a party
exercising judicial or quasi-judicial, non-judicial entities
or ministerial functions
Extends to discretionary functions Extends only to ministerial functions Does not necessarily extend to
ministerial, discretionary or legal
function;
Always the main action Always the main action May be the main action or just a provisional
remedy
May be brought in the Supreme May be brought in the Supreme May be brought in the Regional Trial
Court, Court of Appeals, Court, Court of Appeals, Court which has jurisdiction over the
Sandiganbayan, or in the Regional Sandiganbayan, or in the Regional territorial area where respondent
Trial Court which has jurisdiction Trial Court which has jurisdiction resides.
over the territorial area where over the territorial area where
respondent resides respondent resides

114 Aicka Singson


Notes in Remedial Law Review II
Provisional Remedies, Special Civil Actions, Special Proceedings and Evidence
School Year 2015-2016 REMEDIAL LAW REVIEW II - ATTY. BRONDIAL

The court may dismiss the petition if it finds the same patently without
GENERL RULE: A Motion for Reconsideration is required before a merit or prosecuted manifestly for delay, or if the questions raised
party may resort to the Remedies of Certiorari, Prohibition and therein are too unsubstantial to require consideration.
Mandamus because it is a form of plain, speedy and adequate remedy In such event, the court may award in favor of the respondent treble
which gives courts the opportunity to correct itself costs solidarily against the petitioner and counsel, in addition to
subjecting counsel to administrative sanctions.
EXCEPTIONS TO FILING OF MOTION FOR RECONSIDERATION
BEFORE FILING PETITION: Where should the petition be filed?

1) When the issue is one purely of law or when public interest is


Supreme Court Subject to the doctrine of
involved;
hierarchy of courts and
2) When there is urgency to decide upon the question and any further only when compelling
delay would prejudice the interests of the government or of the reasons exist for not filing
petitioner; the same with the lower
3) Where the subject matter of the action is perishable; courts
4) When order is a patent nullity, as where the court a quo has no Regional Trial Court If the petition relates to
jurisdiction or there was no due process; an act or an omission of
5) When questions have been duly raised and passed upon by the an MTC, corporation,
lower court; board, officer or person
6) When is urgent necessity for the resolution of the question; Court of Appeals only If the petition involves an
7) When Motion for Reconsideration would be useless, e.g. the court act or an omission of a
already indicated it would deny any Motion for Reconsideration; quasi-judicial agency,
unless otherwise provided
8) In a criminal case, where relief from order of arrest is urgent and the
by law or rules
granting of such relief by the trial court is improbable;
9) Where the proceedings was ex parte or in which the petitioner had Court of Appeals or the Whether or not in aid of
Sandiganbayan appellate jurisdiction
no opportunity to object;
10) When petitioner is deprived of due process and there is extreme COMELEC In election cases
involving an act or an
urgency for urgent relief.
omission of an MTC or
RTC
EFFECTS OF FILING OF AN UNMERITORIOUS PETITION
The Court may impose motu proprio, based on res ipsa loquitur, other
disciplinary sanctions or measures on erring lawyers for patently
dilatory an unmeritorious petition for certiorari.

115 Aicka Singson


Notes in Remedial Law Review II
Provisional Remedies, Special Civil Actions, Special Proceedings and Evidence
School Year 2015-2016 REMEDIAL LAW REVIEW II - ATTY. BRONDIAL

E. Quo Warranto (Rule 66) (c) An association which acts as a corporation within the Philippines
without being legally incorporated or without lawful authority so to
Quo warranto is a demand made by the state upon some individual act.
or corporation to show by what right they exercise some franchise or
privilege appertaining to the state which, according to the Sec. 2. When Solicitor General or public prosecutor must commence
Constitution and laws they cannot legally exercise by virtue of a action.
grant and authority from the State.
The Solicitor General or a public prosecutor, when directed by the
It is a special civil action commenced by a verified petition against: President of the Philippines, or when upon complaint or otherwise he
has good reason to believe that any case specified in the preceding
a) a person who usurps a public office, position or franchise; section can be established by proof, must commence such action.

b) a public officer who performs an act constituting forfeiture of a Sec. 3. When Solicitor General or public prosecutor may commence
public office; or action with permission of court.

c) an association which acts as a corporation within the Philippines The Solicitor General or a public prosecutor may, with the permission
without being legally incorporated or without lawful authority to do so. of the court in which the action is to be commenced, bring such an
action at the request and upon the relation of another person; but in
Quo warranto – by what authority.
such case the officer bringing it may first require an indemnity for the
expenses and costs of the action in an amount approved by and to
To whom are you addressing the question “by what authority?” to the
person occupying a public office. be deposited in the court by the person at whose request and upon
whose relation the same is brought.
1. Parties (Sections 1-6)
Sec. 4. When hearing had on application for permission to
Section 1. Action by Government against individuals. commence action.

An action for the usurpation of a public office, position or franchise Upon application for permission to commence such action in
may be commenced by a verified petition brought in the name of the accordance with the next preceding section, the court shall direct
Republic of the Philippines against: that notice be given to the respondent so that he may be heard in
opposition thereto; and if permission is granted, the court shall issue
(a) A person who usurps, intrudes into, or unlawfully holds or an order to that effect, copies of which shall be served on all
exercises a public office, position or franchise; interested parties, and the petition shall then be filed within the
period ordered by the court.
(b) A public officer who does or suffers an act which, by the provision
of law, constitutes a ground for the forfeiture of his office; or Sec. 5. When an individual may commence such an action.

116 Aicka Singson


Notes in Remedial Law Review II
Provisional Remedies, Special Civil Actions, Special Proceedings and Evidence
School Year 2015-2016 REMEDIAL LAW REVIEW II - ATTY. BRONDIAL

A person claiming to be entitled to a public office or position usurped 3) An association which acts a corporation within the Philippines
or unlawfully held or exercised by another may bring an action without being legally incorporated or without lawful authority so to
therefor in his own name. act.

Sec. 6. Parties and contents of petition against usurpation. WHEN INDIVIDUAL MAY COMMENCE AN ACTION

When the action is against a person for usurping a public office, • The petition may be commenced by a private person in his own
position or franchise, the petition shall set forth the name of the name where he claims to be entitled to the public office or position
person who claims to be entitled thereto, if any, with an averment of alleged to have been usurped or unlawfully held or exercised by
his right to the same and that the respondent is unlawfully in another.
possession thereof. All persons who claim to be entitled to the public
office, position or franchise may be made parties, and their • Accordingly, the private person may maintain the action without the
respective rights to such public office, position or franchise intervention of the Solicitor General and without need for any leave
determined, in the same action. of court.

WHEN GOVERNMENT COMMENCE AN ACTION AGAINST • In bringing a petition for quo warranto, he must show that he has a
INDIVIDUALS clear right to the office allegedly being held by another. It is not
enough that he merely asserts the right to be appointed to the office.
Quo warranto is commenced by a verified petition brought in the
If the public office is the subject matter, who is the petitioner in
name of the Government of the Republic of the Philippines by the
quo warranto? The government because it is the government who
Solicitor General, or in some instances, by a public prosecutor.
is concerned with the public office. The election or appointment of
When the action is commenced by the Solicitor General, the petition
private officers is beyond the authority of the government. It is only
may be brought in the Regional Trial Court of the City of Manila, the
public office. This public office may either be through appointment or
Court of Appeals or the Supreme Court.
election. If it is an appointive public office, Rule 66 applies. If it is an
An action for the usurpation of a public office, position or franchise elected public office, omnibus election rules apply.
may be commenced by a verified petition brought in the name of the
The OSG is the person authorized to file cases. This is delegated to
Republic of the Philippines thru the Solicitor General against:
the DOJ. Can a private individual file a case of quo warranto? Yes.
1) A person who usurps, intrudes into, or unlawfully holds or Only when he is entitled to the public office. Without entitlement to
exercises a public office, position or franchise; public office, he cannot file a petition for quo warranto.

2) A public officer who does or suffers an act which, by the provision What/who is a relator? A relator is a private individual who intitiates
of law, constitutes a ground for the forfeiture of his office; a quo warranto proceeding by telling the public or the solicitor
general that a person is not entitled to the public office and he must
be ousted.

117 Aicka Singson


Notes in Remedial Law Review II
Provisional Remedies, Special Civil Actions, Special Proceedings and Evidence
School Year 2015-2016 REMEDIAL LAW REVIEW II - ATTY. BRONDIAL

The quo warranto proceeding is against a usurper; one who usurps a that such respondent be ousted and altogether excluded therefrom, and that
public office or misuses a franchises. A usurper is the supposed the petitioner or relator, as the case may be, recover his costs. Such further
defendant or respondent in a quo warranto proceeding. Against the judgment may be rendered determining the respective rights in and to the
public office, position or franchise of all the parties to the action as justice
quo warranto petition would be the person usurping the public office.
requires.
The mandamus would be against the person who caused the
usurpation.

If Mayor Joseph Estrada removes his City Attorney and replaces him Section 10. Rights of persons adjudged entitled to public office; delivery of
with Jun Estrada as the City Attorney, you can file 2 cases: quo books and papers; damages. — If judgment be rendered in favor of the
warranto against Jun Estrada and mandamus against Joseph person averred in the complaint to be entitled to the public office he may,
Estrada. after taking the oath of office and executing any official bond required by law,
take upon himself the execution of the office, and may immediately
Can you therefore join a petition for quo warranto with a petition thereafter demand of the respondent all the books and papers in the
for mandamus? In mandamus, one of the grounds is one excludes respondent's custody or control appertaining to the office to which the
judgment relates. If the respondent refuses or neglects to deliver any book
another from the enjoyment of an office or a franchise. Yes. This is
or paper pursuant to such demand, he may be punished for contempt as
not violative of joinder of causes of action under Rule 2. You cannot
having disobeyed a lawful order of the court. The person adjudged entitled to
join an ordinary action with a special civil action. But there is no the office may also bring action against the respondent to recover the
prohibition in joining together special civil actions. damages sustained by such person by reason of the usurpation.dence over
any other civil matter pending in the court.
2. PERIOD (Section 8)
3. Limitation (Section 11)
Sec. 8. Period for pleadings and proceedings may be reduced; action given
precedence. Sec. 11. Limitations.

The court may reduce the period provided by these Rules for filing pleadings Nothing contained in this Rule shall be construed to authorize an action
and for all other proceedings in the action in order to secure the most against a public officer or employee for his ouster from office unless the
expeditious determination of the matters involved therein consistent with the same be commenced within one (1) year after the cause of such ouster, or
rights of the parties. Such action may be given prece Section 8. Period for the right of the petitioner to hold such office or position, arose; nor to
pleadings and proceedings may be reduced; action given precedence. — authorize an action for damages in accordance with the provisions of the
The court may reduce the period provided by these Rules for filing pleadings next preceding section unless the same be commenced within one (1) year
and for all other proceedings in the action in order to secure the most after the entry of the judgment establishing the petitioner’s right to the office
expeditious determination of the matters involved therein consistent with the in question.
rights of the parties. Such action may be given precedence over any other
civil matter pending in the court. 4. Judgment for Cost (Section 12)
Section 9. Judgment where usurpation found. — When the respondent is Sec. 12. Judgment for costs.
found guilty of usurping into, intruding into, or unlawfully holding or
exercising a public office, position or franchise, judgment shall be rendered

118 Aicka Singson


Notes in Remedial Law Review II
Provisional Remedies, Special Civil Actions, Special Proceedings and Evidence
School Year 2015-2016 REMEDIAL LAW REVIEW II - ATTY. BRONDIAL

In an action brought in accordance with the provisions of this Rule, the court JUDGMENT IN QUO WARRANTO ACTION
may render judgment for costs against either the petitioner, the relator, or
the respondent, or the person or persons claiming to be a corporation, or Of office, position or franchise, judgment shall be rendered that such
may apportion the costs, as justice requires. respondent be ousted and altogether excluded therefrom, and that
the petitioner recover his costs. Such further judgment may be
rendered determining the respective rights in and to the public office,
QUO WARRANTO QUO WARRANTO position or franchise of the parties to the action as justice requires
(RULE 66) (ELECTION CODE)
RIGHTS OF A PERSON ADJUDGED ENTITLED TO PUBLIC
Subject of the petition is in Subject of the petition is in relation
relation to an appointive to an elective office; OFFICE
office;
The issue is the legality of Grounds relied upon are: (a) If the petitioner is adjudged to be entitled to the office, he may sue
the occupancy of the office ineligibility to the position; or for damages against the alleged usurper within one (1) year from the
by virtue of a legal (b) disloyalty to the Republic. entry of judgment establishing his right to the office in question.
appointment;
Petition is brought either May be instituted with the CASES:
to the Supreme Court, the COMELEC by any voter
Court of Appeals or the contesting the election of any a) Mendoza vs. Allias (302 SCRA)
Regional Trial Court; member of Congress, regional,
provincial or city officer; or to FACTS: Petitioner Pedro Mendoza joined the Bureau of Customs in
the MeTC, MTC or MCTC if
1972.
against any barangay official;
Filed within one (1) year Filed within ten (10) days after In September 1994, petitioner received a letter from Deputy Customs
from the time the cause of the proclamation of the results of
Commissioner Cesar Z. Dario, informing him of his termination from
ouster, or the right of the the election;
petitioner to hold the the Bureau of Customs, in view of respondent Allas' appointment as
office or position arose; Director III by President Fidel V. Ramos.
Petitioner is the person Petitioner may be any voter even if
entitled to the office; he is not entitled to the office Petitioner wrote the Customs Commissioner demanding his
reinstatement with full back wages and without loss of seniority
The court has to declare When the tribunal declares the rights. No reply was made.
who the person entitled candidate-elect as ineligible, he will
to the office is if he is be unseated but the person Petitioner filed a petition for quo warranto against respondent Allas
the petitioner. occupying the second place will not before the RTC Paranaque, Branch 258 – GRANTED. The court
be declared as the one duly elected found that petitioner was illegally terminated from office without due
because the law shall consider only
the person who, having duly filed his process of law and in violation of his security of tenure, and that as
certificate of candidacy, received a he was deemed not to have vacated his office, the appointment of
plurality of votes. respondent Allas to the same office was void ab initio. The court

119 Aicka Singson


Notes in Remedial Law Review II
Provisional Remedies, Special Civil Actions, Special Proceedings and Evidence
School Year 2015-2016 REMEDIAL LAW REVIEW II - ATTY. BRONDIAL

ordered the ouster of respondent Allas from the position of Director petition for quo warranto is a proceeding to determine the right of a
III, and at the same time directed the reinstatement of petitioner to person to the use or exercise of a franchise or office and to oust the
the same position with payment of full back salaries and other holder from its enjoyment, if his claim is not well-founded, or if he has
benefits appurtenant thereto. forfeited his right to enjoy the privilege. The action may be
commenced for the Government by the Solicitor General or the fiscal
Respondent Allas appealed to the CA. While the case was pending against individuals who usurp a public office, against a public officer
before said court, respondent Allas was promoted by President whose acts constitute a ground for the forfeiture of his office, and
Ramos to the position of Deputy Commissioner of Customs for against an association which acts as a corporation without being
Assessment and Operations. As a consequence of this promotion, legally incorporated. The action may also be instituted by an
petitioner moved to dismiss respondent's appeal as having been individual in his own name who claims to be entitled to the public
rendered moot and academic. The CA granted the motion and office or position usurped or unlawfully held or exercised by another.
dismissed the case accordingly. The order of dismissal became final
and entry of judgment was made. Where the action is filed by a private person, he must prove that he
is entitled to the controverted position, otherwise respondent has a
Petitioner filed with the court a quo a Motion for Execution of its right to the undisturbed possession of the office. If the court finds for
decision. The court denied the motion on the ground that the the respondent, the judgment should simply state that the
contested position vacated by respondent Allas was now being respondent is entitled to the office.
occupied by respondent Godofredo Olores who was not a party to
the quo warranto petition. If it is found that the respondent or defendant is usurping or intruding
into the office, or unlawfully holding the same, the court may order:
Petitioner filed a special civil action for certiorari and mandamus with
the CA questioning the order of the trial court. The CA dismissed the (1) The ouster and exclusion of the defendant from office;
petition. Hence, this recourse.
(2) The recovery of costs by plaintiff or relator;
ISSUE: WON the CA grossly erred in holding that a writ of
execution may no longer be issued, considering that (3) The determination of the respective rights in and to the office,
respondent Olores who was not a party to the case now position, right, privilege or franchise of all the parties to the action as
occupies the subject position. justice requires.

HELD: NO. The instant petition arose from a special civil action for The decision of the trial court had long become final and executory,
quo warranto under Rule 66 of the Revised Rules of Court. Quo and petitioner prays for its execution. He alleges that he should
warranto is a demand made by the state upon some individual or have been reinstated despite respondent Olores' appointment
corporation to show by what right they exercise some franchise or because the subject position was never vacant to begin with.
privilege appertaining to the state which, according to the Petitioner's removal was illegal and he was deemed never to have
Constitution and laws of the land, they cannot legally exercise except vacated his office when respondent Allas was appointed to the
by virtue of a grant or authority from the state. In other words, a same. Respondent Allas' appointment was null and void and this

120 Aicka Singson


Notes in Remedial Law Review II
Provisional Remedies, Special Civil Actions, Special Proceedings and Evidence
School Year 2015-2016 REMEDIAL LAW REVIEW II - ATTY. BRONDIAL

nullity allegedly extends to respondent Olores, his successor-in- affirmative defenses. All the parties were then required to submit
interest. their respective memoranda.

Ordinarily, a judgment against a public officer in regard to a public Issue: WON RTC has Jurisdiction
right binds his successor in office. This rule, however, is not
applicable in quo warranto cases. A judgment in quo warranto does WON RTC Br-58 is the proper venue
not bind the respondent's successor in office, even though such
Held:
successor may trace his title to the same source. This follows from
the nature of the writ of quo warranto itself. It is never directed to an It should be noted that allegations in a complaint for quo warranto
officer as such, but always against the person-- to determine whether that certain persons usurped the offices, powers and functions of
he is constitutionally and legally authorized to perform any act in, or duly elected members of the board, trustees and/or officers make out
exercise any function of the office to which he lays claim. In the case a case for an intra-corporate controversy. Prior to the enactment of
at bar, the petition for quo warranto was filed by petitioner solely R.A. No. 8799, the Court, adopting Justice Jose Y. Feria’s view,
against respondent Allas. What was threshed out before the trial declared in Unilongo v. Court of Appeals that Section 1, Rule 66 of
court was the qualification and right of petitioner to the contested the 1997 Rules of Civil Procedure is "limited to actions of quo
position as against respondent Ray Allas, not against Godofredo warranto against persons who usurp a public office, position or
Olores. The CA did not err in denying execution of the trial court's franchise; public officers who forfeit their office; and associations
decision. which act as corporations without being legally incorporated," while
"[a]ctions of quo warranto against corporations, or against persons
b) Calleja vs. Panday (483 SCRA)
who usurp an office in a corporation, fall under the jurisdiction of the
Quo warranto is not the proper petition or action because it was not
Securities and Exchange Commission and are governed by its
directed against the office but against the public officer.
rules.”
Respondents filed a petition with the Regional Trial Court for quo
However, actions of quo warranto against persons who usurp an
warranto with Damages and Prayer for Mandatory and Prohibitory
office in a corporation, which were formerly cognizable by the
Injunction, Damages and Issuance of Temporary Restraining Order
Securities and Exchange Commission under PD 902-A, have been
against herein petitioners. Respondents alleged that petitioners, who
transferred to the courts of general jurisdiction. But, this does not
are also among the incorporators and stockholders of said
change the fact that Rule 66 of the 1997 Rules of Civil Procedure
corporation, forcibly and with the aid of armed men usurped the
does not apply to quo warranto cases against persons who usurp an
powers which supposedly belonged to Respondents.
office in a private corporation.
The RTC-Br. 58 then proceeded to issue and serve summons on
As explained in the Unilongo case, Section 1(a) of Rule 66 of the
herein petitioners. Petitioner Tabora filed his Answer raising therein
present Rules no longer contains the phrase "or an office in a
the affirmative defenses of (1) improper venue, (2) lack of
corporation created by authority of law" which was found in the old
jurisdiction, and (3) wrong remedy of quo warranto. Thereafter, the
Rules. Clearly, the present Rule 66 only applies to actions of quo
other petitioners also filed their Answer, also raising the same

121 Aicka Singson


Notes in Remedial Law Review II
Provisional Remedies, Special Civil Actions, Special Proceedings and Evidence
School Year 2015-2016 REMEDIAL LAW REVIEW II - ATTY. BRONDIAL

warranto against persons who usurp a public office, position or it is the Regional Trial Court designated as Special Commercial
franchise; public officers who forfeit their office; and associations Courts in Camarines Sur which shall have jurisdiction over the
which act as corporations without being legally incorporated despite petition for quo warranto filed by herein Respondents.
the passage of R.A. No. 8799.
Evidently, the RTC-Br. 58 in San Jose, Camarines Sur is bereft of
It is, therefore, The Interim Rules of Procedure Governing Intra- jurisdiction over respondents’ petition for quo warranto. Based on the
Corporate Controversies Under R.A. No. 8799 (hereinafter the allegations in the petition, the case was clearly one involving an
Interim Rules) which applies to the petition for quo warranto filed by intra-corporate dispute. The trial court should have been aware that
respondents before the trial court since what is being questioned is under R.A. No. 8799 and the aforementioned administrative
the authority of herein petitioners to assume the office and act as the issuances of this Court, RTC-Br. 58 was never designated as a
board of directors and officers of St. John Hospital, Incorporated. Special Commercial Court; hence, it was never vested with
jurisdiction over cases previously cognizable by the SEC.
The Interim Rules provide thus:
Such being the case, RTC-Br. 58 did not have the requisite authority
Section 1. (a) Cases covered. – These Rules shall govern the
or power to order the transfer of the case to another branch of the
procedure to be observed in civil cases involving the following:
Regional Trial Court. The only action that RTC-Br. 58 could take on
(2) Controversies arising out of intra-corporate, partnership, or
the matter was to dismiss the petition for lack of jurisdiction.
association relations, between and among stockholders, members,
or associates, and between, any or all of them and the corporation,
c) Lokin Jr. vs COMELEC (621 SCRA)
partnership, or association of which they are stockholders, members,
or associates, respectively; FACTS: The CIBAC manifested their intent to participate in the May
(3) Controversies in the election or appointment of directors, 14, 2007 synchronized national and local elections. Together with its
trustees, officers, or managers of corporations, partnerships, or manifestation of intent to participate, CIBAC, through its president,
associations; Emmanuel Joel J. Villanueva, submitted a list of five nominees from
which its representatives would be chosen should CIBAC obtain the
SEC. 5. Venue. – All actions covered by these Rules shall be required number of qualifying votes. The nominees were: (1)
commenced and tried in the Regional Trial Court which has Emmanuel Joel J. Villanueva; (2) herein petitioner Luis K. Lokin, Jr.;
jurisdiction over the principal office of the corporation, partnership, or (3) Cinchona C. Cruz-Gonzales; (4) Sherwin Tugna; and (5) Emil L.
association concerned. Galang.

The next question then is, which branch of the Regional Trial Prior to the elections, however, CIBAC, still through Villanueva, filed
Court has jurisdiction over the present action for quo warrato? a certificate of nomination, substitution and amendment of the list of
Section 5 of the Interim Rules provides that the petition should be nominees dated May 7, 2007,6 whereby it withdrew the nominations
commenced and tried in the Regional Trial Court that has jurisdiction of Lokin, Tugna and Galang and substituted: (1) Villanueva, (2) Cruz-
over the principal office of the corporation. It is undisputed that the Gonzales, and (3) Borje.
principal office of the corporation is situated at Goa, Camarines Sur.
Thus, pursuant to A.M. No. 00-11-03-SC and A.M. No. 03-03-03-SC,

122 Aicka Singson


Notes in Remedial Law Review II
Provisional Remedies, Special Civil Actions, Special Proceedings and Evidence
School Year 2015-2016 REMEDIAL LAW REVIEW II - ATTY. BRONDIAL

Villanueva sent a letter to COMELEC Chairperson Benjamin Abalos, No. 7804 expanded Section 8 of R.A. No. 7941.18 the law that the
transmitting therewith the signed petitions of more than 81% of the COMELEC seeks to thereby implement.
CIBAC members, in order to confirm the withdrawal of the
nomination of Lokin, Tugna and Galang and the substitution of Borje. ISSUE: WON quo warranto was proper

CIBAC filed with the COMELEC en banc sitting as the National HELD: A special civil action for quo warranto refers to questions of
Board of Canvassers a motion seeking the proclamation of Lokin as disloyalty to the State, or of ineligibility of the winning candidate. The
its second nominee. objective of the action is to unseat the ineligible person from the
office, but not to install the petitioner in his place. Any voter may
The motion was opposed by Villanueva and Cruz-Gonzales. initiate the action, which is, strictly speaking, not a contest where the
Notwithstanding Villanueva’s filing of the certificate of nomination, parties strive for supremacy because the petitioner will not be seated
substitution and amendment of the list of nominees, the COMELEC even if the respondent may be unseated.
failed to act on the matter, prompting Villanueva to file a petition to
confirm the certificate of nomination, substitution and amendment of The controversy involving Lokin is neither an election protest nor an
the list of nominees of CIBAC. action for quo warranto, for it concerns a very peculiar situation in
which Lokin is seeking to be seated as the second nominee of
The COMELEC issued Resolution whereby it resolved to set the CIBAC. Although an election protest may properly be available to
matter pertaining to the validity of the withdrawal of the nominations one party-list organization seeking to unseat another party-list
of Lokin, Tugna and Galang and the substitution of Borje for proper organization to determine which between the defeated and the
disposition and hearing. winning party-list organizations actually obtained the majority of the
legal votes, Lokin’s case is not one in which a nominee of a
The COMELEC en banc approves the withdrawal of the nomination particular party-list organization thereby wants to unseat another
of Lokin, Tugna and Galang and the substitution thereby with Cruz- nominee of the same party-list organization. Neither does an action
Gonzales and Borje as nominees for the party list CIBAC. for quo warranto lie, considering that the case does not involve the
ineligibility and disloyalty of Cruz-Gonzales to the Republic of the
Lokin seeks through mandamus to compel respondent COMELEC to
Philippines, or some other cause of disqualification for her.
proclaim him as the official second nominee of CIBAC.
Lokin has correctly brought this special civil action for certiorari
In G.R. No. 180443, Lokin assails Section 13 of Resolution No. 7804
against the COMELEC to seek the review of the September 14, 2007
promulgated on January 12, 2007;16 and the resolution dated
resolution of the COMELEC in accordance with Section 7 of Article
September 14, 2007 issued in E.M. No. 07-054 (approving CIBAC’s
IX-A of the 1987 Constitution, notwithstanding the oath and
withdrawal of the nominations of Lokin, Tugna and Galang as
assumption of office by Cruz-Gonzales. The constitutional mandate
CIBAC’s second, third and fourth nominees, respectively, and the
is now implemented by Rule 64 of the 1997 Rules of Civil Procedure,
substitution by Cruz-Gonzales and Borje in their stead, based on the
which provides for the review of the judgments, final orders or
right of CIBAC to change its nominees under Section 13 of
resolutions of the COMELEC and the Commission on Audit. As Rule
Resolution No. 7804).17 He alleges that Section 13 of Resolution
64 states, the mode of review is by a petition for certiorari in

123 Aicka Singson


Notes in Remedial Law Review II
Provisional Remedies, Special Civil Actions, Special Proceedings and Evidence
School Year 2015-2016 REMEDIAL LAW REVIEW II - ATTY. BRONDIAL

accordance with Rule 65 to be filed in the Supreme Court within a him to take the oath of office as Mayor of San Antonio, Zambales.
limited period of 30 days. Undoubtedly, the Court has original and Secretary Jesse M. Robredo allowed Aratea to take an oath of office
exclusive jurisdiction over Lokin’s petitions for certiorari and for as "the permanent Municipal Mayor of San Antonio, Zambales
mandamus against the COMELEC. without prejudice however to the outcome of the cases pending
before the COMELEC.
d) Aratea vs. Comelec (683 SCRA) COMELEC En Banc issued a Resolution disqualifying Lonzanida
Facts: Romeo D. Lonzanida (Lonzanida) and Estela D. Antipolo from running for Mayor in the May 2010 elections. Antipolo filed a
(Antipolo) were candidates for Mayor of San Antonio, Zambales in Motion for Leave to Intervene and to Admit Attached Petition-in-
the May 2010 National and Local Elections. Dra. Sigrid S. Rodolfo Intervention. She claimed her right to be proclaimed as Mayor of San
(Rodolfo) filed a petition to disqualify Lonzanida and to deny due Antonio, Zambales because Lonzanida ceased to be a candidate. In
course or to cancel Lonzanida’s certificate of candidacy on the his Comment Aratea asserted that Antipolo, as the candidate who
ground that Lonzanida was elected, and had served, as mayor of received the second highest number of votes, could not be
San Antonio, Zambales for four consecutive terms immediately prior proclaimed as the winning candidate. Since Lonzanida’s
to the term for the May 2010 elections. Rodolfo asserted that disqualification was not yet final during election day, the votes cast in
Lonzanida made a false material representation in his certificate of his favor could not be declared stray. Lonzanida’s subsequent
candidacy when Lonzanida certified under oath that he was eligible disqualification resulted in a permanent vacancy in the Office of
for the office he sought election. Mayor, and Aratea, as the duly-elected Vice-Mayor, was mandated
The COMELEC Second Division rendered a Resolution cancelling by Section 44 of the Local Government Code to succeed as Mayor.
Lonzanida’s certificate of candidacy. Lonzanida’s motion for The COMELEC’s granted the Motion.
reconsideration before the COMELEC En Banc remained pending
during the May 2010 elections. Lonzanida and Efren Racel Aratea The sole issue to be resolved at this juncture is how to fill the
(Aratea) garnered the highest number of votes and were respectively vacancy resulting from Lonzanida’s disqualification.
proclaimed Mayor and Vice-Mayor.
Aratea took his oath of office as Acting Mayor before RTC Judge The Court’s Ruling
Raymond C. Viray. On the same date, Aratea wrote the Department We hold that Antipolo, the alleged "second placer," should be
of Interior and Local Government (DILG) and requested for an proclaimed Mayor because Lonzanida’s certificate of candidacy was
opinion on whether, as Vice-Mayor, he was legally required to void ab initio. In short, Lonzanida was never a candidate at all. All
assume the Office of the Mayor in view of Lonzanida’s votes for Lonzanida were stray votes. Thus, Antipolo, the only
disqualification. DILG Legal Opinion No. 117, S. 2010 stated that qualified candidate, actually garnered the highest number of votes
Lonzanida was disqualified to hold office by reason of his criminal for the position of Mayor.
conviction. As a consequence of Lonzanida’s disqualification, the Indeed, the Court has already likened a proceeding under Section 78
Office of the Mayor was deemed permanently vacant. Thus, Aratea to a quo warranto proceeding under Section 253 of the OEC since
should assume the Office of the Mayor in an acting capacity without they both deal with the eligibility or qualification of a candidate, with
prejudice to the COMELEC’s resolution of Lonzanida’s motion for the distinction mainly in the fact that a "Section 78" petition is filed
reconsideration. In another letter, Aratea requested the DILG to allow

124 Aicka Singson


Notes in Remedial Law Review II
Provisional Remedies, Special Civil Actions, Special Proceedings and Evidence
School Year 2015-2016 REMEDIAL LAW REVIEW II - ATTY. BRONDIAL

before proclamation, while a petition for quo warranto is filed after who discovers such misrepresentations and who would want the
proclamation of the winning candidate. disqualification of the candidate committing the misrepresentations. It
would seem, therefore, that there could indeed be a gap between the
It is obvious from a reading of the laws and jurisprudence that there time of the discovery of the misrepresentation, (when the discovery
is an overlap in the grounds for eligibility and ineligibility vis-à-vis is made after the 25-day period under Sec. 78 of the Code has
qualifications and disqualifications. For example, a candidate may lapsed) and the time when the proclamation of the results of the
represent that he is a resident of a particular Philippine locality when election is made. During this so-called "gap" the would-be petitioner
he is actually a permanent resident of another country. In cases of (who would seek the disqualification of the candidate) is left with
such overlap, the petitioner should not be constrained in his choice nothing to do except to wait for the proclamation of the results, so
of remedy when the Omnibus Election Code explicitly makes that he could avail of a remedy against the misrepresenting
available multiple remedies. Section 78 allows the filing of a petition candidate, that is, by filing a petition for quo warranto against him.
to deny due course or to cancel a certificate of candidacy before the Respondent Commission sees this "gap" in what it calls a procedural
election, while Section 253 allows the filing of a petition for quo gap which, according to it, is unnecessary and should be remedied.
warranto after the election. Despite the overlap of the grounds, one
should not confuse a petition for disqualification using grounds e) De Castro vs. Carlos (696 SCRA)
enumerated in Section 68 with a petition to deny due course or to
cancel a certificate of candidacy under Section 78. FACTS: On 29 July 2009, then President Gloria Macapagal Arroyo
appointed petitioner as AGM0.
The distinction between a petition under Section 68 and a petition
under Section 78 was discussed in Loong v. Commission on Atty. Francis N. Tolentino, chairperson of the MMDA, issued Office
Elections with respect to the applicable prescriptive period. Order No. 106, designating Corazon B. Cruz as officer-in-charge
Respondent Nur Hussein Ututalum filed a petition under Section 78 (OIC) of the Office of the AGMO. Petitioner was then reassigned to
to disqualify petitioner Benjamin Loong for the office of Regional the Legal and Legislative Affairs Office, Office of the General
Vice-Governor of the Autonomous Government of Muslim Mindanao Manager. The service vehicle and the office space previously
for false representation as to his age. The petition was filed 16 days assigned to him were withdrawn and assigned to other employees.
after the election, and clearly beyond the prescribed 25 day period
Subsequently, on 2 November 2010, Chairperson Tolentino
from the last day of filing certificates of candidacy. This Court ruled
designated respondent as OIC of the Office of the AGMO by virtue of
that Ututalum’s petition was one based on false representation under
Memorandum Order No. 24, which in turn cited OP Memorandum
Section 78, and not for disqualification under Section 68. Hence, the
Circular No. 2 as basis. Thereafter, the name of petitioner was
25-day prescriptive period provided in Section 78 should be strictly
stricken off the MMDA payroll, and he was no longer paid his salary
applied. We recognized the possible gap in the law:
beginning November 2010.
It is true that the discovery of false representation as to material facts
required to be stated in a certificate of candidacy, under Section 74 Petitioner sought a clarification from the Career Executive Service
of the Code, may be made only after the lapse of the 25-day period Board (CESB) as to the proper classification of the position of
prescribed by Section 78 of the Code, through no fault of the person

125 Aicka Singson


Notes in Remedial Law Review II
Provisional Remedies, Special Civil Actions, Special Proceedings and Evidence
School Year 2015-2016 REMEDIAL LAW REVIEW II - ATTY. BRONDIAL

AGMO. In sum, she said, he was not covered by OP Memorandum A direct invocation of this Court’s jurisdiction is allowed only when
Circular Nos. 1 and 2. there are special and important reasons that are clearly and
specifically set forth in a petition. The rationale behind this policy
Petitioner was later offered the position of Director IV of MMDA arises from the necessity of preventing (1) inordinate demands upon
Public Health and Safety Services and/or MMDA consultant. He the time and attention of the Court, which is better devoted to those
turned down the offer, claiming that it was a demotion in rank. matters within its exclusive jurisdiction; and (2) further overcrowding
of the Court’s docket.
Demanding payment of his salary and reinstatement in the monthly
payroll, petitioner sent a letter to Edenison Faisan, assistant general Petitioner’s excuses are not special and important circumstances
manager (AGM) for Finance and Administration; and Lydia Domingo, that would allow a direct recourse to this Court. More so, mere
Director III, Administrative Services. For his failure to obtain an speculation and doubt to the exercise of judicial discretion of the
action or a response from MMDA, he then made a formal demand for lower courts are not and cannot be valid justifications to hurdle the
his reinstatement as AGMO through a letter addressed to the Office hierarchy of courts. Thus, the Petition must be dismissed.
of the President on 17 December 2010.
*Petitioner undisputedly lacked CES eligibility. Thus, he did not hold
However, on 4 January 2011, President Benigno S. Aquino III the position of AGMO in a permanent capacity or acquire security of
(President Aquino) appointed respondent as the new AGMO of the tenure in that position. Otherwise stated, his appointment was
MMDA. On 10 January 2011, the latter took his oath of office. temporary and "co-terminus with the appointing authority." Therefore,
we find no violation of security of tenure when petitioner was
ISSUE: Whether respondent Carlos was validly appointed by
replaced by respondent upon the latter’s appointment to the position
President Aquino to the position of AGMO of the MMDA
of AGMO by President Aquino.
HELD:
Even granting for the sake of argument that the position of AGMO is
*Although Section 5(1) of Article VIII of the 1987 Constitution yet to be classified by the CESB, petitioner’s appointment is still
explicitly provides that the Supreme Court has original jurisdiction deemed coterminous pursuant to CESB Resolution.
over petitions for certiorari, prohibition, mandamus, quo warranto,
Therefore, considering that petitioner is an appointee of then
and habeas corpus, the jurisdiction of this Court is not exclusive but
President Arroyo whose term ended on 30 June 2010, petitioner’s
is concurrent with that of the Court of Appeals and regional trial court
term of office was also deemed terminated upon the assumption of
and does not give petitioner unrestricted freedom of choice of court
President Aquino.
forum. The hierarchy of courts must be strictly observed.
Likewise, it is inconsequential that petitioner was allegedly replaced
Settled is the rule that "the Supreme Court is a court of last resort
by another non-CESO eligible. In a quo warranto proceeding, the
and must so remain if it is to satisfactorily perform the functions
person suing must show that he has a clear right to the office
assigned to it by the fundamental charter and immemorial tradition."
allegedly held unlawfully by another. Absent a showing of that right,
A disregard of the doctrine of hierarchy of courts warrants, as a rule,
the outright dismissal of a petition.

126 Aicka Singson


Notes in Remedial Law Review II
Provisional Remedies, Special Civil Actions, Special Proceedings and Evidence
School Year 2015-2016 REMEDIAL LAW REVIEW II - ATTY. BRONDIAL

the lack of qualification or eligibility of the supposed usurper is Mia case: Manila International Airport expanded and a lot of private
immaterial. owners complained. They demanded just compensation. The
government said they did not take their property. SC: Although the
All the foregoing considered, the petition merits an outright dismissal government did not take their property, the use of their properties
for disregarding the hierarchy of courts and petitioner’s lack of cause were diminished because their poultry business was affected as the
of action against respondent for failure to sufficiently show that he chickens were scared by the airplane.
has undisturbed rights to the position of AGMO of the MMDA.
“public use” – the utility should redound to the benefit of the general
F. EXPROPRIATION (Rule 67) public. Even though it caters only to a few, provided it will redound to
the general will, it is still considered public use. Example: Bayani
Expropriation is an exercise of the State’s power of eminent domain
Fernando’s urinals. Although only men could use them, the effect is
wherein the government takes a private property for public purpose to the general public. Women don’t pee just anywhere while men do.
upon payment of just compensation.
Just Compensation=Fair Market Value + Consequential Damages -
Note: To be filed with the RTC; this is action incapable of pecuniary
Consequential Benefit
estimation.
The consequential benefit shall not be greater than the
The Right of Eminent Domain
consequential damages.
1987 Constitution, Article III, Section 9. Private property shall not be Fair Market Value – the price at which the buyer is willing to buy the
taken for public use without just compensation.
property and the vendor is willing to sell.
This is the fundamental or constitutional basis of expropriation. Prior
The determination of just compensation is a judicial function. The
to the amendment of the Rules of Court, the title of this Rule is executive and legislative branches have nothing to do with the
eminent domain. It is now corrected. Eminent domain is the right and determination of just compensation.
the activation of the right of eminent domain is expropriation.
Which court has jurisdiction over expropriation? RTC (as settled in
“property” – refers to private property. Not public property because
Brgy. San Roque vs Heirs of Pastor). Expropriation, foreclosure of
the same already belongs to the State. This may refer to real or real estate mortgage and partition are actions incapable of pecuniary
personal property (example: jewelry, clothes, shoes).
estimation, the jurisdiction of which is with the RTC.
“taking” – does not only mean actual seizure but also includes What is the venue? Where the property is located since it is a real
diminution of benefits or use. In the order of nature, utility is prior to action.
ownership.
The Regalian doctrine states that all properties belong to the State.
He who claims otherwise has the burden of proof.

127 Aicka Singson


Notes in Remedial Law Review II
Provisional Remedies, Special Civil Actions, Special Proceedings and Evidence
School Year 2015-2016 REMEDIAL LAW REVIEW II - ATTY. BRONDIAL

Don’t confuse expropriation with reclamation. The latter is a means 1. An ordinance is enacted by the local legislative council authorizing
of acquiring title over the property but it is not expropriation. the local chief executive, in behalf of the local government unit, to
exercise the power of eminent domain or pursue expropriation
The basis for the exercise of the power of eminent domain by proceedings over a particular private property .
local government units is section 19 of RA 7160 which provides
that: 2. The power of eminent domain is exercised for public use, purpose
or welfare, or for the benefit of the poor and the landless.
A local government unit may, through its chief executive and acting
pursuant to an ordinance, exercise the power of eminent domain for 3. There is payment of just compensation, as required under Section
public use, purpose, or welfare for the benefits of the poor and the 9, Article III of the Constitution, and other pertinent laws.
landless, upon payment of just compensation, pursuant to the
provisions of the Constitution and pertinent laws; Provided, however, 4. A valid and definite offer has been previously made to the owner
That the power of eminent domain may not be exercised unless a of the property sought to be expropriated, but said offer was not
valid and definite offer has been previously made to the owner, and accepted.
such offer was not accepted; Provided, further, That the local
Who may expropriate
government unit may immediately take possession of the property
upon the filing of the expropriation proceedings and upon making a The government and its instrumentalities, including government
deposit with the proper court of at least fifteen percent (15%) of the owned and controlled corporations, may expropriate.
fair market value of the property based on the current tax declaration
of the property to be expropriated; Provided, finally, That the amount The respondent is the owner of the property.
to be paid for the expropriated property shall be determined by the
proper court, based on the fair market value at the time of the taking When the government expropriates, it can immediately take
of the property. possession of the property unless a third party holds possession of
the property adverse to the title of the respondent. A writ of
Despite the existence of this legislative grant in favor of local possession must first be executed.
governments, it is still the duty of the courts to determine whether the
power of eminent domain is being exercised in accordance with the Assuming it is owned by the respondent, can the petitioner
delegating law. In fact, the courts have adopted a more censorious immediately take possession of the property? No, it must first pay
attitude in resolving questions involving the proper exercise of this deposit. Under the Local Government Code, the deposit must be
delegated power by local bodies, as compared to instances when it 15% of the zonal valuation.
is directly exercised by the national legislature.
Asia’s Emerging Dragon vs DOTC: Amending the LGC, the deposit
The courts have the obligation to determine whether the following is now 100% of the zonal value. This is not for all. This refers only to
requisites have been complied with by the local government unit infrastructure projects of the government.
concerned:

128 Aicka Singson


Notes in Remedial Law Review II
Provisional Remedies, Special Civil Actions, Special Proceedings and Evidence
School Year 2015-2016 REMEDIAL LAW REVIEW II - ATTY. BRONDIAL

MATTERS TO ALLEGE IN COMPLAINT FOR EXPROPRIATION What is the mode of appeal in expropriation? By record on
appeal because there are multiple appeals. The appeal is by record
An expropriation proceeding is commenced by the filing of a verified on appeal and not by notice of appeal because it constitutes more
complaint which shall: than one issue. The order of expropriation or condemnation is a final
order which is appealable by record on appeal.
a) State with certainty the right of the plaintiff to expropriation and the
purpose thereof; Note:

b) Describe the real or personal property sought to be expropriated; Clearly, although the determination and award of just compensation
and to the defendant is indispensable to the transfer of ownership in favor
of the plaintiff, it is but the last stage of the expropriation
c) Join as defendants all persons owning or claiming to own, or
proceedings, which cannot be arrived at without an initial finding by
occupying, any part of the property or interest therein showing as far
the court that the plaintiff has a lawful right to take the property
as practicable the interest of each defendant. If the plaintiff cannot
sought to be expropriated, for the public use or purpose described in
with accuracy identify the real owners, averment to that effect must
the complaint. An order of condemnation or dismissal at this stage
be made in the complaint.
would be final, resolving the question of whether or not the plaintiff
TWO STAGES IN EVERY ACTION FOR EXPROPRIATION has properly and legally exercised its power of eminent domain.

1) Determination of the authority of the plaintiff to expropriate Also, it is noted that as soon as the complaint is filed the plaintiff
(appealable already at this stage) – this includes an inquiry into the shall already have the right to enter upon the possession of the real
propriety of the expropriation, its necessity and the public purpose. property involved upon depositing with the court at least fifteen
percent (15%) of the fair market value of the property based on the
This stage will end in the issuance of an order of expropriation if the current tax declaration of the property to be expropriated. Therefore,
court finds for the plaintiff or in the dismissal of the complaint if it an ordinance promulgated by the local legislative body authorizing its
finds otherwise. local chief executive to exercise the power of eminent domain is
necessary prior to the filing by the latter of the complaint with the
2) Determination of just compensation through the court-appointed proper court, and not only after the court has determined the amount
commissioners. of just compensation to which the defendant is entitled

Will the commissioners render judgment? No. They will just DEFENSES AND OBJECTIONS (SECTION 3)
submit a report.
Omnibus Motion Rule — a motion attacking a pleading, order,
Is this report conclusive upon the court? No. The court can do judgment or proceeding shall include all objections then available,
any other thing that would resolve the issue of just compensation. and all objections not so included shall be deemed waived.
They are not limited to accepting or rejecting or recommitting the
report of the commission.

129 Aicka Singson


Notes in Remedial Law Review II
Provisional Remedies, Special Civil Actions, Special Proceedings and Evidence
School Year 2015-2016 REMEDIAL LAW REVIEW II - ATTY. BRONDIAL

If a defendant has no objection or defense to the action or the taking to be determined as of the date of the taking of the property or the
of his property, he may file and serve a notice of appearance and a filing of the complaint, whichever came first.
manifestation to that effect, specifically designating or identifying the
property in which he claims to be interested, within the time stated in A final order sustaining the right to expropriate the property may be
the summons. Thereafter, he shall be entitled to notice of all appealed by any party aggrieved thereby.
proceedings affecting the same.
Such appeal, however, shall not prevent the court from determining
If a defendant has any objection to the filing of or the allegations in the just compensation to be paid.
the complaint, or any objection or defense to the taking of his
After the rendition of such an order, the plaintiff shall not be
property, he shall serve his answer within the time stated in the
permitted to dismiss or discontinue the proceeding except on such
summons. The answer shall specifically designate or identify the
terms as the court deems just and equitable.
property in which he claims to have an interest, state the nature and
extent of the interest claimed, and adduce all his objections and  Appealable with the CA
defenses to the taking of his property. No counterclaim, cross-claim
or third-party complaint shall be alleged or allowed in the answer or ASCERTAINMENT OF JUST COMPENSATION
any subsequent pleading.
(Section 5)
A defendant waives all defenses and objections not so alleged but
the court, in the interest of justice, may permit amendments to the The order of expropriation merely declares that the plaintiff has the
answer to be made not later than ten (10) days from the filing lawful to expropriate the property but contains no ascertainment of
thereof. the compensation to be paid to the owner of the property.

However, at the trial of the issue of just compensation, whether or So upon the rendition of the order of expropriation, the court shall
not a defendant has previously appeared or answered, he may appoint not more than three (3) commissioners to ascertain the just
present evidence as to the amount of the compensation to be paid compensation for the property. Objections to the appointment may
for his property, and he may share in the distribution of the award. be made within 10 days from service of the order of appointment.

ORDER OF EXPROPRIATION (Section 4) The commissioners are entitled to fees and their fees shall be taxed
as part of the costs of the proceedings, and all costs shall be paid by
If the objections to and the defenses against the right of the plaintiff the plaintiff except those costs of rival claimants litigating their
to expropriate the property are overruled, or when no party appears claims. Where the principal issue is the determination of just
to defend as required by this Rule, the court may issue an order of compensation, a hearing before the commissioners is indispensable
expropriation declaring that the plaintiff has a lawful right to take the to allow the parties to present evidence on the issue of just
property sought to be expropriated, for the public use or purpose compensation. Although the findings of the commissioners may be
described in the complaint, upon the payment of just compensation disregarded and the trial court may substitute its own estimate of the
value, the latter may do so only for valid reasons, that is where the

130 Aicka Singson


Notes in Remedial Law Review II
Provisional Remedies, Special Civil Actions, Special Proceedings and Evidence
School Year 2015-2016 REMEDIAL LAW REVIEW II - ATTY. BRONDIAL

commissioners have applied illegal principles to the evidence such report, the clerk of the court shall serve copies thereof on all
submitted to them, where they have disregarded a clear interested parties, with notice that they are allowed ten (10) days
preponderance of evidence, or where the amount allowed is either within which to file objections to the findings of the report, if they so
grossly inadequate or excessive. desire. Action upon the report. Upon the expiration of the period of
ten (10) days referred to in the preceding section, or even before the
APPOINTMENT OF COMMISSIONERS; COMMISSIONER’S expiration of such period but after all the interested parties have filed
REPORT; COURT ACTION UPON COMMISSIONER’S REPORT their objections to the report or their statement of agreement
therewith, the court may, after hearing, accept the report and render
(Sections 6,7,8)
judgment in accordance therewith; or, for cause shown, it may
Appointment. Upon the rendition of the order of expropriation, the recommit the same to the commissioners for further report of facts;
court shall appoint not more than three (3) competent and or it may set aside the report and appoint new commissioners; or it
disinterested persons as commissioners to ascertain and report to may accept the report in part and reject it in part; and it may make
the court the just compensation for the property sought to be taken. such order or render such judgment as shall secure to the plaintiff
The order of appointment shall designate the time and place of the the property essential to the exercise of his right of expropriation,
first session of the hearing to be held by the commissioners and and to the defendant just compensation for the property so taken.
specify the time within which their report shall be submitted to the
RIGHTS OF PLAINTIFF UPON JUDGMENT AND PAYMENT
court. Copies of the order shall be served on the parties. Objections
to the appointment of any of the commissioners shall be filed with the (Section 10)
court within ten (10) days from service, and shall be resolved within
thirty (30) days after all the commissioners shall have received After payment of the just compensation as determined in the
copies of the objections. judgment, the plaintiff shall have the right to enter upon the property
expropriated and to appropriate the same for the public use or
Report. The court may order the commissioners to report when any purpose defined in the judgment or to retain possession already
particular portion of the real estate shall have been passed upon by previously made.
them, and may render judgment upon such partial report, and direct
the commissioners to proceed with their work as to subsequent Title to the property expropriated passes from the owner to the
portions of the property sought to be expropriated, and may from expropriator upon full payment of just compensation.
time to time so deal with such property. The commissioners shall
make a full and accurate report to the court of all their proceedings, EFFECT OF RECORDING OF JUDGMENT (Section 13)
and such proceedings shall not be effectual until the court shall have
The judgment entered in expropriation proceedings shall state
accepted their report and rendered judgment in accordance with their
definitely, by an adequate description, the particular property or
recommendations. Except as otherwise expressly ordered by the
interest therein expropriated, and the nature of the public use or
court, such report shall be filed within sixty (60) days from the date
purpose for which it is expropriated.
the commissioners were notified of their appointment, which time
may be extended in the discretion of the court. Upon the filing of

131 Aicka Singson


Notes in Remedial Law Review II
Provisional Remedies, Special Civil Actions, Special Proceedings and Evidence
School Year 2015-2016 REMEDIAL LAW REVIEW II - ATTY. BRONDIAL

When real estate is expropriated, a certified copy of such judgment immediately expropriated when it could still have looked for some
shall be recorded in the registry of deeds of the place in which the other properties.
property is situated, and its effect shall be to vest in the plaintiff the
title to the real estate so described for such public use or purpose. City Council of Manila enacted Ordinance No. 7833, authorizing the
expropriation of certain properties in Tondo, which are to be sold and
JUDGMENT ON FORECLOSURE FOR PAYMENT OR SALE distributed to qualified occupants pursuant to the Land Use
Development Program of the City of Manila.
If after the trial, the court finds that the matters set forth in the
complaint are true, it shall render a judgment containing the following
One of the properties sought to be expropriated, denominated as Lot
matters:
1-C, consists of 343.10 square meters owned by Feliza De Guia.
a) An ascertainment of the amount due to the plaintiff upon the After her death, the estate of Feliza De Guia was settled among her
mortgage debt or obligation, including interest and other charges as heirs by virtue of a compromise agreement, which was duly
approved by the court, as well as costs; approved by the Regional Trial Court. The said property was
b) A judgment of the sum found due; transferred to Lee Kuan Hui. The property was subsequently sold to
c) An order that the amount found due be paid to the court or to the Demetria De Guia.
judgment obligee within the period of not less than 90 days nor more
than 120 days from the entry of judgment; and
d) An admonition that in default of such payment the property shall Petitioner City of Manila filed an amended complaint for
be sold at public auction to satisfy the judgment. expropriation, against the supposed owners of the lots, all surnamed
Serrano. Respondents filed a consolidated answer, in which they
The judgment of the court on the above matters is considered a final alleged that their mother, the late Demetria De Guia, had acquired
adjudication of the case and hence, is subject to challenge by the Lot 1-C from Lee Kian Hui; that they had been the bona fide
aggrieved party by appeal or by other post-judgment remedies. occupants of the said parcel of land for more than 40 years; that the
expropriation of Lot 1-C would result in their dislocation, it being the
The period granted to the mortgagor for the payment of the amount only residential land left to them by their deceased mother; and that
found due by the court is not just a procedural requirement but s the said lot was exempt from expropriation because dividing the said
substantive right given by law to the mortgagee as his first chance to parcel of land among them would entitle each of them to only about
save his property from final disposition at the foreclosure sale. 50 square meters of land. Respondents, therefore, prayed that
judgment be rendered declaring Lot 1-C exempt from expropriation
CASES:
and ordering the cancellation of the notice annotated regarding the
pendency of Civil Case for eminent domain filed by petitioner.
a. City of Manila vs. Serrano (359 SCRA)
Expropriation should be the final resort or the last recourse. SC did
Upon motion by petitioner, the court ordered the petitioner to deposit
not accept the argument of the Serranos that their land is only 300+
the amount of P1,825,241.00 equivalent to the assessed value of the
sqm (which would be a lot smaller after it has been distributed
properties.
among the family members). The Serranos still won because Manila
Respondents filed a petition for certiorari with the Court of Appeals.

132 Aicka Singson


Notes in Remedial Law Review II
Provisional Remedies, Special Civil Actions, Special Proceedings and Evidence
School Year 2015-2016 REMEDIAL LAW REVIEW II - ATTY. BRONDIAL

Court of Appeals rendered a decision holding that Lot 1-C is not complied with since no evidentiary hearing had yet been conducted
exempt from expropriation because it undeniably exceeds 300 by the trial court.
square meters which is no longer considered a small property within
the framework of R.A. No. 7279. However, it held that in accordance Rule 67, Sec. 2 provides:
with the ruling in Filstream International Inc. v. Court of Appeals, the
other modes of acquisition of lands enumerated in the law must first Upon the filing of the complaint or at any time thereafter and after
be tried by the city government before it can resort to expropriation. due notice to the defendant, the plaintiff shall have the right to take
As petitioner failed to show that it had done so, the Court of Appeals or enter upon the possession of the real property involved if he
gave judgment for respondents and enjoined petitioner from deposits with the authorized government depositary an amount
expropriating Lot 1-C. equivalent to the assessed value of the property for purposes of
taxation to be held by such bank subject to the orders of the court.
Issue: WON the expropriation is proper Such deposit shall be in money, unless in lieu thereof the court
authorizes the deposit of a certificate of deposit of a government
It is clear that respondents raised in issue the propriety of the bank of the Republic of the Philippines payable on demand to the
expropriation of their property in connection with R.A. No. 7279. authorized government depositary.
Although what was discussed at length in their petition before the
Court of Appeals was whether or not the said property could be If personal property is involved, its value shall be provisionally
considered a small property within the purview of the exemption ascertained and the amount to be deposited shall be fixed by the
under the said law, the other provisions of the said law concerning court.
expropriation proceedings need also be looked into to address the
first issue raised by respondents and to determine whether or not After such deposit is made the court shall order the sheriff or other
expropriation of Lot 1-C was proper under the circumstances. The proper officer to forthwith place the plaintiff in possession of the
Court of Appeals properly considered relevant provisions of R.A. No. property involved and promptly submit a report thereof to the court
7279 to determine the issues raised by respondents. Whether or not with service of copies to the parties.
it correctly applied the doctrine laid down in Filstream in resolving the
issues raised by respondents, however, is a different matter Thus, a writ of execution may be issued by a court upon the filing by
altogether, and this brings us to the next point. the government of a complaint for expropriation sufficient in form and
substance and upon deposit made by the government of the amount
Petitioner contends that the Court of Appeals erroneously presumed equivalent to the assessed value of the property subject to
that Lot 1-C has been ordered condemned in its favor when the fact expropriation. Upon compliance with these requirements, the
is that the order of the trial court, merely authorized the issuance of a issuance of the writ of possession becomes ministerial. In this case,
writ of possession and petitioner’s entry into the property pursuant to these requirements were satisfied and, therefore, it became the
Rule 67, Sec. 2. At that stage, it was premature to determine ministerial duty of the trial court to issue the writ of possession.
whether the requirements of R.A. No. 7279, Sec. 9-10 have been

133 Aicka Singson


Notes in Remedial Law Review II
Provisional Remedies, Special Civil Actions, Special Proceedings and Evidence
School Year 2015-2016 REMEDIAL LAW REVIEW II - ATTY. BRONDIAL

The Court of Appeals, however, ruled that petitioner failed to comply ordered the expropriation of the lots upon NPC’s payment of
with the requirements laid down in Sec. 9-10 of R.A. No. 7279 and P207,790.
reiterated in the Filstream ruling. This is error. The ruling in
Filstream was necessitated because an order of condemnation had NPC filed its second expropriation case against Pobre to acquire an
already been issued by the trial court in that case. Thus, the additional 5,554 square meters of the Property
judgment in that case had already become final. In this case, the
Pobre filed a motion to dismiss the second complaint for
trial court has not gone beyond the issuance of a writ of possession.
expropriation. Pobre claimed that NPC damaged his Property.
Hearing is still to be held to determine whether or not petitioner
Pobre prayed for just compensation of all the lots affected by NPC’s
indeed complied with the requirements provided in R.A. No. 7279. It
actions and for the payment of damages.
is, therefore, premature at this stage of the proceedings to find that
petitioner resorted to expropriation without first trying the other NPC filed a motion to dismiss the second expropriation case on the
modes of acquisition enumerated in Sec. 10 of the law. ground that NPC had found an alternative site and that NPC had
already abandoned in 1981 the project within the Property due to
Whether petitioner has complied with these provisions requires the
Pobre’s opposition. The trial court granted NPC’s motion to dismiss
presentation of evidence, although in its amended complaint
but the trial court allowed Pobre to adduce evidence on his claim for
petitioner did allege that it had complied with the requirements. The
damages. The trial court admitted Pobre’s exhibits on the damages
determination of this question must await the hearing on the
because NPC failed to object.
complaint for expropriation, particularly the hearing for the
condemnation of the properties sought to be expropriated. On 30 August 1985, the trial court ordered the case submitted for
Expropriation proceedings consists of two stages: first, decision since NPC failed to appear to present its evidence. The trial
condemnation of the property after it is determined that its acquisition court denied NPC’s motion to reconsider the submission of the case
will be for a public purpose or public use and, second, the for decision.
determination of just compensation to be paid for the taking of
private property to be made by the court with the assistance of not NPC filed a petition for certiorari with the then IAC, questioning the
more than three commissioners. 30 August 1985 Order of the trial court. The IAC dismissed NPC’s
petition but directed the lower court to rule on NPC’s objections to
b. National Power Corp vs. CA (436 SCRA) Pobre’s documentary exhibits.

FACTS: Petitioner NPC is authorized by law to acquire property and The trial court issued its Decision in favor of Pobre. On 13 July 1987,
exercise the right of eminent domain. Private respondent Pobre is NPC filed its MR – DENIED.
the owner of a 68,969 square-meter land Property located in Tiwi,
Albay. NPC appealed to the CA - upheld the decision of the trial court but
deleted the award of attorney’s fees. The CA denied NPC’s motion
NPC filed its expropriation case against Pobre to acquire an for reconsideration.
8,311.60 square-meter portion of the Property. The trial court

134 Aicka Singson


Notes in Remedial Law Review II
Provisional Remedies, Special Civil Actions, Special Proceedings and Evidence
School Year 2015-2016 REMEDIAL LAW REVIEW II - ATTY. BRONDIAL

ISSUE: WON NPC as plaintiff had the right to move for the motion to dismiss where he should present all of his objections and
automatic dismissal of its complaint as Pobre had yet to serve defenses to the taking of his property for the purpose specified in the
an answer or a motion for summary judgment on NPC complaint. In short, in expropriation cases under Section 3 of Rule
67, the motion to dismiss took the place of the answer.
HELD: NO. We must correct NPC’s claim that it filed the notice of
dismissal just “shortly” after it had filed the complaint for The records show that Pobre had already filed and served on NPC
expropriation. While NPC had intimated several times to the trial his “motion to dismiss/answer” even before NPC filed its own motion
court its desire to dismiss the expropriation case, it was only on 2 to dismiss. NPC filed its notice of dismissal of the complaint on 2
January 1985 that NPC filed its notice of dismissal. It took NPC more January 1985. However, as early as 10 December 1984, Pobre had
than five years to actually file the notice of dismissal. Five years is already filed with the trial court and served on NPC his “motion to
definitely not a short period of time. NPC obviously dilly-dallied in dismiss/answer.” A certain Divina Cerela received Pobre’s pleading
filing its notice of dismissal while NPC meanwhile burdened Pobre’s on behalf of NPC. Unfortunately for NPC, even Section 1, Rule 17 of
property rights. the 1964 Rules of Court could not save its cause.

Even a timely opposition against Pobre’s claim for damages would In expropriation cases, there is no such thing as the plaintiff’s matter
not yield a favorable ruling for NPC. It is not Section 1, Rule 17 of of right to dismiss the complaint precisely because the landowner
the 1964 Rules of Court that is applicable to this case but Rule 67 of may have already suffered damages at the start of the taking. The
the same Rules, as well as jurisprudence on expropriation cases. plaintiff’s right in expropriation cases to dismiss the complaint has
Rule 17 referred to dismissal of civil actions in general while Rule 67 always been subject to court approval and to certain conditions. The
specifically governed eminent domain cases. exceptional right that Section 1, Rule 17 of the 1964 Rules of Court
conferred on the plaintiff must be understood to have applied only to
Eminent domain is the authority and right of the state, as sovereign, other civil actions. The 1997 Rules of Civil Procedure abrogated this
to take private property for public use upon observance of due exceptional right.
process of law and payment of just compensation. The power of
eminent domain may be validly delegated to the local governments, NPC must Pay Just Compensation for the Entire Property
other public entities and public utilities such as NPC. Expropriation is
the procedure for enforcing the right of eminent domain. “Eminent Ordinarily, the dismissal of the expropriation case restores
Domain” was the former title of Rule 67 of the 1964 Rules of Court. possession of the expropriated land to the landowner. However,
In the 1997 Rules of Civil Procedure, which took effect on 1 July when possession of the land cannot be turned over to the landowner
1997, the prescribed method of expropriation is still found in Rule 67, because it is neither convenient nor feasible anymore to do so, the
but its title is now “Expropriation.” only remedy available to the aggrieved landowner is to demand
payment of just compensation.
While Section 1, Rule 17 spoke of the “service of answer or
summary judgment,” the Rules then did not require the filing of an In this case, we agree with the trial and appellate courts that it is no
answer or summary judgment in eminent domain cases. In lieu of an longer possible and practical to restore possession of the Property to
answer, Section 3 of Rule 67 required the defendant to file a single Pobre. The Property is no longer habitable as a resort-subdivision.

135 Aicka Singson


Notes in Remedial Law Review II
Provisional Remedies, Special Civil Actions, Special Proceedings and Evidence
School Year 2015-2016 REMEDIAL LAW REVIEW II - ATTY. BRONDIAL

The Property is worthless to Pobre and is now useful only to NPC. The trial court decreed that the plaintiff is legally entitled to its
Pobre has completely lost the Property as if NPC had physically inherent right of expropriation to, it being shown that it is for public
taken over the entire 68,969 square-meter Property. use and purpose.
Both parties appealed to the Court of Appeals. The Republic
c. Republic vs. Andaya (524 SCRA) contested the awards of severance damages and attorney’s fees
Respondent Ismael Andaya is the registered owner of two parcels of while Andaya demanded just compensation for his entire property
land. These properties are subject to a 60-meter wide perpetual minus the easement. The Court of Appeals modified the trial court’s
easement for public highways, irrigation ditches, aqueducts, and decision by imposing a 6% interest on the consequential damages
other similar works of the government or public enterprise, at no cost from the date of the writ of possession or the actual taking, and by
to the government, except only the value of the improvements deleting the attorney’s fees.
existing thereon that may be affected. Hence, the instant petition.
Petitioner Republic of the Philippines (Republic) negotiated with
Andaya to enforce the 60-meter easement of right-of-way. The Is the Republic liable for just compensation if in enforcing the
parties, however, failed to reach an agreement. legal easement of right-of-way on a property, the remaining area
Republic instituted an action before the Regional Trial Court to would be rendered unusable and uninhabitable?
enforce the easement of right-of-way or eminent domain. The trial
court issued a writ of possession. It also constituted a Board of YES. "Taking," in the exercise of the power of eminent domain,
Commissioners (Board) to determine the just compensation. occurs not only when the government actually deprives or
Eventually, the trial court issued an Order of Expropriation upon dispossesses the property owner of his property or of its ordinary
payment of just compensation. Later, the Board reported that there use, but also when there is a practical destruction or material
was a discrepancy in the description of the property sought to be impairment of the value of his property. Using this standard, there
expropriated. The Republic thus amended its complaint, reducing the was undoubtedly a taking of the remaining area of Andaya’s
60-meter easement to 10 meters, or an equivalent of 701 square property. True, no burden was imposed thereon and Andaya still
meters. retained title and possession of the property. But, as correctly
The Board also reported that the easement would diminish the value observed by the Board and affirmed by the courts a quo, the nature
of the remaining 5,937 square meters. As a result, it recommended and the effect of the floodwalls would deprive Andaya of the normal
the payment of consequential damages amounting to P2,820,430 for use of the remaining areas. It would prevent ingress and egress to
the remaining area. the property and turn it into a catch basin for the floodwaters coming
Andaya objected to the report because although the Republic from the Agusan River.
reduced the easement to 10 meters or an equivalent of 701 square For this reason, in our view, Andaya is entitled to payment of just
meters, the Board still granted it 4,443 square meters. He contended compensation, which must be neither more nor less than the
that the consequential damages should be based on the remaining monetary equivalent of the land. One of the basic principles
area of 9,679 square meters. Thus, the just compensation should be enshrined in our Constitution is that no person shall be deprived of
P11,373,405. The Republic did not file any comment, opposition, nor his private property without due process of law; and in expropriation
objection. cases, an essential element of due process is that there must be just

136 Aicka Singson


Notes in Remedial Law Review II
Provisional Remedies, Special Civil Actions, Special Proceedings and Evidence
School Year 2015-2016 REMEDIAL LAW REVIEW II - ATTY. BRONDIAL

compensation whenever private property is taken for public use. Corporation and Fil-Homes Realty and Development Corporation
Noteworthy, Section 9, Article III of our Constitution mandates that could still be given possession of the properties which were already
private property shall not be taken for public use without just expropriated in favor of the City of Parañaque.
compensation. Before the Court of Appeals, respondents filed a petition for review,
For the purpose of determining the final just compensation, the case they maintained that respondents’ "act of allowing several years to
is remanded to the trial court. Said court is ordered to make the pass without requiring them to vacate nor filing an ejectment case
determination of just compensation payable to respondent Andaya against them amounts to acquiescence or tolerance of their
with deliberate dispatch. possession." Appellate court ruled in favor of respondents.
Respecting the issuance of a writ of possession in the expropriation
d. Asia’s Emerging Dragon vs. DOTC (552 SCRA) proceedings, the appellate court, citing Republic v. Gingoyon, held
the same does not signify the completion of the expropriation
e. Abad vs. Fil-Homes Realty (636 SCRA) proceedings. Hence, the filing of the present petition for review.
Respondents are co-owners of two lots situated in Parañaque City a
Issue: Won there was a complete expropriation proceeding
filed a complaint for unlawful detainer against petitioners before the
Parañaque MeTC. Respondents alleged that petitioners, through
Held: No. In the exercise of the power of eminent domain, the State
tolerance, had occupied the subject lots since 1980 but ignored their
expropriates private property for public use upon payment of just
repeated demands to vacate them.
compensation. A socialized housing project falls within the ambit of
Petitioners countered that there is no possession by tolerance for
public use as it is in furtherance of the constitutional provisions on
they have been in adverse, continuous and uninterrupted possession
social justice.
of the lots for more than 30 years; and that respondent’s
The exercise of expropriation by a local government unit is covered
predecessor-in-interest, Pilipinas Development Corporation, had no
by Section 19 of the Local Government Code (LGC):
title to the lots.
SEC. 19. Eminent Domain. – A local government unit may, through
City of Parañaque filed expropriation proceedings covering the lots
its chief executive and acting pursuant to an ordinance, exercise the
before the Regional Trial Court of Parañaque. A writ of possession
power of eminent domain for public use, or purpose, or welfare for
was consequently issued and a Certificate of Turn-over given to the
the benefit of the poor and the landless, upon payment of just
City. MeTC, rendered judgment in the unlawful detainer case against
compensation, pursuant to the provisions of the Constitution and
petitioners. It added that petitioners cannot claim a better right by
pertinent laws: Provided, however, That the power of eminent
virtue of the issuance of a Writ of Possession for the project
domain may not be exercised unless a valid and definite offer has
beneficiaries have yet to be named.
been previously made to the owner, and such offer was not
On appeal, RTC reversed the MeTC decision and dismissed
accepted: Provided, further, That the local government unit may
respondents’ complaint stating that the Writ of Possession shows
immediately take possession of the property upon the filing of the
that possession over the properties subject of this case had already
expropriation proceedings and upon making a deposit with the
been given to the City of Parañaque since January 19, 2006 after
proper court of at least fifteen percent (15%) of the fair market value
they were expropriated. It is serious error for the court a quo (MTC)
of the property based on the current tax declaration of the property to
to rule in the unlawful detainer case that Magdiwang Realty

137 Aicka Singson


Notes in Remedial Law Review II
Provisional Remedies, Special Civil Actions, Special Proceedings and Evidence
School Year 2015-2016 REMEDIAL LAW REVIEW II - ATTY. BRONDIAL

be expropriated: Provided, finally, That the amount to be paid for the met and complied with before they can be considered to be
expropriated property shall be determined by the proper court, based beneficiaries.
on the fair market value of the property.
Lintag v. National Power Corporation clearly outlines the stages of f. NPC vs. YLCA Sugar Development Corp (712 SCRA)
expropriation, viz:
Expropriation of lands consists of two stages: FACTS: Petitioner NPC is a GOCC created for the purpose of
The first is concerned with the determination of the authority of the undertaking the development of hydroelectric power throughout the
plaintiff to exercise the power of eminent domain and the propriety of Philippines. NPC is thus authorized to exercise the power of eminent
its exercise in the context of the facts involved in the suit. It ends with domain to carry out the said purpose. Respondent YCLA is the
an order, if not of dismissal of the action, "of condemnation declaring registered owner of three parcels of land situated in Puerto Galera,
that the plaintiff has a lawful right to take the property sought to be Oriental Mindoro.
condemned, for the public use or purpose described in the
complaint, upon the payment of just compensation to be determined NPC filed a Complaint for expropriation with the RTC against YCLA
as of the date of the filing of the complaint x x x. and several other individuals. The NPC sought the expropriation of a
The second phase of the eminent domain action is concerned with portion of the parcels of land owned by the said defendants for the
the determination by the court of "the just compensation for the acquisition of an easement of right-of-way over areas that would be
property sought to be taken." This is done by the court with the affected by the construction of transmission lines. YCLA filed its
assistance of not more than three (3) commissioners. Answer, alleging that the Complaint should be dismissed outright
due to NPC’s failure to allege the public use for the intended
It is only upon the completion of these two stages that expropriation expropriation of its properties.
is said to have been completed. The process is not complete until
Meanwhile, on June 4, 1999, the RTC, acting on NPC’s urgent ex-
payment of just compensation. Accordingly, the issuance of the writ
parte motion, issued a writ of possession placing NPC in possession
of possession in this case does not write finis to the expropriation
of the properties sought to be expropriated.
proceedings. To effectuate the transfer of ownership, it is necessary
for the NPC to pay the property owners the final just compensation. On May 2, 2001, the Board of Commissioners submitted its Report,
In the present case, the mere issuance of a writ of possession in the which fixed the amount of just compensation of the subject
expropriation proceedings did not transfer ownership of the lots in properties at P500.00 per sq m. YCLA objected to the amount
favor of the City. Such issuance was only the first stage in recommended by the Board of Commissioners, claiming that the
expropriation. There is even no evidence that judicial deposit had amount of just compensation should be fixed at P900.00 per sq m
been made in favor of respondents prior to the City’s possession of considering the improvements in their properties.
the lots, contrary to Section 19 of the LGC.
Petitioners cannot thus claim any right over the lots on the basis of YCLA filed a motion asking the RTC to direct the Board of
the ordinance. Even if the lots are eventually transferred to the City, Commissioners to conduct an ocular inspection over the subject
it is non sequitur for petitioners to claim that they are automatically properties and, thereafter, amend/revise the Board of
entitled to be beneficiaries thereof. For certain requirements must be Commissioner’s Report dated May 2, 2001. YCLA’s motion was

138 Aicka Singson


Notes in Remedial Law Review II
Provisional Remedies, Special Civil Actions, Special Proceedings and Evidence
School Year 2015-2016 REMEDIAL LAW REVIEW II - ATTY. BRONDIAL

granted by the RTC on July 25, 2003. The Board of Commissioners It is settled that the amount of just compensation is to be ascertained
submitted its second Report, which fixed the just compensation of as of the time of the taking, which usually coincides with the
the subject properties at P1,000.00 per sq m. commencement of the expropriation proceedings. Where the
institution of the action precedes entry into the property, the amount
On May 12, 2005, the RTC rendered a Decision, which adopted the of just compensation is to be ascertained as of the time of the filing
report and recommendation of the Board of Commissioners. of the complaint.

Unperturbed, NPC appealed the RTC Decision to the CA, alleging In this case, in arriving at the amount of just compensation, both the
that the RTC erred in relying on the recommendation of the Board of RTC and the CA relied heavily on the Board of Commissioners’
Commissioners as regards the amount of just compensation. NPC Report dated September 15, 2003, which, in turn, was arrived at
claimed that the amount of just compensation, which was adopted by after conducting an ocular inspection of the subject properties on
the RTC, is too excessive considering that the subject properties August 27, 2003. However, the Board of Commissioners’
were barren and undeveloped agricultural lands at the time it recommendation as to the amount of just compensation was based
instituted the action for expropriation. on the prevailing market value of the subject properties in 2003.
What escaped the attention of the lower courts is that the prevailing
The CA affirmed with modification the RTC Decision – just
market value of the subject properties in 2003 cannot be used to
compensation: P900 per sqm.
determine the amount of just compensation considering that the
ISSUE: Whether the RTC and the CA had sufficient basis in Complaint for expropriation was filed by NPC on December 2, 1997.
arriving at the questioned amount of just compensation of the
Further, the Court notes that the Board of Commissioners, in its
subject properties
Report dated September 15, 2003, merely alleged that its members
HELD: NO. In expropriation proceedings, just compensation is arrived at the amount of P1,000.00 per sqm as just compensation for
defined as the full and fair equivalent of the property taken from its the subject properties based on actual sales, presumably of
owner by the expropriator. The measure is not the taker’s gain, but surrounding parcels of land, and on the opinion of "reliable persons"
the owner’s loss. The word "just" is used to intensify the meaning of that were interviewed. However, the Report dated September 15,
the word "compensation" and to convey thereby the idea that the 2003 is not supported by any corroborative documents such as
equivalent to be rendered for the property to be taken shall be real, sworn declarations of the "reliable persons" that were supposedly
substantial, full and ample. The constitutional limitation of "just interviewed.
compensation" is considered to be a sum equivalent to the market
The Court has consistently ruled that just compensation cannot be
value of the property, broadly defined as the price fixed by the seller
arrived at arbitrarily; several factors must be considered such as, but
in open market in the usual and ordinary course of legal action and
not limited to, acquisition cost, current market value of like
competition; or the fair value of the property; as between one who
properties, tax value of the condemned property, its size, shape, and
receives and one who desires to sell it, fixed at the time of the actual
location. But before these factors can be considered and given
taking by the government.
weight, the same must be supported by documentary evidence. The
amount of just compensation could only be attained by using reliable

139 Aicka Singson


Notes in Remedial Law Review II
Provisional Remedies, Special Civil Actions, Special Proceedings and Evidence
School Year 2015-2016 REMEDIAL LAW REVIEW II - ATTY. BRONDIAL

and actual data as bases for fixing the value of the condemned Distinguish judicial from extrajudicial foreclosure
property. A commissioners’ report of land prices which is not based
on any documentary evidence is manifestly hearsay and should be There are 2 kinds of foreclosure proceedings under the Rules:
disregarded by the court.
1. Foreclosure of real estate mortgage under Rule 68
The trial court, in expropriation cases, may accept or reject, whether
2. Extrajudicial RA 3135 as amended by RA 4118
in whole or in part, the report submitted by the Board of
Commissioners, which is merely advisory and recommendatory in Extrajudicial foreclosure is more often resorted to because it is
character. It may also recommit the report or set aside the same and speedier than judicial foreclosure. Here, the mortgagee will simply go
appoint new commissioners. In this case, the lower courts gave full to the Clerk of Court (not the branch Clerk of Court, but with the ex
faith and credence to the Board of Commissioners' Report dated officio sheriff) and apply for the foreclosure of the mortgage. The
September 15, 2003 notwithstanding that it was not supported by Clerk of Court will post it on the bulletin board for a minimum of 30
any documentary evidence. days. Thereafter, the sale will take place.
This case is remanded to the trial court for the proper determination In extrajudicial foreclosure, there is no deficiency judgment so you
of just compensation, in conformity with this Decision. are limited only to the proceeds of the sale.

G. Foreclosure of Real Estate Mortgage (Rule 68) When you get a loan from a bank, they will ask you to sign the
mortgage contract. The contract will state that the bank may
 A real estate mortgage is an accessory contract executed by a
foreclose the sale either judicially or extrajudicially.
debtor in favor of a creditor as security for the principal obligation.
 This principal obligation is a simple loan or mutuum described in In extrajudicial foreclosure, no notice is necessary. Before you know
Art. 1953, Civil Code. To be a real estate mortgage, the contract it, your property has already been sold. So you go to the court, file
must be constituted on either immovables (real property) or annulment of foreclosure but you will probably lose because in
inalienable real rights. extrajudicial foreclosure, there is no need for a notice. The
If constituted on movables, the contract is a chattel mortgage (Art. consolidation of title here is immediate unlike in Rule 39 where the
2124, CC). consolidation of title is after the expiration of the right of redemption.

• A mortgage contract may have a provision in which the mortgage is Under the General Banking Act, financial institutions have the right of
a security for past, present and future indebtedness. This clause redemption for a maximum of 90 days (not 1 year) or at the time of
known as a DRAGNET CLAUSE OR BLANKET mortgage clause sale, whichever is earlier. Hence, the bank sells the property very
has its origins in American jurisprudence. quickly so they can proceed to consolidation.

• The Supreme Court ruled that mortgages given to secure future General rule: The issuance of a writ of possession is a ministerial
advancements are valid and legal contracts (Prudential Bank vs. function. Example: The mortgagor fails to pay the loan and the bank
Alviar, 464 SCRA 353). extrajudicial forecloses the mortgage. The property is sold to the

140 Aicka Singson


Notes in Remedial Law Review II
Provisional Remedies, Special Civil Actions, Special Proceedings and Evidence
School Year 2015-2016 REMEDIAL LAW REVIEW II - ATTY. BRONDIAL

mortgagee. The mortgagee will go to court and ask for a writ of will render the judgment null and void not only as to those excluded
possession. After the issuance of the writ of possession, the sheriff but also to those impleaded.
will go to the house of the mortgagor and tell the latter that the house
no longer belongs to him. Who are these people whose rights are subordinate to that of the
holder of the mortgage? The holder of the mortgage is the
Exception: Dayot vs Shell: The writ of possession ceases to be mortgagee. Those whose rights are subordinate to him must be
ministerial when the property subject of foreclosure belongs to a third impleaded in the action in order to have a complete determination of
party. If the property belongs to a third party, he should have been the case (they are necessary parties). Under Rule 3, if you do not
impleaded. If he is not impleaded, he is not affected by the equity of implead a necessary party, they are not affected by the order of
redemption. confirmation.

1. The Complaint (Section 1)


Who may they be? They can be the owner of the property, the
Section 1. Complaint in action for foreclosure. second mortgagee, or any third party, provided the negotiation or
action relative to the property is subsequent to that of the holder of
In an action for the foreclosure of a mortgage or other encumbrance the mortgage.
upon real estate, the complaint shall set forth the date and due
execution of the mortgage; its assignments, if any; the names and A, a bank, lends to B P10,000,000 secured by a real estate
residences of the mortgagor and the mortgagee; a description of the mortgage over a property a 100 sqm land in Makati. Under banking
mortgaged property; a statement of the date of the note or other procedure, the security is always worth more than the mortgage
documentary evidence of the obligation secured by the mortgage, debt. B was not able to pay 3 installments. The bank sent demand
the amount claimed to be unpaid thereon; and the names and letters. The bank can either file a case for sum of money or to
residences of all persons having or claiming an interest in the foreclosure the mortgage. The bank will go to the RTC and file a
property subordinate in right to that of the holder of the mortgage, all foreclosure suit. The bank can file a collection suit which is not
of whom shall be made defendants in the action. feasible because the mortgaged property is worth more than the
As to whom should be impleaded in an action for foreclosure, section P10,000,000 loan. In foreclosure proceedings, when the time comes,
1 says all persons having or claiming an interest in the property it is the mortgagee who purchases the property. After the usual
subordinate in right to that of the holder of the mortgage. If these requirements (complaint, summons, trial), the judgment in a
parties are not impleaded, there will be no complete determination of foreclosure proceeding is the judgment to pay the mortgage debt. It
the case and these people will not be affected by the equity of is not a judgment to sell.
redemption.
2. 2. The Judgment (Section 2)
In other words, all people whose rights are subordinate to that of the Sec. 2. Judgment on foreclosure for payment or sale.
holder of the mortgage must be impleaded as necessary parties and If upon the trial in such action the court shall find the facts set forth in
not as indispensable parties. An exclusion of an indispensable party the complaint to be true, it shall ascertain the amount due to the
plaintiff upon the mortgage debt or obligation, including interest and

141 Aicka Singson


Notes in Remedial Law Review II
Provisional Remedies, Special Civil Actions, Special Proceedings and Evidence
School Year 2015-2016 REMEDIAL LAW REVIEW II - ATTY. BRONDIAL

other charges as approved by the court, and costs, and shall render possession of the property unless a third party is actually holding the
judgment for the sum so found due and order that the same be paid same adversely to the judgment obligor. The said purchaser or last
to the court or to the judgment obligee within a period of not less redemptioner may secure a writ of possession, upon motion, from
than ninety (90) days nor more than one hundred twenty (120) days the court which ordered the foreclosure.
from the entry of judgment, and that in default of such payment the
property shall be sold at public auction to satisfy the judgment. Notes:

A court that renders a judgment to sell the property is abusing or Equity of redemption – the period from entry of judgment. The one
exceeding the exercise of its jurisdiction. Hence, censurable under a month period within which to pay. The Rule says not less than 90
petition for certiorari, if there is no appeal. days or more than 120 days. This is after entry of judgment. If the
case started with the RTC, when you appeal the case to the CA,
When the court renders a judgment foreclosing the property, the there is no entry of judgment yet. Even if the CA affirmed the
judgment is to pay. B is therefore adjudged to pay P10,000,000 judgment of the RTC, and you go up on petition for review, there will
including interest. After the court has rendered the judgment, after still be no entry of judgment. If you are really willing and able to pay,
the entry of judgment (not final judgment), the mortgagor has “equity you can easily pay. The Rule is very lenient regarding the mortgage
of redemption.” debt. After entry of judgment, the mortgagor still has more than one
month (not less than 90 days but not more than 120 days).
3. Sale of Foreclosed Property (Section 3)
Right of redemption – one year period from the registration of the
Sec. 3. Sale of mortgaged property; effect. certificate of sale.

When the defendant, after being directed to do so as provided in the Notwithstanding the equity of redemption, if B doed not comply or
next preceding section, fails to pay the amount of the judgment avail of the equity of redemption, what should A do? A cannot just
within the period specified therein, the court, upon motion, shall order sell the property. A has to go to court and file a litigated motion to sell
the property to be sold in the manner and under the provisions of the property. This is only the time when the property gets sold.
Rule 39 and other regulations governing sales of real estate under
execution. Such sale shall not affect the rights of persons holding Requirements of sale: (1) notice; (2) posting. If the property is
prior encumbrances upon the property or a part thereof, and when substantial, (3) publication.
confirmed by an order of the court, also upon motion, it shall operate
After all these proceedings are complied with, the sale proceeds.
to divest the rights in the property of all the parties to the action and
Who may buy the property? The same as Rule 39, anybody can buy
to vest their rights in the purchaser, subject to such rights of
the property. But there is priority. The purchaser is usually the
redemption as may be allowed by law.
mortgagee.
Upon the finality of the order of confirmation or upon the expiration of
the period of redemption when allowed by law, the purchaser at the
auction sale or last redemptioner, if any, shall be entitled to the

142 Aicka Singson


Notes in Remedial Law Review II
Provisional Remedies, Special Civil Actions, Special Proceedings and Evidence
School Year 2015-2016 REMEDIAL LAW REVIEW II - ATTY. BRONDIAL

After the foreclosure sale, the only thing necessary is an order of Furthermore, those who are not impleaded as necessary parties are
confirmation. The court will still an issue an order of confirmation. not subject to the judgment of the court. They can still run after the
Without that order of confirmation, the equity of redemption is not cut property. If the mortgagor happens to be a third party who is not the
because the order of confirmation is the only thing that can cut the owner of the property and he is not impleaded in the action, he can
equity of redemption. run after the property.

The confirmation of the sale shall divest the rights in the property of The order of confirmation has to become executory so that this order
all parties to the action and shall vest their rights in the purchaser, of confirmation is appealable (because it is a final order). When you
subject to such rights of redemption as may be allowed by law. The appeal the order of confirmation, the equity of redemption continues
title vests in the purchaser upon a valid confirmation of the sale and because it is only that which cuts the equity of redemption.
retroacts to the date of sale.
DISPOSITION OF PROCEEDS OF SALE
The import of Sec. 3 includes one vital effect: The equity of
redemption of the mortgagor or redemptioner is cut-off and there will The proceeds of the sale of the mortgaged property shall, after
be no further redemption, unless allowed by law (as in the case of deducting the costs of the sale, be paid to the person foreclosing the
banks as mortgagees). mortgage, and when there shall be any balance or residue after
paying off the mortgage debt due, the same shall be paid to junior
The equity of redemption starts from the ninety-day period set in the encumbrancers in the order of their priority. If there be any further
judgment of the court up to the time before the sale is confirmed by balance after paying them or if there be no junior encumbrancers,
an order of the court. Once confirmed, no equity of redemption may the same shall be paid to the mortgagor or any person entitled
further be exercised. thereto. (Section 4)

The order of confirmation is appealable and if not appealed within Section 5. How sale to proceed in case the debt is not all due. — If
the period for appeal becomes final. Upon the finality of the order of the debt for which the mortgage or encumbrance was held is not all
confirmation or upon the expiration of the period of redemption when due as provided in the judgment as soon as a sufficient portion of the
allowed by law, the purchaser at the auction sale or last property has been sold to pay the total amount and the costs due,
redemptioner, if any, shall be entitled to the possession of the the sale shall terminate; and afterwards as often as more becomes
property and he may secure a writ of possession, upon, motion, from due for principal or interest and other valid charges, the court may,
the court which ordered the foreclosure unless a third party is on motion, order more to be sold. But if the property cannot be sold
actually holding the same adversely to the judgment obligor. in portions without prejudice to the parties, the whole shall be
ordered to be sold in the first instance, and the entire debt and costs
In other words, the equity of redemption will continue beyond the one shall be paid, if the proceeds of the sale be sufficient therefor, there
month period provided because it is not cut until the order of being a rebate of interest where such rebate is proper.
confirmation.
4. Deficiency of Judgment (Section 6)
Sec. 6. Deficiency judgment.

143 Aicka Singson


Notes in Remedial Law Review II
Provisional Remedies, Special Civil Actions, Special Proceedings and Evidence
School Year 2015-2016 REMEDIAL LAW REVIEW II - ATTY. BRONDIAL

If upon the sale of any real property as provided in the next INSTANCES WHEN COURT CANNOT RENDER DEFICIENCY
preceding section there be a balance due to the plaintiff after JUDGMENT
applying the proceeds of the sale, the court, upon motion, shall
render judgment against the defendant for any such balance for Where the debtor-mortgagor is a non-resident and who at the time of
which, by the record of the case, he may be personally liable to the the filing of the action for foreclosure and during the pendency of the
plaintiff, upon which execution may issue immediately if the balance proceedings was outside the Philippines, it is believed that a
is all due at the time of the rendition of the judgment; otherwise, the deficiency judgment would not be procedurally feasible.
plaintiff shall be entitled to execution at such time as the balance
Don’t be misled by the word “judgment,” because you can only get a
remaining becomes due under the terms of the original contract,
judgment through a separate action. But in this instance, it is not a
which time shall be stated in the judgment.
separate action that you get a judgment. This is only by a motion.
When does this happen? When after the sale (so this is in favor of
Notes:
the mortgagee), the judgment creditor was not able to recover his
If there be a balance due to the plaintiff after applying the proceeds losses. It can happen that the property mortgaged is worth
of the sale, the court, upon motion, shall render judgment against the P50,000,000 and the mortgage loan is P10,000,000 but at the time
defendant for any such balance. of the sale the property is only worth P8,000,000. The judgment
creditor may avail of a deficiency judgment. He can go to court and
Execution may issue immediately if the balance is all due the plaintiff file a motion (this is not a separate action) and the court will
shall be entitled to execution at such time as the remaining balance determine because this is a litigated motion. When the court gives a
shall become due and such due date shall be stated in the judgment. deficiency judgment, A can now avail of Rule 39.
Note that the deficiency judgment is in itself a judgment hence, also
appealable. If A is only able to recover P8,000,000 out of the P10,000,000 loan,
A ask for the issuance of a writ or levy on the property. The sheriff
No independent action need be filed to recover the deficiency from will levy on the property pursuant to Rule 39 until the judgment is
the mortgagor. The deficiency judgment shall be rendered upon fully satisfied.
motion of the mortgagee. The motion must be made only after the
sale and after it is known that a deficiency exists. Before that, any Section 7. Registration. — A certified copy of the final order of the
court order to recover the deficiency is void. It has been held that the court confirming the sale shall be registered in the registry of deeds.
mortgagor who is not the debtor and who merely executed the If no right of redemption exists, the certificate of title in the name of
mortgage to secure the principal debtor’s obligation is not liable for the mortgagor shall be cancelled, and a new one issued in the name
the deficiency unless he assumed liability for the same in the of the purchaser.
contract. Since a deficiency judgment cannot be obtained against the
Where a right of redemption exists, the certificate of title in the name
mortgagor who is not the debtor in the principal obligation,
of the mortgagor shall not be cancelled, but the certificate of sale and
mortgagee may have to file a separate suit against the principal
the order confirming the sale shall be registered and a brief
debtor.
memorandum thereof made by the registrar of deeds upon the

144 Aicka Singson


Notes in Remedial Law Review II
Provisional Remedies, Special Civil Actions, Special Proceedings and Evidence
School Year 2015-2016 REMEDIAL LAW REVIEW II - ATTY. BRONDIAL
EQUITY OF RIGHT OF
certificate of title. In the event the property is redeemed, the deed REDEMPTION REDEMPTION
of redemption shall be registered with the registry of deeds, and a The right of defendant mortgagor to A right granted to a debtor
brief memorandum thereof shall be made by the registrar of deeds extinguish the mortgage and retain mortgagor, his successor in interest
on said certificate of title. ownership of the property by or any judicial creditor or
paying the debt within 90 to 120 days judgment creditor or any
If the property is not redeemed, the final deed of sale executed by after the entry of judgment or even person having a lien on the
after the foreclosure sale but prior to property subsequent to the
the sheriff in favor of the purchaser at the foreclosure sale shall be
confirmation. mortgage or deed of trust under
registered with the registry of deeds; whereupon the certificate of which the property is sold to
title in the name of the mortgagor shall be cancelled and a new repurchase the property within
one issued in the name of the purchaser. one year even after the
confirmation of the sale and even
Section 8. Applicability of other provisions. — The provisions of after the registration of the
certificate of foreclosure sale.
sections 31, 32 and 34 of Rule 39 shall be applicable to the
judicial foreclosure of real estate mortgages under this Rule
May be exercised even after the There is no right of
insofar as the former are not inconsistent with or may serve to
foreclosure sale provided it is made redemption in a judicial
supplement the provisions of the latter. before the sale is confirmed by foreclosure of mortgage under
order of the court. Rule 68. This right of redemption
exists only in
extrajudicial foreclosures where
there is always a right of
redemption within one year
from the date of sale (Sec. 3,
Act 3135), but interpreted by
the Court to mean one year from
the registration of the sale.
May also exist in favor or other General rule: In judicial
encumbrances. foreclosures there is only an
equity of redemption which can be
exercised prior to the confirmation
of the foreclosure sale. This
means that after the foreclosure
sale but before its confirmation, the
mortgagor may exercise his right of
pay the proceeds of the sale and
prevent the confirmation of the sale.

145 Aicka Singson


Notes in Remedial Law Review II
Provisional Remedies, Special Civil Actions, Special Proceedings and Evidence
School Year 2015-2016 REMEDIAL LAW REVIEW II - ATTY. BRONDIAL

If subsequent lien
EXTRA-JUDICIAL JUDICIAL holders are not
FORECLOSURE (ACT FORECLOSURE impleaded as parties in
3135) (RULE 68) the foreclosure suit, the
judgment in favor of the
foreclosing mortgagee
No complaint is filed; Complaint is filed with
does not bind the other
the courts;
lien holders. In this
There is a right of No right of redemption case, their equity of
redemption. Mortgagor except when mortgagee is redemption remains
has a right of redemption a banking institution; unforeclosed. A
for 1 year from equity of redemption only separate foreclosure
registration of the sale; (90 to 120 days, and any proceeding has to be
time before confirmation brought against them to
of foreclosure sale); require them to redeem
from the first mortgagee or
from the party
Mortgagee has to file a Mortagagee can move for acquiring the title to the
separate action to recover deficiency judgment in mortgaged property.
any deficiency; the same action

Buyer at public auction Buyer at public auction If not by banks, the Exception: there is a right of
becomes absolute owner becomes absolute owner mortgagors merely have redemption if the
only after finality of an only after confirmation of an equity of foreclosure is in favor of
action for consolidation the sale; redemption, which is banks as mortgagees,
of ownership; simply their right, as whether the foreclosure be
mortgagor, to judicial or extrajudicial. This
Mortgagee is given a Mortgagee need not be extinguish the mortgage right of redemption is explicitly
special power of attorney given a special power of and retain ownership of provided in Sec.
in the mortgage contract attorney. the property by paying 47 of the General Banking
to foreclose the the secured debt prior to Law of 2000. While the law
mortgaged property in the confirmation of the mentions the redemption
case of default. foreclosure sale. period to be one year
counted from the date of
registration of the
certificate in the Registry of
Property

146 Aicka Singson


Notes in Remedial Law Review II
Provisional Remedies, Special Civil Actions, Special Proceedings and Evidence
School Year 2015-2016 REMEDIAL LAW REVIEW II - ATTY. BRONDIAL

CASES: We rule that when respondent failed to send the notice of


extrajudicial foreclosure sale to Ramirez, it committed a contractual
a. Ramirez vs. Manila Banking Corp (712 SCRA breach of said paragraph N sufficient to render the extrajudicial
Facts: Petitioner Ramirez mortgaged two parcels of land located in foreclosure sale on September 8, 1994 null and void
Marikina City in favor of respondent The Manila Banking Corporation In Carlos Lim, et al. v. Development Bank of the Philippines, we held
to secure his P265,000 loan. The real estate mortgage provides that that unless the parties stipulate, personal notice to the mortgagor in
all correspondence relative to the mortgage including notifications of extrajudicial foreclosure proceedings is not necessary because
extrajudicial actions shall be sent to petitioner Ramirez at his given Section 3 of Act No. 3135 only requires the posting of the notice of
address. Respondent filed a request for extrajudicial foreclosure of sale in three public places and the publication of that notice in a
real estate mortgage before Atty. Hipolito Sañez on the ground that newspaper of general circulation. In this case, the parties stipulated
Ramirez failed to pay his loan despite demands. During the auction in paragraph N of the real estate mortgage that all correspondence
sale, respondent was the only bidder for the mortgaged properties. relative to the mortgage including notifications of extrajudicial actions
Thereafter, a certificate of sale was issued in its favor as the highest shall be sent to mortgagor Ramirez at his given address.
bidder. Respondent had no choice but to comply with this contractual
Respondent demanded that Ramirez vacate the properties. provision it has entered into with Ramirez. The contract is the law
Ramirez sued respondent for annulment of sale and prayed that the between them. Hence, we cannot agree with the bank that
certificate of sale be annulled on the ground, among others, that paragraph N of the real estate mortgage does not impose an
paragraph N of the real estate mortgage was violated for he was not additional obligation upon it to provide personal notice of the
notified of the foreclosure and auction sale. extrajudicial foreclosure sale to the mortgagor Ramirez.
In its answer, respondent claimed that the foreclosure proceedings As we explained in Metropolitan Bank v. Wong, the bank’s violation
were valid. of paragraph N of the real estate mortgage is sufficient to invalidate
The trial court ruled that the extrajudicial foreclosure proceedings the extrajudicial foreclosure sale:
were null and void and the certificate of sale is invalid. The contract is the law between the parties and absent any showing
The CA reversed the trial court’s decision and ruled that absence of that its provisions are wholly or in part contrary to law, morals, good
personal notice of foreclosure to Ramirez as required by paragraph customs, public order, or public policy, it shall be enforced to the
N of the real estate mortgage is not a ground to set aside the letter by the courts. Section 3, Act No. 3135 reads:
foreclosure sale. Ramirez’s motion for reconsideration was denied in "Sec. 3. Notice shall be given by posting notices of the sale for not
the assailed CA Resolution. less than twenty days in at least three public places of the
municipality or city where the property is situated, and if such
Issue: WON notice of extrajudicial foreclosure sale to Ramirez property is worth more than four hundred pesos, such notice shall
as required by paragraph N of the real estate mortgage will not also be published once a week for at least three consecutive weeks
invalidate the extrajudicial foreclosure sale. (It will invalidate) in a newspaper of general circulation in the municipality and city."
The Act only requires (1) the posting of notices of sale in three public
Held: NO. places, and (2) the publication of the same in a newspaper of
general circulation. Personal notice to the mortgagor is not

147 Aicka Singson


Notes in Remedial Law Review II
Provisional Remedies, Special Civil Actions, Special Proceedings and Evidence
School Year 2015-2016 REMEDIAL LAW REVIEW II - ATTY. BRONDIAL

necessary. Nevertheless, the parties to the mortgage contract are claiming that the same is ministerial on the court’s part following the
not precluded from exacting additional requirements. In this case, consolidation of her and her husband’s title over the subject property.
petitioner and respondent in entering into a contract of real estate Impleaded in said petition are Sps. Gutierrez, including all persons
mortgage, agreed inter alia: claiming rights under them.
"all correspondence relative to this mortgage, including demand
letters, summonses, subpoenas, or notifications of any judicial or The RTC granted Anita’s ex-parte petition and thereby directed the
extra-judicial action shall be sent to the MORTGAGOR…." issuance of a writ of possession in her favor.
Precisely, the purpose of the foregoing stipulation is to apprise
Sps. Alindog sought the issuance of a TRO and/or writ of preliminary
respondent of any action which petitioner might take on the subject
injunction with prayer for damages, (injunction case).
property, thus according him the opportunity to safeguard his rights.
When petitioner failed to send the notice of foreclosure sale to While it appears that the RTC issued a 72-hour TRO on September
respondent, he committed a contractual breach sufficient to render 29, 2005 in Sps. Alindog’s favor, records nonetheless show that said
the foreclosure sale null and void. order was not extended to a full 20-day TRO. To this end, the
Sheriff’s Return shows that Sheriff Cosare was able to implement the
b. Marquez vs Alindog (714 SCRA) writ of possession turning over the possession of the subject
property to Sps. Marquez.
FACTS: Sometime in June 1998, petitioner Anita extended a loan in
the amount of P500,000.00 to Gutierrez. As security therefor, After further proceedings on the injunction case, the RTC issued a
Gutierrez executed a Deed of Real Estate Mortgage dated June 16, writ of preliminary injunction enjoining Sps. Marquez from taking
1998 over a parcel of land located in Tagaytay City. Since Gutierrez possession of the subject property until after the controversy has
defaulted in the payment of his loan obligation, Anita sought the been fully resolved on the merits. Thus, notwithstanding the
extra-judicial foreclosure of the subject property. At the public auction consolidation of Sps. Marquez’s title over the subject property, the
sale held on January 19, 2000, Anita emerged as the highest bidder. RTC granted Sps. Alindog’s prayer for injunctive relief, holding that
Upon Gutierrez’s failure to redeem the same property within the any further dispossession on their part would cause them irreparable
prescribed period therefor, title was consolidated under TCT No. T- injury.
41939 on November 5, 2001 in the name of Sps. Marquez which,
however, bore an annotation of adverse claim dated March 2, 2000 Aggrieved, Sps. Marquez moved for reconsideration, essentially
in the names of respondents- Sps. Alindog. pointing out that, as the confirmed and registered owners of the
subject property, they are entitled to its possession as a matter of
Subsequently, or on March 21, 2000, Sps. Alindog filed a civil case right – DENIED.
for annulment of real estate mortgage and certificate of sale with
prayer for damages against Sps. Marquez and a certain Agripina For their part, Sps. Alindog filed a Motion for Approval of Cash Bond
Gonzales (Gonzales) before the RTC (annulment case). and to Regain Possession of the subject property – GRANTED.

Meanwhile, on March 16, 2005, Anita filed an ex-parte petition for the Unperturbed, Sps. Marquez elevated the case to the CA on certiorari
issuance of a writ of possession (ex-parte petition) before the RTC – DENIED.

148 Aicka Singson


Notes in Remedial Law Review II
Provisional Remedies, Special Civil Actions, Special Proceedings and Evidence
School Year 2015-2016 REMEDIAL LAW REVIEW II - ATTY. BRONDIAL

ISSUE: WON the CA erred in finding no grave abuse of Notably, the property should not only be possessed by a third party,
discretion on the part of the RTC when it issued the injunctive but also held by the third party adversely to the judgment obligor." In
writ which enjoined Sps. Marquez from taking possession of the other words, as mentioned in Villanueva v. Cherdan Lending
subject property Investors Corporation, the third person must therefore claim a right
superior to that of the original mortgagor.
HELD: It is an established rule that the purchaser in an extra-judicial
foreclosure sale is entitled to the possession of the property and can In this case, it is clear that the issuance of a writ of possession in
demand that he be placed in possession of the same either during favor of Sps. Marquez, who had already consolidated their title over
(with bond) or after the expiration (without bond) of the redemption the extra-judicially foreclosed property, is merely ministerial in
period therefor. nature. The general rule as herein stated – and not the exception
found under Section 33, Rule 39 of the Rules – should apply since
In the case of Spouses Espiridion v. CA, the Court expounded on the Sps. Alindog hinged their claim over the subject property on their
ministerial nature of the foregoing issuance as follows: purported purchase of the same from its previous owner, i.e., Sps.
Gutierrez (with Gutierrez being the original mortgagor). Accordingly,
The issuance of a writ of possession to a purchaser in a public
it cannot be seriously doubted that Sps. Alindog are only the latter’s
auction is a ministerial act. After the consolidation of title in the
(Sps. Gutierrez) successors-in-interest who do not have a right
buyer’s name for failure of the mortgagor to redeem the property, the
superior to them.
writ of possession becomes a matter of right. Its issuance to a
purchaser in an extrajudicial foreclosure sale is merely a ministerial
c. Ardiente vs. Provincial Sheriff (436 SCRA)
function. The trial court has no discretion on this matter.
Spouses Rustico Ardiente obtained a loan in the amount of
The ministerial issuance of a writ of possession in favor of the P100,000.00 from the Peninsula Development Bank (the bank). To
purchaser in an extra-judicial foreclosure sale, however, admits of an secure the payment of the loan, the Ardientes executed in favor of
exception. Section 33, Rule 39 of the Rules of Court (Rules) the bank a Real Estate Mortgage.
pertinently provides that the possession of the mortgaged property Out of the proceeds of the loan, the Ardientes purchased a mini bus
may be awarded to a purchaser in an extra-judicial foreclosure costing P81,875.00. After the bus was in operation for several
unless a third party is actually holding the property by adverse title or months, it met an accident as result of which it sustained heavy
right. In the recent case of Rural Bank of Sta. Barbara (Iloilo), Inc. v. damages and rendered the Ardientes unable to meet their obligation
Centeno, citing the case of China Banking Corp., the Court illumined to the bank. Demands for the payment of their obligation to the bank
that "the phrase ‘a third party who is actually holding the property notwithstanding, the Ardientes failed to settle the same.
adversely to the judgment obligor’ contemplates a situation in which The bank thus extra-judicially foreclosed the mortgage and the
a third party holds the property by adverse title or right, such as that parcels of land covered thereby were sold at public auction to the
of a co-owner, tenant or usufructuary. The co-owner, agricultural bank which was the highest bidder.
tenant, and usufructuary possess the property in their own right, and The bank later notified the Ardientes by letter that they had one (1)
they are not merely the successor or transferee of the right of year to redeem the foreclosed mortgage.
possession of another co-owner or the owner of the property.

149 Aicka Singson


Notes in Remedial Law Review II
Provisional Remedies, Special Civil Actions, Special Proceedings and Evidence
School Year 2015-2016 REMEDIAL LAW REVIEW II - ATTY. BRONDIAL

Two days before the period to redeem the foreclosed mortgage application in accordance with law," to thus call for the presentation
expired, the spouses Ardiente filed before the Regional Trial Court of of evidence, they citing again Benavides, the same fails.
Quezon a complaint, denominated as Petition, against the bank Benavides bears on the rendition of judgment on the pleadings. It
anchored to two grounds namely: (a) that it was the respondents, holds that where the defendant’s answer tenders an issue, as where
who had violated the Real Estate Mortgage and Amended Real it does not only deny the material allegations of the complaint but
Estate Mortgage, and (b) that the requisite of notifying the also sets up certain special and affirmative defenses, the nature of
mortgagors of the intended extra-judicial foreclosure sale was not such answer calls for presentation of evidence, hence, it is error to
duly complied with, the FORECLOSURE SALE should be annulled. render a judgment on the pleadings thereon without such evidence.
Ardiente spouses capitalized on the alleged lack of notice to them of No doubt, it is a well-settled rule that statutory provisions governing
the "judicial foreclosure auction sale." publication of notice of mortgage foreclosure sales must be strictly
complied with, and that even slight deviations therefrom will
The trial court, noting the absence of documentary evidence showing invalidate the notice and the sale at least voidable.
strict compliance with the statutory requirements on publication of Despite petitioners’ non-allegation of lack of publication of notice of
notice of extra-judicial foreclosure of mortgage, declared the extra- foreclosure in their Complaint, the bank pleaded in its Answer (1)
judicial foreclosure and the sale of the mortgaged properties null and "that petitioners were duly notified of the extrajudicial foreclosure and
void. The Defendants bank et al. thus appealed to the Court of public auction sale" and "There was sufficient notice and publication
Appeals which reversed the decision of the trial court. served to all concerned of said public auction sale," and (2) that it
Hence, the present petition for review filed by the Ardiente spouses. and the Office of the provincial Sheriff "fully complied with the
requirements of law under Act 3135, more specifically with regard to
Issue: WON THE LACK OF PUBLICATION OF NOTICE WILL notices of the public auction as well as the extra-judicial foreclosure
MAKE THE EXTRA-JUDICIAL FORECLOSURE SALE NULL AND in accordance with law."
VOID Yet petitioners never refuted in their Reply and Answer to
Counterclaim such defense of the bank nor presented evidence
Held: NO. With respect to petitioners’ paragraphs 15 and 16 before the trial court to disprove the same.
allegations in their Complaint, clearly, they were questioning the In fact, in its Comment on petitioners’ Formal Offer of Evidence
validity of the extra-judicial foreclosure of the mortgage on the basis before the trial court, the bank, passing on Exhibit "D" – its letter to
of lack of notice to them as mortgagors. petitioners advising them that they had one year from November 11,
It is settled that personal notice to the mortgagor in extra-judicial 1993 to exercise their right of redemption, stated that said exhibit
foreclosure proceedings is not necessary, hence, not a ground to set was admitted "with the qualification as to the purpose to the effect
aside the foreclosure sale. that said extra-judicial foreclosure was filed in accordance with law
With respect to petitioners’ argument that the bank, in paragraph 25 and that all requirements of said law were complied with and that
of its Answer, in fact put in issue its compliance with the plaintiffs were duly notified of said proceedings."
requirements of Act 3135, "more specifically with regards to the Despite the bank’s repeated claim that the statutory requirements
notices of the public auction sale as well as the extra-judicial governing extra-judicial foreclosure had been complied with, the
bank’s plea of lack of publication of notice of foreclosure was not

150 Aicka Singson


Notes in Remedial Law Review II
Provisional Remedies, Special Civil Actions, Special Proceedings and Evidence
School Year 2015-2016 REMEDIAL LAW REVIEW II - ATTY. BRONDIAL

raised by petitioners either in the Amended Complaint or in the Reply


and Answer to Counterclaim. It was not also raised during the trial as d. LZK Holdings vs. Planters Development Bank (714 SCRA)
the entire transcripts of the stenographic notes of the proceedings
before the trial court show. Nor even in their memorandum filed FACTS: LZK Holdings obtained a P40,000,000.00 loan from Planters
before the trial court, petitioners having merely assailed the lack of Bank on December 16, 1996 and secured the same with a Real
"personal" notification to them of any "intended" extrajudicial Estate Mortgage over its lot located in La Union.
foreclosure and the "grossly and greatly inadequate" purchase price
of the lands. On September 21, 1998, the lot was sold at a public auction after
As the appellate court thus held, the issue of lack of publication of Planters Bank extrajudicially foreclosed the real estate mortgage
notice cannot be raised for the first time on appeal. thereon due to LZK Holdings' failure to pay its loan. Planters Bank
In the Tambunting case cited by petitioners to support their thesis emerged as the highest bidder during the auction sale and its
that failure to strictly comply with statutory requirements governing certificate of sale was registered on March 16, 1999.
publication of notice of mortgage foreclosure sales renders the sale
On April 5, 1999, LZK Holdings filed before the RTC of Makati City, a
at least voidable, the therein mortgagors, in their complaint for
complaint for annulment of extra judicial foreclosure, mortgage
annulment of mortgage and damages, sought to enjoin the extra-
contract, promissory note and damages. LZK Holdings also prayed
judicial foreclosure of mortgage. During the pendency of the case,
for the issuance of a TRO or writ of preliminary injunction to enjoin
the extra-judicial foreclosure pushed thru just the same. The
the consolidation of title over the lot by Planters Bank.
mortgaged property was sold at public auction to the mortgagees,
and the property was eventually sold to the Tambunting Realty. The On December 27, 1999, Planters Bank filed an ex-parte motion for
mortgagors thereupon filed a Supplemental Complaint impleading the issuance of a writ of possession with the RTC-San Fernando.
the realty firm, the provincial sheriff as the officer responsible for
holding the foreclosure, and the Register of Deeds for the On March 13, 2000 or three (3) days before the expiration of LZK
subsequent transfer of the property "despite alleged non-compliance Holdings' redemption period, the RTC-Makati issued a TRO effective
with the requirements of Act 3135, Sec. 3(as amended by Act 4118) for 20 days enjoining Planters Bank from consolidating its title over
on posting and publication of the notice of foreclosure sale." In other the property. On April 3, 2000, the RTC-Makati ordered the issuance
words, the lack of publication was raised in issue by the mortgagors of a writ of preliminary injunction for the same purpose but the writ
in their Supplemental Complaint. was issued only on June 20, 2000 upon LZK Holdings' posting of a
In the case of Go v. Court of Appeals, as in the present case, despite P40,000.00 bond.
the fact that the mortgagees pleaded as a defense in their Answer
the "receipt of the ‘notice of the sale which was published in a In the meantime, Planters Bank succeeded in consolidating its
newspaper of general circulation,’’’ the issue of lack of publication of ownership over the property on April 24, 2000. However, the
the notice of foreclosure was never raised in issue by the proceedings for its ex-parte motion for the issuance of a writ of
mortgagors. possession was suspended by the RTC-San Fernando in view of the
WHEREFORE, there being no reversible error in the assailed TRO and writ of preliminary injunction issued by the RTC-Makati.
decision, the petition is hereby DISMISSED.

151 Aicka Singson


Notes in Remedial Law Review II
Provisional Remedies, Special Civil Actions, Special Proceedings and Evidence
School Year 2015-2016 REMEDIAL LAW REVIEW II - ATTY. BRONDIAL

Planters Bank moved for reconsideration but its motion was denied to issue the writ, therefore, should have acted on the ex parte
by the RTC-San Fernando. petition. The injunction order is of no moment because it should be
understood to have merely stayed the consolidation of title. As
Meanwhile, upon motion of LZK Holdings, the RTC-Makati declared previously stated, an injunction is not allowed to prohibit the issuance
as null and void the consolidated title of Planters Bank. Such ruling of a writ of possession. Neither does the pending case for annulment
was affirmed by the CA. When the matter reached the Court via G.R. of foreclosure sale, mortgage contract, promissory notes and
No. 164563, we sustained the CA's judgment. damages stay the issuance of said writ.

Planters Bank also appealed the May 11, 2000 Order of the RTC- We cannot also uphold the contentions of LZK Holdings that the
San Fernando which held in abeyance the resolution of its ex parte RTC, in issuing the writ of possession, transgressed Act No. 3135.
motion for the issuance of a writ of possession. This time, Planters
Bank was victorious. The CA granted the appeal and annulled the No hearing is required prior to the issuance of a writ of possession.
assailed order of the RTC-San Fernando. This is clear from the following disquisitions in Espinoza v. United
Overseas Bank Phils. which reiterates the settled rules on writs of
Aggrieved, LZK Holdings sought recourse with the Court in a petition possession, to wit:
for review. In Our Decision, we affirmed the CA's ruling and decreed
that Planters Bank may apply for and is entitled to a writ of The proceeding in a petition for a writ of possession is ex parte and
possession as the purchaser of the property in the foreclosure sale. summary in nature. It is a judicial proceeding brought for the benefit
of one party only and without notice by the court to any person
ISSUE: WON the issuance of writ of possession may be adverse of interest. It is a proceeding wherein relief is granted
enjoined by an injunction without giving the person against whom the relief is sought an
opportunity to be heard.
HELD: NO. We have consistently held that the duty of the trial court
to grant a writ of possession is ministerial. Such writ issues as a By its very nature, an ex parte petition for issuance of a writ of
matter of course upon the filing of the proper motion and the possession is a non-litigious proceeding. It is a judicial proceeding
approval of the corresponding bond. No discretion is left to the trial for the enforcement of one's right of possession as purchaser in a
court. foreclosure sale. It is not an ordinary suit filed in court, by which one
party sues another for the enforcement of a wrong or protection of a
To emphasize the writ's ministerial character, we have in previous
right, or the prevention or redress of a wrong.
cases disallowed injunction to prohibit its issuance, just as we have
held that issuance of the same may not be stayed by a pending e. Goldenway Merchandising Corp vs. Equitable PCI
action for annulment of mortgage or the foreclosure itself.
Bank (639 SCRA)
x x x [Planters Bank], as the purchaser in the foreclosure sale, may
Facts: Goldenway Merchandising Corporation (petitioner) executed a
apply for a writ of possession during the redemption period. In fact, it
REM in favor of Equitable PCI Bank (respondent) over its real
did apply for a writ on December 27, 1999, well within the
properties situated now Valenzuela City. The mortgage secured the
redemption period. The San Fernando RTC, given its ministerial duty

152 Aicka Singson


Notes in Remedial Law Review II
Provisional Remedies, Special Civil Actions, Special Proceedings and Evidence
School Year 2015-2016 REMEDIAL LAW REVIEW II - ATTY. BRONDIAL

Two Million Pesos loan granted by respondent to petitioner and was behalf; it was only a certain Chan Guan Pue, the alleged President of
duly registered. petitioner corporation, who gave instruction to Atty. Abat-Vera to
redeem the foreclosed properties.
As petitioner failed to settle its loan obligation, respondent
extrajudicially foreclosed the mortgage. During the public auction, the Aggrieved, petitioner appealed to the CA which affirmed the trial
mortgaged properties were sold for P3,500,000.00 to respondent. court’s decision.
Accordingly, a Certificate of Sale was issued to respondent on
January 26, 2001. On February 16, 2001, the Certificate of Sale was Petitioner’s motion for reconsideration was likewise denied by the
registered and inscribed. CA.

Petitioner’s counsel offered to redeem the foreclosed properties by Issue: WON Act No. 3135 shall govern
tendering a check in the amount of P3,500,000.00. However,
Held: The law governing cases of extrajudicial foreclosure of
petitioner was told that such redemption is no longer possible
mortgage is Act No. 3135, as amended by Act No. 4118. Section 6
because the certificate of sale had already been registered.
thereof provides:
Petitioner also verified with the Registry of Deeds that title to the
foreclosed properties had already been consolidated in favor of SEC. 6. In all cases in which an extrajudicial sale is made under the special
respondent and that new certificates of title were issued in the name power hereinbefore referred to, the debtor, his successors-in-interest or any
of respondent. judicial creditor or judgment creditor of said debtor, or any person having a
lien on the property subsequent to the mortgage or deed of trust under which
Petitioner filed a complaint for specific performance and damages the property is sold, may redeem the same at any time within the term of one
against the respondent, asserting that it is the one-year period of year from and after the date of the sale; and such redemption shall be
redemption under Act No. 3135 which should apply and not the governed by the provisions of sections four hundred and sixty-four to four
shorter redemption period provided in Republic Act No. 8791. hundred and sixty-six, inclusive, of the Code of Civil Procedure, in so far as
these are not inconsistent with the provisions of this Act.
Petitioner argued that applying Section 47 of R.A. 8791 to the real
estate mortgage executed in 1985 would result in the impairment of
The one-year period of redemption is counted from the date of the
obligation of contracts and violation of the equal protection clause
registration of the certificate of sale. In this case, the parties provided
under the Constitution. in their real estate mortgage contract that upon petitioner’s default
and the latter’s entire loan obligation becoming due, respondent may
Trial court rendered its decision dismissing the complaint as well as
the counterclaim. It noted that the issue of constitutionality of Sec. 47 immediately foreclose the mortgage judicially in accordance with the
Rules of Court, or extrajudicially in accordance with Act No. 3135, as
of R.A. No. 8791 was never raised by the petitioner during the pre-
amended.
trial and the trial. Aside from the fact that petitioner’s attempt to
redeem was already late, there was no valid redemption made
However, Section 47 of R.A. No. 8791 otherwise known as "The
because Atty. Judy Ann Abat-Vera who talked to Atty. Joseph E.
General Banking Law of 2000" which took effect on June 13, 2000,
Mabilog of the Legal Division of respondent bank, was not properly
amended Act No. 3135. Said provision reads:
authorized by petitioner’s Board of Directors to transact for and in its

153 Aicka Singson


Notes in Remedial Law Review II
Provisional Remedies, Special Civil Actions, Special Proceedings and Evidence
School Year 2015-2016 REMEDIAL LAW REVIEW II - ATTY. BRONDIAL

SECTION 47. Foreclosure of Real Estate Mortgage. — In the event of and the mortgage foreclosed when R.A. No. 8791 was already in
foreclosure, whether judicially or extrajudicially, of any mortgage on real effect? YES
estate which is security for any loan or other credit accommodation granted,
the mortgagor or debtor whose real property has been sold for the full or When confronted with a constitutional question, it is elementary that
partial payment of his obligation shall have the right within one year after the every court must approach it with grave care and considerable
sale of the real estate, to redeem the property by paying the amount due caution bearing in mind that every statute is presumed valid and
under the mortgage deed, with interest thereon at the rate specified in the
every reasonable doubt should be resolved in favor of its
mortgage, and all the costs and expenses incurred by the bank or institution
from the sale and custody of said property less the income derived
constitutionality. For a law to be nullified, it must be shown that there
therefrom. However, the purchaser at the auction sale concerned whether in is a clear and unequivocal breach of the Constitution. The ground for
a judicial or extrajudicial foreclosure shall have the right to enter upon and nullity must be clear and beyond reasonable doubt. Indeed, those
take possession of such property immediately after the date of the who petition this Court to declare a law, or parts thereof,
confirmation of the auction sale and administer the same in accordance with unconstitutional must clearly establish the basis therefor. Otherwise,
law. Any petition in court to enjoin or restrain the conduct of foreclosure the petition must fail.
proceedings instituted pursuant to this provision shall be given due course
only upon the filing by the petitioner of a bond in an amount fixed by the Petitioner’s contention that Section 47 of R.A. 8791 violates the
court conditioned that he will pay all the damages which the bank may suffer constitutional proscription against impairment of the obligation of
by the enjoining or the restraint of the foreclosure proceeding. contract has no basis.
Notwithstanding Act 3135, juridical persons whose property is being The purpose of the non-impairment clause of the Constitution is to
sold pursuant to an extrajudicial foreclosure, shall have the right to safeguard the integrity of contracts against unwarranted interference
redeem the property in accordance with this provision until, but not by the State. As a rule, contracts should not be tampered with by
after, the registration of the certificate of foreclosure sale with the subsequent laws that would change or modify the rights and
applicable Register of Deeds which in no case shall be more than obligations of the parties. Impairment is anything that diminishes the
three (3) months after foreclosure, whichever is earlier. Owners of efficacy of the contract. There is an impairment if a subsequent law
property that has been sold in a foreclosure sale prior to the changes the terms of a contract between the parties, imposes new
effectivity of this Act shall retain their redemption rights until their conditions, dispenses with those agreed upon or withdraws remedies
expiration. for the enforcement of the rights of the parties.
Under the new law, an exception is thus made in the case of juridical Section 47 did not divest juridical persons of the right to redeem their
persons which are allowed to exercise the right of redemption only foreclosed properties but only modified the time for the exercise of
"until, but not after, the registration of the certificate of foreclosure such right by reducing the one-year period originally provided in Act
sale" and in no case more than three (3) months after foreclosure, No. 3135. The new redemption period commences from the date of
whichever comes first. foreclosure sale, and expires upon registration of the certificate of
sale or three months after foreclosure, whichever is earlier. There is
May the foregoing amendment be validly applied in this case
likewise no retroactive application of the new redemption period
when the real estate mortgage contract was executed in 1985
because Section 47 exempts from its operation those properties

154 Aicka Singson


Notes in Remedial Law Review II
Provisional Remedies, Special Civil Actions, Special Proceedings and Evidence
School Year 2015-2016 REMEDIAL LAW REVIEW II - ATTY. BRONDIAL

foreclosed prior to its effectivity and whose owners shall retain their (ii) affidavit of self-adjudication – there is only 1 heir and there
redemption rights under Act No. 3135. are no debts. There is no one to settle with. Apply with the
Register of Deeds if dealing with real property and with the Office
Having ruled that the assailed Section 47 of R.A. No. 8791 is of the Civil Registrar if dealing with personal property. There are
constitutional, we find no reversible error committed by the CA in certain requirements there including publication and affidavit of
holding that petitioner can no longer exercise the right of redemption self-adjudication then the properties will be transferred to the
over its foreclosed properties after the certificate of sale in favor of heir’s name after payment of certain fees.
respondent had been registered.
This is a form of settlement of estate under judicial settlement of
H. PATRITION (Rule 69) estate. The first issue is the determination of co-ownership. Once
there is no co-ownership, partition will not apply.
Partition is the separation, division and assignment of a thing held in
common among those to whom it may belong. Example: H and W are husband and wife. They have 4 children: A,
B, C and D. H and W died. A, B, C and D are co-owners of the
Partition is a kind of settlement of the estate.
properties belonging to the estate. A is a lawyer; B is a doctor; C did
There are 2 general modes of settlement of estate: not graduate; and D is a teacher. They can resort to judicial or
extrajudicial settlement. A wanted to settle the property. The other
(1) judicial settlement of estate – siblings will oppose. They resort to judicial partition under Rule 69.

(i) summary settlement of estate of small value What should the court do upon filing of the petition for
partition? The court shall determine if there is co-ownership or not.
(ii) judicial partition All of them should file but in actual practice, not all of them files. So if
one files, the other 3 will be unwilling plaintiffs who shall be named
(iii)escheat – no heirs, so the government will take the property. as defendants under Rule 3.
Longest publication (once a week for 6 consecutive weeks in a
newspaper of general circulation). The proceeds from the escheat After the determination of co-ownership, the next step is for the court
proceedings will be given to the charitable institutions, hospitals or to convince them to enter into a project of partition. The project of
public schools. partition is what is called extrajudicial settlement. The court will try to
convince them to settle things there. A will say that they went to court
(iv) Conventional mode of settlement of estate – governed by because they cannot agree.
Rules 75-90
The court will move on to accounting and distribution. There will be a
(2) extrajudicial settlement of estate – outside the court, there are trial by commissioner. The committee is advised to come up with a
2 kinds: settlement. If this is not possible, the commissioner’s next option is
assignment. Assignment is like a buy-out.
(i)extrajudicial partition

155 Aicka Singson


Notes in Remedial Law Review II
Provisional Remedies, Special Civil Actions, Special Proceedings and Evidence
School Year 2015-2016 REMEDIAL LAW REVIEW II - ATTY. BRONDIAL

Example: The estate is worth P10,000,000 so P2,500,000 per heir. A Accordingly, an action will not lie without the joinder of all co-owners
should just give P2,500,000 to each sibling. and other persons having interest in the property. All the co-owners,
therefore, are indispensable parties.
But B refuses. He says he wants to be the one to buy it. They should
go now to the next option: sale. Even in that particular instance, MATTERS TO ALLEGE IN THE COMPLAINT FOR PARTITION
there is another problem: who will sell? It is not the commission who
sells. A will sell it at P2,000 per sqm but the other siblings do not The plaintiff shall state in his complaint, the nature and extent of his
assent. D also has a buyer for P2,100 per sqm. title, an adequate description of the real estate of which partition is
demanded, and shall join as defendants all other persons interested
They go back to court and the court will render judgment on in the property. He must also include a demand for the accounting of
whatever the siblings came up with. the rents, profits and other income from the property which he may
be entitled to. These cannot be demanded in another action because
Under the settlement of estate of proceedings, no sale of every they are parts of the cause of action for partition. They will be barred
property will be valid unless approved by the court. if not set up in the same action pursuant to the rule against splitting a
single cause of action
1. THE COMPLAINT (Section 1)
2. THE ORDER (Section 2)
Section 1. Complaint in action for partition of real estate.
Sec. 2. Order for partition, and partition by agreement thereunder.
A person having the right to compel the partition of real estate may do so as
provided in this Rule, setting forth in his complaint the nature and extent of
If after the trial the court finds that the plaintiff has the right thereto, it shall
his title and an adequate description of the real estate of which partition is
order the partition of the real estate among all the parties in interest.
demanded and joining as defendants all other persons interested in the
Thereupon the parties may, if they are able to agree, make the partition
property. among themselves by proper instruments of conveyance, and the court shall
confirm the partition so agreed upon by all the parties, and such partition,
Note: together with the order of the court confirming the same, shall be recorded in
the registry of deeds of the place in which the property is situated.
WHO MAY FILE COMPLAINT; WHO SHOULD BE MADE
DEFENDANTS A final order decreeing partition and accounting may be appealed by any
party aggrieved thereby.
The action shall be brought by the person who has a right to compel
the partition of real estate or of an estate composed of personal Notes:
property, or both real and personal property. The plaintiff is a person
who is supposed to be a co-owner of the property or estate sought to During the trial, the court shall determine whether or not the plaintiff
be partitioned. The defendants are all the co-owners. All the co- is truly a co-owner of the property, that there is indeed a co-
owners must be joined. ownership among the parties, and that a partition is not legally
proscribed thus may be allowed. If the court so finds that the facts

156 Aicka Singson


Notes in Remedial Law Review II
Provisional Remedies, Special Civil Actions, Special Proceedings and Evidence
School Year 2015-2016 REMEDIAL LAW REVIEW II - ATTY. BRONDIAL

are such that a partition would be in order, and that the plaintiff has a Sec. 3. Commissioners to make partition when parties fail to agree.
right to demand partition, the court will issue an order of partition.
If the parties are unable to agree upon the partition, the court shall appoint
The court shall order the partition of the property among all the not more than three (3) competent and disinterested persons as
commissioners to make the partition, commanding them to set off to the
parties in interest, if after trial it finds that the plaintiff has the right to
plaintiff and to each party in interest such part and proportion of the property
partition. It was held that this order of partition including an order
as the court shall direct.
directing an accounting is final and not interlocutory and hence,
appealable; thus, revoking previous contrary rulings on the matter. A Sec. 4. Oath and duties of commissioners.
final order decreeing partition and accounting may be appealed by
any party aggrieved thereby. Before making such partition, the commissioners shall take and subscribe an
oath that they will faithfully perform their duties as commissioners, which
Partition by agreement. The order of partition is one that directs the oath shall be filed in court with the other proceedings in the case. In making
parties or co-owners to partition the property and the parties may the partition, the commissioners shall view and examine the real estate, after
due notice to the parties to attend at such view and examination, and shall
make the partition among themselves by proper instruments of
hear the parties as to their preference in the portion of the property to be set
conveyance, if they agree among themselves. If they do agree, the apart to them and the comparative value thereof, and shall set apart the
court shall then confirm the partition so agreed upon by all of the same to the parties in lots or parcels as will be most advantageous and
parties, and such partition, together with the order of the court equitable, having due regard to the improvements, situation and quality of
confirming the same, shall be recorded in the registry of deeds of the the different parts thereof.
place in which the property is situated. There always exists the
possibility that the co-owners are unable to agree on the partition. If Sec. 5. Assignment or sale of real estate by commissioners.
they cannot partition the property among themselves, the next stage
When it is made to appear to the commissioners that the real estate, or a
in the action will follow the appointment of commissioners. portion thereof, cannot be divided without prejudice to the interests of the
parties, the court may order it assigned to one of the parties willing to take
3. STAGES OF PARTITION the same, provided he pays to the other parties such amounts as the
commissioners deem equitable, unless one of the interested parties asks
A reading of the Rules will reveal that there are actually three (3) that the property be sold instead of being so assigned, in which case the
stages in the action, each of which could be the subject of appeal: court shall order the commissioners to sell the real estate at public sale
under such conditions and within such time as the court may determine.
1) The order of partition where the property of the partition is
determined; Sec. 6. Report of commissioners; proceedings not binding until confirmed.

2) The judgment as to the accounting of the fruits and income of the The commissioners shall make a full and accurate report to the court of all
property; and their proceedings as to the partition, or the assignment of real estate to one
of the parties, or the sale of the same. Upon the filing of such report, the
3) The judgment of partition. clerk of court shall serve copies thereof on all the interested parties with
notice that they are allowed ten (10) days within which to file objections to
4. RULE OF COMMISIONERS (Sections 3-7) the findings of the report, if they so desire. No proceeding had before or

157 Aicka Singson


Notes in Remedial Law Review II
Provisional Remedies, Special Civil Actions, Special Proceedings and Evidence
School Year 2015-2016 REMEDIAL LAW REVIEW II - ATTY. BRONDIAL

conducted by the commissioners shall pass the title to the property or bind Section 12. Neither paramount rights nor amicable partition affected by this
the parties until the court shall have accepted the report of the Rule. — Nothing in this Rule contained shall be construed so as to prejudice,
commissioners and rendered judgment thereon. defeat, or destroy the right or title of any person claiming the real estate
involved by title under any other person, or by title paramount to the title of
Sec. 7. Action of the court upon commissioners’ report. the parties among whom the partition may have been made, nor so as to
restrict or prevent persons holding real estate jointly or in common from
Upon the expiration of the period of ten (10) days referred to in the making an amicable partition thereof by agreement and suitable instruments
preceding section, or even before the expiration of such period but after the of conveyance without recourse to an action.
interested parties have filed their objections to the report or their statement
of agreement therewith, the court may, upon hearing, accept the report and Section 13. Partition of personal property. — The provisions of this Rule
render judgment in accordance therewith; or, for cause shown, recommit the shall apply to partitions of estates composed of personal property, or of both
same to the commissioners for further report of facts; or set aside the report real and personal property, in so far as the same may be applicable.
and appoint new commissioners; or accept the report in part and reject it in
part; and may make such order and render such judgment as shall Note:
effectuate a fair and just partition of the real estate, or of its value, if
assigned or sold as above provided, between the several owners thereof. The judgment shall state definitely, by metes and bounds and
adequate description, the particular portion of the real estate
5. THE JUGDMENT (Section 11)
assigned to each party, the effect of the judgment shall be to vest in
Sec. 11. The judgment and its effect; copy to be recorded in registry of each party to the action in severalty the portion of the real estate
deeds. assigned to him.

If actual partition of property is made, the judgment shall state definitely, by If the whole property is assigned to one of the parties upon his
metes and bounds and adequate description, the particular portion of the paying to the others the sum or sums ordered by the court, the
real estate assigned to each party, and the effect of the judgment shall be to judgment shall state the fact of such payment and of the assignment
vest in each party to the action in severalty the portion of the real estate of the real estate to the party making the payment, and the effect of
assigned to him. If the whole property is assigned to one of the parties upon
the judgment shall be to vest in the party making the payment the
his paying to the others the sum or sums ordered by the court, the judgment
whole of the real estate free from any interest on the part of the other
shall state the fact of such payment and of the assignment of the real estate
to the party making the payment, and the effect of the judgment shall be to
parties to the action.
vest in the party making the payment the whole of the real estate free from
If the property is sold and the sale confirmed by the court, the
any interest on the part of the other parties to the action. If the property is
sold and the sale confirmed by the court, the judgment shall state the name judgment shall state the name of the purchaser or purchasers and a
of the purchaser or purchasers and a definite description of the parcels of definite description of the parcels of real estate sold to each
real estate sold to each purchaser, and the effect of the judgment shall be to purchaser, and the effect of the judgment shall be to vest the real
vest the real estate in the purchaser or purchasers making the payment or estate in the purchaser or purchasers making the payment or
payments, free from the claims of any of the parties to the action. A certified payments, free from the claims of any of the parties to the action.
copy of the judgment shall in either case be recorded in the registry of deeds
of the place in which the real estate is situated, and the expenses of such
recording shall be taxed as part of the costs of the action.

158 Aicka Singson


Notes in Remedial Law Review II
Provisional Remedies, Special Civil Actions, Special Proceedings and Evidence
School Year 2015-2016 REMEDIAL LAW REVIEW II - ATTY. BRONDIAL

A certified copy of the judgment shall in either case be recorded in the period allowed by law and the sheriff executed a Definite Deed of
the registry of deeds of the place in which the real estate is situated, Sale in the Bank's favor.
and the expenses of such recording shall be taxed as part of the
costs of the action. On October 10, 1989, herein petitioner and respondents executed an
Extrajudicial Settlement of Estate adjudicating to each of them a
PARTITION OF PERSONAL PROPERTY specific one-third portion of the subject property consisting of 10,246
square meters. The Extrajudicial Settlement also contained
The provisions of this Rule shall apply to partitions of estates provisions wherein the parties admitted knowledge of the fact that
composed of personal property, or of both real and personal their father mortgaged the subject property to the Bank and that they
property, in so far as the same may be applicable. intended to redeem the same at the soonest possible time.

PRESCRIPTION OF ACTION Three years after the execution of the Extrajudicial Settlement,
herein respondents bought the subject property from the Bank, a
 Prescription of action does not run in favor of a co-owner or co-
Deed of Sale of Registered Land was executed by the Bank in favor
heir against his co-owner or co-heirs as long as there is a
of respondents. Subsequently, new title was issued in favor of
recognition of the co-ownership expressly or impliedly.
respondents but petitioner continued possession of the subject lot.
 The action for partition cannot be barred by prescription as long
as the co-ownership exists. Respondents filed a Complaint for Recovery of Possession and
 But while the action to demand partition of a co-owned property Damages against petitioner, contending that they had already
does not prescribe, a co-owner may acquire ownership thereof by informed petitioner of the fact that they were the new owners of the
prescription where there exists a clear repudiation of the co- disputed property, but the petitioner still refused to surrender
ownership and the co-owners are apprised of the claim of adverse possession of the same to them.
and exclusive ownership.
The RTC rendered a Decision ordering the plaintiffs to execute a
CASES: Deed of Sale in favor of the defendant, the one-third share of the
property in question, presently possessed by him, and described in
a. Balus vs. Balus (610 SCRA)
the deed of partition.
FACTS: Herein petitioner and respondents are the children of the
spouses Rufo and Sebastiana Balus. Sebastiana. The RTC held that the right of petitioner to purchase from the
respondents his share in the disputed property was recognized by
Rufo mortgaged a parcel of land, which he owns, as security for a
the provisions of the Extrajudicial Settlement of Estate, which the
loan he obtained from the Rural Bank of Maigo.
parties had executed before the respondents bought the subject lot
from the Bank.
Rufo failed to pay his loan, the mortgaged property was foreclosed
and was subsequently sold to the Bank as the sole bidder at a public
Aggrieved by the Decision of the RTC, herein respondents filed an
auction held for that purpose. Certificate of Sale was executed by the
appeal with the CA.The CA reversed and set aside RTC’s decision
sheriff in favor of the Bank. The property was not redeemed within

159 Aicka Singson


Notes in Remedial Law Review II
Provisional Remedies, Special Civil Actions, Special Proceedings and Evidence
School Year 2015-2016 REMEDIAL LAW REVIEW II - ATTY. BRONDIAL

contending that when petitioner and respondents did not redeem the Moreover, petitioner's asseveration of his and respondents' intention
subject property within the redemption period and allowed the of continuing with their supposed co-ownership is negated by no less
consolidation of ownership and the issuance of a new title in the than his assertions in the present petition that on several occasions
name of the Bank, their co-ownership was extinguished. Hence, the he had the chance to purchase the subject property back, but he
instant petition. refused to do so. In fact, he claims that after the Bank acquired the
disputed lot, it offered to re-sell the same to him but he ignored such
ISSUE: Whether co-ownership by petitioner and respondents offer. How then can petitioner now claim that it was also his intention
over the subject property persisted even after the lot was to purchase the subject property from the Bank, when he admitted
purchased by the Bank and title thereto transferred to its name, that he refused the Bank's offer to re-sell the subject property to him?
and even after it was eventually bought back by the
respondents from the Bank Furthermore, petitioner's contention that he and his siblings intended
to continue their supposed co-ownership of the subject property
HELD: NO. contradicts the provisions of the subject Extrajudicial Settlement
where they clearly manifested their intention of having the subject
Petitioner and respondents are arguing on the wrong premise that, at
property divided or partitioned by assigning to each of the petitioner
the time of the execution of the Extrajudicial Settlement, the subject
and respondents a specific 1/3 portion of the same.
property formed part of the estate of their deceased father to which
they may lay claim as his heirs. Partition calls for the segregation and conveyance of a determinate
portion of the property owned in common. It seeks a severance of
The rights to a person's succession are transmitted from the moment
the individual interests of each co-owner, vesting in each of them a
of his death. In the present case, since Rufo lost ownership of the
sole estate in a specific property and giving each one a right to enjoy
subject property during his lifetime, it only follows that at the time of
his estate without supervision or interference from the other. In other
his death, the disputed parcel of land no longer formed part of his
words, the purpose of partition is to put an end to co-ownership, an
estate to which his heirs may lay claim. Stated differently, petitioner
objective which negates petitioner's claims in the present case.
and respondents never inherited the subject lot from their father.
b. Feliciano vs. Canosa (629 SCRA)
Petitioner and respondents, therefore, were wrong in assuming that
they became co-owners of the subject lot. Thus, any issue arising When Antonio Feliciano passed away, he left behind his only
from the supposed right of petitioner as co-owner of the contested property, a parcel of land located in Bulacan. On March 28, 1972,
parcel of land is negated by the fact that, in the eyes of the law, the Leona, Maria, Pedro and Salina, all surnamed Feliciano, declared
disputed lot did not pass into the hands of petitioner and respondents themselves to be the only surviving heirs of Antonio Feliciano, with
as compulsory heirs of Rufo at any given point in time. the exception of Salina. They executed an extrajudicial settlement of
Antonio Feliciano’s estate and appropriated among themselves the
There is no co-ownership to talk about and no property to partition,
said parcel of land, to the exclusion of the heirs of Esteban Feliciano
as the disputed lot never formed part of the estate of their deceased
and Doroteo Feliciano, deceased children of Antonio Feliciano. On
father.
even date, Leona, Maria, Pedro and Salina executed a deed of

160 Aicka Singson


Notes in Remedial Law Review II
Provisional Remedies, Special Civil Actions, Special Proceedings and Evidence
School Year 2015-2016 REMEDIAL LAW REVIEW II - ATTY. BRONDIAL

absolute sale or Kasulatan sa Ganap Na Bilihan over the property in Respondents appealed to the CA rendered the assailed Decision
favor of the late Jacinto Feliciano (Pedro’s portion), Felisa Feliciano reversing the trial court’s decision. The CA held that prescription had
(Salina’s portion) and Pedro Canoza (Leona and Maria’s portions). set in, citing the case of Pedrosa v. Court of Appeals, which held
that the applicable prescriptive period to annul a deed of extrajudicial
On October 18, 1993, Eugenio Feliciano and Angelina Feliciano-de settlement is four (4) years from the discovery of the fraud. It
Leon, surviving heirs of the late Esteban Feliciano, and Trinidad reasoned that when petitioners filed the instant complaint for the
Feliciano-Valiente and Basilia Feliciano-Trinidad, surviving children annulment of the extrajudicial settlement of Antonio Feliciano’s
of the late Doroteo Feliciano, filed a complaint against Salina estate, more than four (4) years had elapsed from the issuance of
Feliciano, Felisa Feliciano, Pedro Canoza and the heirs of the late the free patents. Petitioners filed a motion for reconsideration of the
Jacinto Feliciano, namely Delia, Rosauro, Elsa, Nardo and Ponciano, aforesaid Decision but it was denied by the CA in the Resolution
all surnamed Feliciano, for the Declaration of Nullity of Documents dated January 15, 2004 for lack of merit.
and Title, Recovery of Real Property and Damages. They alleged
that the settlement of the estate and sale were done without their Issue: Whether the CA erred in reversing the trial court’s
participation and consent as heirs of Esteban and Doroteo. Likewise, decision in declaring that prescription had set in
they averred that the ancestral home of the Felicianos is erected on
the subject property and that they have occupied the same since Held: The Supreme Court affirmed the ruling of the CA. As the
birth. Canoza and Jacinto falsely declared that the property was not records show, the heirs of Doroteo and Esteban did not participate in
occupied, so their titles to the property should be declared null and the extrajudicial partition executed by Salina with the other
void on the ground that they have made false statements in their compulsory heirs, Leona, Maria and Pedro. Undeniably, the said
respective applications for free patent. deed was fraudulently obtained as it deprived the known heirs of
Doroteo and Esteban of their shares in the estate. A deed of
Before an Answer could be filed, the petitioners amended their extrajudicial partition executed without including some of the heirs,
complaint to include the allegation that they sought to recover the who had no knowledge of and consent to the same, is fraudulent and
shares of their fathers, Esteban and Doroteo, which they could have vicious. Hence, an action to set it aside on the ground of fraud could
acquired as heirs of Antonio Feliciano. be instituted. Such action for the annulment of the said partition,
however, must be brought within four (4) years from the discovery of
In their Answer, respondent Pedro Canoza and his spouse, the fraud. In Gerona v. De Guzman, respondents therein executed a
respondent Delia Feliciano, alleged that they were buyers in good deed of extrajudicial settlement declaring themselves to be the sole
faith and for value. They likewise contended that assuming that there heirs of the late Marcelo de Guzman. They secured new transfer
was preterition of legal heirs, they never took part in it. certificates of title in their own names, thereby excluding the
petitioners therein from the estate of the deceased. The petitioners
The trial court rendered judgment in favor of the plaintiffs and against
brought an action for the annulment of the said deed upon the
the defendants declaring the extra-judicial settlement of estate of
ground that the same is tainted with fraud. The Court held,
Antonio Feliciano null and void.
Inasmuch as petitioners seek to annul the aforementioned deed of
"extra-judicial settlement" upon the ground of fraud in the execution

161 Aicka Singson


Notes in Remedial Law Review II
Provisional Remedies, Special Civil Actions, Special Proceedings and Evidence
School Year 2015-2016 REMEDIAL LAW REVIEW II - ATTY. BRONDIAL

thereof, the action therefor may be filed within four (4) years from the their love and affection for respondent and the sum of P150,000.00,
discovery of the fraud (Mauricio v. Villanueva, L-11072, September waived and ceded their respective shares over the three parcels of
24, 1959). Such discovery is deemed to have taken place, in the land in favor of respondent.
case at bar, on June 25, 1948, when said instrument was filed with
the Register of Deeds and new certificates of title were issued in the According to petitioner, respondent promised to give her an
name of respondents exclusively, for the registration of the deed of additional amount for her share in her father’s estate. Thus, after the
extra-judicial settlement constitute constructive notice to the whole signing of the Deed, petitioner demanded from respondent the
world.27 (Emphasis and underscoring supplied.) promised additional amount, but respondent refused to pay, claiming
that she had no more money.
Evidently, the applicable prescriptive period to institute the action to
annul the deed of extrajudicial settlement was four (4) years counted A year later, while processing her tax obligations with the BIR,
from the discovery of fraud as held in the case of Gerona v. De respondent was required to submit an original copy of the Deed. Left
Guzman. However, the records show that petitioners’ complaint was with no more original copy of the Deed, respondent summoned
filed only on October 18, 1993, or almost sixteen (16) years after petitioner to her office and asked her to countersign a copy of the
Jacinto Feliciano was issued Free Patent No. (IV-4) 012293 on Deed. Petitioner refused to countersign the document, demanding
November 28, 1977, and almost fourteen (14) years from the time that respondent first give her the additional amount that she
Pedro Canoza was issued OCT No. P-364 on November 28, 1979. promised. Petitioner agreed to sign the Deed when respondent
As petitioners are deemed to have obtained constructive notice of signed the promissory note.
the fraud upon the registration of the Free Patent, they clearly failed
When the promissory note fell due, respondent failed and refused to
to institute the present civil action within the allowable period. The
pay despite demand. Petitioner made several more demands upon
same result obtains even if their complaint is treated as one (1)
respondent but the latter kept on insisting that she had no money.
essentially for reconveyance as more than ten (10) years have
passed since petitioners’ cause of action accrued. The CA Petitioner filed a Complaint for Specific Performance with Damages
committed no error in dismissing their complaint. against respondent.

c. Mangahas vs. Brobio (634 SCRA) In her Answer with Compulsory Counterclaim, respondent admitted
that she signed the promissory note but claimed that she was forced
FACTS: On January 10, 2002, Pacifico S. Brobio died intestate,
to do so. She also claimed that the undertaking was not supported
leaving three parcels of land. He was survived by his wife,
by any consideration.
respondent Eufrocina A. Brobio, and four legitimate and three
illegitimate children; petitioner Carmela Brobio Mangahas is one of The RTC rendered a decision in favor of petitioner.
the illegitimate children.
The CA reversed the RTC decision and dismissed the complaint.
The heirs of the deceased executed a Deed of Extrajudicial The CA found that there was a complete absence of consideration in
Settlement of Estate of the Late Pacifico Brobio with Waiver. In the the execution of the promissory note, which made it inexistent and
Deed, petitioner and Pacifico’s other children, in consideration of without any legal force and effect. The CA went on to hold that if

162 Aicka Singson


Notes in Remedial Law Review II
Provisional Remedies, Special Civil Actions, Special Proceedings and Evidence
School Year 2015-2016 REMEDIAL LAW REVIEW II - ATTY. BRONDIAL

petitioner disagreed with the amount she received, then she should I. FORCIBLE ENTRY AND UNLAWFUL DETAINER (RULE
have filed an action for partition. 70)
Petitioner moved for the reconsideration of the CA Decision – 1. Distinction between accion publiciana, accion reivindicatoria and
DENIED. accion interdictal

ISSUE: WON CA erred when it stated that petitioner should have These are the 3 kinds of recovery actions.
filed an action for partition instead of a case for specific
performance. Accion Publiciana -Recovery of plenary possession, may be based
on ownership or not
HELD: YES.
Accion Reivindicatoria -Recovery of ownership
Respondent failed to prove that the promissory note was not
supported by any consideration. From her testimony and her Accion Interdictal-Recovery of possession de facto only
assertions in the pleadings, it is clear that the promissory note was
issued for a cause or consideration, which, at the very least, was Forcible Entry- An entry effected by force, intimidation, threat,
petitioner’s signature on the document. stealth, strategy, and the action is to recover possession founded
upon illegal possession from the beginning.
It may very well be argued that if such was the consideration, it was
inadequate. Nonetheless, even if the consideration is inadequate, The person must be deprived of possession and the action must be
the contract would not be invalidated, unless there has been fraud, brought within one year from the unlawful deprivation.
mistake, or undue influence. As previously stated, none of these
grounds had been proven present in this case. Unlawful Detainer – Unlawful detention by a person who has
acquired possession rightfully, by who detains the property after the
The foregoing discussion renders the final issue insignificant. Be that right to keep possession has ended.
as it may, we would like to state that the remedy suggested by the
CA is not the proper one under the circumstances. An action for Note: Action must be brought within one year after such unlawful
partition implies that the property is still owned in common. deprivation or withholding of possession and demand to pay or
Considering that the heirs had already executed a deed of comply with the conditions of the lease and to vacate is made upon
extrajudicial settlement and waived their shares in favor of the lessee.
respondent, the properties are no longer under a state of co-
ownership; there is nothing more to be partitioned, as ownership had
already been merged in one person.

163 Aicka Singson


Notes in Remedial Law Review II
Provisional Remedies, Special Civil Actions, Special Proceedings and Evidence
School Year 2015-2016 REMEDIAL LAW REVIEW II - ATTY. BRONDIAL

FORCIBLE ENTRY UNLAWFUL *** If you use tolerance as a basis for unlawful detainer, the tolerance
DETAINER must be existing at the time of possession. If it is not existing, as
Grounds: (1) fraud; (2) Grounds: (1) Termination of the when tolerance came after possession, unlawful detainer cannot be
intimidation; (3) strategy; contract;** (2) tolerance***
a ground.
(4) stealth; (5) threat****
The possession of the The possession of the Example: A has a piece of land. There are a lot of squatters. A said
defendant is unlawful defendant is lawful from the to them they may stay but if he demands that they should leave,
from the beginning; beginning becomes illegal by they’ll have to vacate. Thus, tolerance is at the time of possession.
issue is which party has reason of the expiration or
prior de facto termination of his right to the
If A sells the property and they are in possession of the property, the
possession; possession of the property;
new owner of the property cannot file an unlawful detainer case
against them on the ground of tolerance because they were in
The law does not Plaintiff must first make
require previous such demand which is possession before the new owner “tolerated” them. How could he
demand for the jurisdictional in nature;***** have really tolerated these people when he was not the owner of the
defendant to vacate; property?
The plaintiff must The plaintiff need not have
prove that he was in been in prior physical This is the new doctrine now in unlawful detainer: tolerance must be
prior physical possession; existing at the time of the possession. Otherwise, it cannot be a
possession of the ground for unlawful detainer.
premises until he was
deprived by the
**** this list is exclusive
defendant; and

The one year period is The one-year period is ***** After demand, you have to vacate within 5 days (in case of
generally counted counted from the date of land/building, within 15 days)
from the date of actual last demand.
entry on the property. These are originally cognizable by the MTCs only and governed by
the Rules on Summary Procedure.
Layman term: ejectment proceedings*
Go back to the salient points of Summary Procedure:
Issue: Who has the better right of possession?
(1) there are prohibited pleadings
* But ejectment proceedings are not anchored only on Rule 70. Rule
70 are the grounds for ejectment but those are not the only ground. Demurrer to evidence is not a prohibited pleading.
** Termination of the contract, whether expressed or implied, which (2) the periods are much shorter
may be oral. The oral contract may be terminated by notifying. This
may be on a week-to-week, month-to-month, or year-to-year basis, When you file the unlawful detainer case, the defendant must answer
without any reason because you are the owner of the property. within a period of 10 days rather than 15 days

164 Aicka Singson


Notes in Remedial Law Review II
Provisional Remedies, Special Civil Actions, Special Proceedings and Evidence
School Year 2015-2016 REMEDIAL LAW REVIEW II - ATTY. BRONDIAL

(3) because it is summary in nature, there is no trial WHO MAY INSTITUTE THE ACTION AND WHEN; AGAINST
WHOM THE ACTION MAY BE MAINTAINED
There is only submission of position papers, affidavits, depositions
and other documents in support of one’s position. On the basis of A person deprived of the possession of any land or building by force,
that, judgment will be rendered. intimidation, threat, strategy, or stealth, or a lessor, vendor, vendee,
or other person against whom the possession of any land or building
1. PARTIES (Section 1) is unlawfully withheld after the expiration or termination of the right to
hold possession, by virtue of any contract, express or implied, or the
Section 1. Who may institute proceedings, and when.
legal representatives or assigns of any such lessor, vendor, vendee,
Subject to the provisions of the next succeeding section, a person or other person, may, at any time within one (1) year after such
deprived of the possession of any land or building by force, unlawful deprivation or withholding of possession, bring an action in
intimidation, threat, strategy, or stealth, or a lessor, vendor, vendee, the proper Municipal Trial Court against the person or persons
or other person against whom the possession of any land or building unlawfully withholding or depriving of possession, or any person or
is unlawfully withheld after the expiration or termination of the right to persons claiming under them, for the restitution of such possession,
hold possession, by virtue of any contract, express or implied, or the together with damages and costs.
legal representatives or assigns of any such lessor, vendor, vendee,
Unless otherwise stipulated, such action by the lessor shall be
or other person, may, at any time within one (1) year after such
commenced only after demand to pay or comply with the conditions
unlawful deprivation or withholding of possession, bring an action in
of the lease and to vacate is made upon the lessee, or by serving
the proper Municipal Trial Court against the person or persons
written notice of such demand upon the person found on the
unlawfully withholding or depriving of possession, or any person or
premises, or by posting such notice on the premises if no person be
persons claiming under them, for the restitution of such possession,
found thereon, and the lessee fails to comply therewith after fifteen
together with damages and costs.
(15) days in the case of land or five (5) days in the case of buildings.
NOTE: 2. PROCEDURE; SUMMARY (Sections 3-15)
Don’t miss the rent control law. This is also a ground for Sec. 3. Summary procedure.
ejectment. Except in cases covered by the agricultural tenancy laws or when the law
otherwise expressly provides, all actions for forcible entry and unlawful
• Recovery of possession due to the use of a family by an detainer, irrespective of the amount of damages or unpaid rentals sought to
immediate member. be recovered, shall be governed by the summary procedure hereunder
provided.
• Renovation is a ground Sec. 4. Pleadings allowed.
The only pleadings allowed to be filed are the complaint, compulsory
• Termination of contract counterclaim and cross-claim pleaded in the answer, and the answers
thereto. All pleadings shall be verified.
Sec. 5. Action on complaint.

165 Aicka Singson


Notes in Remedial Law Review II
Provisional Remedies, Special Civil Actions, Special Proceedings and Evidence
School Year 2015-2016 REMEDIAL LAW REVIEW II - ATTY. BRONDIAL

The court may, from an examination of the allegations in the complaint and Within five (5) days after the termination of the preliminary conference, the
such evidence as may be attached thereto, dismiss the case outright on any court shall issue an order stating the matters taken up therein, including but
of the grounds for the dismissal of a civil action which are apparent therein. If not limited to:
no ground for dismissal is found, it shall forthwith issue summons. 1. Whether the parties have arrived at an amicable settlement, and if so, the
Sec. 6. Answer. terms thereof;
Within ten (10) days from service of summons, the defendant shall file his 2. The stipulations or admissions entered into by the parties;
answer to the complaint and serve a copy thereof on the plaintiff. Affirmative 3. Whether, on the basis of the pleadings and the stipulations and
and negative defenses not pleaded therein shall be deemed waived, except admissions made by the parties, judgment may be rendered without the
lack of jurisdiction over the subject matter. Cross-claims and compulsory need of further proceedings, in which event the judgment shall be rendered
counterclaims not asserted in the answer shall be considered barred. The within thirty (30) days from issuance of the order;
answer to counterclaims or cross-claims shall be served and filed within ten 4. A clear specification of material facts which remain controverted; and
(10) days from service of the answer in which they are pleaded. 5. Such other matters intended to expedite the disposition of the case.
Sec. 7. Effect of failure to answer. Sec. 10. Submission of affidavits and position papers.
Should the defendant fail to answer the complaint within the period above Within ten (10) days from receipt of the order mentioned in the next
provided, the court, motu proprio or on motion of the plaintiff, shall render preceding section, the parties shall submit the affidavits of their witnesses
judgment as may be warranted by the facts alleged in the complaint and and other evidence on the factual issues defined in the order, together with
limited to what is prayed for therein. The court may in its discretion reduce their position papers setting forth the law and the facts relied upon by them.
the amount of damages and attorney’s fees claimed for being excessive or Sec. 11. Period for rendition of judgment.
otherwise unconscionable, without prejudice to the applicability of section 3 Within thirty (30) days after receipt of the affidavits and position papers, or
(c), Rule 9 if there are two or more defendants. the expiration of the period for filing the same, the court shall render
Sec. 8. Preliminary conference; appearance of parties. judgment.
Not later than thirty (30) days after the last answer is filed, a preliminary However, should the court find it necessary to clarify certain material facts, it
conference shall be held. The provisions of Rule 18 on pre-trial shall be may, during the said period, issue an order specifying the matters to be
applicable to the preliminary conference unless inconsistent with the clarified, and require the parties to submit affidavits or other evidence on the
provisions of this Rule. said matters within ten (10) days from receipt of said order. Judgment shall
The failure of the plaintiff to appear in the preliminary conference shall be be rendered within fifteen (15) days after the receipt of the last affidavit or
cause for the dismissal of his complaint. The defendant who appears in the the expiration of the period for filing the same.
absence of the plaintiff shall be entitled to judgment on his counterclaim in The court shall not resort to the foregoing procedure just to gain time for the
accordance with the next preceding section. All cross-claims shall be rendition of the judgment.
dismissed. Sec. 12. Referral for conciliation.
If a sole defendant shall fail to appear, the plaintiff shall likewise be entitled Cases requiring referral for conciliation, where there is no showing of
to judgment in accordance with the next preceding section. This procedure compliance with such requirement, shall be dismissed without prejudice, and
shall not apply where one of two or more defendants sued under a common may be revived only after that requirement shall have been complied with.
cause of action who had pleaded a common defense shall appear at the Sec. 13. Prohibited pleadings and motions.
preliminary conference. The following petitions, motions, or pleadings shall not be allowed:
No postponement of the preliminary conference shall be granted except for 1. Motion to dismiss the complaint except on the ground of lack of
highly meritorious grounds and without prejudice to such sanctions as the jurisdiction over the subject matter, or failure to comply with section 12;
court in the exercise of sound discretion may impose on the movant. 2. Motion for a bill of particulars;
Sec. 9. Record of preliminary conference.

166 Aicka Singson


Notes in Remedial Law Review II
Provisional Remedies, Special Civil Actions, Special Proceedings and Evidence
School Year 2015-2016 REMEDIAL LAW REVIEW II - ATTY. BRONDIAL

3. Motion for new trial, or for reconsideration of a judgment, or for reopening resolve the issue of possession. When that happens, possession
of trial; becomes an attribute of ownership.
4. Petition for relief from judgment;
5. Motion for extension of time to file pleadings, affidavits or any other paper; Default is a prohibited pleading. But there is a provision here
6. Memoranda; regarding partial default. This happens when there are multiple
7. Petition for certiorari, mandamus, or prohibition against any interlocutory parties. Cross refer to Rule 9 because if there are multiple party-
order issued by the court;
defendants, and one answers while the others do not, those who did
8. Motion to declare the defendant in default;
9. Dilatory motions for postponement;
not answer will be declared in partial default. But the case can move
10. Reply; on. Especially if there is a common defense, the case must be
11. Third-party complaints; resolved.
12. Interventions.
Sec. 14. Affidavits. But what if the defense is not a common defense? Suppose the
The affidavits required to be submitted under this Rule shall state only facts issue is non-payment of rentals. Payment will be a common defense.
of direct personal knowledge of the affiants which are admissible in Example: X filed a case against A, B and C. A said he paid. His
evidence, and shall show their competence to testify to the matters stated defense accrues to the others because the defense is common. But
therein. how about where the defense is not a common defense, like
A violation of this requirement may subject the party or the counsel who minority? A vs X, Y and Z. X puts up the defense of minority. It does
submits the same to disciplinary action, and shall be cause to expunge the
not mean that because X is a minor, the other two are minors. That
inadmissible affidavit or portion thereof from the record.
is not a common defense.
Section 16. Resolving defense of ownership. — When the defendant raises
the defense of ownership in his pleadings and the question of possession
5. Judgment; character as immediately executory
cannot be resolved without deciding the issue of ownership, the issue of
ownership shall be resolved only to determine the issue of possession.
Judgment is given by MTC. The Rules say the judgment is
immediately executory. Why is it that there is no execution if it is
Notes: The only issue is possession de facto; never ownership. But
immediately executory? Because you can stall the execution of the
Section 16 states that when the issue of ownership is raised in the
judgment.
pleading, the court is not divested of its jurisdiction and must resolve
the issue of ownership only to resolve the issue of possession. Section 17. Judgment. — If after trial court finds that the allegations of the
Hence, the resolution of the trial court with regard to ownership is not complaint are true, it shall render judgment in favor of the plaintiff for the
res judicata. Litis pendentia will not even apply in unlawful detainer if restitution of the premises, the sum justly due as arrears of rent or as
there is an issue of accion reivindicatoria before the RTC at the reasonable compensation for the use and occupation of the premises,
same time that a case of accion interdictal is pending with the MTC. attorney's fees and costs. If a counterclaim is established, the court shall
render judgment for the sum found in arrears from either party and award
You cannot file a motion to dismiss on the ground of litis pendentia
costs as justice requires.
because the issues are absolutely different. While in the MTC, it is
possession, in the RTC, it is ownership. But the issue of ownership Section 18. Judgment conclusive only on possession; not conclusive in
may be raised in the MTC and must be resolved by the MTC only to actions involving title or ownership. — The judgment rendered in an action

167 Aicka Singson


Notes in Remedial Law Review II
Provisional Remedies, Special Civil Actions, Special Proceedings and Evidence
School Year 2015-2016 REMEDIAL LAW REVIEW II - ATTY. BRONDIAL

for forcible entry or detainer shall be conclusive with respect to the execution will not be stalled because there are 2 more requirements:
possession only and shall in no wise bind the title or affect the ownership of the posting of the supersedeas bond and the regular monthly deposit
the land or building. Such judgment shall not bar an action between the for the use and occupancy of the premises.
same parties respecting title to the land or building.
Once a notice of appeal is filed by the obligor, then the Clerk of Court
The judgment or final order shall be appealable to the appropriate Regional
of the MTC will elevate the records within 15 days. Even if the
Trial Court which shall decide the same on the basis of the entire record of
the proceedings had in the court of origin and such memoranda and/or briefs records are already in the appellate court, if there is no supersedeas
as may be submitted by the parties or required by the Regional Trial Court. bond, there will be an execution pending appeal under Section 2 of
Rule 39. You can file an execution pending appeal for non-
Section 21. Immediate execution on appeal to Court of Appeals or Supreme compliance with Section 19 of Rule 70.
Court. — The judgment of the Regional Trial Court against the defendant
shall be immediately executory, without prejudice to a further appeal that How much should the supersedeas bond be? The supersedeas bond
may be taken therefrom. refers to the unpaid rentals. If there are no unpaid rentals, there is no
need for the posting of supersedeas bond. Its amount should be
6. Ways to stall execution of judgment
according to the judgment.
How can you stall the execution of the judgment? Under section 19,
Suppose the judgment does not talk of the unpaid rentals, how much
in order to stall the execution of a judgment, the following are
will be paid for the supersedeas bond? According to the contract.
needed: (1) file a notice of appeal; (2) post supersedeas bond; and
(3) pay monthly deposit for the use and occupancy of the premises. Suppose there is no contract? According to the last payment.

The first 2 should be filed with the RTC. The regular monthly deposit Where there is no payment, there is no rental.
must be filed or paid with the appellate court because of the
timeframe (on or before the 10th day of every succeeding month). Section 19. Immediate execution of judgment; how to stay same. —
Example: Rentals for January shall be paid on or before February If judgment is rendered against the defendant, execution shall issue
10. immediately upon motion unless an appeal has been perfected and
the defendant to stay execution files a sufficient supersedeas bond,
What is the rationale? The records have already been elevated to approved by the Municipal Trial Court and executed in favor of the
the appellate court pursuant to Rule 40. Upon perfection of appeal, plaintiff to pay the rents, damages, and costs accruing down to the
you have to elevate the records within 15 days. time of the judgment appealed from, and unless, during the
pendency of the appeal, he deposits with the appellate court the
If a notice of appeal is filed by the defendant-appellant or the
amount of rent due from time to time under the contract, if any, as
judgment obligor, is appeal perfected? Yes. Section 19 does not
determined by the judgment of the Municipal Trial Court. In the
prohibit appeal. What it stalls is the execution so that non-
absence of a contract, he shall deposit with the Regional Trial Court
compliance with any of the 3 requirements will render immediate
the reasonable value of the use and occupation of the premises for
execution of the judgment. But when you file a notice of appeal,
the preceding month or period at the rate determined by the
appeal is perfected but will that stall the execution? Yes. The

168 Aicka Singson


Notes in Remedial Law Review II
Provisional Remedies, Special Civil Actions, Special Proceedings and Evidence
School Year 2015-2016 REMEDIAL LAW REVIEW II - ATTY. BRONDIAL

judgment of the lower court on or before the tenth day of each If the RTC affirms the decision of the MTC in favor of the plaintiff,
succeeding month or period. The supersedeas bond shall be there is no more appeal. If you want to stall it, file a petition for
transmitted by the Municipal Trial Court, with the papers, to the clerk review. Even in your petition for review, you can ask for execution. It
of the Regional Trial Court to which the action is appealed. will not stall anymore the execution unless the appellant or petitioner
here is able to get injunction. Without injunction, there will be
All amounts so paid to the appellate court shall be deposited with execution.
said court or authorized government depositary bank, and shall be
held there until the final disposition of the appeal, unless the court, Section 15. Preliminary injunction. — The court may grant
by agreement of the interested parties, or in the absence of preliminary injunction, in accordance with the provisions of Rule 58
reasonable grounds of opposition to a motion to withdraw, or for hereof, to prevent the defendant from committing further acts of
justifiable reasons, shall decree otherwise. Should the defendant fail dispossession against the plaintiff.
to make the payments above prescribed from time to time during the
pendency of the appeal, the appellate court, upon motion of the A possessor deprived of his possession through forcible from the
plaintiff, and upon proof of such failure, shall order the execution of filing of the complaint, present a motion in the action for forcible entry
the judgment appealed from with respect to the restoration of or unlawful detainer for the issuance of a writ of preliminary
possession, but such execution shall not be a bar to the appeal mandatory injunction to restore him in his possession. The court
taking its course until the final disposition thereof on the merits. shall decide the motion within thirty (30) days from the filing thereof.

After the case is decided by the Regional Trial Court, any money Section 15 is about injunctive relief and who can avail of injunctive
paid to the court by the defendant for purposes of the stay of relief. It may be availed of pending the case with the MTC. Correlate
execution shall be disposed of in accordance with the provisions of this with section 20, which is an injunctive relief before the RTC.
the judgment of the Regional Trial Court. In any case wherein it Even on appeal, the appellant can move for injunction. This will be
appears that the defendant has been deprived of the lawful mandatory injunction (not prohibitory) because you want to return to
possession of land or building pending the appeal by virtue of the the status quo.
execution of the judgment of the Municipal Trial Court, damages for
Section 20. Preliminary mandatory injunction in case of appeal. —
such deprivation of possession and restoration of possession and
Upon motion of the plaintiff, within ten (10) days from the perfection
restoration of possession may be allowed the defendant in the
of the appeal to the Regional Trial Court, the latter may issue a writ
judgment of the Regional Trial Court disposing of the appeal.
of preliminary mandatory injunction to restore the plaintiff in
7. Appeals possession if the court is satisfied that the defendant's appeal is
frivolous or dilatory or that the appeal of the plaintiff is prima facie
But when you go up to the appellate court (which is RTC under Rule meritorious.
42), before the RTC, there is no summary procedure. Even if it is an
unlawful detainer case or a forcible entry case, once it has reached
the RTC, it is no longer summary procedure.

169 Aicka Singson


Notes in Remedial Law Review II
Provisional Remedies, Special Civil Actions, Special Proceedings and Evidence
School Year 2015-2016 REMEDIAL LAW REVIEW II - ATTY. BRONDIAL

CASES: wrested from another who had been in the physical or material
possession of the same for more than one year by resorting to a
a. Suarez vs. Emboy Jr (718 SCRA 677, 3/12/2014) summary action of ejectment.” The respondents also invoked the
doctrine enunciated in Amagan v. Marayag that the pendency of
At the center of the dispute is a 222–square meter parcel of land,
another action anchored on the issue of ownership justifies the
situated in Cebu City, and covered by Transfer Certificate of Title
suspension of an ejectment suit involving the same real property.
issued in the name of Carmencita.
The foregoing is especially true in the case at bar where the issue of
possession is so interwoven with that of ownership. Besides, the
Respondents were asked by their cousins, who are the Heirs of
Vicente, to vacate the subject lot but they refused to comply. resolution of the question of ownership would necessarily result in
the disposition of the issue of possession.
Respondents received from Carmencita’s counsel, (Atty. Pareja), a
The respondents also averred that Carmencita’s complaint lacked a
demand letter, requiring them to vacate the subject lot. They were
informed that Carmencita had already purchased the subject lot from cause of action. The certification to file an action was issued by the
the former’s relatives. However, the respondents did not heed the officials of Barangay Duljo in the name of James Tan Suarez,
Carmencita’s brother, who had no real rights or interests over the
demand.
subject lot. Further, while Carmencita based her claim over the
Carmencita filed before the MTCC and against the respondents a subject lot by virtue of a deed of sale executed on April 1, 2004, no
complaint for unlawful detainer, the origin of the instant petition. She demand to vacate was made upon the respondents after that date.
alleged that she bought the subject lot from Remedios, Moreno, The absence of such demand rendered the complaint fatally
Veronica and Dionesia, the registered owners thereof and the defective, as the date of its service should be the reckoning point of
persons who allowed the respondents to occupy the same by mere the one–year period within which the suit can be filed.
tolerance.
The Ruling of the CA. Reversing the disquisitions of the courts a quo
The MTCC upheld Carmencita’s claims in its decision rendered. On and dismissing Carmencita’s complaint for unlawful detainer.
appeal, the RTC affirmed in its entirety the MTCC ruling.
The distinction between forcible entry and unlawful detainer was
The respondents challenged the MTCC and RTC judgments through lucidly explained in Sarmiento vs. Court of Appeals,:
a Petition for Review filed before the CA.
Forcible entry and unlawful detainer cases are two distinct actions
The respondents argued that her (Carmencita) complaint for unlawful defined in Section 1, Rule 70 of the Rules of Court. [In] forcible
detainer was filed on December 8, 2004 subsequent to the entry, one is deprived of physical possession of land or building by
respondents’ institution on August 13, 2004 of a petition for means of force, intimidation, threat, strategy, or stealth. In unlawful
nullification of the partition. detainer, one unlawfully withholds possession thereof after the
expiration or termination of his right to hold possession under any
Citing Sarmiento v. CA, the respondents emphasized that “even if contract, express or implied. In forcible entry, the possession is
one is the owner of the property, the possession thereof cannot be illegal from the beginning and the basic inquiry centers on who has

170 Aicka Singson


Notes in Remedial Law Review II
Provisional Remedies, Special Civil Actions, Special Proceedings and Evidence
School Year 2015-2016 REMEDIAL LAW REVIEW II - ATTY. BRONDIAL

the prior possession de facto. In unlawful detainer, the possession complaint fails to aver facts constitutive of forcible entry or unlawful
was originally lawful but became unlawful by the expiration or detainer, as where it does not state how entry was effected or how
termination of the right to possess, hence the issue of rightful and when dispossession started, as in the case at bar, the remedy
possession is decisive for, in such action, the defendant is in actual should either be an accion publiciana or an accion reivindicatoria in
possession and the plaintiffs cause of action is the termination of the the proper RTC.
defendant’s right to continue in possession.
Munoz vs. Court of Appeals enunciated:
What determines the cause of action is the nature of defendant’s
entry into the land. If the entry is illegal, then the action which may It must be stated that regardless of actual condition of the title to the
be filed against the intruder within one (1) year therefrom is forcible property, the party in peaceable quiet possession shall not be turned
entry. If, on the other hand, the entry is legal but the possession out by a strong hand, violence or terror. Thus, a party who can
thereafter became illegal, the case is one of unlawful detainer which prove prior possession can recover such possession even against
must be filed within one (1) year from the date of the last demand. the owner himself. Whatever may be the character of his prior
possession, if he has in his favor priority in time, he has the security
A close perusal of Carmencita’s complaint a quo reveals that the that entitles him to remain on the property until he is lawfully ejected
action was neither one of forcible entry nor unlawful detainer but by a person having a better right by accion publiciana or accion
essentially involved an issue of ownership which must be resolved in reivindicatoria.
an accion reivindicatoria. It did not characterize the respondents’
alleged entry into the land: whether the same was legal or illegal. It CA denied Carmencita’s Motion for Reconsideration.
did not state how the respondents entered the land and constructed
Whether or not Carmencita’s complaint against the respondents
a house thereon. It was also silent on whether the respondents’
had sufficiently alleged and proven a cause of action for
possession became legal before Carmencita demanded from them
unlawful detainer NO!
to vacate the land. The complaint merely averred that their relatives
previously owned the lot the respondents were occupying and that “Without a doubt, the registered owner of real property is entitled to
after Carmencita purchased it, she, as its new owner, demanded for its possession. However, the owner cannot simply wrest possession
the respondents to vacate the land. Moreover, it is undisputed that thereof from whoever is in actual occupation of the property. To
the respondents and their ancestors have been occupying the land recover possession, he must resort to the proper judicial remedy
for several decades already. There was no averment as to how or and, once he chooses what action to file, he is required to satisfy the
when Carmencita’s predecessors tolerated the respondents’ conditions necessary for such action to prosper.”
possession of the land. Consequently, there was no contract to
speak of, whether express or implied, between the respondents, on In Spouses Valdez, Jr., the Court is instructive anent the three kinds
one hand, and Carmencita or her predecessors, on the other, as of actions available to recover possession of real property, viz:
would qualify [the respondents’] possession of the land as a case of
unlawful detainer. Neither was it alleged that the respondents took (a) accion interdictal; (b) accion publiciana; and (c) accion
possession of the land through force, intimidation, threat, strategy or reivindicatoria.
stealth to make out a case of forcible entry. Indeed, when the

171 Aicka Singson


Notes in Remedial Law Review II
Provisional Remedies, Special Civil Actions, Special Proceedings and Evidence
School Year 2015-2016 REMEDIAL LAW REVIEW II - ATTY. BRONDIAL

Accion interdictal comprises two distinct causes of action, namely, (1) initially, possession of property by the defendant was by contract
forcible entry (detentacion) and unlawful detainer (desahuico) [sic]. with or by tolerance of the plaintiff;
In forcible entry, one is deprived of physical possession of real
property by means of force, intimidation, strategy, threats, or stealth (2) eventually, such possession became illegal upon notice by
whereas in unlawful detainer, one illegally withholds possession after plaintiff to defendant of the termination of the latter’s right of
the expiration or termination of his right to hold possession under any possession;
contract, express or implied. The two are distinguished from each
(3) thereafter, the defendant remained in possession of the property
other in that in forcible entry, the possession of the defendant is
and deprived the plaintiff of the enjoyment thereof; and
illegal from the beginning, and that the issue is which party has prior
de facto possession while in unlawful detainer, possession of the (4) within one year from the last demand on defendant to vacate the
defendant is originally legal but became illegal due to the expiration property, the plaintiff instituted the complaint for ejectment.
or termination of the right to possess.
In the case at bar, the first requisite mentioned above is markedly
The jurisdiction of these two actions, which are summary in nature, absent. Carmencita failed to clearly allege and prove how and when
lies in the proper municipal trial court or metropolitan trial court. Both the respondents entered the subject lot and constructed a house
actions must be brought within one year from the date of actual entry upon it. Carmencita was likewise conspicuously silent about the
on the land, in case of forcible entry, and from the date of last details on who specifically permitted the respondents to occupy the
demand, in case of unlawful detainer. The issue in said cases is the lot, and how and when such tolerance came about. Instead,
right to physical possession. Carmencita cavalierly formulated a legal conclusion, sans factual
substantiation, that (a) the respondents’ initial occupation of the
Accion publiciana is the plenary action to recover the right of
subject lot was lawful by virtue of tolerance by the registered owners,
possession which should be brought in the proper regional trial court
and (b) the respondents became deforciants unlawfully withholding
when dispossession has lasted for more than one year. It is an
the subject lot’s possession after Carmencita, as purchaser and new
ordinary civil proceeding to determine the better right of possession
registered owner, had demanded for the former to vacate the
of realty independently of title. In other words, if at the time of the
property. It is worth noting that the absence of the first requisite
filing of the complaint more than one year had elapsed since
assumes even more importance in the light of the respondents’ claim
defendant had turned plaintiff out of possession or defendant’s
that for decades, they have been occupying the subject lot as
possession had become illegal, the action will be, not one of the
owners thereof.
forcible entry or illegal detainer, but an accion publiciana. On the
other hand, accion reivindicatoria is an action to recover ownership The jurisdictional facts must appear on the face of the complaint.
also brought in the proper regional trial court in an ordinary civil When the complaint fails to aver facts constitutive of forcible entry or
proceeding. unlawful detainer, as where it does not state how entry was effected
or how and when dispossession started, the remedy should either be
In a complaint for unlawful detainer, the following key
an accion publiciana or accion reivindicatoria.
jurisdictional facts must be alleged and sufficiently established:

172 Aicka Singson


Notes in Remedial Law Review II
Provisional Remedies, Special Civil Actions, Special Proceedings and Evidence
School Year 2015-2016 REMEDIAL LAW REVIEW II - ATTY. BRONDIAL

b. Alconera vs. Pallanan (714 SCRA 204, 1/2014) Supersedeas Bond filed with it. Instead, what was filed not with the
RTC but with the MTCC was a "NOTICE OF APPEAL – and –
Facts: FACTS: Complainant was the counsel for Morito Rafols, the MOTION TO APPROVE PROPERTY SUPERSEDEAS BOND,"
defendant in Civil Case No. 5967-2, an unlawful detainer case which was not granted.
entitled Cua Beng a.k.a. Manuel Sy and Ka Kieng v. Morita Rafols, et
al., filed before the MTCC, Branch 2 in General Santos City, South ISSUE: Whether there was compliance with Sec.19, Rule 70?
Cotabato. After trial, the MTCC ruled against Rafols and his co-
defendants in a Judgment. HELD: NONE. It must be borne in mind that the case at bar traces its
roots to an unlawful detainer case wherein the MTCC ruled against
Therefrom, Rafols, through complainant Alconera, appealed the case Rafols, complainant’s client. In ejectment cases, the rulings of the
to the RTC, Branch 36. Pending appeal, the court issued an Order courts are immediately executory and can only be stayed via
granting Cua Beng’s motion for execution she filed in Civil Case No. compliance with Section 19, Rule 70 of the Rules of Court.
5967-2, the unlawful detainer case. Alconera sought reconsideration
– DENIED. Clearly then under said Sec. 19, Rule 70, a judgment on a forcible
entry and detainer action is made immediately executory to avoid
Evelyn Rafols, Rafols’ daughter-in-law, called up Alconera to report further injustice to a lawful possessor. The defendant in such a case
that the sheriff, respondent Pallanan, was about to implement the may have such judgment stayed only by (a) perfecting an appeal; (b)
adverted writ of execution. After explaining the matter to Alconera, filing a supersedeas bond; and (c) making a periodic deposit of the
Evelyn Rafols passed her phone to respondent sheriff. Over the rental or reasonable compensation for the use and occupancy of the
phone, a verbal disagreement between the two ensued. Alconera property during the pendency of the appeal. The failure of the
claims that he has a pending motion for reconsideration on the defendant to comply with any of these conditions is a ground for the
issuance of the writ of execution, but the respondent said that the outright execution of the judgment, the duty of the court in this
motion has already been denied. And since no TRO has been issued respect being ministerial and imperative. Hence, if the defendant-
enjoining the implementation, respondent claimed that he is legally appellant has perfected the appeal but failed to file a supersedeas
mandated to perform his ministerial duty of enforcing the writ. bond, the immediate execution of the judgment would automatically
follow. Conversely, the filing of a supersedeas bond will not stay the
Complainant countered that he has not yet received a copy of the execution of the judgment if the appeal is not perfected. Necessarily
denial of the motion, rendering the execution premature and, at the then, the supersedeas bond should be filed within the period for the
same time, preventing him from securing a TRO from the higher perfection of the appeal.
courts. Nevertheless, respondent still pushed through with the
execution of the judgment. In the case at bar, complainant lost his client’s case and appealed to
the RTC. His client has also been periodically depositing rental with
On March 18, 2011, complainant returned to General Santos City the court for the use of the property pending appeal. However, as
and, at his law office, found a copy of the Order denying his Motion ruled by the RTC, the bond filed did not meet the legal requirements
for Reconsideration, which was only served that very same day. The because first and foremost, the bond posted was a property bond,
RTC ruled that there was no pending Motion to Approve not cash nor surety. Furthermore, Rafols did not own the property he

173 Aicka Singson


Notes in Remedial Law Review II
Provisional Remedies, Special Civil Actions, Special Proceedings and Evidence
School Year 2015-2016 REMEDIAL LAW REVIEW II - ATTY. BRONDIAL

posted as bond and besides, it was also not issued in favour of the possession; Petra, Mariano, Teodoro Teodoro’s (petitioners’) father;
plaintiff in the ejectment case. Because of the non-compliance with and Ana. Genaro and his children are all deceased.
the requirements under the above-quoted rule, the execution of the
judgment was not effectively stayed. The only exceptions to non- Of all Genaro’s children, only Petra occupied the subject property,
compliance are the existence of fraud, accident, mistake or living at the ancestral house. After Petra’s death, her purported will, a
excusable negligence which prevented the defendant from posting holographic will, was probated in Special Proceeding, which Decision
the supersedeas bond or making the monthly deposit, or the on the will’s extrinsic validity has become final and executory. In the
occurrence of supervening events which brought about a material will, Petra, asserting ownership, devised the subject property to
change in the situation of the parties and which would make the Teodoro Teodoro.
execution inequitable. But whether or not these obtain in the case at
Teodoro Teodoro effected the demolition of the ancestral house,
bar is an issue best left to the court that issued the writ of execution.
intending to use the subject property for other purposes.
Given the above circumstances, there was no legal impediment
Respondents erected a fence on the surrounding portion, barricaded
preventing respondent sheriff from performing his responsibility of
its frontage, and put up a sign thereat, effectively dispossessing
enforcing the writ of execution. Since Rafols failed to comply with the
Teodoro.
requirements under the Rules, Cua Beng who prevailed in the
unlawful detainer case is entitled as a matter of right to the After Teodoro Teodoro’s demand for respondents to vacate the
immediate execution of the court’s judgment both as to the subject property went unheeded, he filed the complaint for forcible
restoration of possession and the payment of the accrued rentals or entry against respondents.
compensation for the use and occupation of the premises.
In their Answer, respondents asserted their own ownership and
c. Teodoro vs. Espino (715 SCRA 435, 2/20/2014) possession of the subject property.
Facts: We exercise judicial restraint: we simply delineate the After trial, the MTC dismissed the complaint, ruling on the issue of
possessory rights of the warring parties and refrain from ruling on ownership and ultimately resolving the issue of who between
these squabbling heirs’ respective claims of ownership. Teodoro and respondents had a better right to possess the subject
property:
The subject property is a portion registered in the name of Genaro,
long deceased ascendant of all the parties. The subject property The RTC, in its appellate jurisdiction over forcible entry cases, acting
pertains to the vacant lot where the old ancestral house of Genaro on Teodoro Teodoro’s appeal, adopted the factual findings of the
stood until its demolition in June 2004, at the instance of Teodoro MTC, but reversed the ruling, ruled in favor of Teodoro Teodoro and
Teodoro. ordered the ejectment of respondents from the subject property. It
pithily ruled, thus:
Genaro had five children: Santiago; Maria, from whom respondents
descended and trace their claim of ownership and right of But the bottom line for resolution in this case is who has the prior
physical possession of the subject parcel.

174 Aicka Singson


Notes in Remedial Law Review II
Provisional Remedies, Special Civil Actions, Special Proceedings and Evidence
School Year 2015-2016 REMEDIAL LAW REVIEW II - ATTY. BRONDIAL

Analyzing the facts of the case, the lower [court] concluded that the No. The ground rules in forcible entry cases:
subject parcel is a part of the estate of the late Genaro Teodoro and
in the absence of an approved partition among the heirs, remains a (1) One employs force, intimidation, threat, strategy or stealth to
community property over which the legal heirs of Genaro Teodoro deprive another of physical possession of real property.
have the right to inherit. All therefore are entitled to exercise the right
(2) Plaintiff (Teodoro Teodoro) must allege and prove prior physical
of dominion including the right of possession.
possession of the property in litigation until deprived thereof by the
This Court disagrees with the said ruling applying the plethora of defendant (herein respondents). This requirement implies that the
cases decisive of the issue and consistent with the established possession of the disputed land by the latter was unlawful from the
jurisprudence that the lower court cannot dispose with finality the beginning.
issue of ownership–such issue being inutile in an ejectment suit
(3) The sole question for resolution hinges on the physical or
except to throw light on the question of possession.
material possession (possession de facto) of the property. Neither a
Given the foregoing, Teodoro Teodoro has established a valid claim claim of juridical possession (possession de jure) nor an averment of
to institute the eviction suit against [respondents] over the disputed ownership by the defendant can, at the outset, preclude the court
area. from taking cognizance of the case.

With the reversal of the MTC’s ruling, respondents then appealed the (4) Ejectment cases proceed independently of any claim of
RTC’s decision to the Court of Appeals. The appellate court reversed ownership, and the plaintiff needs merely to prove prior possession
the RTC, likewise dismissed the complaint as the MTC had done, but de facto and undue deprivation thereof.
did not reach the same result as that of the inferior court.
In this case, both parties assert prior and exclusive physical
In all, the appellate court found that Teodoro Teodoro (substituted by possession in the concept of owner acquired through succession
his heirs Nelson and Rolando Teodoro at that juncture) “failed to from the same decedent, their aunt and grand aunt, respectively,
discharge the burden of proof that he had prior actual physical Petra. In turn, Petra inherited the property from her father Genaro, in
possession of the subject [property] before it was barricaded by whose name the subject property is still registered.
[respondents] to warrant the institution of the forcible entry suit.”
Teodoro Teodoro’s assertion of physical possession comprises
Hence, this appeal by certiorari filed by the heirs of Teodoro mainly of his claimed ownership of the subject property acquired
Teodoro. through testate succession, or via the holographic will of Petra.
Teodoro Teodoro then points, as an exercise of his ownership and
Whether or not the act of respondents in barricading the incident of his physical possession of the subject property, to his act
frontage of the portion of the Lot on which stood the ancestral of demolition of the ancestral house.
house occupied by Petra amounted to Teodoro Teodoro’s
unlawful dispossession thereof through the forcible entry of On the other hand, respondents assert possession likewise by virtue
respondents of ownership manifested in their residence spanning more than five
(5) decades, reckoned even from the time Maria, respondents’

175 Aicka Singson


Notes in Remedial Law Review II
Provisional Remedies, Special Civil Actions, Special Proceedings and Evidence
School Year 2015-2016 REMEDIAL LAW REVIEW II - ATTY. BRONDIAL

grandmother and sister of Petra, was alive and resided thereat. and two appellate reviews of the forcible entry case filed by Teodoro
Respondents trace their possession from the extrajudicial partition of Teodoro. The RTC said:
the commingled properties of the siblings Maria, respondents’ direct
ascendant, Petra and Mariano, father of Teodoro Teodoro, progeny Analyzing the facts of the case, the lower [court] concluded that the
and heirs of Genaro. According to respondents, from the partition, subject parcel is a part of the estate of the late Genaro Teodoro and
the heirs of all three Genaro children possessed and occupied their in the absence of an approved partition among the heirs, remains a
respective shares: respondents received Lot No. 2476 which community property over which the legal heirs of Genaro Teodoro
encompasses herein subject property, while Teodoro Teodoro and have the right to inherit. All therefore are entitled to exercise the right
his siblings received a different property. of dominion including the right of possession.

Given both parties respective claims of ownership over the subject d. Ferrer vs. Rabaca (632 SCRA )
property via succession from their ascendants Maria, Petra and
FACTS: The complainants were the President and the Executive
Mariano Teodoro, who are all compulsory heirs of Genaro in whose
Director of the plaintiff in an ejectment suit entitled Young Women’s
name the subject property is still registered, the MTC ruled that
Christian Association, Inc. v. Conrado Cano. After trial, Civil Case
respondents cannot be disturbed in their possession of the subject
No. 176394-CV was decided on June 22, 2004 by respondent
property “until and unless the question of ownership over the same
Judge, who Rendered judment in favor of the plaintiff and against the
[is] finally resolved before the appropriate court.”
defendant ordering the latter (a) to vacate the premises and
In contrast, the RTC, without categorically resolving the issue of surrender possession thereof to plaintiff; (b) to pay plaintiff the sum
ownership of Lot No. 2476, ruled that on the portion of Lot No. 2476 of Php45,211.80 representing his arrears in rentals.
where the ancestral house used to stand, Teodoro did establish his
On July 12, 2004, the plaintiff’s counsel filed a motion for immediate
prior physical possession over the subject property resulting in his
execution, praying that a writ of execution be issued "for the
right to institute the ejectment suit against respondents. Significantly,
immediate execution of the aforesaid Judgment." The plaintiff cited
the RTC confirmed respondents’ physical possession of, and
Section 19, Rule 70 of the Rules of Court as basis for its motion.
residency at, Lot No. 2476.
However, respondent Judge denied the motion for immediate
There would yet be another turn of events. The appellate court, albeit
execution.
refusing to touch and rule on the issue of ownership, declared that
there lacked conclusive evidence of Teodoro Teodoro’s prior actual According to the complainants, their counsel talked with respondent
physical possession over the subject property. Judge about the matter. Allegedly, respondent Judge told their
counsel that "if you think the court is wrong, file a motion for
We reverse the decision of the Court of Appeals and restore the
reconsideration." With that, the plaintiff filed a motion for
decision of the RTC on the appeal reversing the MTC.
reconsideration, which respondent Judge nonetheless denied and
We affirm the finding of fact by the RTC which is decisive of the directed the Branch Clerk of Court to immediately forward the
issue that has remained unresolved inspite of a summary procedure records of the case to the Regional Trial Court, Manila.

176 Aicka Singson


Notes in Remedial Law Review II
Provisional Remedies, Special Civil Actions, Special Proceedings and Evidence
School Year 2015-2016 REMEDIAL LAW REVIEW II - ATTY. BRONDIAL

The complainants pointed out that respondent Judge apparently did time during the pendency of the appeal. Otherwise, execution
not know that appeal in forcible entry and detainer cases was not becomes ministerial and imperative.
perfected by the mere filing of a notice of appeal (as in ordinary
actions) but by filing of a notice of appeal and a sufficient In the case at bar, defendant seasonably filed his Notice of Appeal
supersedeas bond approved by the trial judge executed to the dated 9 July 2004 on 13 July 2004; he however failed to file any
plaintiff to pay the rents, damages and costs accruing down to the supersedeas bond. Prior to the filing of such notice of appeal, more
time of the judgment appealed from. They asserted that respondent specifically on 12 July 2004, complainants have already filed their
Judge’s invocation of good faith and error of judgment did not Motion for Execution dated 8 July 2004. Instead of acting on the
absolve him of liability, because he had grossly neglected his duties Motion for Execution, respondent Judge Rabaca gave due course to
mandated by law by failing and refusing to act on their motion for the appeal in an Order dated 14 July 2004 and directed his Branch
immediate execution and motion for reconsideration and by giving Clerk of Court to elevate the records of the case to the Regional Trial
due course to the appeal despite no supersedeas bond having been Court (RTC). The Branch Clerk of Court however failed to forward
filed and approved by the trial court. the records to the RTC. This fact is clear from Judge Rabaca’s Order
dated 28 July 2004 wherein he directed the Branch Clerk of Court to
ISSUE: WON complainants’ contention was correct forward the records of the case to the Manila Regional Trial Court
immediately.
HELD: YES. Section 19, Rule 70 of the 1997 Revised Rules on Civil
Procedure provides: From the foregoing, it is clear that when the complainant moved for
the immediate execution of Judge Rabaca’s decision, the latter still
"SEC. 19. If judgment is rendered against the defendant, execution had jurisdiction over the case. He therefore clearly erred when he
shall issue immediately upon motion, unless an appeal has been refused to act on the Motion for Execution.
perfected and the defendant to stay execution files a supersedeas
bond, approved by the Municipal Trial Court and executed in favor of This case is an opportune occasion to remind judges of the first level
the plaintiff to pay the rents, damages, and costs accruing down to courts to adhere always to the mandate under Section 19, Rule 70,
the time of the judgment appealed from, and unless, during the of the Rules of Court to issue writs of execution upon motion of the
pendency of the appeal, he deposits with the appellate court the plaintiffs in actions for forcible entry or unlawful detainer when the
amount of rent due from time to time under the contract, if any, as defendant has appealed but has not filed a sufficient supersedeas
determined by the judgment of the Municipal Trial Court." bond. The summary nature of the special civil action under Rule 70
and the purpose underlying the mandate for an immediate execution,
It is clear from the foregoing that the perfection of an appeal by itself which is to prevent the plaintiffs from being further deprived of their
is not sufficient to stay the execution of the judgment in an ejectment rightful possession, should always be borne in mind.
case. The losing party should likewise file a supersedeas bond
executed in favor of the plaintiff to answer for rents, damages and e. CGR Corp vs. Treyes (522 SCRA 765)
costs, and, if the judgment of the court requires it, he should likewise
deposit the amount of the rent before the appellate court from the

177 Aicka Singson


Notes in Remedial Law Review II
Provisional Remedies, Special Civil Actions, Special Proceedings and Evidence
School Year 2015-2016 REMEDIAL LAW REVIEW II - ATTY. BRONDIAL

Whether a complainant in a forcible entry case can file an independently institute and maintain an action for damages
independent action for damages arising after the act of which they claim arose from incidents occurring after the
dispossession had occurred. YES dispossession by respondent of the premises.

CGR Corporation, Herman M. Benedicto and Alberto R. Benedicto YES. The petition is impressed with merit.
(petitioners) claimed to have occupied 37.3033 hectares of public
land in Negros Occidental even before the notarized separate Section 17, Rule 70 of the Rules of Court provides:
Fishpond Lease Agreement in their respective favor were approved
SEC. 17. Judgment. – If after trial the court finds that the allegations
in October 2000 by the Secretary of Agriculture for a period of
of the complaint are true, it shall render judgment in favor of the
twenty-five (25) years or until December 31, 2024.
plaintiff for the restitution of the premises, the sum justly due as
Treyes, Jr. (respondent) allegedly forcibly and unlawfully entered the arrears of rent or as reasonable compensation for the use and
leased properties and once inside barricaded the entrance to the occupation of the premises, attorney’s fees and costs. If it finds that
fishponds, set up a barbed wire fence along the road going to said allegations are not true, it shall render judgment for the
petitioners’ fishponds, and harvested several tons of milkfish, fry and defendant to recover his costs. If a counterclaim is established, the
fingerlings owned by petitioners. court shall render judgment for the sum found in arrears from either
party and award costs as justice requires.
Petitioners promptly filed with the Municipal Trial Court (MTC)
separate complaints for Forcible Entry With Temporary Restraining The recoverable damages in forcible entry and detainer cases thus
Order And/Or Preliminary Injunction And Damages against Ernesto refer to "rents" or "the reasonable compensation for the use and
M. Treyes, Sr. and respondent. occupation of the premises" or "fair rental value of the property" and
attorney’s fees and costs.
Petitioners filed in with the Bacolod RTC a complaint for damages
against respondent, docketed as Civil Case No, 04-12284, alleging, The 2006 case of Dumo v. Espinas reiterates the long-established
inter alia, rule that the only form of damages that may be recovered in an
action for forcible entry is the fair rental value or the reasonable
Respondent filed a Motion to Dismiss petitioners’ complaint for compensation for the use and occupation of the property.
damages on three grounds – litis pendentia, res judicata and forum
shopping. It bears noting, however, that as reflected in the earlier-quoted
allegations in the complaint for damages of herein petitioners, their
Bacolod RTC dismissed petitioners’ complaint on the ground of claim for damages have no direct relation to their loss of possession
prematurity, it holding that a complaint for damages may only be of the premises. It had to do with respondent’s alleged harvesting
maintained "after a final determination on the forcible entry cases and carting away several tons of milkfish and other marine products
has been made." in their fishponds, ransacking and destroying of a chapel built by
petitioner CGR Corporation, and stealing religious iconsand even
The only issue is whether, during the pendency of their
separate complaints for forcible entry, petitioners can

178 Aicka Singson


Notes in Remedial Law Review II
Provisional Remedies, Special Civil Actions, Special Proceedings and Evidence
School Year 2015-2016 REMEDIAL LAW REVIEW II - ATTY. BRONDIAL

decapitating the heads of some of them, after the act of A. Contempt (Rule 71)
dispossession had occurred.
1. Kinds; nature; grounds
Surely, one of the elements of litis pendentia - that the identity
between the pending actions, with respect to the parties, rights There are 2 kinds of contempt: direct and indirect contempt.
asserted and reliefs prayed for, is such that any judgment rendered
Direct Contempt Indirect Contempt
on one action will, regardless of which is successful, amount to res
judicata in the action under consideration - is not present, hence, it Performed or acted within the
may not be invoked to dismiss petitioners’ complaint for damages. vicinity of the court. It is a show of
disrespect to the tribunal or court.
Res judicata may not apply because the court in a forcible entry case
has no jurisdiction over claims for damages other than the use and There is no proceeding There should be notice and
occupation of the premises and attorney’s fees. hearing. This cannot be done
motu proprio
Neither may forum-shopping justify a dismissal of the complaint for
damages, the elements of litis pendentia not being present, or where Wide latitude on the part The rule provides for the grounds
a final judgment in the forcible entry case will not amount to res of the court as to when it should
judicata in the former. exercise the power of direct
contempt.
Petitioners’ filing of an independent action for damages other than
those sustained as a result of their dispossession or those caused by Section 1. Direct contempt punished summarily. — A person guilty of
the loss of their use and occupation of their properties could not thus misbehavior in the presence of or so near a court as to obstruct or
be considered as splitting of a cause of action. interrupt the proceedings before the same, including disrespect
toward the court, offensive personalities toward others, or refusal to
be sworn or to answer as a witness, or to subscribe an affidavit or
deposition when lawfully required to do so, may be summarily
adjudged in contempt by such court and punished by a fine not
exceeding two thousand pesos or imprisonment not exceeding ten
(10) days, or both, if it be a Regional Trial Court or a court of
equivalent or higher rank, or by a fine not exceeding two hundred
pesos or imprisonment not exceeding one (1) day, or both, if it be a
lower court.

Section 2. Remedy therefrom. — The person adjudged in direct


contempt by any court may not appeal therefrom, but may avail
himself of the remedies of certiorari or prohibition. The execution of
the judgment shall be suspended pending resolution of such petition,

179 Aicka Singson


Notes in Remedial Law Review II
Provisional Remedies, Special Civil Actions, Special Proceedings and Evidence
School Year 2015-2016 REMEDIAL LAW REVIEW II - ATTY. BRONDIAL

provided such person files a bond fixed by the court which rendered (g) The rescue, or attempted rescue, of a person or property in the
the judgment and conditioned that he will abide by and perform the custody of an officer by virtue of an order or process of a court held
judgment should the petition be decided against him. by him.

Section 3. Indirect contempt to be punished after charge and But nothing in this section shall be so construed as to prevent the
hearing. — After a charge in writing has been filed, and an court from issuing process to bring the respondent into court, or from
opportunity given to the respondent to comment thereon within such holding him in custody pending such proceedings.
period as may be fixed by the court and to be heard by himself or
counsel, a person guilty of any of the following acts may be punished 2. Procedure
for indirect contempt;
Penalty for contempt may be imprisonment or a fine, or both. The
(a) Misbehavior of an officer of a court in the performance of his fine imposable before the RTC is much higher than in the MTC. The
official duties or in his official transactions; imprisonment too (up to 6 months maximum in RTC but only up to 30
days in MTC).
(b) Disobedience of or resistance to a lawful writ, process, order, or
judgment of a court, including the act of a person who, after being Section 4. How proceedings commenced. — Proceedings for indirect
dispossessed or ejected from any real property by the judgment or contempt may be initiated motu propio by the court against which the
process of any court of competent jurisdiction, enters or attempts or contempt was committed by an order or any other formal charge
induces another to enter into or upon such real property, for the requiring the respondent to show cause why he should not be
purpose of executing acts of ownership or possession, or in any punished for contempt.
manner disturbs the possession given to the person adjudged to be
In all other cases, charges for indirect contempt shall be commenced
entitled thereto;
by a verified petition with supporting particulars and certified true
(c) Any abuse of or any unlawful interference with the processes or copies of documents or papers involved therein, and upon full
proceedings of a court not constituting direct contempt under section compliance with the requirements for filing initiatory pleadings for
1 of this Rule; civil actions in the court concerned. If the contempt charges arose
out of or are related to a principal action pending in the court, the
(d) Any improper conduct tending, directly or indirectly, to impede, petition for contempt shall allege that fact but said petition shall be
obstruct, or degrade the administration of justice; docketed, heard and decided separately, unless the court in its
discretion orders the consolidation of the contempt charge and the
(e) Assuming to be an attorney or an officer of a court, and acting as principal action for joint hearing and decision.
such without authority;
Section 5. Where charge to be filed. — Where the charge for indirect
(f) Failure to obey a subpoena duly served; contempt has been committed against a Regional Trial Court or a
court of equivalent or higher rank, or against an officer appointed by
it, the charge may be filed with such court. Where such contempt has

180 Aicka Singson


Notes in Remedial Law Review II
Provisional Remedies, Special Civil Actions, Special Proceedings and Evidence
School Year 2015-2016 REMEDIAL LAW REVIEW II - ATTY. BRONDIAL

been committed against a lower court, the charge may be filed with Section 9. Proceeding when party released on bail fails to answer. —
the Regional Trial Court of the place in which the lower court is When a respondent released on bail fails to appear on the day fixed
sitting; but the proceedings may also be instituted in such lower court for the hearing, the court may issue another order of arrest or may
subject to appeal to the Regional Trial Court of such place in the order the bond for his appearance to be forfeited and confiscated, or
same manner as provided in section 11 of this Rule. both; and, if the bond be proceeded against, the measure of
damages shall be the extent of the loss or injury sustained by the
Section 6. Hearing; release on bail. — If the hearing is not ordered to aggrieved party by reason of the misconduct for which the contempt
be had forthwith, the respondent may be released from custody upon charge was prosecuted, with the costs of the proceedings, and such
filing a bond, in an amount fixed by the court, for his appearance at recovery shall be for the benefit of the party injured. If there is no
the hearing of the charge. On the day set therefor, the court shall aggrieved party, the bond shall be liable and disposed of as in
proceed to investigate the charge and consider such comment, criminal cases.
testimony or defense as the respondent may make or offer.
Section 10. Court may release respondent. — The court which
Section 7. Punishment for indirect contempt. — If the respondent is issued the order imprisoning a person for contempt may discharge
adjudged guilty of indirect contempt committed against a Regional him from imprisonment when it appears that public interest will not
Trial Court or a court of equivalent or higher rank, he may be be prejudiced by his release.
punished by a fine not exceeding thirty thousand pesos or
imprisonment not exceeding six (6) months, or both. If he is 3. Judgment and Review
adjudged guilty of contempt committed against a lower court, he may
be punished by a fine not exceeding five thousand pesos or In case of direct contempt, there is no appeal. Remedy is certiorari
imprisonment not exceeding one (1) month, or both. If the contempt but you have to post a bond. This is the only kind of certiorari where
consists in the violation of a writ of injunction, temporary restraining you have to post a bond. In the case of indirect contempt, because
order or status quo order, he may also be ordered to make complete there is notice and hearing, there is an appeal. Appeal it to the higher
restitution to the party injured by such violation of the property court (if MTC, go to RTC; if RTC, go to CA).
involved or such amount as may be alleged and proved.
Section 11. Review of judgment or final order; bond for stay. — The
The writ of execution, as in ordinary civil actions, shall issue for the judgment or final order of a court in a case of indirect contempt may
enforcement of a judgment imposing a fine unless the court be appealed to the proper court as in criminal cases. But execution
otherwise provides. of the judgment or final order shall not be suspended until a bond is
filed by the person adjudged in contempt, in an amount fixed by the
Section 8. Imprisonment until order obeyed. — When the contempt court from which the appeal is taken, conditioned that if the appeal
consists in the refusal or omission to do an act which is yet in the be decided against him he will abide by and perform the judgment or
power of the respondent to perform, he may be imprisoned by order final order.
of the court concerned until he performs it.
Section 12. Contempt against quasi-judicial entities. — Unless
otherwise provided by law, this Rule shall apply to contempt

181 Aicka Singson


Notes in Remedial Law Review II
Provisional Remedies, Special Civil Actions, Special Proceedings and Evidence
School Year 2015-2016 REMEDIAL LAW REVIEW II - ATTY. BRONDIAL

committed against persons, entities, bodies or agencies exercising CASES:


quasi-judicial functions, or shall have suppletory effect to such rules
as they may have adopted pursuant to authority granted to them by a. SEC CHAIRMAN PERFECTO R. YASAY, JR., vs. MANUEL D.
law to punish for contempt. The Regional Trial Court of the place RECTO, PELAGIO T. RICALDE and CESAR P. MANALAYSAY.R.
wherein the contempt has been committed shall have jurisdiction G No. 129521 September 7, 1999
over such charges as may be filed therefor.
FACTS: On June 28, 1996, SEC Chairman Yasay upon request of
Yasay vs Recto: Interport Resource Development Corporation certain stockholders of Interport Resources Corporation. On the
allegedly continued to meet and elect their board of officers when same date, the SEC issued a TRO enjoining the Interport Resources
there was an injunctive on the part of Perfecto Yasay, the Chairman Corporation from holding the July 9, 1996 scheduled annual meeting
of SEC. He was chastised by the SC because he did not know that of the stockholders. Notwithstanding the SEC's TRO, the
the injunction that he issued was also enjoined by the CA. SC got stockholders proceeded with the meeting on July 9, 1996, presided
mad at Yasay because he suspended the lawyer from appearing over by respondent Manalaysay.
before the SEC for 6 months. Only the SC can suspend lawyers and
On July 10, 1996, the SEC declared the stockholders meeting of
they even meet en banc.
Interport Resources Corporation held on July 9, 1996, null and void
Espanol vs Formoso: During the presentation of evidence before and directed respondents to appear before the SEC to show cause
Judge Espanol, Atty. Formoso was presenting a document to which why no disciplinary action should be taken against them or why they
Judge Espanol said was fraudulent. Judge Espanol said the should not be cited in contempt.
document has been pronounced a spurious document in another
At the hearing on July 15, 1996, respondent Manalaysay questioned
sala. This court takes judicial notice of this case and that document
the validity of the TRO as well as the contempt proceedings in light
is fraudulent. They had an argument. Atty. Formoso insisted on
of the TRO issued by the CA restraining the SEC from enforcing its
presenting the document. He was cited in direct contempt. He was
TRO. After the hearing, on July 15, 1996, the SEC issued an order
sent to jail. SC: The power of contempt must be exercised
declaring Atty. Cesar Manalaysay, Manuel D. Recto and Atty.
cautiously, judiciously, not in the vindictive but rather in the
Pelagio T. Ricalde GUILTY OF CONTEMPT
preservative way.
In due time, respondents appealed from the aforesaid order to the
Sison vs Caoibes, Jr.: You must exercise the power of contempt
Court of Appeals.
cautiously, judiciously, not in the vindictive but rather in the
preservative way. After due proceedings, the CA promulgated its decision reversing
and setting aside the SEC order declaring respondents guilty of
Montenegro vs Montenegro: The party was cited in contempt
contempt.
because he’s always late or absent.
Petitioners filed a MR – DENIED. Hence, this appeal.

182 Aicka Singson


Notes in Remedial Law Review II
Provisional Remedies, Special Civil Actions, Special Proceedings and Evidence
School Year 2015-2016 REMEDIAL LAW REVIEW II - ATTY. BRONDIAL

In the main, respondents submit that contempt is criminal in prosecution. The distinction is for the purpose only of determining the
character and their exoneration from a charge of contempt amounts character of punishment to be administered."
to an acquittal from which an appeal would not lie.
In this case, the contempt is not civil in nature, but criminal, imposed
ISSUE: WON the CA erred in setting aside the order of the SEC to vindicate the dignity and power of the Commission; hence, as in
criminal proceedings, an appeal would not lie from the order of
HELD: NO. We agree with respondents that the charge of contempt dismissal of, or an exoneration from, a charge of contempt."
partakes of the nature of a criminal offense. The exoneration of the
contemner from the charge amounts to an acquittal from which an At any rate, the SEC order directing respondents to show cause why
appeal would not lie. they should not be cited in contempt was highly improper. The CA
issued a TRO against the order of the SEC directing the Interport
A distinction is made between a civil and criminal contempt. Civil Resources Corporation to desist from holding the stockholders'
contempt is the failure to do something ordered by a court to be done meeting. Contrary to the view of petitioners, the effect of the TRO of
for the benefit of a party. A criminal contempt is any conduct directed the CA directing the SEC to desist from enforcing its own TRO was
against the authority or dignity of the court. to allow such meeting to proceed as scheduled. More, the CA in its
final decision nullified the SEC's order. Hence, there was no willful
"Civil contempt proceedings are generally held to be remedial and
disobedience to a lawful order of the SEC. Respondents were not
civil in their nature; that is, they are proceedings for the enforcement
guilty of contempt.
of some duty, and essentially a remedy for coercing a person to do
the thing required." In this case, the SEC issued the citation for contempt sua sponte.
There was no charge filed by a private party aggrieved by the acts of
"In general, civil contempt proceedings should be instituted by an
respondents. Strictly speaking, there was no disobedience to the
aggrieved party, or his successor, or someone who has a pecuniary
SEC's temporary restraining order. The Court of Appeals enjoined
interest in the right to be protected." If the contempt is initiated by the
that order. Consequently, respondents' act in proceeding with the
court or tribunal exercising the power to punish a given contempt, it
scheduled stock-holders' meeting was not contumacious as there
is criminal in nature, and the proceedings are to be conducted in
was no willful disobedience to an order of the SEC. The
accordance with the principles and rules applicable to criminal cases.
disobedience which the law punishes as constructive contempt
The State is the real prosecutor.
implies willfulness. For, at bottom, contempt is a willful disregard or
"The real character of the proceedings in contempt cases is to be disobedience.
determined by the relief sought or by the dominant purpose. The
The SEC was rather hasty in asserting its power to punish for
proceedings are to be regarded as criminal when the purpose is
contempt. The chairman and commissioners of the SEC must
primarily punishment, and civil when the purpose is primarily
exercise the power of contempt judiciously and sparingly with utmost
compensatory or remedial."
self-restraint.
"But whether the first or the second, contempt is still a criminal
proceeding in which acquittal, for instance, is a bar to a second

183 Aicka Singson


Notes in Remedial Law Review II
Provisional Remedies, Special Civil Actions, Special Proceedings and Evidence
School Year 2015-2016 REMEDIAL LAW REVIEW II - ATTY. BRONDIAL

b. SALVADOR SISON, complainant, vs. JUDGE JOSE F. the complainant never appeared to prove the charges against the
CAOIBES, JR., Presiding Judge, and TEODORO S. ALVAREZ, respondent judge, the facts averred in the complaint appear to be
substantially correct and true. Thus, the respondent judge abused
The instant administrative complaint arose when Salvador Sison, a his authority to charge and punish any person for indirect contempt
Metropolitan Manila Development Authority traffic enforcer, filed a under Rule 71 of the Rules of Civil Procedure. The Investigating
verified Complaint, charging Judge Jose F. Caoibes, Jr. and Sheriff Justice recommended that the respondent (judge) be admonished
Teodoro Alvarez of the Regional Trial Court with grave abuse of and warned, pursuant to Section 10(1), Rule 140 of the Rules of
authority. Court, and Section 11(c) of the same rule.

In turn, the complaint stemmed from an Order in Criminal Case The respondent judge anchors the justification of his acts against the
which the respondent judge issued, requiring the complainant to complainant on Section 3, Rule 71 of the Rules of Civil Procedure,
appear before him to explain a traffic incident involving his son and viz.:
the complainant. And giving Sison within a non-extendible period of
twenty-four (24) hours from receipt hereof, the accused is ordered to Sec. 3. Indirect contempt to be punished after charge and hearing. – After a
show cause why he should not be cited as in contempt of court and charge in writing has been filed, and an opportunity given to the respondent
dealt with accordingly. Because of the complainant’s failure to to comment thereon within such period as may be fixed by the court and to
be heard by himself or counsel, a person guilty of any of the following acts
appear before the respondent judge as directed, the latter, issued
may be punished for indirect contempt:
another Order for the complainant’s arrest and commitment, and for
the latter to appear for hearing before his sala. The respondent (a) Misbehavior of an officer of a court in the performance of his official
sheriff then served the order on the complainant. On the scheduled duties or in his official transactions;
hearing, the complainant appeared and executed an affidavit
admitting to the court that he made a mistake and that it was all a (b) Disobedience of or resistance to a lawful writ, process, order, or
judgment of a court, including the act of a person who, after being
misunderstanding. The respondent judge, thereafter, lifted Order.
dispossessed or ejected from any real property by the judgment or process
In his comment, the respondent judge vehemently denied the of any court of competent jurisdiction, enters or attempts or induces another
to enter into or upon such real property, for the purpose of executing acts of
accusations against him, contending that he was merely preserving
ownership or possession, or in any manner disturbs the possession given to
the dignity and honor due to the courts of law. The respondent judge
the person adjudged to be entitled thereto;
also alleged that he initiated the complaint for contempt pursuant to
the following provisions of the Revised Rules of Court: a) Section (c) Any abuse of or any unlawful interference with the processes or
3(d) and Section 4 of Rule 71; b) Section 5(c) of Rule 135; and, c) proceedings of a court not constituting direct contempt under section 1 of
the last paragraph of Section 3 of Rule 71. this Rule;

Thereafter, the complainant executed a Sinumpaang Salaysay ng (d) Any improper conduct tending, directly or indirectly, to impede, obstruct,
or degrade the administration of justice;
Pagbawi ng Reklamo, where he indicated that he was no longer
interested in pursuing the administrative complaint against the (e) Assuming to be an attorney or an officer of a court, and acting as such
respondent judge. According to the Investigating Justice, although without authority;

184 Aicka Singson


Notes in Remedial Law Review II
Provisional Remedies, Special Civil Actions, Special Proceedings and Evidence
School Year 2015-2016 REMEDIAL LAW REVIEW II - ATTY. BRONDIAL

(f) Failure to obey a subpoena duly served; was deliberately inserted so that the respondent could confront and
harass the complainant.
(g) The rescue, or attempted rescue, of a person or property in the custody
of an officer by virtue of an order or process of a court held by him. It is clear that the respondent Judge had no legitimate basis by which
to consider Sison’s apprehension of his son as indirect contempt. As
But nothing in this section shall be so construed as to prevent the court from
issuing process to bring the respondent into court, or from holding him in indicated earlier, the act complained against must be any of those
custody pending such proceedings. specified in Sec. 3, Rule 71, 1997 Rules of Civil Procedure;
otherwise, there is no contempt of court, which requires that the
Issue: WON the respondent judge committed grave abuse in person obstructed should be performing a duty connected with
declaring the complainant in contempt judicial functions. As such, the respondent Judge acted oppressively
and vindictively.
Held: YES. What triggered the contempt charge was, in fact, the
traffic violation incident involving the respondent judge’s son. Parenthetically, it is odd that the respondent Judge would even
Furthermore, the record shows that when the complainant filed his propose herein that Caoibes III, already 25 years at the time of the
reply to the charge as required by the respondent judge, the same apprehension, was serving his father as the latter’s personal driver,
was refused by some staff member in the latter’s sala. albeit not officially employed in the Judiciary. Most likely, therefore,
Caoibes III might not be doing anything for his father at the time of
In Cortes v. Bangalan, we held that a judge may not hold a party in his apprehension but was in the place for his own purposes.
contempt of court for expressing concern on the judge’s impartiality
through a motion for voluntary inhibition, even if the latter may have The act of a judge in citing a person in contempt of court in a manner
felt insulted therein. The Court also declared, thus: which smacks of retaliation, as in the case at bar, is appalling and
violative of Rule 2.01 of the Code of Judicial Conduct which
…[W]hile the power to punish in contempt is inherent in all courts so mandates that “a judge should so behave at all times to promote
as to preserve order in judicial proceedings and to uphold due public confidence in the integrity and impartiality of the judiciary.” The
administration of justice, judges, however, should exercise their very delicate function of administering justice demands that a judge
contempt powers judiciously and sparingly, with utmost restraint, and should conduct himself at all times in a manner which would
with the end in view of utilizing their contempt powers for correction reasonably merit the respect and confidence of the people, for he is
and preservation not for retaliation and vindication. the visible representation of the law. The irresponsible or improper
conduct of judges erodes public confidence in the judiciary; as such,
We agree with the Investigating Justice when he opined that the
a judge must avoid all impropriety and the appearance thereof.
respondent judge should have refrained from ordering the arrest and
detention of the complainant, since the incident involved his own
son, and the matter was very personal to him. The fact that the
respondent judge insisted that the complainant personally file his
comment in court gives rise to doubts as to the motive behind it; as
the Investigating Justice puts it, the requirement of personal filing

185 Aicka Singson


Notes in Remedial Law Review II
Provisional Remedies, Special Civil Actions, Special Proceedings and Evidence
School Year 2015-2016 REMEDIAL LAW REVIEW II - ATTY. BRONDIAL

c. JUDGE DOLORES L. ESPAÑOL vs. ATTY. BENJAMIN S. that Sharcons' TCT and other supporting documents are falsified and
FORMOSO and SPOUSES BENITO SEE and MARLY SEE, that respondents are responsible therefor.

G.R. No. 150949 June 21, 2007 Petitioner issued warrants of arrest against respondents. They were
confined in the municipal jail of Dasmariñas, Cavite. That same day,
FACTS: Sharcons Builders Philippines, Inc. (Sharcons) bought from respondents filed a motion for bail and a motion to lift the order of
Evanswinda Morales a piece of land in Paliparan, Dasmariñas, arrest. But they were denied outright by petitioner.
Cavite. However, when the latter’s workers tried to fence and take
possession of the lot, they were prevented by the caretaker of Respondents then filed with the CA a petition for a writ of habeas
spouses Joseph and Enriqueta Mapua. The caretaker claimed that corpus – GRNTED.
spouses Mapua are the owners of the land.
The CA ruled that Judge Español erred in taking cognizance of the
Sharcons filed with the RTC, Branch 90, Dasmariñas, Cavite a Decision rendered by then Judge Tagle in Civil Case No. 623-92
complaint for quieting of title, docketed as Civil Case No. 2035-00. since it was not offered in evidence in Civil Case No. 2035-00 for
Impleaded as defendants were spouses Mapua, Evanswinda quieting of title. Moreover, as the direct contempt of court is criminal
Morales, and the Register of Deeds of Trece Martires City. in nature, petitioner should have conducted a hearing. Thus, she
could have determined whether respondents are guilty as charged.
In their answer, spouses Mapua alleged, among others, that all the
documents relied upon by Sharcons are spurious and falsified. Petitioner filed a motion for reconsideration – DENIED.

In the course of the proceedings, Judge Dolores L. Español, Hence, this petition.
petitioner, issued an Order stating that Benito See and Marly See,
president and treasurer, respectively, of Sharcons, and its counsel, ISSUE: Whether petitioner erred in ruling that respondents are
Atty. Benjamin Formoso, respondents, have used a spurious guilty of direct contempt of court for using falsified documents
certificate of title and tax declaration when it (Sharcons) filed with the when Sharcons filed its complaint for quieting of title
RTC its complaint for quieting of title. Consequently, petitioner
HELD: YES.
declared respondents guilty of direct contempt of court and ordered
their confinement for ten (10) days in the municipal jail of The early case of In re Jones defined contempt of court as "some act
Dasmariñas, Cavite. or conduct which tends to interfere with the business of the court, by
a refusal to obey some lawful order of the court, or some act of
Petitioner stated that in determining the merits of Sharcons'
disrespect to the dignity of the court which in some way tends to
complaint for quieting of title, she "stumbled" upon Civil Case No.
interfere with or hamper the orderly proceedings of the court and
623-92 for cancellation of title and damages filed with the RTC,
thus lessens the general efficiency of the same." It has also been
Branch 20, Imus, Cavite, presided by then Judge Lucenito N. Tagle.
described as "a defiance of the authority, justice or dignity of the
Petitioner then took judicial notice of the judge’s Decision declaring
court; such conduct as tends to bring the authority and administration
of the law into disrespect or to interfere with or prejudice parties

186 Aicka Singson


Notes in Remedial Law Review II
Provisional Remedies, Special Civil Actions, Special Proceedings and Evidence
School Year 2015-2016 REMEDIAL LAW REVIEW II - ATTY. BRONDIAL

litigants or their witnesses during litigation." Simply put, it is despising rules of evidence in contempt proceedings are assimilated as far as
of the authority, justice, or dignity of the court. practicable to those adapted to criminal prosecutions. Perforce,
petitioner judge erred in declaring summarily that respondents are
In Narcida v. Bowen, this Court characterized direct contempt as one guilty of direct contempt and ordering their incarceration. She should
done "in the presence of or so near the court or judge as to obstruct have conducted a hearing with notice to respondents.
the administration of justice." It is a contumacious act done facie
curiae and may be punished summarily without hearing. In other d. ATTY. JOSE MANUEL DIOKNO and MONIQUE CU-UNJIENG
words, one may be summarily adjudged in direct contempt at the LA'O
very moment or at the very instance of the commission of the act of
contumely. Before the Court is a petition to cite respondents in contempt of
Court.
Indirect or constructive contempt, in turn, is one perpetrated outside
of the sitting of the court and may include misbehavior of an officer of Petitioner P/Supt. (Marantan) is the respondent in G.R. No. 199462,
a court in the performance of his official duties or in his official a petition filed but already dismissed although the disposition is not
transactions, disobedience of or resistance to a lawful writ, process, yet final. Respondent (La ‘O) is one of the petitioners in the said
order, judgment, or command of a court, or injunction granted by a case, while respondent (Atty. Diokno) is her counsel therein.
court or a judge, any abuse or any unlawful interference with the
G.R. No. 199462 relates to Criminal Cases pending before the
process or proceedings of a court not constituting direct contempt, or
(RTC), where Marantan and his co-accused are charged with
any improper conduct tending directly or indirectly to impede,
homicide. The criminal cases involve an incident, where Anton Cu-
obstruct or degrade the administration of justice.
Unjieng (son of respondent La’O), Francis Xavier Manzano, and
We agree with petitioner that the use of falsified and forged Brian Anthony Dulay, were shot and killed by police officers in front
documents is a contumacious act. However, it constitutes indirect of the AIC Gold Tower at Ortigas Center, which incident was
contempt not direct contempt. Pursuant to the above provision, such captured by a television crew from UNTV 37 (Ortigas incident).
act is an improper conduct which degrades the administration of
Marantan submits that the respondents (La’O) violated the sub judice
justice. In Santos v. Court of First Instance of Cebu, Branch VI, we
rule, making them liable for indirect contempt under Section 3(d) of
ruled that the imputed use of a falsified document, more so where
Rule 71 of the Rules of Court, for their contemptuous statements and
the falsity of the document is not apparent on its face, merely
improper conduct tending directly or indirectly to impede, obstruct or
constitutes indirect contempt, and as such is subject to such
degrade the administration of justice. He argues that their
defenses as the accused may raise in the proper proceedings. Thus,
pronouncements and malicious comments delved not only on the
following Section 3, Rule 71, a contemner may be punished only
supposed inaction of the Court in resolving the petitions filed, but
after a charge in writing has been filed, and an opportunity has been
also on the merits of the criminal cases before the RTC and
given to the accused to be heard by himself and counsel. Moreover,
prematurely concluded that he and his co-accused are guilty of
settled is the rule that a contempt proceeding is not a civil action, but
murder.
a separate proceeding of a criminal nature in which the court
exercises limited jurisdiction. Thus, the modes of procedure and the

187 Aicka Singson


Notes in Remedial Law Review II
Provisional Remedies, Special Civil Actions, Special Proceedings and Evidence
School Year 2015-2016 REMEDIAL LAW REVIEW II - ATTY. BRONDIAL

The respondents, in their Comment, argue that there was no For a comment to be considered as contempt of court "it must really
violation of the sub judice rule as their statements were legitimate appear" that such does impede, interfere with and embarrass the
expressions of their desires, hopes and opinions which were taken administration of justice. What is, thus, sought to be protected is the
out of context and did not actually impede, obstruct or degrade the all-important duty of the court to administer justice in the decision of
administration of justice in a concrete way; that no criminal intent a pending case. The specific rationale for the sub judice rule is that
was shown as the utterances were not on their face actionable being courts, in the decision of issues of fact and law should be immune
a fair comment of a matter of public interest and concern; and that from every extraneous influence; that facts should be decided upon
this petition is intended to stifle legitimate speech. evidence produced in court; and that the determination of such facts
should be uninfluenced by bias, prejudice or sympathies.
WON THE RESPONDENTS VIOLATED THE SUB JUDICE RULE
AND THEREFORE MUST BE CITED FOR INDIRECT CONTEMPT The power of contempt is inherent in all courts in order to allow them
to conduct their business unhampered by publications and
NO. The sub judice rule restricts comments and disclosures comments which tend to impair the impartiality of their decisions or
pertaining to the judicial proceedings in order to avoid prejudging the otherwise obstruct the administration of justice. As important as the
issue, influencing the court, or obstructing the administration of maintenance of freedom of speech, is the maintenance of the
justice. A violation of this rule may render one liable for indirect independence of the Judiciary. The "clear and present danger" rule
contempt under Sec. 3(d), Rule 71 of the Rules of Court,4 which may serve as an aid in determining the proper constitutional
reads: boundary between these two rights.

Section 3. Indirect contempt to be punished after charge and The "clear and present danger" rule means that the evil
hearing. – x x x a person guilty of any of the following acts may be consequence of the comment must be "extremely serious and the
punished for indirect contempt: degree of imminence extremely high" before an utterance can be
punished. There must exist a clear and present danger that the
(d) Any improper conduct tending, directly or indirectly, to impede,
utterance will harm the administration of justice. Freedom of speech
obstruct, or degrade the administration of justice.
should not be impaired through the exercise of the power of
The proceedings for punishment of indirect contempt are criminal in contempt of court unless there is no doubt that the utterances in
nature. This form of contempt is conduct that is directed against the question make a serious and imminent threat to the administration of
dignity and authority of the court or a judge acting judicially; it is an justice. It must constitute an imminent, not merely a likely, threat.
act obstructing the administration of justice which tends to bring the
The contemptuous statements made by the respondents allegedly
court into disrepute or disrespect. Intent is a necessary element in
relate to the merits of the case, particularly the guilt of petitioner, and
criminal contempt, and no one can be punished for a criminal
the conduct of the Court as to its failure to decide G.R. No. 199462.
contempt unless the evidence makes it clear that he intended to
commit it. As to the merits, the comments seem to be what the respondents
claim to be an expression of their opinion that their loved ones were
murdered by Marantan. This is merely a reiteration of their position in

188 Aicka Singson


Notes in Remedial Law Review II
Provisional Remedies, Special Civil Actions, Special Proceedings and Evidence
School Year 2015-2016 REMEDIAL LAW REVIEW II - ATTY. BRONDIAL

G.R. No. 199462, which precisely calls the Court to upgrade the For his part, respondent, on October 7, 2002, filed an Omnibus
charges from homicide to murder. The Court detects no malice on Motion to immediately allow him to inspect and photocopy the
the face of the said statements. The mere restatement of their documents and to compel petitioners to deposit with the court the
argument in their petition cannot actually, or does not even tend to, documents subject of the September 10, 2002 Order.
influence the Court.
On December 9, 2002, then Presiding Judge Bruselas issued an
As to the conduct of the Court, a review of the respondents' Order denying petitioners’ MR of the Order dated August 9, 2002
comments reveals that they were simply stating that it had not yet and considered respondent’s omnibus motion as a reiteration of his
resolved their petition. There was no complaint, express or implied, earlier motion for inspection and production of documents; thus, the
that an inordinate amount of time had passed since the petition was immediate implementation of the September 10, 2002 Order was
filed without any action from the Court. There appears no attack or simultaneously ordered.
insult on the dignity of the Court either.
Petitioners elevated the case to the CA via a petition for certiorari
e. CAPITOL HILLS GOLF & COUNTRY CLUB, INC. and PABLO assailing the Orders dated August 9, 2002 and December 9, 2002.
B. ROMAN, JR. vs. MANUEL O. SANCHEZ However, the CA denied the same in its Decision dated June 29,
2004. Petitioners’ MR was likewise denied on November 3, 2004. A
G.R. No. 182738 February 24, 2014 petition for review was filed before this Court, but We denied it per
Resolution dated January 10, 2005.
FACTS: On July 1, 2002, respondent Sanchez, a stockholder of
petitioner Capitol Hills filed a petition for the nullification of the annual In the meantime, respondent sought to enforce the September 10,
meeting of stockholders of May 21, 2002 and the special meeting of 2002 Order. The supposed inspection on September 30, 2002 was
stockholders. Respondent filed a Motion for Production and not held per the trial court’s Order dated September 27, 2002. The
Inspection of Documents, which the court granted. January 22, 2003 inspection also did not push through after
petitioners and their co-defendants again moved for its deferment.
Petitioners filed a MR which denied their motion for preliminary
When the court eventually denied their motion on June 16, 2003,
hearing. Subsequently, they filed a Supplement to Defendants’
respondent set the inspection to August 1, 2003. On said date,
Motion for Reconsideration, attaching therewith an alleged
however, Atty. Matias V. Defensor, then Corporate Secretary of the
certification issued by the National Printing Office to support their
Corporation, was alleged to be out of town and petitioner Pablo B.
contention of lack of cause of action on the grounds, among others,
Roman, Jr. (Roman) purported to have shown no willingness to
that the SEC Memorandum Circular No. 5, Series of 1996, as
comply with the directive. The matter was reported to the trial court,
amended, has not been duly published in accordance with law and
which merely noted respondent’s Report and Manifestation. On
jurisprudence. Pending resolution of the MR, petitioners filed on
November 3, 2003, respondent moved for the issuance of an order
January 21, 2003 a Motion for Deferment of Implementation of the
for immediate implementation of the September 10, 2002 Order, as
September 10, 2002 Order.
reiterated in the Order dated June 16, 2003, but the court denied the
same in its May 24, 2004 Order. Respondent’s motion for issuance

189 Aicka Singson


Notes in Remedial Law Review II
Provisional Remedies, Special Civil Actions, Special Proceedings and Evidence
School Year 2015-2016 REMEDIAL LAW REVIEW II - ATTY. BRONDIAL

of writ of execution suffered the same fate when the trial court denied Further sanctions shall be meted upon defendants should the Court
it on February 10, 2005. find that defendants have been in bad faith in complying with the
order of September 10, 2002 despite the order of this Court.
Respondent filed a Manifestation with Omnibus Motion for
Clarification and to Resolve Plaintiff’s Pending Motion for the SO ORDERED.”
Issuance of a Writ of Execution and to Set the Case for Pre-Trial
Conference. Acting thereon, Judge Ramon Paul L. Hernando, Petitioners questioned the aforesaid Resolution via Petition for
likewise now a member of the CA issued the July 10, 2006 Order, Certiorari (With Application for TRO and/or Writ of Preliminary
which directed the immediate execution of the September 10, 2002 Injunction). In resolving the petition, the CA ruled that there is no
Order, and set the case for pre-trial. indication that the RTC committed grave abuse of discretion
amounting to lack or excess of jurisdiction. It noted that, except for
During the January 11, 2007 inspection, the only document produced the sanctions contained therein, the assailed Resolution merely
by the Acting Corporate Secretary, Atty. Antonio V. Meriz, and one of reiterated the September 10, 2002 Order of Judge Bruselas, which
the staff, Malou Santos, was the Stock and Transfer Book of the petitioners did not dispute in accordance with Section 2, Rule 3 of
Corporation. They alleged that they could not find from the corporate the Interim Rules or via petition for certiorari. The CA further held
records the copies of the proxies submitted by the stockholders, that petitioners were not denied due process as they were able to
including the tape recordings taken during the stockholders’ move for a reconsideration of the September 10, 2002 Order, but not
meetings, and that they needed more time to locate and find the list opted to file the same with respect to the September 3, 2007
of stockholders as of March 2002, which was in the bodega of the Resolution.
Corporation. This prompted respondent to file a Manifestation with
Omnibus Motion praying that an order be issued in accordance with Petitioners moved to reconsider the CA Decision, but it was denied.
Section 3, Paragraphs (a) to (d) of Rule 29 of the Rules of Court
ISSUE: WON the recourse availed of by the petitioners was
(Rules), in relation to Section 4, Rule 3 of the Interim Rules of
proper
Procedure Governing Intra-Corporate Controversies under Republic
Act No. 8799 (Interim Rules). HELD: NO. A person guilty of disobedience of or resistance to a
lawful order of a court or commits any improper conduct tending,
On September 3, 2007, the trial court issued a Resolution “x x x
directly or indirectly, to impede, obstruct, or degrade the
ordering the defendants to strictly comply with the September 10,
administration of justice may be punished for indirect contempt. In
2002 Order. Failure of the defendants to comply with all the
particular, Section 4, Rule 3 of the Interim Rules states that, in
requirements of the order dated September 10, 2002 will result in
addition to a possible treatment of a party as non-suited or as in
this court citing all the defendants in contempt of court. This Court
default, the sanctions prescribed in the Rules for failure to avail of, or
shall order defendants solidarily to pay a fine of P10,000.00 for every
refusal to comply with, the modes of discovery shall apply. Under
day of delay to comply with the order of September 10, 2002 until the
Section 3, Rule 29 of the Rules, if a party or an officer or managing
defendants shall have fully and completely complied with the said
agent of a party refuses to obey an order to produce any document
order.
or other things for inspection, copying, or photographing or to permit

190 Aicka Singson


Notes in Remedial Law Review II
Provisional Remedies, Special Civil Actions, Special Proceedings and Evidence
School Year 2015-2016 REMEDIAL LAW REVIEW II - ATTY. BRONDIAL

it to be done, the court may make such orders as are just. The quoted provision of second paragraph, Section 4, Rule 71 of the
enumeration of options given to the court under Section 3, Rule 29 of Rules of Court, must be filed.
the Rules is not exclusive, as shown by the phrase "among others."
The Rules itself is explicit on this point:
If adjudged guilty of indirect contempt, the respondent who
committed it against a Regional Trial Court or a court of equivalent or In all other cases, charges for indirect contempt shall be commenced
higher rank may be punished with a fine not exceeding thirty by a verified petition with supporting particulars and certified true
thousand pesos, or imprisonment not exceeding six (6) months, or copies of documents or papers involved therein, and upon full
both. In this case, the threatened sanction of possibly ordering compliance with the requirements for filing initiatory pleadings for
petitioners to solidarily pay a fine of P10,000.00 for every day of civil actions in the court concerned. If the contempt charges arose
delay in complying with the September 10, 2002 Order is well within out of or are related to a principal action pending in the court, the
the allowable range of penalty. petition for contempt shall allege that fact but said petition shall be
docketed, heard and decided separately, unless the court in its
As far as the proceedings for indirect contempt is concerned, the discretion orders the consolidation of the contempt charge and the
case of Baculi v. Judge Belen is instructive: principal action for joint hearing and decision.

x x x Under the Rules of Court, there are two ways of initiating In this case, the proceedings for indirect contempt have not been
indirect contempt proceedings: (1) motu proprio by the court; or (2) initiated. To the Court’s mind, the September 3, 2007 Resolution
by a verified petition. could be treated as a mere reiteration of the September 10, 2002
Order. It is not yet a "judgment or final order of a court in a case of
First Mode. Sections 3 and 4, Rule 71 of the Rules of Court provide indirect contempt" as contemplated under the Rules. The penalty
the procedure to be followed in case of indirect contempt. First, there mentioned therein only serves as a reminder to caution petitioners of
must be an order requiring the respondent to show cause why he the consequence of possible non-observance of the long-overdue
should not be cited for contempt. Second, the respondent must be order to produce and make available for inspection and
given the opportunity to comment on the charge against him. Third, photocopying of the requested records/documents. In case of
there must be a hearing and the court must investigate the charge another failure or refusal to comply with the directive, the court or
and consider respondent’s answer. Finally, only if found guilty will respondent could formally initiate the indirect contempt proceedings
respondent be punished accordingly. pursuant to the mandatory requirements of the Rules and existing
jurisprudence.
As to the second mode of initiating indirect contempt proceedings,
that is, through a verified petition, the rule is already settled in Even if We are to treat the September 3, 2007 Resolution as a
Regalado v. Go: "judgment or final order of a court in a case of indirect contempt," this
would still not work to petitioners’ advantage. Section 11, Rule 71 of
In cases where the court did not initiate the contempt charge, the
the Rules of Court lays down the proper remedy from a judgment in
Rules prescribe that a verified petition which has complied with the
indirect contempt proceedings.
requirements of initiatory pleadings as outlined in the heretofore

191 Aicka Singson


Notes in Remedial Law Review II
Provisional Remedies, Special Civil Actions, Special Proceedings and Evidence
School Year 2015-2016 REMEDIAL LAW REVIEW II - ATTY. BRONDIAL

The recourse provided for in the above-mentioned provision is clear


enough: the person adjudged in indirect contempt must file an
appeal under Rule 41 (Appeal from the Regional Trial Courts) and
post a bond for its suspension pendente lite. Obviously, these were
not done in this case. Instead, petitioners filed a petition for certiorari
under Rule 65 of the Rules and did not post the required bond,
effectively making the September 3, 2007 Resolution final and
executory.

192 Aicka Singson


Notes in Remedial Law Review II
Provisional Remedies, Special Civil Actions, Special Proceedings and Evidence

Das könnte Ihnen auch gefallen